Anda di halaman 1dari 357

Captulo 1

Heurstica
La estrategia o la tactica para resolver problemas es llamada heurstica. En este
captulo nos ocuparemos de la heurstica para resolver problemas matematicos.
Aquellos que han pensado sobre la heurstica han descrito un n umero de ideas
basicas que son tpicamente usuales. Los cinco clasicos de la resoluci on de prob-
lemas de George Polya son obras maestras dedicadas enteramente al estudio
practico de la heurstica en matematicas. Entre las ideas desarrolladas en estos
libros, nos enfocaremos a las siguientes:
1. B usqueda de un patr on.
2. Dibujo de una gura.
3. Formulacion de un problema equivalente.
4. Modicaci on del problema.
5. Eleccion de una notacion efectiva.
6. Explotacion de la simetra.
7. Division en casos.
8. Trabajo hacia atr as.
9. Argumentaci on por contradiccion.
10. B usqueda de paridad.
11. Consideracion de casos extremos.
12. Generalizacion.
1
2 CAP

ITULO 1. HEUR

ISTICA
Nuestro interes en esta lista de ideas para resolver problemas no es describirlas
sino implementarlas. Al examinar ejemplos de c omo otros han utilizado estas
sencillas pero poderosas ideas, podemos esperar mejorar nuestras habilidades
para resolver problemas.
Antes de comenzar, un consejo acerca de los problemas al nal de las secciones:
no se preocupe por aplicar la heurstica tratada en esa seccion. Aunque los
problemas estan escogidos para practicar el uso de la heurstica en cuestin, un
enfoque estrecho puede ser psicol ogicamente debilitante. Un mismo problema
admite usualmente varias soluciones, a veces empleando diferentes heursticas.
As, es mejor atacar cada problema con la mente abierta que con una nocion
preconcebida de c omo una heurstica en particular debiera ser aplicada. Al tra-
bajar en un problema, resolverlo es lo importante. Es la experiencia acumulada
de todas las ideas trabajando simult aneamente lo que terminar a en una mejor
comprensi on de las posibilidades de resoluci on de un problema.
1.1. B usqueda de un Patron
Virtualmente todos los que resuelven un problema comienzan su analisis dandose
una idea del problema, convenciendose de la plausibilidad del resultado. Esto
se hace mejor examinando los casos especiales m as inmediatos; cuando esta
exploraci on es llevada a cabo de manera sistematica, pueden surgir patrones
que sugieren ideas para proceder con el problema.
1.1.1 Demuestre que un conjunto con n elementos (diferentes) tiene exacta-
mente 2
n
subconjuntos (diferentes).
Cuando el problema esta dado en este modo imperativo, un principiante puede
asustarse y no saber como proceder. Suponga, sin embargo, que el problema
fuera propuesto como una indagacion, tal como:
(i) Cu antos subconjuntos pueden formarse de un conjunto de n elementos?
(ii) Demuestre o de un contraejemplo: Un conjunto con n elementos tiene 2
n
subconjuntos.
En cualquiera de estas formas ya esta implcita la sugerencia de que deber a comen-
zar revisando unos cuantos casos particulares. As es como cada problema de-
biera ser atacado: permanecer esceptico respecto al resultado hasta no estar
plenamente convencido.
Solucion 1. Empezaremos por analizar lo que sucede cuando el conjunto con-
tiene 0, 1, 2, 3 elementos; los resultados se muestran en la siguiente tabla:
1.1. B

USQUEDA DE UN PATR

ON 3
n elementos subconjuntos n um.de subconjuntos
0 ninguno 1
1 x
1
, {x
1
} 2
2 x
1
, x
2
, {x
1
}, {x
2
}, {x
1
, x
2
} 4
3 x
1
, x
2
, x
3
{x
3
}, {x
1
, x
3
}, {x
2
, x
3
}, {x
1
, x
2
, x
3
}, , {x
1
}, {x
2
}, {x
1
, x
2
} 8
Nuestro prop osito al construir esta tabla no es s olo vericar el resultado, sino
tambien buscar patrones que pudieran sugerir c omo proceder en el caso general.
As, nuestro objetivo es ser tan sistematico como sea posible. En este caso, notese
que cuando n = 3, hemos enlistado primero los subconjuntos de x
1
, x
2
y luego,
en la segunda lnea, cada uno de los subconjuntos aumentados por el elemento
x
3
. Esta es la idea clave que nos permite proseguir con valores m as grandes de n.
Por ejemplo, cuando n = 4, los subconjuntos de S = x
1
, x
2
, x
3
, x
4
son los ocho
subconjuntos de x
1
, x
2
, x
3
(mostrados en la tabla) junto con los ocho formados
al agregar x
4
a cada uno de estos. Estos dieciseis subconjuntos construyen toda
la coleccion de posibilidades; as, un conjunto de 4 elementos tiene 2
4
(= 16)
subconjuntos.
Una demostracion basada en esta idea es una aplicaci on facil de induccion
matematica (ver seccion 2.1)
Solucion 2. Otro camino para presentar la idea de la ultima soluci on es argu-
mentar como sigue. Para n, sea A
n
el n umero de subconjuntos (diferentes) de un
conjunto con n elementos. Sea S un conjunto con n + 1 elementos, y desgnese
uno de sus elementos como x. Hay una correspondencia uno a uno entre aque-
llos subconjuntos de S que no contienen a x (propiamente, un subconjunto T
del tipo anterior se corresponde con T x ). Los tipos anteriores son todos
subconjuntos de S x, un conjunto con n elementos, y as, debe darse el caso
de que A
n+1
= 2A
n
.
Esta relacion de recurrencia, verdadera para n = 0, 1, 2, 3, ..., combinando con el
hecho A
0
= 1, implica que A
n
= 2
n
. (A
n
= 2A
n1
= 2
2
A
n2
= ... = 2
n
A
0
= 2
2
)
Solucion 3. Otra enumeracion sistematica de los subconjuntos puede ser llevada
a cabo construyendo un arbol .
4 CAP

ITULO 1. HEUR

ISTICA
Cada rama del arbol corresponde a un subconjunto diferente de S (la barra
sobre el nombre del elemento signica que no esta incluido en el subconjunto
correspondiente a esa rama). El arbol esta construido en tres etapas, correspon-
dientes a los tres elementos de S. Cada elemento de S lleva a dos posibilidades:
esta contenido en el subconjunto de S o no lo esta, y estas opciones estan rep-
resentadas por dos ramas. Como cada elemento es considerado, el n umero de
ramas se duplica. As, para un conjunto de tres elementos el n umero de ramas
es 2 2 2 = 8. Para un conjunto de n elementos el n umero de elementos es
2 2 ... 2
. .
n veces
= 2
n
;
as, un conjunto con n elementos tiene 2
n
subconjuntos.
Solucion 4. Supongase que enumeramos los subconjuntos de acuerdo a su
tama no. Por ejemplo, cuando S = a, b, c, d, los subconjuntos son:
n um. de elementos n um. de subconjuntos
0 1
1 {a}, {b}, {c}, {d} 4
2 {a, b}, {a, c}, {a, d}, {b, c}, {b, d}, {c, d} 6
3 {a, b, c}, {a, b, d}, {a, c, d}, {b, c, d} 4
4 {a, b, c, d} 1
Este comienzo podra propiciar el siguiente argumento. Sea S un conjunto con
n elementos. Entonces el n umero de subconjuntos de S =

n
0
(n umero de
subconjuntos de S con k elementos).
=
n

0
_
n
k
_
= 2
n
El paso nal en esta cadena de igualdades se sigue del teorema del binomio,
(x +y)
n
=
n

0
_
n
k
_
x
k
y
nk
,
haciendo x = 1 y y = 1
Solucion 5. Otro comienzo sistematico esta ilustrado en la siguiente tabla,
donde se enlistan los subconjuntos de S = x
1
, x
2
, x
3
. Para entender el pa-
tron aqu, notese la correspondencia entre los objetos de la columna extrema
izquierda y la ocurrencia de 1s en la segunda columna de triadas
1.1. B

USQUEDA DE UN PATR

ON 5
Subconjunto Triada N um. binario N um. decimal
(0,0,0) 0 0
x
3
(0,0,1) 1 1
x
2
(0,1,0) 10 2
x
2
, x
3
(0,1,1) 11 3
x
1
(1,0,0) 100 4
x
1
, x
3
(1,0,1) 101 5
x
1
, x
2
(1,1,0) 110 6
x
1
, x
2
, x
3
(1,1,1) 111 7
Especcamente, si A es un subconjunto de S = x
1
, x
2
, ...., x
n
, defnase a
i
,
para i = 1, 2, ..., n, como
a
i
=
_
1 si xi A
0 si xi / A
Es claro que ahora podemos identicar un subconjunto Ade S , con (a
1
, a
2
, ..., a
n
),
una n-ada de 0s y 1s. Inversamente a cada n-upla le corresponde un unico sub-
conjunto de S. Luego el n umero de subconjuntos de S es igual al n umero de
n-uplas de 0s y 1s. Este ultimo conjunto esta obviamente en correspondencia
uno-a-uno con el conjunto de los n umeros binarios no negativos menores que 2
n
corresponde a exactamente un subconjunto de S, e inversamente. Por lo tanto,
debe suceder que S tenga 2
n
subconjuntos.
Normalmente, daremos una soluci on a cada ejemplo, una soluci on que sirva
para ilustrar la heurstica bajo consideraci on. En este primer ejemplo, sin em-
bargo, sencillamente queramos reiterar la observaci on anterior de que un mismo
problema pueda ser trabajado en una variedad de formas. La leccion que debe
aprenderse es que uno debe permanecer exible en las primeras etapas de explo-
raci on de problemas. Si un acercamiento no parece llevar a ninguna parte, no
hay que desesperarse, sino hay que buscar una idea nueva. No se quede jo en
una sola idea hasta que no haya tenido la oportunidad de pensar ampliamente
en una variedad de acercamientos alternativos.
1.1.2 Den otese por S
n,0
, S
n,1
, S
n,2
a la suma de cada tercer elemento en el n-
esimo rengl on del tri angulo de pascal, comenzando por la izquierda con el primer
elemento, el segundo elemento, y el tercer elemento respectivamente. Haga una
6 CAP

ITULO 1. HEUR

ISTICA
conjetura sobre el valor de S
100,1
Tri angulo de Pascal n S
n,0
S
n,1
S
n,2
1 0 1
+
0 0
1 1 1 1 1 0

1 2 1 2 1 2
+
1
1 3 3 1 3 2

3 3
1 4 6 4 1 4 5 5 6
+
1 5 10 10 5 1 5 11 10

11
1 6 15 20 15 61 6 22
+
21 21
1 7 21 35 35 21 7 1 7 43 43 42

Solucion Empezaremos por examinar las casillas de orden inferior con la es-
peranza de encontrar patrones que pudieran generalizarse. En la tabla 1.2, los
terminos que estan subrayados con una sola raya y doblemente subrayados son
los sumandos de S
n,1
, S
n,2
, respectivamente. Las tres columnas de la derecha
muestran que, en cada caso, dos de las sumas son iguales, mientras que la tercera
es mayor (indicada por un suprandice +) o menor (indicada por un suprandice
-). Tambien se ve que el termino desigual en esta sucesion cambia en un ciclo
de seis. As, del patr on establecido en los primeros renglones, esperamos que la
anomala n = 8 ocurra en la columna de enmedio y sea uno m as que las otras
dos.
Sabemos que S
n,0
+ S
n,1
+ S
n,2
= 2
n
(ver 1.1.1). Ya que 100 = 6 16 + 4,
esperemos que el termino desigual ocurra en la tercera columna (S
100,2
) y que
sea uno m as que las otras dos. As S
100,0
= S
100,1
= S
100,2
1 y S
100,1
+S
100,1
+
S
100,1
= 2
100
. Estas ecuaciones nos llevan a conjeturar que S
100,1
=
(2
100
1)
3
Una demostracion formal de esta conjetura es una aplicaci on directa de induc-
cion matematica (ver captulo 2).
1.1.3 Establezca las condiciones necesarias y sucientes sobre x
1
y x
2
para que
x
n
sea un n umero entero para una innidad de valores de n. Donde x
1
, x
2
, x
3
, ...
es una sucesi on de n umeros reales distintos de cero que satisfacen
x
n
=
x
n2
x
n1
2x
n2
x
n1
n = 3, 4, 5, ...
Soluci on. Para darse una idea de la sucesion, calcularemos los primeros termi-
nos, expresandolos en terminos de x
1
y x
2
. Tenemos (omitiendo el algebra)
1.1. B

USQUEDA DE UN PATR

ON 7
x
3
=
x
1
x
2
2x
1
x
2
,
x
4
=
x
1
x
2
3x
1
2x
2
,
x
5
=
x
1
x
2
4x
1
3x
2
.
Somos afortunados en este caso en particular de que los c alculos sean realizables
y de que emerja un patr on. Un sencillo argumento por induccion establece que
x
n
=
x
1
x
2
(n 1) x
1
(n 2) x
2
,
lo cual, al aislar el coeciente para n toma la forma
x
n
=
x
1
x
2
(x
1
x
2
) n + (2x
2
x
1
)
.
De este modo, vemos que si x
1
,= x
2
, el denominador exceder a eventualemente
al numerador, en magnitud, de forma que x
n
no sera un entero. Sin embargo, si
x
1
= x
2
, todos lo terminos de la sucesion son iguales. As, x
n
es un entero para
un n umero innito de valores de n si y s olo si x
1
= x
2
.
1.1.4 Encuentre n umeros positivos n y a
1
, a
2
..., a
n
tales que a
1
+a
2
+... +a
n
=
1000 y el producto a
1
a
2
...a
n
sea tan grande como se pueda.
Soluci on Cuando un problema involucra un parametro que hace el analisis
complicado, frecuentemente es de ayuda en la etapa de descubrimiento el reem-
plazo temporal con algo m as manejable. En este problema, podemos comenzar
por examinar una sucesion de casos particulares obtenidos al reemplazar 1000
por 2, 3, 4, 5, 6, 7, 8, 9, ... . En esta forma llegamos a descubrir que en un producto
m aximo
(i) ning un a
i
sera mayor que 4,
(ii) ning un a
i
sera igual a 1,
(iii) todas las a
i

s pueden ser tomadas como 2 o 3 (porque 4 = 22 y 4 = 2+2),
(iv) a los m as dos a
i

s seran iguales a 2 (porque 2 2 2 < 3 3 y 2 +2 +2 =


3 + 3).
Cada una de estas es facil de establecer. As cuando el par ametro es 1000 como
en el problema en cuestion, el producto m aximo debe ser 3
332
2
2
.
8 CAP

ITULO 1. HEUR

ISTICA
1.1.5 Muestre que x y = y x para toda x, y en S. Donde S un conjunto y
una operaci on binaria sobre S que satisface las dos leyes
x x = x
(x y) z = (y z) x
Soluci on La soluci on, que aparece tan transparentemente abajo, es en realidad
el resultado de una cantidad considerable de trabajo; el procedimento puede ser
descrito s olo como la b usqueda de un patr on (el patr on gua esta en la naturaleza
cclica de los factores en la segunda condici on). Tenemos, para toda x, y en S,
x y = (x y) (x y) = [y (x y)] x = [(x y) x] y
= [(y x) x] y = [(x x) y] y = [(y y)] (x x)
= y x.
Problemas Desarrolle una idea para los siguientes problemas mediante la
b usqueda de patrones.
Haga conjeturas apropiadas, y piense c omo podran llevarse a cabo las demostra-
ciones.
1.1.6 Comenzando con 2 y 7, la sucesi on 2, 7, 1, 4, 7, 4, 2, 8, ... se construye al
multiplicar parejas sucesivos de sus terminos y a nadiendo el resultado como el
siguiente termino o los dos siguientes terminos de la sucesi on, dependiendo de
si el producto es un n umero de uno o dos dgitos. Demuestre que el dgito 6
aparece un n umero innito de veces en la sucesi on.
1.1.7 Den otese por S
1
a la sucesi on de los enteros positivos 1, 2, 3, 4, 5, 6, ..., y
defnase S
n+1
en terminos de S
n
a nadiendo 1 a aquellos enteros en S
n
que son
divisibles por n. As, por ejemplo, S
2
es 2, 3, 4, 5, 6, 7, ..., S
3
es 3, 3, 5, 5, 7, 7, ...
Determine aquellos enteros n con la propiedad de que los primeros n1 enteros
en S
n
son n.
1.1.8 Demuestre que puede hacerse una lista de todos los subconjuntos de un
conjunto nito de tal forma que
1.2. DIBUJO DE UNA FIGURA 9
(i) El conjunto vaco sea primero en la lista.
(ii) Cada subconjunto aparezca s olo una vez, y
(iii) Cada subconjunto en la lista se obtenga ya sea al a nadir un elemento al
subconjunto precedente o al borrar un elemento del subconjunto precedente.
1.1.9 Determine el n umero de coecientes binomiales impares en la expansi on
de (x +y)
100
.
1.1.10 Un teorema bien conocido asegura que un primo p > 2 puede ser escrito
como la suma de dos cuadrados perfectos (p = m
2
+ n
2
, con m y n enteros) si
y s olo si p es uno m as que un m ultiplo de 4. Haga una conjetura sobre cu ales
primos p > 2 pueden ser escritos en cada una de las siguientes formas, utilizando
enteros x y y (no necesariamente positivos): (a) x
2
+16y
2
, (b) 4x
2
+4xy +5y
2
.
(ver 1.5.10)
1.1.11 Si (a
n
) es una sucesi on tal que para n 1, (2a
n
)a
n+1
= 1, que sucede
con a
n
cuando n tiende hacia innito?
1.1.12 Sea S un conjunto, y sea una operaci on binaria sobre S que satisface
las leyes
x (x y) = y
(y x) x = y
Demuestre que x y = y x para todas x, y en S.
Ejemplos adicionales
La mayor parte de los problemas de induccion estan basados en el descubrim-
iento de un patr on. As, los problemas en las secciones 2.1, 2.2, 2.3, 2.4 ofrecen
practica adicional para esta heurstica. Vease tambien 1.7.2, 1.7.7, 1.7.8, 2.5.6.
1.2. Dibujo de una Figura
Es de mucha ayuda describir un problema gr acamente siempre que sea posible,
por medio de una gura, un diagrama, o una gr aca. Una representaci on gr aca
usualmente hace m as facil asimiliar los datos relevantes y notar las relaciones y
las dependencias.
10 CAP

ITULO 1. HEUR

ISTICA
1.2.1 Una cuerda de longitud constante se desliza a lo largo de un semicrculo.
El punto medio de la cuerda y las proyecciones de sus extremos son los vertices
de un tri angulo. Demuestre que el tri angulo es is osceles y que nunca cambia de
forma.
Figura 1.1.
Soluci on. Denotese por AB la base del semicrculo, y sean XY la cuerda, M
el punto medio de XY , C y D las proyecciones de X y Y sobre AB (g. 1.2)
Denotese la proyeccion de M sobre AB como N. Entonces N es el punto medio
de CD y se sigue que el tri angulo CMD es isosceles.
Para mostrar que la forma del tri angulo es independiente de la posici on de la
cuerda, basta mostrar que el MCD permanece sin cambio, o equivalentemente,
que el XCM es constante, para todas las posiciones de XY . Para ver que
tal es el caso, extiendase XC hasta cortar el crculo completo en Z (g. 1.3).
Entonces CM es paralela a ZY (C y M son los puntos medios de XZ y XY
respectivamente), y consecuentemente XCM = XZY es igual a la mitad
del arco XY , y este arco depende s olo de la longitud de la cuerda XY . Esto
completa la demostracion.
Uno podra preguntarse: Como fue que se le ocurri o a alguien extender XC de
esta manera? Este es precisamente el paso que hace el argumento tan bonito, y
es ciertamente un paso difcil de motivar. Casi todo lo que puede decirse es que
el uso de lneas y arcos auxiliares (frecuentemente encontrados por reexion,
extensi on, o rotacion) es una practica com un en geometra. El s olo hecho de
tomar esto en cuenta contribuir a a las posibles aproximaciones a un problema
dado.
Otro acercamiento interesante a este problema es dar coordenadas a los puntos
y proceder analticamente. Para mostrar que el tri angulo es independiente de la
posici on de la cuerda, basta con mostrar que la razon entre la altura y la base,
MN/CD, es constante.
1.2. DIBUJO DE UNA FIGURA 11
Figura 1.2.
Figura 1.3.
Sea O el punto medio de AB, y sea = Y OB. Es claro que toda la congu-
raci on esta determinada por (g. 1.4)
Sea = XOY. Usando esta notacion,
CD = cos cos( +)
MN =
sen +sen( +)
2
.
y la razon altura-base es
F() =
sen +sen( +)
2(cos cos ( +))
, 0
No es inmediatamente claro que esta cantidad es independiente de ; este es el
contenido de 1.8.1 y 6.6.7
12 CAP

ITULO 1. HEUR

ISTICA
1.2.2 Una partcula que se mueve sobre una lnea recta comienza del reposo y
alcanza una velocidad v
0
despues de recorrer una distancia s
0
. Si el movimiento
es tal que la aceleraci on nunca aumenta, encuentre el tiempo m aximo para el
recorrido.
Figura 1.4.
Soluci on Concentre la atencion en la gr aca de la velocidad v = v(t) (g.
1.5). Tenemos que v(0) = 0, y la gr aca de v nunca es c oncava hacia arriba
(distancia recorrida=
_
t
0
v(t)dt). A partir de esta representaci on, es claro que
maximizaremos el tiempo de recorrido cuando la curva v(t) de 0 a P sea una
lnea recta (g. 1.6). En el tiempo m aximo t
0
,
1
2
t
0
v
0
= s
0
, o equivalentemente,
t
0
= 2s
0
/v
0
.
Figura 1.5.
1.2.3 Si a y b son enteros positivos sin factores comunes, muestre que
__
a
b

+
__
2a
b

+... +
__
(b1)a
b
__
=
(a 1)(b 1)
2
Soluci on Cuando b = 1, vemos que la suma de la izquierda es 0 y as se cumple
el resultado.
No es claro c omo una gura podra ser de ayuda para establecer esta identidad
puramente aritmetica. Ademas, la armaci on involucra dos variables indepen-
1.2. DIBUJO DE UNA FIGURA 13
dientes, a y b, y
a
b
,
2a
b
,
3a
b
, .... son los valores de la funci on f(x) =
ax
b
cuando
x = 1, 2, 3, ...., respectivamente.
Es posible interpretar
_
a
b

,
_
2a
b

, .... geometricamente?
Para concretar, considerese el caso a = 5 y b = 7. Los puntos P
k
= (k, 5k/7), k =
1, 2, ..., 6, estan cada uno en la recta y = 5k/7, y
_
5k
7

es igual al n umero de
puntos de la retcula en la lnea vertical a traves de P
k
que esta por encima
del eje x y abajo de P
k
. As,

k=1
6
_
5k
7

es igual al n umero de puntos de la


retcula que estan dentro del ABC (ver g. 1.7). Por simetra, este n umero
es la mitad del n umero de p untos de la retcula que estan en el interior del
rectangulo ABCD. Hay 4 6 = 24 puntos de la retcula en ABCD, lo que
signica que el tri angulo ABC contiene 12 puntos de la retcula en su interior.
Figura 1.6.
El mismo argumento puede aplicarse en el caso general. La condici on de que a y
b no tiene factores comunes nos asegura que ninguno de los puntos de la retcula
interior de ABCD caer a en la recta y = ax/b. As,
k=1

b1
_
ka
b
_
=
1
2
k
(donde k es el n umero de puntos de la retcula en el interior de ABCD)
=
(a 1)(b 1)
2
1.2.4 (El problema de los saludos de mano)El Sr. y la Sra. Adams fueron
hace poco a una esta en la que haba otras tres parejas. Ocurrieron varios
saludos de mano. Nadie salud o a su propio c onyugue, ni a la misma persona
dos veces, y por supuesto, nadie se salud o a si mismo.
14 CAP

ITULO 1. HEUR

ISTICA
Despues de que los saludos hubieron terminado, el Sr. Adams le pregunt o a cada
persona, incluida su esposa, a cu antas personas haba saludado?. Para sorpresa
suya, cada quien di o una respuesta diferente. A cu antas personas salud o la Sra.
Adams?
Soluci on. Aunque un diagrama no es propiamente la soluci on , es de gran
ayuda visualizar los datos gr acamente de la siguiente manera. Representese a
los ocho individuos con los ocho puntos que muestra la gura 1.7
Figura 1.7.
Figura 1.8.
Ahora bien, las respuestas a la encuesta del Sr. Adams deben haber sido los
n umeros 0, 1, 2, 3, 4, 5, 6. As, uno de los individuos, A digamos, salud o a otros
seis, digamos B, C, D, E, F, G. Indquese esto en la gr aca dibujando segmentos
desde A a estos puntos, como en la gura 1.8.
En este diagrama vemos que H debe haber sido la persona que no salud o a
nadie. Ademas, A y H deben ser esposos, pues A ha saludado a seis personas,
sin contar a su c onyugue.
1.2. DIBUJO DE UNA FIGURA 15
Por suposici on, alguno entre B, C, D, E, F, G ha saludado a cinco personas. Re-
etiquetando de ser necesario, podemos suponer que se trata de la persona B
salud o son A, C, D, E, F. Esto se muestra en la gura 1.9. A partir de este
dibujo vemos facilmente que G es la unica persona que pudo haber contestado
uno , y B y G deben estar casados.
De nuevo, como antes, re-etiquetando los puntos C, D, E, F de ser necesario,
podemos suponer que C salud o a cuatro personas y que estas eran A, B, D, E.
El diagrama correspondiente se muestra en la gura 1.10. Usando el mismo
razonamiento, F y C son esposos, y consecuentemente, D y E son esposos.
D y E han ambos saludado a otros tres. Dado que el Sr. Adams no recibi o dos
respuestas tres , D y E deben corresponder al Sr. y la Sra. Adams ; es decir,
el Sr. Adams salud o a otros tres.
Figura 1.9.
Figura 1.10.
Problemas
16 CAP

ITULO 1. HEUR

ISTICA
1.2.5 Dos postes, de alturas a y b, est an a una distancia d (con respecto al
nivel del suelo). Un alambre se extiende desde la parte superior de cada uno de
ellos a un punto P en el suelo entre ellas. En d onde debera estar localizado P
para minimizar la longitud total del cable? (Sugerencia: Suponga que los postes
se encuentran en los puntos C y D, que sus partes superiores son A y B, respec-
tivamente. Deseamos minimizar AP +PB. Aumente este diagrama reej andolo
en la lnea base CD. Suponga que B se reeja a B

(PB = PB

). Ahora el
problema es: En d onde debera estar localizado P para minimizar AP +PB

?)
1.2.6 Sea ABC un tri angulo de angulos agudos, y sea D en el interior del seg-
mento AB. Localice los puntos E sobre AC y F sobre CB tales que el tri angulo
inscrito DEF tenga permetro mnimo.(Sugerencia: Reeje D sobre la lnea
AC a un punto D

; reeje D sobre CB a un punto D

y considere el segmento
D

).
1.2.7 Un cuarto rectangular mide 30 pies de largo y 12 pies de altura, y los
extremos se encuentran a 12 pies de ancho. Una mosca, con un ala rota, se
encuentra en un punto un pie abajo del techo y a la mitad de un extremo. Un
pedazo de comida se encuentra localizado un pie arriba del piso y a la mitad
del otro extremo. La mosca tiene unicamente la energa para caminar 40 pies.
Muestre que hay un camino por el cual la mosca pueda caminar y conseguir la
comida.
1.2.8 Los tri angulos equilateros ABP y ACQ est an construidos externamente
sobre los lados AB y AC del tri angulo ABC. Pruebe que CP = BQ. (Sugeren-
cia: Para encontrar una soluci on bonita, rote el plano del tri angulo 60
0
por el
punto A, en una direcci on que lleve B en la direcci on de C. Que sucede con el
segmento CP?)
1.2.9 Sean a y b n umeros enteros positivos tales que a < b. Si dos puntos son
seleccionados al azar de un segmento de longitud b, Cu al es la probabilidad de
que la distancia entre ellos sea al menos a?(Sugerencia: Sean x y y los n umeros
escogidos al azar del intervalo [0, b], y considere a estas variables aleatorias
independientes sobre dos ejes separados. Que area corresponde a [x y[ a?
).
1.2.10 De una interpretaci on geometrica al siguiente problema. Sea f difer-
enciable con f continua en [a, b]. Muestre que si existe un n umero c en (a, b] tal
que f

(c) = 0, entonces podemos encontrar un n umero d en (a, b) tal que


f

(d) =
f(d) f(a)
b a
.
1.3. FORMULACI

ON DE UN PROBLEMA EQUIVALENTE 17
1.2.11 Sea a, b n umeros reales, a < b. Indique geometricamente la ubicaci on
precisa de cada uno de los siguientes n umeros:
(a+b)
2
(=
1
2
a +
1
2
b);
2
3
a +
1
3
b;
1
3
a +
2
3
b; [m/(m + n)]a + [n/(m + n)]b, donde m > 0 y n > 0. Este ultimo n umero
corresponde al centro de gravedad de un sistema de dos masas, uno de masa m,
localizado en a, y el otro, de masa n, localizado en b.
1.2.12 Use la gr aca de y = senx para mostrar lo siguiente. Dado un tri angulo
ABC,
(a)
SenB+SenC
2
Sen
B+C
2
(b)
m
m+n
SenB +
n
m+n
SenC Sen(
m
m+n
B +
m
m+n
C), m > 0, n > 0
1.2.13 Use un diagrama (un arreglo rectangular (a
i
a
j
)) para mostrar que
(a)

n
i=0

n
j=0
a
i
a
j
=

n
j=0

n
i=0
a
i
a
j
,
(b)

n
j=0

n
i=j
a
i
a
j
=

n
i=0

n
j=i
a
i
a
j
,
(c) (

n
i=0
a
i
)
2
=

n
i=0

n
j=0
a
i
a
j
= 2

n
i=0

i
j=0
a
i
a
j

n
i=0
a
2
i
.
Ejemplos adicionales
1.3.11, 1.9.2, 1.9.4, 1.11.3, 2.1.3, 2.5.5, 2.6.11
1.3. Formulacion de un Problema Equivalente
El mensaje de la seccion precedente es que el primer paso para resolver un prob-
lema es juntar informaci on, explorar, entender, relacionar, conjeturar, analizar.
Pero, que pasa cuando no es posible hacer esto de manera signicativa, sea
porque los c alculos se vuelven muy complicados o porque el problema no admite
casos especiales que nos hagan vislumbrar la soluci on? En esta seccion consid-
eraremos algunos problemas de este tipo. La recomendaci on de esta seccion es
tratar de reformular el problema a una forma equivalente pero m as simple. El
llamado es a la imaginacion y creatividad propia. Algunas tecnicas estandar de
reformulacion involucran c alculos algebracos o manipulaciones trigonometric-
as, substitucion o cambio de variable, uso de correspondencias biunvocas, y
reinterpretacion del lenguaje de otra materia (algebra, geometra, analisis, com-
binatoria, etc.)
18 CAP

ITULO 1. HEUR

ISTICA
1.3.1 Encuentre una f ormula general para la n-esima derivada de f(x) =
1/(1 x
2
).
Soluci on Un paso simplicador com un al trabajar con funciones racionales es
escribir la funci on como una suma de fracciones parciales. En este caso,
f(x) =
1
2
_
1
1 x
+
1
1 +x
_
y en esta forma es facil mostrar que
f
(n)
(x) =
n!
2
_
1
(1 x)
n+1
+
(1)
n
(1 +x)
n+1
_
1.3.2 Encuentre todas las soluciones de x
4
+x
3
+x
2
+x + 1 = 0
Soluci on Esta ecuaci on puede resolverse dividiendo entre x
2
, luego substituyen-
do y = x+1/x, y aplicando la formula para resolver ecuaciones de segundo grado.
Tenemos
x
2
+
1
x
2
+x +
1
x
+ 1 = 0,
_
x
2
+ 2 +
1
x
2
_
+
_
x +
1
x
_
+ (1 2) = 0,
_
x +
1
x
_
2
+
_
x +
1
x
_
1 = 0,
y
2
+y 1 = 0,
Las races de esta ecuaci on son
y
1
=
1 +

5
2
, y
2
=
1

5
2
.
Solo queda determinar x resolviendo las ecuaciones
x +
1
x
= y
1
y x +
1
x
= y
2
,
Que son equivalentes a
x
2
y
1
x + 1 = 0 y x
2
y
2
x + 1 = 0.
1.3. FORMULACI

ON DE UN PROBLEMA EQUIVALENTE 19
Las cuatro races encontradas al resolver estas son
x
1
=
1 +

5
4
+i
_
10 + 2

5
4
,
x
2
=
1 +

5
4
i
_
10 + 2

5
4
,
x
3
=
1

5
4
+i
_
10 2

5
4
,
x
4
=
1

5
4
i
_
10 2

5
4
.
Otra aproximacion a este problema es multiplicar cada lado de la ecuaci on
original por x 1. Ya que
(x 1)(x
4
+ x
3
+ x
2
+ x + 1) = x
5
1, un problema equivalente es encontrar
todas las x que satisfagan x
5
= 1. Estas son las cinco races de la unidad.
Como una consecuencia de haber trabajado este problema en dos formas difer-
entes, vemos que
Cos
2
5
+iSen
2
5
=
1 +

5
4
+i
_
10 + 2

5
4
.
Al igualar las partes real e imaginarias se obtiene
Cos72
0
=
1 +

5
4
, Sen72
0
= i
_
10 + 2

5
4
(Formulas similares pueden ser obtenidas para x
2
, x
3
, y x
4
)
1.3.3 Sea P un punto dentro de un tri angulo dado ABC, D, E, F son los
pies de las perpendiculares desde P a las lneas BC, CA, AB, respectivamente.
Encuentre todos los P para los cuales
BC
PD
+
CA
PE
+
AB
PF
es mnimo.
20 CAP

ITULO 1. HEUR

ISTICA
Figura 1.11.
Soluci on Denotense las longitudes de BC, AC, AB, por a, b, c, respectivamente
(ver gura 1.11). Queremos minimizar
a
p
+
b
q
+
c
r
.
Notar que
Area ABC = Area BPC +Area CAP +Area ABP
=
1
2
ap +
1
2
bq +
1
2
c
=
ap +bq +cr
2
As, ap+bq+cr es una constante, independientemente de la colocacion de P. Por
lo tanto, en vez de minimizar
a
p
+
b
q
+
c
r
, minimizaremos (ap+bq+cr)(
a
p
+
b
q
+
c
r
).
Se obtiene
(ap +bq +cr)(
a
p
+
b
q
+
c
r
) = a
2
+b
2
+c
2
+ab(
p
q
+
q
p
) +bc(
q
r
+
r
q
) +ac(
p
r
+
r
p
)
a
2
+b
2
+c
2
+ 2ab + 2bc + 2ac = (a +b +c)
2
La desigualdad en el segundo paso se sigue del hecho de que para cualesquiera
dos n umeros positivos x y y tenemos
x
y
+
y
x
2, con igualdad si y s olo si x = y.
Como resultado de este hecho, (ap+bq+cr)(
a
p
+
b
q
+
c
r
) alcanzara su valor mnimo
(a + b + c)
2
cuando, y s olo cuando, p = q = r. Equivalentemente,
a
p
+
b
q
+
c
r
alcanza un valor mnimo cuando P se encuentra en el incentro del tri angulo.
1.3.4 Probar que si m y n son enteros positivos y 1 k n, entonces

k
i=0
_
n
i
__
m
k i
_
=
_
n +m
k
_
1.3. FORMULACI

ON DE UN PROBLEMA EQUIVALENTE 21
Soluci on La armaci on del problema constituye una de las ideas fundamentales
que involucran coecientes binomiales. En el lado izquierdo esta una suma de
productos de coecientes binomiales. Obviamente, una substitucion directa de
factoriales en los coecientes binomiales no nos da ninguna introspeccion.
Muy seguido, las series nitas (especialmente aquellas que involucran coe-
cientes binomiales) pueden ser sumadas combinatoriamente. Para entender lo
que se quiere decir aqu, transforme el problema de series en un problema de
conteo de la siguiente manera. Sea S = A B, donde A es un conjunto con n
elementos y B es un conjunto, ajeno a A, con m elementos. Contaremos, de dos
formas diferentes, el n umero de subconjuntos (distintos) de S con k elementos.
Por un lado este n umero es
_
m+n
k
_
. Por otro lado, el n umero de subconjuntos
de S con exactamente i elementos de A (y ki elementos de B) es
_
n
i
__
m
k i
_
.
Se sigue que
_
m+n
k
_
= No. de k subconjuntos de S.
=
k

i=0
(No. de k subconjuntos de S con i elementos de A)
=
k

i=0
_
n
i
__
m
k i
_
Los problemas de conteo frecuentemente pueden ser simplicados Identican-
do (por medio de una correspondencia uno-a-uno) los elementos del conjunto
con los de otros cuyos elementos puedan ser contados m as facilmente. Los sigu-
ientes tres ejemplos ilustran esta idea.
1.3.5 Sobre un crculo se seleccionan n puntos y se dibujan las cuerdas que los
unen por pares. Suponiendo que no hay tres de estas cuerdas que sean concur-
rentes (excepto en los extremos), cu antos puntos de intersecci on hay?
Soluci on Los casos para n = 4, 5, 6 se muestran en la gura 1.12. Notese que
cada punto de intersecci on (interior) determina, y esta determinado por, cuatro
de los n puntos dados alrededor del crculo (estos cuatro puntos produciran en
forma unica dos cuerdas que se intersectaran en el interior del crculo). As, el
n umero de puntos de intersecci on es
_
n
4
_
.
1.3.6 Dado un entero positivo n, encuentre el n umero de cu adruplas de enteros
22 CAP

ITULO 1. HEUR

ISTICA
Figura 1.12.
(a, b, c, d) tales que 0 a b c d n.
Soluci on La idea clave que hace el problema transparente es notar que hay
una correspondencia uno-a-uno entre las cu adruplas de nuestro conjunto y el de
subconjuntos de cuatro elementos tomados de 0, 1, ..., n +3. Especcamente,
identifquese a (a, b, c, d), 0 a b c d n, con el subconjunto a, b +
1, c+2, d+3. Es facil ver que esta correpondencia es uno-a-uno. Cada elemento
de nuestro conjunto corresponde exactamente a un subconjunto de cuatro de
0, 1, ..., n + 3, y viceversa. As, el n umero deseado es
_
n + 4
4
_
.
1.3.7 El n umero cinco puede ser expresado como una suma de 3 n umeros
naturales, tomando en cuenta el orden, en 6 maneras, especcamente, como
5 = 1 + 1 + 3 = 1 + 3 + 1 = 3 + 1 + 1 = 1 + 2 + 2 = 2 + 1 + 2 = 2 + 2 + 1
Sean m y n n umeros naturales tales que m n. De cu antas maneras puede
ser escrito n como una suma de m n umeros naturales, tomando en cuenta el
orden?
Soluci on Escribir n como una suma de n unos:
n = 1 + 1 +... + 1
. .
n
El n umero que buscamos es el n umero de maneras de escoger m 1 signos de
suma de los n 1; esto es
_
n 1
m1
_
.
1.3. FORMULACI

ON DE UN PROBLEMA EQUIVALENTE 23
Problemas
1.3.8 Mostrar que x
7
+ 2x
5
+ 10x
2
1 no tiene ninguna raz mayor que 1.
(Sugerencia: ya que generalmente es m as f acil probar que una ecuaci on no tiene
races positivas, nos vemos empujados a considerar el problema equivalentemente
obtenido al hacer la sustituci on algebr aica x = y + 1.)
1.3.9 El n umero tres puede ser expresado como la suma de uno o m as enteros
positivos, tomando en cuenta el orden, de cuatro maneras, especcamente, como
3, 1 + 2, 2 + 1, y 1 + 1 + 1. Mostrar que cualquier entero positivo n puede ser
as expresado en 2
n1
formas.
1.3.10 De cu antas maneras puede ser expresado el 10 como una suma de 5
enteros no negativos, cuando se toma en cuenta el orden? (Sugerencia: Encon-
trar un problema en el que la frase 5 enteros no negativos sea reemplazada
por 5 enteros positivos).
1.3.11 Para cu ales valores de a el sistema de ecuaciones
x
2
= y
2
(x a)
2
+y
2
= 1
tiene exactamente cero, una, dos, tres, cuatro soluciones, respectivamente? (Sug-
erencia: Traduzca el problema a un problema geometrico equivalente).
1.3.12 Dado n objetos arreglados en un rengl on, un subconjunto de estos ob-
jetos es llamado no amigable si nunca dos de sus elementos est an contiguos.
Mostrar que el n umero de subconjuntos no amigables con k elementos cada uno
es
_
n k + 1
k
_
. (Sugerencia: Adoptar una idea similar a la utilizada en 1.3.6)
1.3.13 Sea a(n) el n umero de representaciones del entero positivo n como una
suma de 1s y 2s tomando el orden en cuenta. Sea b(n) el n umero representa-
ciones de n como una suma de enteros mayores que 1 , nuevamente tomando
el orden en cuenta y contando el sumando n. La tabla de abajo muestra que
a(4) = 5 y b(6) = 5 :
24 CAP

ITULO 1. HEUR

ISTICA
a-sumas b- sumas
1 + 1 + 2 4 + 2
1 + 2 + 1 3 + 3
2 + 1 + 1 2 + 4
2 + 2 2 + 2 + 2
1 + 1 + 1 + 1 6
(a) Mostrar que a(n) = b(n + 2) para toda n, describiendo una correspondencia
uno-a-uno entre las a-sumas y las b- sumas.
(b) Mostrar que a(1) = 1, a(2) = 2, y para n > 2, a(n) = a(n 1) +a(n 2).
1.3.14 Hallando el area de un tri angulo de dos formas diferentes, mostrar que
si p
1
, p
2
, p
3
son las alturas de un tri angulo y r es el radio del crculo inscrito,
entonces 1/p
1
+ 1/p
2
+ 1/p
3
= 1/r.
1.3.15 Usar un argumento de conteo para probar que para enteros r, n, 0 <
r n,
_
r
r
_
+
_
r + 1
r
_
+
_
r + 2
r
_
+ .... +
_
n
r
_
=
_
n + 1
r + 1
_
.
Ejemplos adicionales
Hay tantos ejemplos de esta heurstica que es difcil se nalar aquellos que son
los m as tpicos. Dignos de atencion son las demostraciones indirectas en las
secciones 1.10,1.11.
1.4. Modicacion del Problema
En el transcurso del trabajo sobre un problema A podemos vernos llevados a
considerar otro problema B. Com unmente, este cambio de problemas es anunci-
ado con frases tales como basta con mostrar que..... o sin perdida de generali-
dad..... En la ultima seccion vimos ejemplos en los cuales A y B eran problemas
equivalentes, esto es, la soluci on de cualquiera de ellos implica la soluci on del
problema modicado (o auxiliar), el problema B, implica la soluci on de A, pero
no necesariamente viceversa.
1.4.1 Dados los n umeros positivos a, b, c, d, demuestre
1.4. MODIFICACI

ON DEL PROBLEMA 25
a
3
+b
3
+c
3
a +b +c
+
b
3
+c
3
+d
3
b +c +d
+
c
3
+d
3
+a
3
c +d +a
+
d
3
+a
3
+b
3
d +a +b
a
2
+b
2
+c
2
+d
2
.
Soluci on Debido a la simetra del problema, basta con demostrar que para
cualesquiera n umeros positivos x, y y z
x
3
+y
3
+z
3
x +y +z

x
2
+y
2
+z
2
3
.
Si este fuera el caso, el lado izquierdo de la desigualdad original es al menos
a
2
+b
2
+c
2
3
+
b
2
+c
2
+d
2
3
+
c
2
+d
2
+a
2
3
+
d
2
+a
2
+b
2
3
a
2
+b
2
+c
2
+d
2
Ahora, para probar esta ultima desigualdad, no hay perdida de generalidad
al suponer que x+y +z = 1. Pero si no, simplemente dividamos cada lado de la
desigualdad por (x +y +z)
2
, y hagamos X = x/(x +y +z), Y = y/(x +y +z)
y Z = z/(x +y +z).
De esta manera , el problema original se reduce al siguiente problema modicado:
Dados n umeros positivos X, Y, Z tales que X +Y +Z = 1, probar que
X
3
+Y
3
+Z
3

X
2
+Y
2
+Z
2
3
.
1.4.2 Sea C cualquier punto en el segmento AB entre A y B, y dib ujense
semicrculos sobre el mismo lado de AB con AB, AC y CB como di ametro
(gura 1.13). Sea D un punto en el semicrculo de di ametro AB tal que CD es
perpendicular a AB, y sean E y F puntos sobre los semicrculos de di ametros
AC y CB, respectivamente, de forma que EF sea un segmento de su tangente
com un. Probar que ECFD es un rect angulo.
Soluci on Notese que basta con mostrar qu A, E, y D son colineales (el mismo
argumento mostrara que B, F y D son colineales). Si este fuera el caso, AEC =
90
0
(E esta en el crculo AEC), ADB = 90
0
, y el resultado es verdadero. Se
hace evidente, sin embargo, que con poca introspeccion, hay muchas formas
de encaminarse mal con esta aproximacion; es difcil evitar dar por cierta la
conclusion.
26 CAP

ITULO 1. HEUR

ISTICA
Figura 1.13.
Una forma de ganar introspeccion en las relaciones entre los par ametros de un
problema es notar el efecto cuando a uno de ellos se le hace variar (modicaci on
del problema). En este problema, se hace variar a D a lo largo de la circunfer-
encia. Sean G y H (gura 1.14) las intersecciones de los segmentos AD y BD
con los crculos de di ametro AC y CB (y centros O y O

) respectivamente. En-
tonces AGC = ADB = CHB = 90
0
, de tal forma que GDHC es un rectangulo.
Ademas, OGC = OCG (OGC es isosceles), y CGH = GCD porque GH y
CD son diagonales de un rectangulo. As, OGH = OCD. Ahora bien, mientras
D se mueve para hacer CD perpendicular AB, OGH se mover a tambien 90
0
,
de forma que GH sea tangente al crculo O, y G conicida con E. Un argumen-
to similar muestra que GH es tangente al crculo O

, entonces H = F. Esto
completa la demostracion. (Notese que la frease un argumento similar, otra
tecnica de simplicacion, tiene el mismo efecto cuando es colocada despues de
un argumento que el que tiene basta mostrar que cuando esta colocada antes
del argumento.)
Figura 1.14.
Notese que se ha resuelto el problema resolviendo un problema m as general.
1.4. MODIFICACI

ON DEL PROBLEMA 27
Esta es una tecnica com un de resoluci on de problemas; se ver an m as ejemplos
en la seccion 1.12.
1.4.3 Demuestre que no existen enteros positivos x, y, z tales que
x
2
+y
2
+z
2
= 2xyz
Soluci on Suponga que x, y, z son enteros positivos tales que x
2
+y
2
+z
2
= 2xyz.
Ya que x
2
+ y
2
+ z
2
es par (= 2xyz), dos entre x, y, z son pares y el otro
es par , o los tres son pares. Suponga que x, y, z son pares. Entonces existen
enteros positivos x
1
, y
1
, z
1
tales que x = 2x
1
, y = 2y
1
, z = 2z
1
. Del hecho que
(2x
1
)
2
+ (2y
1
)
2
+ (2z
1
)
2
= 2(2x
1
)
2
(2y
1
)
2
(2z
1
)
2
se sigue que x
1
, y
1
, z
1
satisfacen
que x
1
2
+ y
1
2
+ z
1
2
= 2
2
x
1
y
1
z
1
. De nuevo, de esta ecuaci on, si x
1
, y
1
, z
1
son
pares un argumento similar muestra que existen enteros positivos x
2
, y
2
, z
2
tales que x
2
2
+y
2
2
+z
2
2
= 2
3
x
2
y
2
z
2
.
Contin uese de esta forma. Eventualmente se llegar a a una ecuaci on de la forma
a
2
+ b
2
+ c
2
= 2
n
abc donde no todos a, b, c son pares (y de aqu que dos entre
a, b, c sean pares y uno impar).
As, hemos sido llevados a considerar el siguiente problema modicado: De-
muestre que no existen enteros positivos x, y, z y n, con x, y pares, tales que
x
2
+y
2
+z
2
= 2
n
xyz
(Este es el problema 1.9.3)
1.4.4 Eval ue
_

0
e
x
2
dx.
Soluci on Las tecnicas usuales de integraci on estudiadas en el primer a no de
c alculo no funcionar an para esta integral. Para evaluar la integral transformare-
mos la integral simple en una integral doble.
Sea I =
_

0
e
x
2
dx. Entoces
I
2
=
_
_

0
e
x
2
dx
_ _
_

0
e
y
2
dy
_
=
_

0
_
_

0
e
x
2
dx
_
e
y
2
dy
=
_

0
_

0
e
x
2
e
y
2
dxdy
. =
_

0
_

0
e
(x
2
+y
2
)
dxdy
28 CAP

ITULO 1. HEUR

ISTICA
Ahora c ambiese a una integral equivalente utilizando coordenadas polares. En-
tonces tenemos
I
2
=
_
2
0
_

0
e
r
2
rdrd
=
_
2
0

1
2
e
r
2
[

0
d
=
1
2
_
2
0
d
=

4
Se sigue que I =

2
Un problema modicado (auxiliar) puede surgir de muchas maneras. Puede
aparecer mediante un cambio de notacion (como en 1.4.4; ver seccion 1.5) o
por simetra (como en 1.4.1; ver seccion 1.6). frecuentemente es el resultado
de trabajar hacia atr as (ver seccion 1.8) o de argumentar por contradicci on
(como 1.4.3; ver secci on 1.9). No es extra no considerar un problema m as general
en el contexto (como 1.4.2; ver seccion 1.12). De esta manera vemos que la
modicaci on de un problema es una heurstica muy general. Por esta razon,
haremos a un lado m as ejemplos y problemas, dejandolos m as apropiadamente
para las secciones m as especializadas que siguen.
1.5. Eleccion de una Notacion Apropiada
Uno de los primeros pasos al trabajar sobre un problema de matematicas es
traducir el problema a terminos simbolicos. Al empezar, todos los conceptos
claves deberian ser identicados y etiquetados; las redundancias en la notaci on
pueden ser eliminadas al ir descubriendo las relaciones.
1.5.1 Una ma nana comenz o a nevar de manera fuerte y constante. Una re-
movedora de nieve comenz o a las 8:00am. A las 9:00am se haba alejado 2 millas.
A las 10:00am se haba alejado 3 millas. Suponiendo que la m aquina remueve
un volumen constante de nieve por hora, determine la hora en que comenz o a
nevar.
Soluci on Es difcil creer que hay suciente informaci on en el problema para
resolver la pregunta. Sin embargo, si hay una soluci on, debemos proceder sis-
tematicamente identicando primero aquellas cantidaddes que son desconocidas.
Introducimos la siguiente notacion: sea t el tiempo que ha transcurrido desde que
comenz o a nevar, y sea T el tiempo en la que la removedora sale (medido desde
t = 0) . Sea x(t) la distancia que ha avanzado la removedora al tiempo t (s olo
1.5. ELECCI

ON DE UNA NOTACI

ON APROPIADA 29
estamos interesados en x(t) para t T). Finalmente, sea h(t) la profundidad
de la nieve en el tiempo t.
Estamos ahora listos para traducir el problema a terminos simbolicos. El hecho
de que la nieve caiga a razon constante signica que la profundidad esta incre-
mentandose constantemente; esto es,
dh
dt
= c, c constante.
Integrando cada lado se obtiene
h(t) = ct +d, c, d constantes.
Ya que h(0) = 0, obtenemos d = 0. As h(t) = ct.
El hecho de que la removedora quite la nieve a una razon constante signica que
la velocidad de la removedora es inversamente proporcional a la profundidad en
cualquier tiempo t (por ejemplo, el doble de profundidad corresponde a la mitad
de la velocidad).
Simbolicamente, para t T,
dx
dt
=
k
h(t)
, k constante
=
k
ct
=
K
t
, K =
k
c
constante.
Integrando ambos lados se obtiene
x(t) = Klogt +C, C constante.
Tenemos tres condiciones: x = 0 cuando t = T, x = 2, cuando t = T +1, y x = 3,
cuando t = T +2. Con estas dos condiciones podemos evaluar las constantes K
y C, y con la tercera, podemos resolver para T. Resulta que (los detalles no son
de interes aqu)
T =

5 1
2
0,618 hrs. 37 minutos, 5 segundos.
As, comenz o a llover a las 7:22:55 AM.
1.5.2 (a) Si n es un entero positivo tal que 2n + 1 es un cuadrado perfecto,
muestre que n + 1 es la suma de dos cuadrados perfectos sucesivos.
30 CAP

ITULO 1. HEUR

ISTICA
(b) Si 3n + 1 es un cuadrado perfecto, muestre que n + 1 es la suma de tres
cuadrados perfectos
Soluci on Introduciendo la notacion adecuada esto se reduce a un problema
sencillo de algebra. Para la parte (a), suponga que 2n + 1 = s
2
, s es un entero.
Ya que s
2
es un n umero impar, tambien lo es s. Sea t un entero tal que s = 2t+1.
Entonces 2n + 1 = (2n + 1)
2
, y resolviendo para n tenemos que
n =
(2t + 1)
2
1
2
=
4t
2
+ 4t
3
= 2t
2
+ 2t.
Consecuentemente,
n + 1 = 2t
2
+ 2t + 1 = t
2
+ (t + 1)
2
(b) Suponga que 3n + 1 = s
2
, s es un entero. Evidentemente , s no es m ultiplo
de 3, as que s = 3t 1 para alg un entero t. Entonces 3n + 1 = (3t 1)
2
, y as
n =
(3t 1)
2
1
2
=
9t
2
6t
3
= 3t
2
2t.
De esta manera,
n + 1 = 3t
2
2t + 1 = 2t
2
+ (t 1)
2
= t
2
+t
2
+ (t 1)
2
1.5.3 En el tri angulo ABC, AB = AC, D es el punto medio de BC, E es el
pie de la perpendicular dibujada desde D a AC, y F es el punto medio de DE
(gura 1.15). Demuestre que AF es perpendicular a BE.
Soluci on Podemos transformar el problema a terminos algebraicos asociando
coordenadas a los puntos relevantes y mostrando que las pendientes m
BE
y
m
AF
son recprocos negativos.
Una forma de proceder es tomar el tri angulo como aparece en la gura 1.15:
tomar D como el origen (0, 0), A = (0, a), B = (b,0). Esta es una forma nat-
ural de se nalar la gura porque aprovecha la simetra bilateral del tri angulo
isoseles (ver los ejemplos en la seccion 1.6). Sin embargo; en este caso en par-
ticular, esta notacion lleva a algunas complicaciones menores cuando buscamos
las coordenadas de E y F.
Una mejor asignaci on de coordenadas es tomar A = (0, 0), B = (4a, ab), C =
(4c, 0), como en la gura 1.16. Entonces a
2
+ b
2
= c
2
, D = (2a + 2c + 2b),
1.5. ELECCI

ON DE UNA NOTACI

ON APROPIADA 31
Figura 1.15.
Figura 1.16.
E = (2a + 2c, 0), y F = (2a + 2c + b). (casi no hay c alculos aqu; todos los
puntos relevantes tienen coordenadas asignadas). Se sigue que
m
BE
m
AF
=
_
b
2 (a +c)
__
4b
4a (2a +c)
_
=
b
2
a
2
c
2
= 1,
y la prueba queda concluida.
1.5.4 Sea 1 < a
0
< 1 y defnase recursivamente
32 CAP

ITULO 1. HEUR

ISTICA
a
n
=
(1 +a
n1
)
1/2
2
, n > 0.
Sea A
n
= 4
n
(1 a
n
) . Que pasa con A
n
cuando n tiende a innito?
Soluci on Intentar expresar directamente a
n
en terminos de a
0
lleva a compli-
cadas expresiones sin esperanza que contiene sucesiones anidadas de radicales,
y no hay forma de condensarlas en una forma cerrada.
La introspeccion clave requerida es observar que hay un unico angulo , 0 < <
, tal que a
0
= cos . Para este ,
a
1
=
_
1 + cos
2
_1
2
= cos
_

2
_
.
Similarmente,
a
2
=
_
1 + cos (/2)
2
_1
2
= cos
_

4
_
, ..., a
n
= cos
_

2
n
_
.
Ahora podemos calcular
A
n
= 4
n
_
1 cos
_

2
n
__
=
4
n
_
1 cos
_

2
n
__ _
1 +cos
_

2
n
__
_
1 +cos
_

2
n
__
=
4
n
sin
2
_

2
n
_
1 +cos
_

2
n
_
=
_

2
1 +cos
_

2
n
_
__
sin
_

2
n
_

2
n
_
2
.
Conforme n crece,
2
/(1 + cos(/2
n
)) tiende a
2
/2, y (sen(/2
n
))/(/2
n
) se
aproxima a 1 (recuerde que senx/x 1 conforme x 0), y as , A
n
converge
a
2
/2 conforme n tiende a innito.
Problemas
1.5. ELECCI

ON DE UNA NOTACI

ON APROPIADA 33
1.5.5 Escriba una ecuaci on para representar las siguientes armaciones:
(a) En el restaurante Mindy, por cada cuatro personas que ordenaron pastel de
queso, hubo cinco que pidieron pay de manzana.
(b) Hay seis veces la cantidad de profesores en estudiantes en este colegio.
1.5.6 Unos cables son extendidos desde la punta de cada uno de los polos a
la base del otro. Cu al es la altura medida desde el piso al punto en que estos
alambres se cruzan?
1.5.7 Una pieza de papel de 8 pulgadas de ancho es doblada como en la gura
1.17 de forma que una esquina es puesta en el lado opuesto. Exprese la longitud
del doblez L en terminos del angulo solamente.
1.5.8 Sean P
1
, P
2
, ....., P
12
los vertices sucesivos de un dodecaedro regular (12
lados). Concurren las diagonales P
1
P
9
, P
2
P
11
, P
4
P
12
.?
Figura 1.17.
1.5.9 Utilice algebra para respaldar sus respuestas a cada uno de los siguientes.
Un carro viaja desde A hasta B a raz on de 40 millas por hora y luego regresa de
B a A a raz on de 60 millas por hora? 50 millas por hora es apr oximadamente
la velocidad promedio par el viaje redondo?
Le dan a usted una taza de cafe y una de crema que contienen la misma cantidad
de lquido. Una cucharada de la mezcla es extrada de la taza y se devuelve a la
de crema. Hay m as o menos crema ahora en la taza de cafe que cafe en la taza
de crema? ( Este problema tiene una elegante soluci on no algebr aica basda en
34 CAP

ITULO 1. HEUR

ISTICA
la observaci on de que el cafe en la taza de crema ha desplazado una cantidad
igual de crema que debe estar ahora en la taza de cafe.)
Imagine que la tierra es una esfera suave y que se le enreda una cuerda alrededor
del Ecuador. Suponga ahora que la cuerda es alargada 6 pies y que la nueva
longitud es empujada al parejo para formar un nuevo crculo justo encima del
Ecuador. Es la distancia entre la cuerda y la supercie de la tierra m as o menos
de 1 pulgada?
1.5.10 Un teorema bien conocido arma que un primo p > 2 puede ser escrito
como la suma de dos cuadrados perfectos
_
p = m
2
+n
2
, con m y n enteros
_
si
y s olo si p es mayor por 1 que un m ultiplo de 4. Dando este resultado por cierto,
demuestre que:
Todo primo mayor por 1 que un m ultiplo de 8 puede ser escrito en la forma
x
2
+ 16y
2
, x y y enteros.
Todo primo mayor por 5 que un m ultiplo de 8 puede ser escrito en la forma
(2x +y)
2
+ 4y
2
, x y y enteros.
Ejemplos adicionales
1.1.10, 2.5.10, 3,2,15, 3.3.11, 3.3.28, 3.4.2, .3.4.4, 4.1.5, 6.4.2, 7.2.4, 8.1.15, 8.2.3,
8.2.17. Tambien, ver las Secciones 2.5 (Relaciones de Recurrencia), 3.2 (Ar-
itmetica Modular), 3.4 (Notacion Posicional), 8.3 (Geometra Vectorial), 8.4
(Geometra en los N umeros Complejos).
1.6. Explotacion de la Simetra
La presencia de simetra de un problema provee usualmente de medios para
reducir la cantidad de trabajo para llegar a una soluci on. Por ejemplo, consid-
erese el producto (a +b +c)
_
a
2
+b
2
+c
2
ab ac bc
_
. Ya que cada factor
es simetrico en a, b, c (la expresi on no cambia cuando cualquier par de sus vari-
ables son intercambiadas), lo mismo suceder a con el producto. Como resultado
de ello, si a
3
aparece en el producto, tambien apareceran b
3
y c
3
. Similarmente,
si a
2
b aparece en el producto, tambien lo har an a
2
c, b
2
a, b
2
c, c
2
b, y cada uno
aparecera con el mismo coeciente, etc. De esta manera, una revisada rapida
muestra que el producto tendr a la forma
A
_
a
3
+b
3
+c
3
_
+B
_
a
2
b +a
2
c +b
2
c +c
2
a +c
2
b
_
+C (abc) .
1.6. EXPLOTACI

ON DE LA SIMETR

IA 35
Es facil checar que A = 1, B = 0, y C = 3.
1.6.1 Se construyen tri angulos equilateros ABK, BCL, CDM, DAN dentro
del cuadrado ABCD. Demuestre que los puntos medios de los cuatro segmentos
KL, LM, MN, NK y los puntos medios de los ocho segmentos AK, BK, BL,
CL, CM, DM, DN, AN son los 12 vertices de un dodecaedro regular.
Figura 1.18.
Soluci on Los doce vertices estan indicados en la Figura 1.18 con puntos gruesos;
dos de estos vertices estan se nalados como a y b seg un se muestra.
Utilizando la simetra de la gura, basta mostrar que bOK = 15

, aOb = 30

,
y [aO[ = [bO[ .
Notese que AN es parte del bisector perpendicular de BK, y entonces [KN[ =
[NB[ . Utilizando simetra que sigue que CBN = 15

. Considerese ahora el
tri angulo equilatero DBN; notese que Ob une los puntos medios de DB y DN,
as qu Ob es la paralela a BN y de la mitad de longitud. De esta manera
[Ob[ = s/2 y bOK = 15

. Desde aqu es facil vericar que


aOb = DOK bOK = 45

15

= 30

, y [Oa[ = [KN[ /2 = s/2.


La presencia de simetra en un problema tambien provee de vision clara que nos
36 CAP

ITULO 1. HEUR

ISTICA
permite ver y descubir relaciones que pueden ser m as difciles de encontrar por
otros medios. Por ejemplo, considerando s olo la simetra se ocurre que el valor
m aximo de xy, bajo la restriccion x+y = 1, x > 0, y > 0, debera ocurrir cuando
x = y = 1/2 (x y y estan simetricamente relacionadas). Este es un ejemplo del
principio de raz on insuficiente, que puede resumirse de la siguiente manera:
Donde no hay suciente razon para distinguir, no puede haber distincion. As,
no hay razon para esperar que el valor m aximo ocurra cuando x tiene el valor
distinto de 1/2, esto es, cercano a 0 o a 1. Para vericar esto, sea x = 1/2 + e.
Entonces y = 1/2 e, y, xy = (1/2 +e) (1/2 e) = 1/4 e
2
. De esta forma es
claro que el m aximo ocurre cuando e = 0; esto es, x = y = 1/2.
El siguiente problema ofrece varios Ejemplos adicionales de este principio.
1.6.2 .
(a) De todos los rect angulos que pueden inscribirse en un crculo dado, cu al
tiene mayor area?
Figura 1.19.
(b) Maximizar senA + senB + senC, donde A, B, C son las medidas de los tres
angulos de un tri angulo.
(c) De todos los tri angulos con un permetro dado, cu al tiene mayor area?
(d) De todos los paraleleppedos de volumen 1, cu al tiene la menor area de
supercie?
(e) De todos los n agonos que pueden ser inscritos en un crculo dado, cu al
tiene mayor area?
Soluci on .(a) El principio de raz on insuciente nos lleva a sospechar que el
rect angulo con mayor area que puede inscribirse en un crculo es un cuadrado
(Figura 1.19). Para vericar esto, sean x y y el largo y el ancho del rect angulo,
1.6. EXPLOTACI

ON DE LA SIMETR

IA 37
y sup ongase sin perdida de generalidad que las unidades han sido escogidas de
manera que el di ametro del crculo es la unidad. Queremos maximizar xy sujetos
a que x
2
+ y
2
= 1. Esto es equivalente a maximizar x
2
y
2
bajo la restricci on
x
2
+ y
2
= 1. Pero este es el mismo problema que se consider o previamente a
este ejemplo; el valor m aximo ocurre cuando x
2
+ y
2
= 1/2, esto es, cuando el
rect angulo es un cuadrado.
(b) N otese que la suma, senA+senB +senC, es siempre positiva (ya que cada
uno de los terminos es positivo), y que puede ser hecha arbitrariamente peque na
(en magnitud) haciendo A arbitrariamente cercano a 180

. No hay raz on para


esperar que el m aximo ocurra en otro punto que no sea A = B = C = 60

(un
tri angulo equil atero). Una demostraci on de esto se sigue de la discusi on en 2.4.1.
De manera similar, sospechamos que las respuestas a (c),(d), y (e) son un
tri angulo equil atero, un cubo, y un n agono regular.
1.6.3 Evaluar
_
2
0
dx
1 + (tan x)

2
Figura 1.20.
Soluci on . He aqu un problema que no puede resolverse utilizando las tecnicas
usuales de integraci on; es decir, el integrando no tiene una antiderivada. Sin
embargo, el problema puede ser manejado si notamos que el integrando (Figura
1.20) es simetrico alrededor del punto (/4, 1/2) .
Para mostrar que esto es as (no es obvio), sea f (x) = 1/
_
1 + (tan x)

2
_
. Basta
mostrar que f (x) +f (/2 x) = 1 para toda x, 0 x /2. As, calculamos,
para r =

2,
38 CAP

ITULO 1. HEUR

ISTICA
f (/2 x) +f (x) =
1
1 + tan (/2 x)
+
1
1 + tan
r
(x)
=
1
1 + cot
r
(x)
+
1
1 + tan
r
(x)
=
tan
r
(x)
1 + tan
r
(x)
+
1
1 + tan
r
(x)
= 1.
Se sigue de la simetra recientemente demostrada que el area bajo la curva en
[0, /2] es la mitad de area en el rectangulo (ver Figura 1.20); esto es, la integral
es (/2) /2 = /4.
Otra manera de sacar ventaja de la simetra es la elecci on de notacion. He
aqu un par de ilustraciones.
1.6.4 Sea P un punto en la gr aca de y = f (x), donde f es un polinomio de
grado tres; sup ongase que la tangente en P intersecta de nuevo a la curva en
Q; y sea A el area de la regi on limitada por la curva y el segmento PQ. Sea
B el area de la regi on denida si comenzamos por Q en vez de P. Cu al es la
relaci on entre A y B?.
Figura 1.21.
Soluci on . Sabemos que un polinomio c ubico es simetrco respecto a su punto
de inexion. Ya que las areas de interes no son afectadas por la elecci on del sis-
1.6. EXPLOTACI

ON DE LA SIMETR

IA 39
tema de coordenadas, jaremos el origen en el punto de inexion. As, podemos
suponer que la funci on c ubica es
f (x) = ax
3
+bx, a ,= 0
(ver Figura 1.21).
Supongase que x
0
es la abcisa de P. Resulta que la abcisa de Q es 2x
0
. (No nos
detendremos en los detalles de estos c alculos tan directos. Existe, ciertamente,
una manera elegante de llegar a este hecho, pero utiliza ideas que no se ver an
en esta seccion).
Integrando directamente se muestra que el area A es igual a Kx
4
0
, donde K es
independiente de x
0
. (De nuevo no nos detendremos en los detalles del c alculo.)
Podemos aplicar ahora nuestras conclusiones previas al punto Q. La tangente en
Q intersectara a la curva en R, la abcisa del cual es evidentemente 2 (2x
0
) =
4x
0
, y el area B es igual a K (2x
0
)
4
= 16Kx
4
0
= 16A.
1.6.5 Determine todos los valores de x que satisfacen
tan x = tan (x + 10

) tan (x + 20

) tan (x + 30

) .
Soluci on . Introduciremos simetra haciendo un simple cambio de variable. As,
sea y = x + 15

. La ecuaci on es entonces
tan (y 15

) = tan (y 5

) tan (y + 5

) tan (y + 15

) ,
que equivale a
sen(y 15

) cos (y + 15

)
cos (y 15

) sen(y + 15

)
=
sen(y 5

) sen(y + 5

)
cos (y 5

) cos (y + 5

)
usando las identidades
sen(A) cos (B) =
1
2
[sen(AB) +sen(A+B)] ,
sen(A) sen(B) =
1
2
[cos (AB) cos (A+B)] ,
cos (A) cos (B) =
1
2
[cos (AB) + cos (A+B)] ,
40 CAP

ITULO 1. HEUR

ISTICA
obtenemos
sen(30

) +sen2y
sen(30

) +sen2y
=
cos (10

) cos 2y
cos (10

) + cos 2y
,
o equivalentemente,
2sen2y 1
2sen2y + 1
=
cos 10

cos 2y
cos 10

+ cos 2y
.
Esto se simplica en
sen(4y) = cos 10

,
lo cual implica que
4y = 80

+ 360

k, 100

+ 360

k, k = 0, 1, 2, ... ,
x = 5

+ 90

k, 10

+ 90

k, k = 0, 1, 2, ... .
Problemas
1.6.6 .
(a) Explote la simetra para expandir el producto
_
x
2
y +y
2
z +z
2
x
_ _
xy
2
+yz
2
+zx
2
_
.
(b) Si x +y +z = 0, demuestre que
_
x
2
+y
2
+z
2
2
__
x
5
+y
5
+z
5
5
_
=
x
7
+y
7
+z
7
7
(substituya z = x y y aplique el teorema del binomio.)
1.6.7 Las caras de cada una de las quince monedas, acomodadas como se mues-
tra en la Figura 1.22, son pintadas de blanco o de negro. Demuestre que exiten
tres monedas del mismo color cuyos centros son los vertices de un tri angulo
equil atero. (Hay muchas maneras de explotar la simetra y de crear argumentos
del tipo sin perdida de generalidad.)
1.6.8 Haga uso del principio de raz on insuciente para minimizar x
2
1
+ x
2
2
+
... +x
2
n
, bajo la condici on de que 0 < x
i
< 1, y x
1
+x
2
+... +x
n
= 1. Demuestre
su conjetura (Para la demostraci on, tome x
i
=
1
n
+e
i
.)
1.7. DIVISI

ON EN CASOS 41
Figura 1.22.
1.6.9 Un punto P est a localizado en el interior de un tri angulo equil atero
ABC. Desde P se trazan perpendiculares que intersectan a cada lado en los
puntos D, E, y F, respectivamente. D onde debera estar P para que PD+PE+
PF sea un m aximo? D onde debera estar P para que PD + PE + PF sea un
mnimo? Justique sus respuestas. (Sugerencia: Es de gran ayuda reejar la
gura respecto a uno de los lados. Que pasa con PD +PE +PF conforme P
se mueve paralelamente a la lnea de reexi on?)
1.6.10 En la Figura 1.23, ABCD es un cuadrado, EDC = ECD = 15

.
Demuestre que el tri angulo AEB es equil atero. (La clave para este bello problema
es crear simetra central. Especcamente, agregar angulos identicos de 15

sobre
los lados AB, BC, y AD (como en el lado CD) y crear un diagrama muy parecido
al construido en 1.6.1.)
1.6.11 El producto de cuatro terminos consecutivos de una serie aritmetica de
enteros m as la cuarta potencia de la diferencia com un es siempre un cuadrado
perfecto. Verique esta identidad incorporando simetra en la notaci on.
Ejemplos adicionales
1.4.1, 8.1.4, 8.1.5, 8.1.8, 8.2.3.
1.7. Division en Casos
Sucede con frecuencia que un problema pueda ser dividido en un n umero peque no
de subproblemas, cada uno de los cuales puede ser manejado por separado a la
manera de caso por caso. Esto es cierto especialmente cuando el problema con-
tiene un cuanticador universal (para todo x ...). Por ejemplo, la demostracion
42 CAP

ITULO 1. HEUR

ISTICA
Figura 1.23.
de una proposici on de la forma para todos los enteros ... puede hacerse ar-
gumentando para los pares y los impares por separado. De forma similar, un
teorema sobre tri angulos puede probarse dividiendolo en tres casos dependiendo
de si el tri angulo es agudo, recto u obtuso. Ocasionalmente, los subproblemas
pueden ser arreglados jerarquicamente en submetas, de forma que los primeros
casos, una vez establecidos, pueden ser utilizados para probar las siguientes
etapas. Tal procedimiento es llamado escalada.
En las primeras etapas del analisis, es util pensar en c omo subdividir un prob-
lema en un n umero peque no de subproblemas (con suerte) m as sencillos. La
heurstica de esta seccion se da muy con frecuencia de la siguiente manera: Si
no puede resolver el problema, encuentre un problema relacionado m as sencillo
y resuelvalo.
1.7.1 Demuestre que un angulo inscrito en un crculo es igual a la mitad del
angulo central que subtiende el mismo arco.
Soluci on. Tenemos un crculo, con centro O digamos, y un angulo inscrito
APB; se muestran algunos ejemplos en la gura 1.24. Tenemos que demostrar
que para todos los casos APB =
1
2
AOB. Las tres guras precedentes repre-
sentan situaciones esenciales diferentes. Especcamente, el centro del crculo, O,
esta en el interior de APB (diagrama 2), o fuera de APB (diagrama 3), o en
uno de los rayos de APB (diagrama 1). Probaremos el teorema considerando
cada uno de estos casos por separado.
Caso1. Supongamos que el centro O esta en PA. Entonces AOB = OPB +
OBP (un angulo exterior es igual a la suma de los angulos interiores no
adyasentes) = 2DPB (OPB es isosceles)= 2APB. El resultado es in-
1.7. DIVISI

ON EN CASOS 43
Figura 1.24.
mediato.
Caso2. Si O esta en el interior del APB (diagrama 2), extendemos la lnea PO
hasta cortar el crculo en D. Acabamos de demostrar que 2APB = AOD y
2DPB = DOB. La suma de estas ecuaciones dar a el resultado deseado.
Caso 3. Si O es exterior APB (diagrama 3), extendemos PO hasta cortar el
crculo en

D. Entonces, utilizando el caso 1, 2DPB = DOB y 2DPA =
DOA. Restando la segunda ecuaci on de la primera se obtiene el resultado
deseado. Esto completa la demostracion.
1.7.2 Una funci on real de variable racional f satisface
f (x +y) = f (x) +f (y)
para todos x y y racionales. Demuestre que f (x) = f (1) x para todo x racional.
Soluci on . Procedemos por pasos. Primero probaremos el resultado para los
enteros positivos, luego los negativos, luego los recprocos de los enteros, y -
nalmente para todos los n umeros racionales.
Caso 1 (Enteros positivos). El resultado se cumple cuando x = 1. Para x = 2,
tenemos f (2) = f (1 + 1) = f (1) +f (1) = 2f (1) . Para
x = 3, f (3) = f (2 + 1) = f (2) + f (1) = 2f (1) + f (1) = 3f (1) . Es claro
que este proceso puede ser continuado, y que para cualquier entero positivo n,
f (n) = nf (1) . (Puede darse una demostracion formal basada en el principio
de induccion matematica, ver Captulo2).
Caso 2 (Enteros no positivos). Primero, f (0) = f (0 + 0) = f (0) + f (0) . Re-
stando f (0) de ambos lados se obtiene 0 = f (0) ; esto es f (0) = 0f (1) . Ahora,
44 CAP

ITULO 1. HEUR

ISTICA
0 = f (0) = f (1 + (1)) = f (1)+f (1) . De aqu que f (1) = f (1) . De man-
era similar, para cualquier entero positivo n, f (n) + f (n) = f (n + (n)) =
f (0) = 0, de forma que f (n) = nf (1) .
Caso 3 (Recprocos). Para x =
1
2
, procedemos como sigue: f (1) = f
_
1
2
+
1
2
_
= f
_
1
2
_
+ f
_
1
2
_
= 2f
_
1
2
_
. dividiendo entre 2 se obtiene f
_
1
2
_
= f (1) /2. Para
x =
1
3
,
f (1) = f
_
1
3
+
1
3
+
1
3
_
= f
_
1
3
_
+ f
_
1
3
_
+ f
_
1
3
_
= 3f
_
1
3
_
, o equivalentemente,
f (1) /3. De forma similar, para cualquier entero positivo n, f
_
1
n
_
= f (1) /n.
Para x =
1
n
, tenemos f
_
1
n
_
+ f
_
1
n
_
= f
_
1
n
+
_
1
n
__
= f (0) = 0, de forma
que f
_
1
n
_
= f (1) /n.
Caso 4 (todos los racionales). Sea n un entero. Entonces f
_
2
n
_
= f
_
1
n
+
1
n
_
=
f
_
1
n
_
+f
_
1
n
_
= 2f
_
1
n
_
=
2
n
f (1) . De forma similar, si
m
n
es cualquier racional,
com m un entero positivo y n un entero, entonces
f
_
m
n
_
= f
_
1
n
+... +
1
n
_
. .
mveces
= f
_
1
n
_
+... +f
_
1
n
_
. .
mveces
= mf
_
1
n
_
=
m
n
f (1) .
Esto establece el resultado, un buen ejemplo de escalada.
1.7.3 Demuestre que el area de un tri angulo retcula es igual a I +
1
2
B 1,
donde I y B representan respectivamente el n umero de puntos interiores y de
frontera de la retcula en el tri angulo. (Un tri angulo retcula es un tri angulo en
el plano con puntos de una retcula como vertices.)
Soluci on . Este es un caso especial del teorema de Pick (ver 2.3.1.). Hay varias
pruebas ingeniosas, cada una de las cuales divide al conjunto de los tri angulos
retcula en unos pocos de tipos especiales. Una forma de hacer esto es circun-
scribir sobre el tri angulo un rectangulo con bordes paralelos a los ejes coor-
denados. Al menos un vertice del rectangulo debe coincidir con un vertice del
tri angulo. Ahora es facil ver que cada tri angulo retcula puede ser clasicado en
una de las clases no equivalentes esbozadas en la Figura 1.25.
En la primera clase estan aquellos tri angulos rectos cuyos catetos son paralelos a
los ejes coordenados. La segunda clase incluye tri angulos de angulos agudos uno
de cuyos lados es paralelo a un eje coordenado. Tales tri angulos son la suma
de dos tri angulos de la primera clase. En la tercera clase estan los tri angulos
obtusos con un lado paralelo a un eje coordenado. Ellos son la resta de dos
tri angulos de la primera clase. Las clases cuarta y quinta abarcan a aquellos
tri angulos que no tienen lados paralelos a los ejes de coordenadas.
La demostracion del resultado sigue un patr on de escalada. Para empezar, con-
sideremos el rectangulo ABCD del caso 1. Supongamos que los segmentos AB
1.7. DIVISI

ON EN CASOS 45
Figura 1.25.
y AD contienen a y b puntos de la retcula, respectivamente, sin contar sus
extremos. Entonces, con I y B los puntos interiores y de frontera de ABCD,
I +
1
2
B 1 = ab +
1
2
(2a + 2b + 4) 1
= ab +a +b + 1
= (a + 1) (b + 1)
= Area ABCD
Ahora supongamos que AB, BC, y AC contienen a, b, y c puntos de la retcula,
respectivamente, sin contar sus extremos, y supongamos que ABC contiene i
puntos interiores. Entonces el rectangulo ABCD tiene 2i +c puntos interiores,
y tenemos, con I y B los puntos interiores y frontera de ABC,
I +
1
2
B 1 = i +
1
2
(a +b +c + 3) 1
=
1
2
(2i +a +b +c + 1)
=
1
2
_
(2i +c) +
1
2
(2a + 2b + 4) 1
_
=
1
2
Area ABCD
= Area ABC.
46 CAP

ITULO 1. HEUR

ISTICA
Los otros casos pueden ser manejados de forma similar; dejamos los detalles al
lector.
Problemas
1.7.4 ( Desiguladad del tri angulo ).
(a) Demuestre que para cualesquiera n umeros reales x y y, [x +y[ [x[ +[y[ .
(b) Demuestre que para cualesquiera n umeros reales x, y, y z, [x y[ [x z[+
[y z[ .
1.7.5 Encuentre todos los valores de x que satisfacen
3
x 1
<
2
x + 1
.
1.7.6 Sean S = i (3, 8) +j (4, 1) +k (5, 4) / i, j, k son enteros, y T =
m(1, 5) +n(0, 7) / m, n son enteros . Demuestre que S = T. (Nota: las pare-
jas ordenadas de enteros se suman componente a componente: (s, t) + (s

, t

) =
(s +s

, t +t

) , y n(s, t) = (ns, nt) .)


1.7.7 Una funci on real f, denida en los n umeros racionales positivos, sat-
isface f (x +y) = f (x) f (y) para cualesquiera x y y racionales positivos. De-
muestre que f (x) = [f (1)]
x
para todo racional positivo x.
1.7.8 Determine F (x) si, para cualesquiera reales x y y, F (x) F (y)+F (xy) =
x +y.
Ejemplos adicionales
1.1.7,2.5.11c,2.5.12,2.5.13,2.6.3, 3.2.14, 3.2.15, 3.2.16, 3.2.17, 3.2.18, 3.4.1, 4.1.3,
4.1.4, 4.4.14, 4.4.29, 5.2.1, 5.3.14c, 6.5.4, 7.4.3, 7.6.2, 7.6.4, 7.6.10, 8.2.4. Algunos
ejemplos particulares que se reducen a estudiar casos muy especiales son 3.3.8,
3.3.9, 3.3.21, 3.3.22, 3.3.26.
1.8. TRABAJO HACIA ATR

AS 47
1.8. Trabajo Hacia Atras
Trabajar hacia atras signica dar cierta conclusion y entonces obtener deduciones
de la conclusion hasta que llegamos al algo conocido o algo que puede de-
mostrarse facilmente. Despues de que llegamos a lo dado o a lo conocido, en-
tonces invertimos los pasos del argumento y procedemos hacia la conclusion.
Este procedimiento es com un en algebra y trigonometra de secundaria. Por
ejemplo, para encontrar todos los n umeros reales que satisfacen 2x + 3 = 7,
argumentos como sigue. Supongamos que x satisface 2x + 3 = 7. Entonces,
restamos 3 de cada lado de la ecuaci on y dividimos cada lado entre 2, para
obtener x = 2. Ya que cada paso en esta deduccion puede ser invertido, con-
cluimos que 2 ciertamente satisface 2x + 3 = 7 y es el unico n umero que lo
hace.
Con frecuencia, en manipulaciones de rutina, tales como las del ejemplo previo,
no se hace una reescritura explcita de los pasos. Sin embargo, es importante
estar atento a lo que puede, y no puede, ser invertido. Por ejemplo, considerese
la ecuaci on

x + 1

x 1 = 2. (Aqu, como es usual, la raz cuadrada es la


raz cuadrada positiva.) Escriba la ecuaci on en la forma

x + 1 =

x 1 + 2,
y eleve ambos lados al cuadrado para obtener x + 1 = x 1 + 4

x 1 + 4, o
equivalentemente,

x 1 = 1/2. Eleve al cuadrado una vez m as para obtener


x 1 = 1/4, o x = 5/4. Sin embargo, 5/4 no satisface la ecuaci on original. La
razon de esto es que no todos los pasos son reversibles. As, en este ejemplo,
procedemos desde

x 1 = 1/2 a x 1 = 1/4. Cuando invertimos esto, sin


embargo, el argumento va desde x 1 = 1/4 a

x 1 = 1/2.
1.8.1 Sea un n umero real jo, 0 < < , y sea
F () =
sen +sen( +)
cos cos ( +)
, 0 .
Demuestre que F es constante. (Este problema surgi o en 1.2.1.)
Soluci on. Supongamos que F es constante. Entonces F () = F (0) para todo
, 0 . Esto es,
sen +sen( +)
cos cos ( +)
=
sen
1 cos
, (1.1)
[sen +sen( +)] [1 cos ] = sen[cos cos ( +)] , (1.2)
sen +sen( +) sen cos sen( +) cos
= sencos sencos ( +) , (1.3)
48 CAP

ITULO 1. HEUR

ISTICA
sen +sen( +) [sen cos +sencos ]
[sen( +) cos sencos ( +)] = 0, (1.4)
sen +sen( +) sen( +) sen( + ) = 0. (1.5)
La ultima ecuaci on es una identidad. Para la demostraci on, debemos invertir
los pasos. El unico paso cuestionable es de 1.2 a 1.1: la prueba s olo es valida si
no dividimos por cero al ir de 1.2 a 1.1. Pero (1 cos ) ,= 0 ya que 0 < < ,
y cos cos( + ) > 0 ya que 0 < + . La demostracion puede
entonces llevarse a cabo; esto es, comenzando por la identidad conocida en 1.5,
podemos argumentar (va los pasos 1.4,1.3,1.2,1.1) que para todo , 0
, F () = sen/ (1 cos ) = constante.
1.8.2 Si a, b, c denotan las longitudes de los lados de un tri angulo, demuestre
que
3 (ab +bc +ca) (a +b +c)
2
4 (ab +bc +ca) .
Soluci on. Considerese la desigualdad del extremo izquierdo:
3 (ab +bc +ca) (a +b +c)
2
,
3 (ab +bc +ca) a
2
+b
2
+c
2
+ 2 (ab +bc +ca) ,
(ab +bc +ca) a
2
+b
2
+c
2
,
a
2
+b
2
+c
2
(ab +bc +ca) 0
2a
2
+ 2b
2
+ 2c
2
2 (ab +bc +ca) 0
_
a
2
2ab +b
2
_
+
_
b
2
2bc +c
2
_
+
_
c
2
2ca +a
2
_
0
(a b)
2
+ (b c)
2
+ (c a)
2
0
Esta desigualdad es verdadera para todos los valores de a, b, c. Considerese ahora
la desigualdad de la derecha:
(a +b +c)
2
4 (ab +bc +ca)
a
2
+b
2
+c
2
+ 2 (ab +bc +ca) 4 (ab +bc +ca)
a
2
+b
2
+c
2
2 (ab +bc +ca)
a
2
+b
2
+c
2
a (b +c) +b (a +c) +c (b +a)
Esta desigualdad nal es verdadera, ya que la suma de cualesquiera dos de los
lados de un tri angulo es mayor que el lado restante. As, a
2
a (b +c) , b
2

b (a +c) , y c
2
c (b +a) .
Los pasos en cada uno de estos argumentos pueden ser invertidos, as que la
prueba esta completa.
1.8. TRABAJO HACIA ATR

AS 49
1.8.3 Se tiene: AOB es el di ametro del crculo O; BM es tangente al crculo
en B; CF es tangente al crculo en E e intersecta BM en C; la cuerda AE, al
ser prolongada, intersecta BM en D. Demuestre que BC = CD. (Ver la Figura
1.26)
Figura 1.26.
Soluci on. Supongamos que BC = CD. Entonces CE = CD, ya que BC = CE
(tangentes al crculo desde C en B y E son iguales). As, CED = CDE (los
angulos base de un tri angulo isosceles son iguales). Esto nos lleva a considerar
los angulos de la manera en que estan etiquetados en la Figura 1.26.
Ahora bien, D es el complemento de a ya que el ABD es un tri angulo
rectangulo, y el e es complemento del c ya que BEA es un angulo recto
(AOB es un di ametro). As, a = c. Pero sabemos que a = c, ya que ambos
subtienden el mismo arco BE en el crculo O.
La prueba puede ser completada invirtiendo los pasos. De esta manera (omitimos
las razones),partimos de que a = c, y por lo tanto, e = d. As CD =
CE, CE = BC, y por lo tanto BC = CD.
1.8.4 En un torneo con n jugadores P
1
, P
2
, ..., P
n
, con n > 1, cada jugador
juega un juego con cada uno de los otros jugadores y las reglas son de tal manera
que no puede haber empates. Sean W
r
y L
r
el n umero de juegos ganados y
perdidos, respectivamente, por el jugador P
r
. Demuestre que
n

r=1
W
2
r
=
n

r=1
L
2
r
.
50 CAP

ITULO 1. HEUR

ISTICA
Soluci on. Supongamos que

n
r=1
W
2
r
=

n
r=1
W
2
r
. Entonces,
n

r=1
_
W
2
r
L
2
r
_
= 0,
n

r=1
(W
r
L
r
) (W
r
+L
r
) = 0.
Pero W
r
+L
r
= n 1 para cada r, as que
(n 1)
n

r=1
(W
r
L
r
) = 0,
n

r=1
(W
r
L
r
) = 0,
n

r=1
W
r
=
n

r=1
L
r
.
Esta ultima ecuaci on es verdadera, ya que el n umero total de juegos ganados
por n jugadores es igual al n umero total de juegos perdidos. La demostraci on
se sigue al invertir el argumento precedente.
Problemas
1.8.5 .
(a) Dados n umeros positivos reales x y y, demuestre que
2
1/x + 1/y

xy
x +y
2
.
(b) Dados a y b, n umeros reales positivos tales que a +b = 1, demuestre que
2
a
x
+
b
y
ax +by, x > 0, y > 0.
1.8.6 .
1.8. TRABAJO HACIA ATR

AS 51
(a) Si a, b, c son n umeros reales positivos, y a < b +c, muestre que
a
1 +a
<
b
1 +b
+
c
1 +c
.
(b) Si a, b, c son las longitudes de tres segmentos que pueden formar un tri angu-
lo, muestre que lo mismo se vale para 1/ (a +c) , 1/ (b +c) , 1/ (a +b) .
1.8.7 Dos crculos son tangentes externamente en A, y una tangente com un
externa los toca en B y C. El segmento BA se prolonga hasta intersectar al
segundo crculo en D. Demuestre que CD es un di ametro.
1.8.8 Considerese el siguiente argumento. Suponga que satisface
cot + tan 3 = 0.
Entonces, como
tan ( +) =
tan + tan
1 + tan tan
,
se sigue que
cot +
tan + tan 2
1 tan tan 2
= 0
cot (1 tan tan 2) + tan + tan 2 = 0
cot tan 2 + tan + tan 2 = 0
cot + tan = 0
1 + tan
2
= 0
tan
2
= 1.
Ya que la ultima ecuaci on no puede ser cierta, la ecuaci on original no tiene
soluci on (no necesitamos invertir ning un paso porque el paso nal no lleva a
nada). Sin embargo, =
1
4
si satisface cot + tan 3 = 0. Que est a mal en la
argumentaci on?
1.8.9 Con las herramientas Euclidianas (regla y comp as), inscriba un cuadra-
do en un tri angulo dado de manera que un lado del cuadrado quede en un la-
do dado del tri angulo. (Sugerencia: Comience con el cuadrado y construya un
tri angulo alrededor de el semejante al tri angulo dado. Luego haga uso del hecho
de que guras semejantes tienen partes proporcionales.)
Ejemplos adicionales
2.1.5, 7.1.1, 7.4.6. Tambien ver la Seccion 2.2 (Inducci on) y la Seccion 2.5 (Re-
currencia).
52 CAP

ITULO 1. HEUR

ISTICA
1.9. Argumentacion por Contradiccion
Arguentar por contradiccion signica suponer que la conclusion es falsa y en-
tonces obtener deducciones hasta que lleguemos a algo que sea contradictorio,
ya sea con lo que esta dado (el m etodo indirecto) o a lo que se sabe que es
verdadero (reductio ad absurdum) . As, por ejemplo, para probar que

2 es
irracional, podemos suponer que es racional y proceder hasta obtener una con-
tradiccion. El metodo es con frecuencia apropiado cuando la conclusion es facil
de negar, cuando las hip otesis ofrecen poco material susceptible de ser manipu-
lado, o cuando se tienen pocas ideas sobre c omo proceder.
Como un ejemplo sencillo de este metodo de demostracion, considere la siguiente
argumentaci on que muestra que la serie armonica diverge. Suongase, por el
contrario, que converge, a r digamos. Entonces
r = 1 +
1
2
+
1
3
+
1
4
+
1
5
+
1
6
+
1
7
+
1
8
+...
>
1
2
+
1
2
+
1
4
+
1
4
+
1
6
+
1
6
+
1
8
+
1
8
+...
= 1 +
1
2
+
1
3
+
1
4
+...
= r,
una contradiccion. Nos vemos forzados a concluir que la serie diverge.
1.9.1 Dado que a, b, c son enteros impares, demuestre que la ecuaci on ax
2
+
bx +c = 0 no puede tener races racionales.
Soluci on . Supongamos que p/q es una raz racional, donde (sin perdida de
generalidad) p y q son enteros pares. Primero estableceremos que ninguno entre
p y q puede ser par. Supongamos que p es par. De ax
2
+ bx + c = 0 tenemos
que ap
2
+ bpq + cq
2
= 0. Ya que ap
2
+ bpq es par, cq
2
debe ser par, pero esto
es imposible, ya que c y q son ambos impares. Obtenemos una contradicci on
similar si suponemos que q es par. Por lo tanto, ambos p y q son impares y
ap
2
+ bpq + cq
2
= 0. Pero esta ultima ecuaci on establece que la suma de tres
n umeros impares es igual a cero, lo cual es imposible. Por lo tanto, la ecuaci on
no tiene races racionales.
Es ilustrativo considerar otra demostracion de este resultado. Las races de ax
2
+
bx + c = 0 son racionales si y s olo si b
2
4ac es un cuadro perfecto. As,
supongamos que b
2
4ac = (2n + 1)
2
para alg un entero n (por hip otesis, b
2
4ac
es impar, y por lo tanto, si es un cuadrado, debe ser el cuadrado de un entero
impar). Agrupando m ultiplos de 4 tenemos que
1.9. ARGUMENTACI

ON POR CONTRADICCI

ON 53
b
2
1 = 4[n(n + 1) +ac].
Ya que ni n ni n+1 son pares, n(n + 1) +ac es impar. As, el lado derecho de la
ultima ecuaci on es divisible por 4 pero no por 8. Sin embargo, el lado izquierdo
es divisible por 8, ya que b
2
1 = (b 1) (b + 1) y entre b 1 y b +1 es divisible
por 4, mientras que el otro es divisible por 2. Por lo tanto la ecuaci on antes
escrita no puede ser cierta, y tenemos una contradiccion. (En esta demostracion,
hemos llegado a una contradiccion al examinar como se relacionan dos n umeros
respecto a m ultiplos de 8, m as que respecto a m ultiplos de 2 como en la primera
demostracion.)
En las dos secciones siguientes se dan Ejemplos adicionales de demostracion por
contradiccion.
Problemas
1.9.2 En una esta con 2000 personas, en cada grupo de 4 hay al menos una
persona que conoce a las otras tres. Hay tres personas que no se conocen entre s.
Demuestre que las otras 1997 conocen a todo mundo en la esta. (Suponga que
conocer es una relaci on simetrica; esto es, si A conoce B entonces B tambien
conoce a A. Cu al es la respuesta si conocer no necesariamente es una relaci on
simetrica?)
1.9.3 Demuestre que no existen enteros positivos a, b, c y n tales que a
2
+b
2
+
c
2
= 2
n
abc. (De 1.4.3, podemos suponer que a y b son impares y c es par. C omo
son los lados de la ecuaci on respecto a 4?)
1.9.4 Cada par de comunidades de un condado est an enlazadas por exacta-
mente un medio de transporte: cami on, tren o avi on. Los tres medios son uti-
lizados en el condado; ninguna de las comunidades cuenta con los tres medios
simult aneamente, y ninguna tercia de comunidades est a enlazada por parejas
por el mismo medio. Cuatro comunidades pueden estar conectadas de acuerdo a
lo estipulado de la siguiente forma: cami on, AB, BC, CD, DA; tren, AC; avi on,
BD.
(a) De un argumento para mostrar que ninguna comunidad puede tener un
medio de transporte en especial que la conecte a tres comunidades difer-
entes.
(b) De una demostraci on que muestre que cinco comunidades no pueden ser
conectadas de acuerdo a lo estipulado.
54 CAP

ITULO 1. HEUR

ISTICA
1.9.5 Sea S un conjunto de n umeros racionales que es cerrado bajo la suma
y producto (esto es, si a y b son elementos de S, tambien lo son a + b y ab), y
que tiene la propiedad de que para cada n umero racional r se cumple una y s olo
una de las siguientes armaciones: r S, r S, r = 0.
(a) Demuestre que 0 no es elemento de S.
(b) Demuestre que todos los enteros positivos son elementos de S.
(c) Demuestre que S es el conjunto de todos los n umeros racionales positivos.
Ejemplos adicionales
1.5.10,1.6.7, 3.2.1, 3.2.6, 3.2.11, 3.2.13, 3.2.15, 3.2.17, 3.2.18, 3.3.4, 3.3.11, 3.3.15,
3.3.28, 3.4.2, 4.1.3, 4.4.6, 5.4.1. Tambien ser Seccion 1.10 (Paridad) y la Seccion
1.11 (Casos Extremos).
1.10. B usqueda de Paridad
La sencilla idea de paridad, ser par o ser impar, es un concepto poderoso en
la resoluci on de problemas con una gran cantidad de aplicaciones. Revisaremos
algunos ejemplos en esta seccion.
1.10.1 Sean nueve puntos de una retcula en el espacio euclidiano de tres di-
mensiones. Muestre que hay un punto de la retcula en uno de los segmentos
que unen dos de estos puntos.
Soluci on . Solo hay ocho patrones de paridad para estos puntos de la retcula:
(par, par, par), (par, par, impar),...,(impar, impar, impar). Ya que hay nueve
puntos dados, dos de ellos deben tener el mismo patr on de paridad. Su punto
medio es un punto de la retcula, y as queda terminada la demostracion.
1.10.2 Coloque un pe on en cada silla de un tablero de ajedrez de 7 por 7. Es
posible que cada pe on haga simult aneamente un movimiento permitido?
Soluci on . Supongamos que un tablero esta iluminado de la forma usual. El
tablero tiene 49 casillas; supongamos que 24 de ellas son balncas y 25 son negras.
Consideremos los 25 peones que estan en las casillas negras. Si cada uno de ellos
hiciera un movimiento permitido, tendran que moverse a 25 casillas blancas.
Sin embargo, s olo hay 24 casillas blancas disponibles, por lo que dicha jugada
no puede llevarse a cabo.
1.10. B

USQUEDA DE PARIDAD 55
1.10.3 Coloque un pe on sobre el tablero de ajedrez de 4 por n. Es posible, en
4n movimientos consecutivos del pe on, visitar cada casilla del tablero y regresar
a la casilla de partida?
Figura 1.27.
Soluci on . Antes de considerar este problema, es interesante analizar la misma
pregunta para un tablero de 7 por 7. Suponga que se intenta dicho paseo cer-
rado. En el primer movimiento el pe on se mueve a una casilla de color opuesto;
en el segundo regresa a una casilla del mismo color; y as se sigue. Vemos que
despues de un n umero impar de movimientos el pe on ocupara una casilla del
color opuesto al de su casilla original. Un paseo cerrado por el tablero de 7 por
7 requiere de 49 movimientos, un n umero impar. As, el pe on no puede regresar
a su casilla original, y el paseo cerrado es imposible.
Consideremos ahora el tablero de 4 por n. La argumentaci on utilizada para el
de 7 por 7 no puede ser aplicada en este caso, porque 4n es un n umero par. Para
manejar este caso, iluminemos el tablero de la forma que se indica en la Figura
1.27.
observemos que los movimientos del pe on hechos desde una casilla blanca en las
las superior e inferior llevan a casillas blancas en las las segunda y tercera.
Inversamente, en un paseo como el requerido, los movimientos del pe on desde
las dos las interiores llevan a casillas blancas en las dos las exteriores. Esto se
debe a que hay exactamente n casillas blancas en las las exteriores, y estas s olo
pueden ser alcanzadas desde las n casillas blancas en las las inteiores. As pues,
el camino del pe on nunca puede pasar de las casillas blancas a las negras, y por
lo tanto el paseo cerrado es imposible.
1.10.4 Sea n un entero impar mayor que 1, y sea A una matriz simetrica de n
por n tal que cada rengl on y cada columna son una permutaci on de los enteros
1, ..., n. Muestre que cada uno de los enteros 1, ..., n debe aparecer en la diagonal
de A.
Soluci on . Los elementos fuera de la diagonal estan por pares ya que A es
simetrica. Cada n umero aparece exactamente n veces, y esto, junto con el hecho
56 CAP

ITULO 1. HEUR

ISTICA
de que n es impar, no lleva al resultado.
1.10.5 Sea a
1
, a
2
, ..., a
2n+1
un conjunto de enteros con la siguiente propiedad
(P) : si alguno de ellos es eliminado, los restantes pueden ser separados en dos
conjuntos de n enteros con sumas iguales. Demuestre que a
1
= a
2
= ... = a
2n+1
.
Soluci on . Primero, observamos que todos los enteros a
1
, a
2
, ..., a
2n+1
tienen
la misma paridad. Para ver esto, sea A = a
1
+ a
2
+ ... + a
2n+1
. La armaci on
se sigue despues de observar que para cada i, A + a
i
es par (de otra forma los
n umeros restantes no podran ser separados de la manera requerida).
Sea a el n umero m as peque no de a
1
, a
2
, ..., a
2n+1
, y para cada i, sea b
i
= a
i
+a.
El problema es equivalente a mostrar que b
i
= 0 para toda i.
Ahora bien b
1
, b
2
, ..., b
2n+1
cumplen la propiedad (P) . Ya que ninguno de ellos es
cero, positivo m as grande tal que 2
k
divide a cada b
i
. Para cada i, sea c
i
= b
i
/2
k
.
Entonces c
1
, c
2
, ..., c
2n+1
cumplen (P) ; sin embargo, no todos tienen la misma
paridad ( ya que uno de ellos es cero, y otro es impar dada la elecci on de k).
Por lo tanto, todos los b
i
son cero y la demostracion queda terminada.
Problemas
1.10.6 .
(a) Quite la casilla inferior izquierda y la superior derecha de un tablero nor-
mal de ajedrez de 8 por 8. Se puede cubrir el tablero resultante con 31
chas de domin o ? Suponga que cada cha cubre exactamente dos casillas
adyacentes del tablero.
(b) Sean P
1
, ..., P
13
puntos del plano y suponga que estan unidos por los seg-
mentos P
1
P
2
, P
2
P
3
, ..., P
12
P
13,
P
13
P
1
. Es posible trazar una lnea recta que
pase por el interior de cada uno de estos segmentos?
1.10.7 .
(a) Es posible trazar una trayectoria que pase por los arcos de la Figura
1.28(a) que atraviese cada arco una vez y s olo una ? (Sugerencia: Cuente
el n umero de arcos que salen de cada vertice.)
(b) Es posible trazar una trayectoria por las lneas de la Figura 1.28(b) que
pase por cada uni on una y s olo una vez? (Sugerencia: Ilumine los vertices
de forma alternada.)
1.10. B

USQUEDA DE PARIDAD 57
Figura 1.28.
1.10.8 Sea a
1
, a
2
, ..., a
n
un arreglo arbitrario de los n umeros 1, ...., n. De-
muestre que, si n es impar, el producto
(a
1
1) (a
2
2) ... (a
n
n)
es un n umero par.
1.10.9 Muestre que (2
a
1)
_
2
b
1
_
= 2
c
+ 1 es imposible para enteros no
negativos a, b, y c. (Sugerencia: Escriba la ecuaci on de la forma equivalente
2
a+b
2
a
2
b
= 2
c
e investigue las posibilidades de a, b y c.)
1.10.10 Muestre que x
2
y
2
= a
3
siempre tiene soluciones enteras para x y
y siempre que a sea un n umero positivo.
Ejemplos adicionales
1.5.10,1.9.1, 2.2.7, 3.2.13, 3.3.4, 3.3.20, 4.2.16(a), 4.3.4, 7.4.6. Ver seccion 3.2
para una generalizaci on de este metodo.
58 CAP

ITULO 1. HEUR

ISTICA
1.11. Consideracion de Casos Extremos
En las primeras etapas de exploraci on de un problema, con frecuencia es de gran
ayuda considerar las consecuencias de variar los par ametros del problema de un
valor extremo a otro. En esta seccion veremos que la existencia de posiciones
extremas son frecuentemente la clave para entender resultados de existencia
(problemas del estilo pruebe que existe x tal que P (x)).
1.11.1 Dado un n umero nito de puntos en un plano, no todos colineales,
pruebe que existe una lnea recta que pasa exactemente por dos de ellos.
Figura 1.29.
Soluci on. Si P es un punto y L una lnea, sea d (P, L) la distancia desde P a
L. Sea S el conjunto de las distancias positivas d (P, L) conforme P vara por
los puntos dados, y L vara por las lneas que no pasan por P pero que si pasan
por al menos dos de los puntos dados. El conjunto S es no vaco (porque los
puntos dados no son todos colineales ) y nito (hay s olo un n umero nito de
puntos y un n umero nito de rectas que pasan por al menos dos puntos ). Por
lo tanto S tiene un elemento al minimal, digamos d (P, M) . Armamos que M
pasa exactamente por dos de los puntos dados.
Supongamos que M pasa por tres de los puntos dados, digamos P
1
, P
2
y P
3
.
Sea Q el punto de M m as cercano a P. Al menos dos de los puntos P
1
, P
2
y P
3
estan del mismo lado de Q (uno de ellos podra ser Q), digamos P
2
y P
3
(ver
Figura 1.29). Supongamos que los puntos estan etiquetados de forma que P
2
esta m as cerca de P que P
3
. Sea N la lnea atraves de P y P
3
, y observamos
que d (P
2
, N) < d (P, M), una contradiccion respecto a nuestra elecci on de P y
M. Se sigue que M s olo puede pasar por dos de los puntos dados.
Supongamos que M pasa por tres de los puntos dados, digamos P
1
, P
2
y P
3
.
Sea Q el punto de M m as cercano a P. Al menos dos de los puntos P
1
, P
2
, P
3
estan del mismo lado que Q (uno de llos podra ser Q ), digamos P
1
y P
3
(ver
Figura 1.27). Supongamos que los puntos estan etiquetados de forma que P
2
esta m as cerca de P que P
3
. Sea N la lnea atraves de P y P
3
, y observemos
1.11. CONSIDERACI

ON DE CASOS EXTREMOS 59
que d (P
2
, N) < d (P, N) , una contradiccion respecto a nuestra elecci on de P y
M. Se sigue que M s olo p uede pasar por dos de los puntos dados.
1.11.2 Sea A un conjunto de 2n puntos en el plano, de los cuales no hay tres
que sean colineales. Suponga que n de ellos son rojos y los otros n son azules.
Pruebe o refute: Existen n segmentos de lnea recta cerrados, de los cuales no
hay dos que tengan un punto en com un, tales que los extremos de cada segmento
son puntos de A de distinto color.
Soluci on. Si ignoramos las intersecciones de lneas, existe una variedad de
formas en las que los puntos rojos pueden ser apareados con lo azules con n
segmentos de lnea cerrados. Asignemos a cada uno de estos apareamientos la
longitud total de los segmentos de lnea en la conguracion. Como s olo hay un
n umero nito de tales apareamientos, una de estas conguraciones tendr a una
longitud total minimal. Este apareamiento no tendr a cruces entre lneas. (Si
los segmentos R
1
B
1
y R
2
B
2
se intersectan, con R
1
, R
2
puntos rojos y B
1
, B
2
puntos azules, entonces podramos reducir la longitud total de la conguracion
reemplazando estos segmentos por R
1
B
2
y R
2
B
1
.)
1.11.3 En una esta, ning un muchacho baila con todas las muchachas, pero
cada muchacha baila al menos con un muchacho. Pruebe que existen dos parejas
bg y b

que bailan, considerando que si b no baila con g

, tampoco g baila con


b

.
Soluci on . Aunque no es necesario, el problema podra ser m as comprensible si
lo ponemos en terminos matriciales. Hagamos que los renglones de una matriz
correspondan a los muchachos y las columnas a las muchachas. Escribamos un
1 o un 0 en el rengl on b y la columna g dependiendo de si b y g bailan juntos o
no. La condici on de que no hay un muchacho que baile con todas las muchachas
implica que (i) cada rengl on tiene al menos una entrada 0. Similarmente, (ii)
cada columna tiene al menos una entrada 1. Queremos probar que existen dos
renglones, b y b

, y dos columnas, g y g

, cuyas entradas de intersecci on tienen


el patr on
_
_
_
_
_
_
_
_
_
.
.
.
.
.
.
1 0
.
.
.
.
.
.
0 1
.
.
.
.
.
.
_
_
_
_
_
_
_
_
_
o
_
_
_
_
_
_
_
_
_
.
.
.
.
.
.
0 1
.
.
.
.
.
.
1 0
.
.
.
.
.
.
_
_
_
_
_
_
_
_
_
Sea h un rengl on arbitrario. Por (i) hay una entrada 0 en este rengl on, digamos
60 CAP

ITULO 1. HEUR

ISTICA
en la columna k, y por (ii) hay una entrada 1 en la columna k, digamos en el
rengl on m :
k
h
m
_
_
_
_
_
_
_
_
_
.
.
.
.
.
.
1 0
.
.
.
.
.
.
0? 1
.
.
.
.
.
.
_
_
_
_
_
_
_
_
_
Ahora bien, ya que los hicimos si hay una columna que tenga 1 en el rengl on
h y 0 en la columna m, En general, tal columna podra existir. Sin embargo, si
h hubiera sido escogido con anticipacion como rengl on con un n umero maximal
de 1s, entonces tal columna no hubiera existido y el problema no hubiera sido
resuelto.
Con estos antecedentes, podemos reescribir la soluci on en lenguaje independi-
ente de la notacion matricial. Sea b un muchacho que baila con un n umero
maximal de muchachas. Sea g

una muchacha con quien b no baila, y b

un
muchacho con el que g

baila. Entre los compa neros de b, debe haber al menos


una muchacha g que no baile con b

(de otra forma b

tendra m as compa neras


que b ). Entonces las parejas bg y b

resuelven el problema.
1.11.4 Demuestre que el producto de n enteros sucesivos siempre es divisible
por n!.
Soluci on. Primero, observamos que es suciente demostrar el resultado para
n enteros positivos sucesivos. El resultado es obviamente cierto si uno de los
enteros es 0, mientras que si todos los enteros son negativos, basta mostrar que
n! divide a su valor absoluto.
As pues, supongamos que existen n enteros sucesivos cuyo producto no es divis-
ible por n!. De todos estos n n umeros, escojamos al m as peque no; llamemosle N.
Observemos que N > 2, ya que el producto de dos enteros sucesivos siempre es
par. Estamos suponiendo, por lo tanto, que existe un n umero no negativo m tal
que (m+ 1) (m+ 2) ... (m+n) no es divisible por N!. De todos estos n umeros
m, sea M el m as chico. Observemos que M > 0, ya que N! es divisible por N!.
As, estamos suponiendo que (M + 1) (M + 2) ... (M +N) no es divisible por
N!. Ahora,
1.11. CONSIDERACI

ON DE CASOS EXTREMOS 61
(M + 1) (M + 2) ... (M +N 1) (M +N)
= M[(M + 1) (M + 2) ... (M +N 1)] +
N[(M + 1) (M + 2) ... (M +N 1)]
Por la forma en que escogimos M, N! divide a M [(M + 1) (M + 2) ... (M +N 1)].
Por la forma en que escogimos N, (N 1)! divide a (M + 1) (M + 2) ... (M +N 1),
y en consecuencia N! divide a N [(M + 1) (M + 2) ... (M +N 1)]. Combinan-
do, vemos que N! divide al lado derecho de la ultima ecuaci on, en contradiccion
con nuestra suposici on. Esta contradiccion establece el resultado.
(Una prueba ingeniosa de este resultado es identicar al cociente (m+ 1) (m+ 2) ...
(m+n) /n! como el coeciente binomial
_
m+n
n
_
, y por lo tanto es un entero
si m es un entero.)
Problemas
1.11.5 Sea f (x) un polinomio de grado n con coecientes reales y tal que
f (x) 0 para todo n umero real x. Muestre que f (x) + f

(x) + ... + f
(n)
(x)
para todo real x. ( f
(k)
(x) denota la k-esima derivada de f (x) . )
1.11.6 De un ejemplo para mostrar que el resultado de 1.11.1 no necesaria-
mente se cumple para un n umero innito de puntos en el plano. En d onde falla
la demostraci on de 1.11.1 para el caso innito?
1.11.7 Muestre que existe un n umero racional, c/d, con d < 100, tal que
_
k
c
d
_
=
_
k
73
100
_
para k = 1, 2, ..., 99.
1.11.8 Suponga que P
n
es una armaci on, para n = 1, 2, 3, ... . Suponga
adem as que
(i) P
1
es cierto, y
(ii) para cada entero positivo m, P
m+1
es cierto si P
m
es cierto.
62 CAP

ITULO 1. HEUR

ISTICA
Demuestre que P
n
es cierto para todo n. (Sugerencia: Sea S el conjunto de
todos los enteros positivos para los cuales P
n
no es cierto. Sea m el elemento
m as peque no en S, suponiendo que S es no vaco.)
Ejemplos adicionales
3.1.9, 3.3.11, 3.3.28, 4.4.7, 4.4.10, y los referentes dados en 6.3.7. Tambien, ver
seccion 7.6 (El Principio del Emparedado) y 6.2 (El teorema del valor interme-
dio) para ejemplos que requieren la consideraci on de casos extremales.
1.12. Generalizacion
Puede parecer paradojico, pero con frecuencia se presenta el caso en que un
problema puede ser simplicado, y hecho accesible y comprensible, cuando se
le generaliza. Este hecho de la vida es muy apreciado por los matematicos;
de hecho, la abstraccion y la generalizaci on son caractersticas basicas de la
matematica moderna. Un contexto m as general provee de una perspectiva m as
amplia, hace a un lado los rasgos no esenciales, y proporciona todo un arsenal
de tecnicas nuevas.
1.12.1 Eval ue la suma

n
k=1
k
2
2
k
.
Soluci on. Evaluaremos en cambio la suma S (x) =

n
k=1
k
2
x
k
y entonces
calcularemos S
_
1
2
_
. La razon para introducir la variable x es que ahora podemos
utilizar tecnicas del analisis. Sabemos que
n

k=1
x
k
=
1 x
n+1
1 x
, x ,= 1.
Derivando ambos lados obtenemos
n

k=1
kx
k1
=
(1 x) ((n + 1) x
n
) +
_
1 +x
n+1
_
(1 x)
2
=
1 (n + 1) x
n
+nx
n+1
(1 x)
2
.
Multiplicando ambos lados de esta ecuaci on por x, derivando una segunda vez,
y multiplicando el resultado por x obtenemos
1.12. GENERALIZACI

ON 63
S (x) =
n

k=1
k
2
x
k
=
x(1 +x) x
n+1
(nx n 1)
2
x
n+2
(1 x)
3
.
Se sigue que
S
_
1
2
_
=
n

k=1
k
2
2
k
= 6
1
2
n2
_
1
2
n n 1
_
2

1
2
n1
= 6
_
n
2
+ 4n + 6
2
n
_
.
1.12.2 Eval ue el siguiente determinante (determinante de Vandermonde):
det
_
_
_
_
_
1 a
1
a
2
1
a
n1
1
1 a
2
a
2
2
a
n1
2
.
.
.
.
.
.
.
.
.
.
.
.
1 a
n
a
2
n
a
n1
n
_
_
_
_
_
.
Soluci on. Supondremos que a
i
,= a
j
, i ,= j, porque de otra forma el determinate
sera cero. Con el objeto de enfocarnos m as claramente en la idea principal,
consideraremos el caso n = 3 :
det
_
_
1 a a
2
1 b b
2
1 c c
2
_
_
.
En este determinante, reemplacemos c por una variable x. Entonces el deter-
minate es un polinomio P (x) de grado 2. Ademas, P (a) = 0 y P (b) = 0, ya
que la matriz correspondiente, con c sustituda por a o b respectivamente, tiene
entonces dos renglones identicos. Por lo tanto,
P (x) = A(x a) (x b)
para alguna constante A. Ahora, A es el coeciente de x
2
, y, regresando al
determinante, encontramos que este coeciente es
det
_
1 a
1 b
_
.
64 CAP

ITULO 1. HEUR

ISTICA
As A = b a, y el determinante original de 3 por 3 es
P (c) = (b c) [(c a) (c b)].
El caso general es analogo. Sea D
n
el determinante buscado (de orden n ).
Sustituimos a
n
en el ultimo rengl on de la matriz por la variable x. El deter-
minante resultante es un polinomio P
n
(x) de grado n 1, que se hace cero en
a
1
, a
2
, ..., a
n1
. Luego, por el Teorema del Factor,
P
n
(x) = A(x a
1
) (x a
2
) ... (x a
n1
) ,
donde A es constante. Como en el caso anterior, A es el coeciente de x
n
, y
expandiendo sobre el ultimo rengl on queda claro que A = D
n1
. Esto es,
D
n
= P
n
(a
n
) = D
n1
[(a
n
a
1
) (a
n
a
2
) ... (a
n
a
n1
)].
Podemos repetir el argumento para D
n1
; etc. El resultado nal sera
D
n
=
n

k=2
_
k1

i=1
(a
k
a
i
)
_
.
1.12.3 Dado que
_

0
(senx) /x dx =
1
2
, eval ue
_

0
_
sen
2
x
_
/x
2
dx.
Soluci on . Evaluaremos la integral m as general
I (a) =
_

0
_
sen
2
ax
_
/x
2
dx, a 0.
utilizando una tecnica llamada diferenciacion de par ametros.
Derivando cada lado de la ecuaci on con respecto a a; obtenemos
I

(a) =
_

0
2senaxcos ax x
x
2
dx
=
_

0
sen2ax
x
dx.
Ahora, con y = 2ax, obtenemos dy = 2adx, y
1.12. GENERALIZACI

ON 65
I

(a) =
_

0
seny
y
=
1
2
.
Integrando ambos lados queda
I (a) =
1
2
a +C. C constante.
Ya que I (0) = 0, tenemos que C = 0, As I (a) =
1
2
a, a 0. Haciendo a = 1 se
tiene I (1) =
_

0
_
sen
2
x
_
/x
2
dx =
1
2
. (Casualmente, el valor de
_

0
(senx) /x
dx puede ser encontrado al evaluar una curva en el plano complejo.)
Problemas
1.12.4 Tomando x los valores apropiados en la expansi on binomial
(1 +x)
n
=
n

k=1
_
n
k
_
x
k
(o en una de sus derivadas) eval ue cada uno de los siguientes
(a)

n
k=1
k
2
_
n
k
_
, (b)

n
k=1
3
k
_
n
k
_
,
(c)

n
k=1
1
k+1
_
n
k
_
, (d)

n
k=1
(2k + 1)
_
n
k
_
.
1.12.5 Eval ue
det
_

_
1 a a
2
a
4
1 b b
2
b
4
1 c c
2
c
4
1 d d
2
d
4
_

_
.
(Sustituya d por una variable x; haga uso del hecho de que la suma de las races
de un polinomio de cuarto grado es igual al coeciente de x
3
.)
1.12.6 .
(a) Eval ue
_

0
(e
x
senx) /x dx. (Considere G(k) =
_

0
(e
x
senkx) /x dx y
utilice diferenciaci on parametrica.)
(b) Eval ue
_
1
0
(x 1) / ln x dx. (Considere H (m) =
_
1
0
(x
m
1) / ln x dx y
utilice diferenciaci on parametrica.)
66 CAP

ITULO 1. HEUR

ISTICA
(c) Eval ue
_

0
arctan (x) arctan x
x
dx.
(Considere F (a) =
_

0
arctan(ax)arctan x
x
dx y utilice diferenciaci on parametri-
ca.)
1.12.7 Cu al n umero es m as grande.
3

60 o 2 +
3

7? (Elevar al cubo cada


n umero lleva a complicaciones que no son de f acil resoluci on. Considere en
cambio el problema m as general: Cu al cantidad es mayor.
3
_
4 (x +y) o
3

x +
3

y, donde x, y 0? Haga x = a
3
, y = b
3
.)
Ejemplos adicionales
1.4.2, 2.2.6, 2.2.7, 4.1.4, 5.1.3, 5.1.4, 5.1.9, 5.1.11, 5.4.4, 5.4.5, 5.4.6, 5.4.7, 6.9.2,
7.4.4. Tambien, ver la seccion 2.4 (Inducci on y Generalizacion)
Captulo 2
Dos Importantes Principios:
El de Inducccion y el del
Palomar.
Las proposiciones matematicas vienen en dos formas: proposiciones universales
las cuales establecen que algo es verdadero para todos los valores de X en
alg un conjunto especicado, y proposiciones existenciales las cuales establecen
que algo es verdadero para alg un valor de X en alg un valor especicado. Las
primeras son expresables en la forma para todo X (en un conjunto S), P(X).,
las del segundo tipo son expresables en la forma all existe un X (en el conjunto
S)tal que P(X), donde P(X) es una armaci on a cerca de X. En este captulo
vanos a considerar dos importantes tecnicas para el trato con esos dos tipos
de armaciones: (i) el principio de induccion matematica, para proposiciones
universales, y (ii) el principio del palomar para proposiciones existenciales.
2.1. Induccion: Construccion Sobre P(k).
Sea a un entero y P(n) una proposici on (armaci on) sobre n para cada entero
n a. El principio de induccion matematica establece que:
Si
(i) P(a) es verdadero, y
67
68CAP

ITULO2. DOS IMPORTANTES PRINCIPIOS: EL DE INDUCCCI

ONYEL DEL PALOMAR.


(ii) para cada entero k a, P(k) verdadero implica P(k+1) verdadero entonces
P(n) es verdadero para todo entero n a.
Notese que el prinicipio lo habilitamos, en dos simples pasos, probar un n umero
innito de proposiciones (a saber, P(n) es verdadero para todo entero n a).
El metodo es especialmente conveniente cuando un modelo ha sido establecido
(ver seccion 1.1 b usqueda de un modelo) para los primeros casos especiales
(P(a), P(a + 1), P(a + 2),.....).
En esta seccion vamos a considerar argumentos de induccion como, en el paso
(ii), procediendo directamente de la verdad de P(k) hacia la verdad de P(k+1),
esto es la verdad de P(k + 1) es construida sobre la consideraci on inicial de
la verdad de P(k).
Esto es en ligero contraste con los argumentos (considerados en la siguiente
seccion) en los que se comienza con una consideraci on de P(k + 1).
2.1.1 Use inducci on matem atica para probar el teorema del binomio:
(a +b)
n
=

n
i=0
_
n
i
_
a
i
b
ni
, n un entero positivo.
Soluci on. Es facil checar que la formula vale para n = 1.
Suponiendo que la formula vale para el entero k (estamos trabajando sobre la
verdad de P(k)), multiplicando ambos lados por (a +b) obtenemos
(a +b)
k
(a +b) =
_

n
i=0
_
k
i
_
a
i
b
ki
_
(a +b)
=

k
i=0
_
k
i
_
a
i+1
b
ki
+

k
i=0
_
k
i
_
a
i
b
k+1i
en la primera suma, hacemos el cambio de variables j = i + 1, y obtenemos.
2.1. INDUCCI

ON: CONSTRUCCI

ON SOBRE P(K). 69
=

k+1
j=1
_
k
j 1
_
a
j
b
k+1j
+

k
i=0
_
k
i
_
a
i
b
k+1i
=
_

k
j=1
_
k
j 1
_
a
j
b
k+1j
+a
k+1
_
+
_

k
i=1
_
k
i
_
a
i
b
k+1i
+b
k+1
_
= a
k+1
+
_

k
i=1
__
k
i 1
_
+
_
k
i
__
a
i
b
k+1i
_
+b
k+1
= a
k+1
+

k
i=1
_
k + 1
i
_
a
i
b
k+1i
+b
k+1
=

k+1
i=0
_
k + 1
i
_
a
i
b
k+1i
donde hemos hecho uso de la identidad basica
_
k
i 1
_
+
_
k
i
_
=
_
k + 1
i
_
(ver
seccion 2.5.2). Esta es la forma para P(k+1), entonces por induccion, la prueba
se ha completado.
2.1.2 Suponga que 0 < a
1
< a
2
< .... < a
n
, y suponga e
i
= 1. Probar
que

n
i=1
e
i
a
i
toma al menos
_
n + 1
2
_
valores distintos seg un varie e
i
sobre 2
n
posibles combinaciones de signos.
Soluci on. Cuando n = 1, hay exactamente 2 valores distintos (a
1
y a
1
), y
_
2
2
_
= 1, el resultado es valido.
Suongase que este resultado es verdadero cuando n = k; esto es, que

n
i=1
e
i
a
i
tiene al menos
_
k + 1
2
_
valores distintos, y suponga que a
k+1
> a
k
. Nosotros
necesitamos generar
_
k + 2
2
_

_
k + 1
2
_
= k + 1 sumas adicionales. Esto lo
podemos conseguir en la siguiente forma: Suponga que S =

k
i=1
a
i
y note
que S + a
k+1
, S + (a
k+1
2a
1
) , ..., S + (a
k+1
2a
k1
) , S + (a
k+1
2a
k
) , son
distintos y mayores que cada una de las sumas obtenidas anteriormente. (Para
ver esto note que S +(a
k+1
2a
k
) > S +(a
k+1
2a
k+1
) = S a
k+1
). Alli hay
k + 1 n umeros en esa lista, entonces el resultado se sigue por induccion.
La induccion matematica es un metodo que se puede utilizar en un problema de
la forma Probar que P (n) es verdadero para todo n a. Pero se debe notar
que la induccion tambien se aplica a muchos problemas donde la cuanticaci on
es sobre conjuntos mas generales. Por ejemplo, una proposici on acerca de todos
los polinomios puede ser probada por induccion sobre el grado del polinomio. Un
teorema acerca de todas las matrices puede ser manejado por induccion sobre
el tama no de la matriz. Varios resultados referentes a proposiciones en l ogica
simbolica son realizados por induccion sobre el n umero de conectivos l ogicos en
70CAP

ITULO2. DOS IMPORTANTES PRINCIPIOS: EL DE INDUCCCI

ONYEL DEL PALOMAR.


la proposici on. La lista de conjuntos inductivos poco comunes puede continuar
indenidamente. Nosotros estamos satisfechos de ver exactamente dos ejemplos
aqui., otros ejemplos son distribuidos a traves del libro. (ver las cuatro secciones
siguientes y las listas en los ejmplos adicionales).
2.1.3 Si V , E y F son, respectivamente, los n umeros de vertices, aristas y
caras de un mapa plano conexo, entonces
V E +F = 2.
Soluci on. El entendimiento intuitivo de los terminos es este resultado es prob-
ablemente exacto, pero para asegurarse, aqui estan las deniciones.
Una red es una gura (en un plano o en el espacio) consistente de un n umero
nito y mayor que cero de arcos, donde dos arcos no se intersectan excepto
posiblemente en sus puntos nales. Los puntos nales de esos arcos son llamados
vertices de la red.
Un camino en una red es una sucesion de diferentes arcos en la red que puede ser
recorrido continuamente sin repasar un arco. Una red es conexa si cualesquiera
dos diferentes vertices de la red son vertices de alg un camino en la red. Un
mapa es una red, junto con una supercie la cual contiene a la red. Si esta
supercie es plana el mapa es llamado un mapa plano, los arcos de un mapa
plano son llamados aristas. Las caras de un mapa plano son las regiones que
estan denidas por las fronteras (aristas) de el mapa (el oceano es tomado en
cuenta como una cara).
Figura 2.1.
La gura 2.1. muestra tres ejemplos de redes conexas. Las primeras dos son
mapeos planares. En el primero, V = 4, E = 4, F = 2., en el segundo, V =
5, E = 6, F = 3. La tercera red es un mapa no plano. Sin embargo, si nosotros
lo aplastamos sobre un plano y llamamos vertices a los nuevos puntos de inter-
seccion, entonces tenemos V = 10, E = 20, F = 12.
2.1. INDUCCI

ON: CONSTRUCCI

ON SOBRE P(K). 71
Ahora volviendo a la consideraci on del teorema. La idea clave en la prueba de
este resultado es que es realizable porque los mapas planos conexos se pueden
construir de un vertice singular por una sucesion de las siguientes construcciones
(cada uno de los cuales es un mapeo conexo):
i)] sumando un vertice en una arista existente (. . quedando as . . .).
ii)] sumando una arista a un vertice la cual parte de el y regresa a el (. quedando
as _).
iii)] sumando una arista entre dos vertices existentes (: : quedando as ).
iv)] sumando una arista y un vertice a otro vertice existente (. quedando as . .).
Vamos aplicar induccion sobre el n umero de pasos requeridos para la construc-
cion del mapeo planar conexo. Si la red consiste de un solo punto, entonces
V = 1, F = 1, E = 0, teniendose V E +F = 2.
Suongase que el resultado es valido cuando se requieren K pasos en la con-
strucci on. El cambio neto para cada uno de los pasos es dado en la siguiente
tabla.
Operacion V E F (V E +F)
(i)
(ii)
(iii)
(iv)
+1 +1 0
0 +1 +1
0 +1 +1
+1 +1 0
0
0
0
0
Observese que la cantidad V F + E permanece invariable cuando es tomado
el (k + 1) esimo paso. As la prueba se completa por induccion.
2.1.4 Dado un entero positivo n y un n umero real x, probar que:
[[x]] +
__
x +
1
n
__
+
__
x +
2
n
__
+ +
__
x +
n 1
n
__
= [[nx]] .
Soluci on. Aunque hay un parametro n entero en este problema, no podemos
usar induccion sobre n para un x jo. Tambien por regla no podemos usar
induccion sobre x, ya que x toma rangos sobre los n umeros reales (pues dado
un n umero x real no podemos determinar el n umero real y mayor inmediato a
x). Por tanto no es claro como se puede aplicar la induccion a este problema.
72CAP

ITULO2. DOS IMPORTANTES PRINCIPIOS: EL DE INDUCCCI

ONYEL DEL PALOMAR.


La idea es probar la formula para un n jo y para todo x en el subintervalo
_
k
n
(k+1)
n
_
para k = 0, 1, 2, . . . .
Primero sup ongase que x pertenece al subintervalo [0 1/n) entonces
_
x +
l
n

= 0
para todo l = 0, 1, . . . , n1, tambien que

n1
i=0
_
x +
1
n

= 0. Ademas [nx] = 0.
Luego la formula es valida en el primer subintervalo.
Ahora sup ongase que la formula es cierta en el subintervalo
_
k1
n
k
n

, donde k
es un entero positivo y x se supone un n umero real en este intervalo. Entonces
[[x]] +
__
x +
1
n
__
+
__
x +
2
n
__
+ +
__
x +
n 1
n
__
= [[nx]] .
Ahora sumando 1/n a x (lo que nos da un nuevo n umero en
_
k
n
,
k+1
n

) en cada
uno de los terminos, excepto en el termino nal, del lado izquierdo se repite
cada uno de los terminos de la ecuaci on previa, y el termino nal
_
x +
n1
n

,
nos queda [x + 1] que excede a [x] por 1. Luego reemplazando en la ecuaci on
previa a x por x +
1
n
el lado izquierdo crece en 1.
Al mismo tiempo, cuando x en [nx] es reemplazado por x +
1
n
, el valor se
incrementa en 1. Ya que ambos lados de la ecuaci on se incrementan en 1 cuando
x es reemplazado por x +
1
n
, el resultado continua siendo verdadero para todos
los n umeros en el intervalo
_
k
n
,
k+1
n

.
Por induccion la formula es valida para todos los valores positivos de x. Un
argumento similar muestra que es valida para todos los valores negativos de x
(reemplazando x por x
1
n
).
El siguiente ejemplo es una buena ilustraci on de la construcci on de P (k + 1) a
partir de P (k) .
2.1.5 Si a > 0 y b > 0, entonces (n 1) a
n
+ b
n
na
n1
b, n es un entero
positivo, con la igualdad solo si a = b.
Soluci on. El resultado es verdadero para n = 1, suponiendo que la formula es
verdadera para el entero k. Para construir P (k + 1) , debemos, obtener el lado
izquierdo apropiado
(i) multiplicar por a:
(k 1) a
k+1
+b
k
a ka
k
b
2.1. INDUCCI

ON: CONSTRUCCI

ON SOBRE P(K). 73
(ii) sumando a
k+1
:
ka
k+1
+b
k
a ka
k
b +a
k+1
(iii) restando b
k
a :
ka
k+1
ka
k
b +a
k+1
b
k
a
(iv) sumando b
k+1
:
ka
k+1
+b
k+1
ka
k
b +a
k+1
b
k
a +b
k+1
Estamos suponiendo que esta desigualdad es una igualdad solo si a = b. Solo
queda mostrar que ka
k
b +a
k+1
b
k
a +b
k+1
(k + 1) a
k
b con la igualdad solo
si a = b. Para ello trabajaremos hacia atr as:
ka
k
b +a
k+1
b
k
a +b
k+1
(k + 1) a
k
b
a
k
b +a
k+1
b
k
a +b
k+1
0,
a
k
(a b) +b
k
(b a) 0,
_
a
k
b
k
_
(a b) 0
el ultimo paso es verdadero (
_
a
k
b
k
_
y (a b) tienen el mismo signo) con
la igualdad se da solo si a = b, entonces la prueba se sigue por induccion.
(Observe que: este resultado es un caso especial de el signicado geometrico de
una desigualdad aritmetica, ver seccion 7.2.).
Problemas
2.1.6 .
Use inducci on para probar que 1 + 1/

2 + 1/

3 + + 1/

n < 2

n.
Use inducci on para probar que 2!4! . . . (2n!) ((n + 1)!)
n
.
74CAP

ITULO2. DOS IMPORTANTES PRINCIPIOS: EL DE INDUCCCI

ONYEL DEL PALOMAR.


2.1.7 El plano euclidiano es dividido en regiones por el trazo de un n umero
nito de lineas rectas. Mostrar que es posible colorear cada una de esas regiones
de azul o rojo en una manera tal que no haya dos regiones adyacentes que tengan
el mismo color.
2.1.8 Probar que la ecuaci on x
2
+ y
2
= z
n
tiene una soluci on en los enteros
positivos (x, y, z) para todo n = 1, 2, 3, . . . . (Para una buena prueba, divida en
dos casos: n par y n impar. Para una prueba no inductiva, ver 3.5.1.).
2.1.9 Un grupo de n personas juega un torneo robin-round. Cada juego nal-
iza en una victoria o una dorrota. Mostrar que es posible etiquetar a los ju-
gadores con p
1
, p
2
, p
3
, . . . , p
n
en una manera tal que p
1
derrote a p
2
, p
2
derrote
a p
3
,. . .,p
n1
derrote a p
n
.
2.1.10 Si cada persona, en un grupo de n personas, es amigo de al menos la
mitad de personas en el grupo, entonces es posible sentar las n personas en un
crculo de manera que cada uno este sentado solo junto a amigos.
2.1.11 Los siguientes pasos conducen a otra prueba del teorema del binomio.
Conocemos que (a +x)
n
se puede escribir como un polinomio de grado n, de
manera que ahi hay constantes A
0
, A
1
, . . . A
n
tales que
(a +x)
n
= A
0
+A
1
x +A
2
x
2
+ +A
n
x
n
.
Use inducci on para describir la ecuaci on que resulta tomando progresivamente
la k esima derivada de cada lado de esta ecuaci on (k = 1, 2, . . . , n) .
Evaluar A
k
para k = 0, 1, . . . , n poniendo x = 0 en la k esima ecuaci on
obtenida en la parte (a).
2.1.12 Su ongase que f : R R es una funci on para la cual f (2x f (x)) = x
para todo x, suponga que r es un n umero real jo.
Probar que si f (x) = x+r, entonces f (x nr) = (x nr) +r para todo entero
positivo n.
Probar que si f es una funci on uno a uno (o sea f (x) = f (y) implica x = y )
entonces la propiedad en (a) es v alida para todo entero n.
Ejemplos adicionales
1.1.2, 1.1.8, 3.2.8, 6.5.13, 7.1.4.
2.2. INDUCCI

ON: PENSANDO SOBRE P (K + 1) 75


2.2. Induccion: Pensando sobre P (k + 1)
En esta seccion vamos a considerar argumentos de induccion en los cuales em-
pezamos con un avance directo sobre p (k + 1) y en los cuales maniobramos
hacia atras para explotar la hipotetica verdad de P (k) . Teoricamente, los argu-
mentos en esta seccion se puedieron todos refundar como en la seccion previa y
viceversa. Sin embargo, desde un punto de vista practico, muchas veces es mas
convincente pensar un camino, que es mejor que el otro.
2.2.1 Probar que
n
5
5
+
n
4
2
+
n
3
3

n
30
es un entero para n = 0, 1, 2, . . . .
Soluci on. El resultado es obvio cuando n = 0, suponiendo que la formula da
un entero cuando n = k. Necesitamos probar que
(k + 1)
5
5
+
(k + 1)
4
2
+
(k + 1)
3
3

(k + 1)
30
es un entero. Expandiendo la formula
k
5
+ 5k
4
+ 10k
3
+ 10k
2
+ 5k + 1
5
+
k
4
+ 4k
3
+ 6k
2
+ 4k + 1
2
+
k
3
+ 3k
2
+ 3k + 1
3

k + 1
30
y recombinando (para hacer uso de P (k)):
_
k
5
5
+
k
4
2
+
k
3
3

k
30
_
+
__
k
4
+ 2k
3
+ 2k
2
+k
_
+
_
2k
3
+ 3k
2
+ 2k
_
+
_
k
2
+k
_
.
La primera parte es un entero por la hip otesis inductiva y la segunda parte
es tambien un entero porque es la suma de enteros. Luego la prueba se sigue
por induccion. (Observe que dicil hubiera sido llegar a la verdad de P (k + 1)
partiendo de la de P (k) .)
2.2.2 Sean a, b, p
1
, p
2
, . . . , p
n
n umeros reales con a ,= b, Denimos f (x) =
(p
1
x) (p
2
x) (p
3
x) (p
n
x) . Mostrar que
76CAP

ITULO2. DOS IMPORTANTES PRINCIPIOS: EL DE INDUCCCI

ONYEL DEL PALOMAR.


det
_

_
p
1
a a a a a
b p
2
a a a a
b b p
3
a a a
b b b p
4
a a
.
.
.
.
.
.
.
.
.
.
.
.
.
.
.
.
.
.
.
.
.
b b b b p
n1
a
b b b b b p
n
_

_
=
bf (a) af (b)
b a
.
Soluci on. Este es similar a muchos problemas de determinantes que se pueden
resolver con induccion matematica. Cuando n = 1, tenemos det (p
1
) = p
1
y
f (x) = (p
1
x) entonces
bf (a) af (b)
b a
=
b (p
1
a) a (p
1
b)
b a
= p
1
y el resultado se cumple.
Suongase que la igualdad es cierta para k 1, k > 1, considere el caso para k
n umeros reales p
1
, . . . p
k
. (Comenzamos por la situaci on para P (k) y el metodo
recae hacia atras sobre la verdad de P (k 1) para completar el paso inductivo.)
Deseamos evaluar:
det
_

_
p
1
a a a a a
b p
2
a a a a
b b p
3
a a a
b b b p
4
a a
.
.
.
.
.
.
.
.
.
.
.
.
.
.
.
.
.
.
.
.
.
b b b b p
k1
a
b b b b b p
k
_

_
Restando la segunda columna a la primera (esto no hace cambiar el determi-
nante):
det
_

_
p
1
a a a a a a
b p
2
p
2
a a a a
0 b p
3
a a a
0 b b p
4
a a
.
.
.
.
.
.
.
.
.
.
.
.
.
.
.
.
.
.
.
.
.
0 b b b p
k1
a
0 b b b b p
k
_

_
y expandiendo la primera columna obtenemos
2.2. INDUCCI

ON: PENSANDO SOBRE P (K + 1) 77


(p
1
a) det
_

_
p
2
a a a
b p
3
a a
.
.
.
.
.
.
.
.
.
.
.
.
.
.
.
b b p
k1
a
b b b p
k
_

_
(b p
2
) det
_

_
a a a a
b p
3
a a
.
.
.
.
.
.
.
.
.
.
.
.
.
.
.
b b p
k1
a
b b b p
k
_

_
Estos dos nuevos determinantes (de matrices (k 1) por (k 1)) son de la forma
para la cual podemos aplicar la suposici on inductiva P (k 1) . Para hacer esto,
necesitamos introducir alguna notacion. Para el primer determinate, estable-
cemos F (x) = (p
2
x) (p
3
x) . . . (p
k
x) y para el segundo, establecemos
G(x) = (a x) (p
3
x) . . . (p
k
x) . Entonces por la suposici on inductiva,
(p
1
a)
_
bF (a) aF (b)
b a
_
(b p
2
)
_
bG(a) aG(b)
b a
_
.
Pero G(a) = 0 y (p
1
a) F (a) = f (a) , y por lo tanto tenemos
bf (a) a (p
1
a) (p
2
b) . . . (p
k
b) a (a b) (p
2
b) . . . (p
k
b) ,
b a
bf (a) a (p
2
b) . . . (p
k
b) [(p
1
a) + (a b)]
b a
bf (a) af (b)
b a
El resultado se sigue por induccion.
Problemas
2.2.3 De una prueba para el paso inductivo en 1.1.3.
78CAP

ITULO2. DOS IMPORTANTES PRINCIPIOS: EL DE INDUCCCI

ONYEL DEL PALOMAR.


2.2.4 Para todo x en el intervalo 0 x , probar que [sen nx[ n sen x,
n un entero no negativo.
2.2.5 Suponga que Q denota el conjunto de los n umeros racionales. Hallara
todas las funciones f : Q Q las cuales satisfacen las dos condiciones sigu-
ientes: (i) f (1) = 2 y (ii) f (xy) = f (x) f (y) f (x +y) + 1 para todo x, y en
Q.
2.2.6 Si a, b, c 1, probar que 4 (abc + 1) (1 +a) (1 +b) (1 +c) . (Sugeren-
cia: Probar, en forma mas general, que 2
n1
(a
1
a
2
. . . a
n
+ 1) (1 +a
1
) (1 +a
2
) . . . (1 +a
n
).)
2.2.7 Dado un conjunto de 51 enteros entre 1 y 100 (inclusive), mostrar que
al menos un elemento de el conjunto debe dividir a otro elemento de el conjunto.
(Sugerencia: probar en forma m as general, que la misma propiedad se cumple
cuando n+1 enteros son elegimos de los enteros entre 1 y 2n (inclusive).) Para
una prueba no inductiva ver 2.6.1.
2.2.8 Critique la prueba dada abajo para el siguiente teorema: Una matriz
nn de enteros no negativos tiene la propiedad de que para cualquier elemento
que sea cero, la suma de la la mas la columna que contienen a dicho elemento
es al menos n. Mostrar que la suma de todos los elementos de la matriz es al
menos n
2
/2.
Prueba ( ?): El resultado se cumple para n = 1. Suponiendo que el resultado se
cumple para n = k 1, considere una matriz k por k. Si no tiene elementos
iguales a cero, el resultado se cumple obviamente. Si a
ij
= 0, la suma de la la
i y la columna j es al menos k, por hip otesis, y la suma de los elementos en la
submatriz (k 1) (k 1) obtenida al suprimir la la i y la columna j es al
menos (k 1)
2
/2 (por hip otesis inductiva). Se sigue la suma de los elementos en
la matriz kk es al menos (k1)
2
/2+k =
_
k
2
2k + 1
_
/2+k =
_
k
2
+ 1
_
/2
k
2
/2. El resultado se sigue por inducci on.
Ejemplos adicionales.
1.1.11, 1.12.2, 3.1.11, 4.2.21, 4.3.5, 4.3.24, 6.5.12, 6.6.1, 7.1.6, 7.1.13, 7.2.5, 7.3.5.
2.3. Induccion Forma Fuerte
Suponga que a es un entero y P (n) una proposici on acerca de n para todo
entero n a. La forma fuerte de induccion matematica estable que:
2.3. INDUCCI

ON FORMA FUERTE 79
si
(i) P (a) es verdadero,
(ii) para cada entero k a, P (a) , P (a + 1) , . . . , P (k) verdaderos implica P (k + 1)
sea verdadero,
Entonces P (n) es verdadero para todo n a.
La diferencia de la forma de induccion anterior esta en que ahora estamos dando
una suposici on fuerte en el paso (ii), a saber, podemos suponer P (a) , P (a + 1) , . . . , P (k) ,
en lugar solamente de P (k), para probar P (k + 1) . Te oricamente, las dos for-
mas de induccion son equivalentes, pero en la practica hay problemas los cuales
son mas faciles de resolver con esta induccion fuerte.
2.3.1 (Teorema de Pick) Probar que el area de un polgono retcula simple
(un polgono con puntos reticulares como vertices cuyos lados no se cruzan) es
dado por I +
1
2
B1, donde I y B denotan respectivamente el n umero de puntos
interiores y puntos reticulares frontera del polgono.
Soluci on. Nosotros usaremos induccion sobre el n umero de lados del polgono.
El caso de un tri angulo es dado en 1.7.3. Considere, entonces, un polgono retic-
ula simple P con k lados, k > 3. polgono reticula simple P con k lados, k > 3.
Primero establecemos que un tal polgono tiene una diagonal interior. Esto es
claro si el polgono es convexo (equivalente, si todos
As que su ongase el angulo interior de alg un vertice, llamado V, con mas de 180
0
.
Entonces un rayo que nace en V y barre el interior del polgono debe tocar otro
vertice (de no ser as el polgono encierra un area innita), y esto determina una
diagonal interior D con V como un punto nal.
Suongase que nuestro polgono P tiene I puntos interiores y B puntos frontera.
La diagonal interior D divide a P en dos polgonos retculas simples P
1
y P
2
con
I
1
y I
2
puntos interiores respectivamente, tambien con B
1
y B
2
puntos frontera
respectivamente. Supongase que hay x puntos reticulares sobre D, excluyendo
sus puntos extremos. Entonces B = B
1
+B
2
2 2x, I = I
1
+I
2
+x.
Ahora, si A, A
1
, A
2
denotan las areas de P, P
1
y P
2
respectivamente. Entonces,
80CAP

ITULO2. DOS IMPORTANTES PRINCIPIOS: EL DE INDUCCCI

ONYEL DEL PALOMAR.


A = A
1
+A
2
=
_
I
1
+
1
2
B
1
1
_
+
_
I
2
+
1
2
B
2
1
_
= (I
1
+I
2
) +
1
2
(B
1
+B
2
) 2
= (I
1
+I
2
+x) +
1
2
(B
1
+B
2
2x) 2
= I +
1
2
(B + 2) 2
= I +
1
2
B 1.
El resultado se sigue por induccion.
Observe en este ejemplo que es el primer paso del argumento de induccion el mas
difcil (realizado en 1.7.3); el paso inductivo (paso (ii) ) es concepcionalmente
mas simple.
Problemas
2.3.2 .
(a) Probar que cualquier entero positivo y mayor que uno puede ser escrito
como un producto de n umeros primos.
(b) El postulado de Bertrands, alguna vez postulado pero ahora un conocido
teorema, establece que para cualquier n umero x > 1. alli existe un n umero
primo entre x y 2x. Use este hecho para mostrar que cualquier entero
positivo puede ser escrito como una suma de distintos primos. (Para esta
prueba suponga que uno es un primo).
2.3.3 .
(a) Mostrar que cualquier entero positivo puede ser escrito como una suma de
distintos n umeros de Fibonacci.
(b) Suponga que k m signicando esto k m + 2. Mostrar que cualquier
entero positivo n tiene una representaci on de la forma n = F
k1
+ F
k2
+
. . .+F
kr
, donde F
ki
son n umeros de Fibonacci y k
1
k
2
. . . k
r
0.
(c) Mostrar que la representaci on en la parte (b) es unica.
2.4. INDUCCI

ON Y GENERALIZACI

ON 81
Ejemplos adicionales
3.1.1, 3.1.2, 3.1.18, 3.5.5, 6.2.3.
2.4. Induccion y Generalizacion
Hemos visto (en la seccion 1.12) que un problema es algunas veces m as facil de
manejar cuando se reconsidera en forma mas general. Esto es verdadero tambien
en problemas de induccion. Por ejemplo, puede suceder que las proposiciones
originales P (1) , P (2) , P (3) , . . . , no contengan suciente informaci on que per-
mitan a uno llevar a cabo el paso inductivo (paso (ii) ). En este caso es natural
reformular las proposiciones en una forma m as consistente, m as generalmente
Q(1) , Q(2) , . . . (donde Q(n) implica P (n) para cada n), y mirando nuevamente
hacia una prueba inductiva.
2.4.1 Si A
1
+ +A
n
= , 0 < A
i
, i = 1, . . . , n., entonces
senA
1
+ +senA
n
nsen

n
.
Figura 2.2.
Soluci on. Suponga que P (k) es la proposici on de el teorema para un k dado
y suponiendo que P (k) es verdadera. Para el paso inductivo, suponga que A
1
+
A
2
+ A
k
+ A
k+1
= , 0 < A
i
, i = 1, . . . , k + 1. En esta forma, no es
claro como hacer uso de P (k) ., sin embargo podemos, por ejemplo, agrupar
A
k
y A
k+1
juntos, as que A
1
+ .....+ A
k1
+ (A
k
+A
k+1
) = y aplicando la
suposici on inductiva obtenemos
82CAP

ITULO2. DOS IMPORTANTES PRINCIPIOS: EL DE INDUCCCI

ONYEL DEL PALOMAR.


sen A
1
+ +sen A
k1
+ sen (A
k
+A
k+1
) k sen

k
.
Pero todava no vemos claro que esto implique P (k + 1) :
sen A
1
+ +sen A
k
+ sen A
k+1
(k + 1) sen

k + 1
.
El requisito de que las A

i
s sumen parece muy restrictivo. Considere inmedi-
atamente la siguiente proposici on Q(n) :
Si 0 < A
i
, i = 1, . . . , entonces
sen A
1
+ +sen A
n
n sen
_
A
1
+ +A
n
n
_
.
(Observe que Q(n) implica P (n) . ) Obviamente Q(1) es verdadera. Suongase
que Q(k) es verdadera, tambien que 0 < A
i
, , i = 1, . . . , k + 1. Entonces
sen A
1
+ +sen A
k
+sen A
k+1
k sen
_
A
1
+ +A
k
k
_
+sen A
k+1
= (k + 1)
_
k
k + 1
sen
_
A
1
+ +A
k
k
_
+
1
k + 1
sen A
k+1
_
(k + 1)
_
sen
_
k
k + 1
(A
1
+ +A
k
) +
1
k + 1
sen A
k+1
__
= (k + 1) sen
_
A
1
+ +A
k+1
k + 1
_
.
(la desigualdad se mantiene en sentido favorable, en estos pasos gracias al re-
sultado de 1.2.12b).)
El resultado se sigue ahora por induccion.
Ahora ya podemos probar la conjetura creada en 1.6.2 e): El polgono de mayor
area que puede ser inscrito en un crculo es el polgono regular, para vericar
esto, suponga que P
1
, P
2
, . . . , P
n
, n 3, son los vertices sucesivos de un polgono
inscrito (inscrito en un crculo de radio r). Suponga que O denota el centro del
crculo; suponga que T
i
denota el area del tri angulo P
1
OP
i+1
, i = 1, . . . , n
(establecemos que P
n+1
= P
1
); Sea A
i
= P
i
OP
i+1
(gura 2.2). Entonces
2.4. INDUCCI

ON Y GENERALIZACI

ON 83
T
i
= 2
_
1
2
_
r cos
1
2
A
i
__
r sen
1
2
A
i
__
= r
2
cos
1
2
A
i
sen
1
2
A
i
=
1
2
r
2
sen A
i
.
El polgono de area m axima debe satisfacer 0 < A
i
< para cada i. nuestro
resultado precedente muestra que
Area del polgono =
n

i=1
T
i
=
n

i=1
1
2
r
2
sen A
i
=
1
2
r
2
n

i=1
sen A
i

n
2
r
2
sen
_
1
n
n

i=1
A
i
_
= n
_
1
2
r
2
sen
_
2
n
__
.
El lado derecho representa el area de un n agono regular, esto completa la
prueba.
2.4.2 Sea f (x) =
_
x
2
1
_1
2
, x > 1; probar que f
(n)
(x) > 0 para n impar y
f
(n)
(x) < 0 para n par.
Soluci on. Podemos suponer que se puede expresar f
(k+1)
(x) en terminos de
f
(k)
(x) . Pero una mirada a las primeras derivadas hace que este plan paresca
imposible:
f

(x) =
x
(x
2
1)
1/2
, f

(x) =
1
(x
2
1)
3/2
,
f

(x) =
3x
(x
2
1)
5/2
, f
(iv)
(x) =
12x
2
+1
(x
2
1)
7/2
,
f
(v)
(x) =
60x
3
+31x
(x
2
1)
9/2
, f
(vi)
(x) =
522x
4
+266x
2
+31
(x
2
1)
11/2
.
Considerese inmediatamente la siguiente reformulacion: si f (x) =
_
x
2
1
_
1/2
,
x > 1, entonces
84CAP

ITULO2. DOS IMPORTANTES PRINCIPIOS: EL DE INDUCCCI

ONYEL DEL PALOMAR.


f
n
(x) =
g
n
(x)
(x
2
1)
(2n1)/2
,
donde g
n
(x) es un polinomio de grado n 2, adem as
g
n
(x) es
_

_
una funci on impar cuyos coecientes todos
son no negativos si n es impar,
una funci on par cuyos coecientes todos
son no positivos si n es par.
Esta proposici on puede ser establecida por induccion (omitimos los confusos
detalles), y esto implica el resultado original.
2.4.3 Suonga que F
i
denota el i-esimo termino en la suseci on de bonacci.
Probar que F
2
n+1
+F
2
n
= F
2n+1
.
Soluci on. La formula vale para n = 1, tambien suponiendo que la formula vale
para el entero k > 1, entonces
F
2
k+2
+F
2
k+1
= (F
k+1
+F
k
)
2
+F
2
k+1
= F
2
k+1
+ 2F
k+1
F
k
+F
2
k
+F
2
k+1
= (F
2
k+1
+F
2
k
) + (2F
k+1
F
k
+F
2
k+1
)
= F
2k+1
+ (2F
k+1
F
k
+F
2
k+1
),
El ultimo paso uso la hip otesis inductiva.
Habremos terminado si podemos mostrar que 2F
k+1
F
k
+ F
2
k+1
= F
2k+2
, podi-
endo entonces continuar los argumentos previos, F
2k+1
+ (2F
k+1
F
k
+ F
2
k+1
) =
F
2k+1
+ F
2k+2
= F
2k+3
, y esto completa el paso inductivo. Por lo tanto, que-
da por probar que 2F
k+1
F
k
+ F
2
k+1
= F
2k+2
. Procederemos por induccion. La
formula es cierta si n = 1, suponiendo que es cierta para n = k, entonces
tenemos:
2F
k+2
F
k+1
+F
2
k+2
= 2(F
k+1
+F
k
)F
k+1
+F
2
k+2
= 2F
2
k+1
+ 2F
k+1
F
k
+F
2
k+2
= (2F
k+1
F
k
+F
k+1
) + (F
2
k+1
+F
2
k+2
)
= F
2k+2
+ (F
2
k+1
+F
2
k+2
).
Pero ahora estamos de vuelta con el problema anterior : Como hacer F
2
k+2
+
F
2
k+1
= F
2k+3
?. Si es as, entonces F
2k+2
+ (F
2
k+1
+ F
2
k+2
) = F
2k+2
+ F
2k+3
=
2.4. INDUCCI

ON Y GENERALIZACI

ON 85
F
2k+4
y la induccion es completa. De esta manera, los problemas estan interrela-
cionados: la verdad de lo primero depende nalmente de la verdad de lo segundo,
y a la inversa, la verdad de lo segundo depende nalmente de la verdad de lo
primero.
Podemos resolver la dicultad con una prueba de las dos en la siguiente forma.
Considere las dos proposiciones
P(n) : F
2
n+1
+F
2
n
= F
2n+1
,
Q(n) : 2F
n+1
F
n
+F
2
n+1
= F
2n+2
.
P(1) y Q(1) son cada una verdadera, los argumentos previos muestran que P(k)
y Q(k) implican P(k + 1), ahora P(k + 1) y Q(k) implica Q(k + 1). De ello se
sigue que P(k) y Q(k) implican P(k +1) y Q(k +1), la prueba se completa as.
2.4.4 Sea f(x) = a
1
senx + a
2
sen2x + .... + a
n
sennx, donde a
1
, ....a
n
son
n umeros reales y donde n es un entero positivo. Suponga que [f(x)[ < [senx[
para todo x real, probar que [a
1
+ 2a
2
+.... +na
n
[ 1.
Soluci on. Supongase que intentamos usar induccion sobre el n umero de termi-
nos en f(x). Cuando n = 1, f(x) = a
1
senx y de la hip otesis [f(x)[ [senx[, se
sigue que [a
1
[ =

a
1
sen
_

2
_

f
_

2
_

sen
_

2
_

= 1
Supongase que el resultado se cumple para k, y considere la funci on
f(x) = a
1
senx +a
2
sen2x +.... +a
k
senkx +a
k+1
sen(k + 1)x,
Para alguna elecci on de n umeros reales a
1
, a
2
, ..., a
k+1
, sup ongase que [f(x)[
[senx[ para todo x real. Como sen(k +1)x = senkxcosx+senxcoskx, podemos
escribir
f(x) = (a
1
+a
k+1
coskx)senx +a
2
sen2x +....
+a
k1
sen(k 1)x + (a
k
+a
k+1
cosx)senkx.
Tenemos ahora reescrito f(x) como una suma de k termionos, m as o menos
del mismo tipo con lo cual ya podemos aplicar la hip otesis de induccion. La
dicultad es que los coecientes de los terminos seno en esta expresi on son no
constantes; la razon es que algunos de ellos contiene funciones de x. Esto sugiere
considerar el siguiente problema m as general.
Sean a
1
(x), .....a
n
(x) funciones diferenciables de x, y sea f(x) = a
1
(x)senx +
+a
2
(x)sen2x+.... +a
n
(x)sennx. Suponga que [f(x)[ [senx[ para todo x real,
probar que [a
1
(0) + 2a
2
(0) +..... +na
n
(0)[ 1.
86CAP

ITULO2. DOS IMPORTANTES PRINCIPIOS: EL DE INDUCCCI

ONYEL DEL PALOMAR.


Si podemos probar esta proposici on, tendremos resuelto tambien el problema
original, porque, tomando a
i
(x) a
i
, a
i
una constante, i = 1, 2, ....., n, para
todo x, cubrimos el problema original.
Nuevamente procederemos por induccion. Estamos suponiendo [a
1
(x)senx[
[senx[. Mientras x se aproxima a 0, senx ,= 0, para tales x [a
1
(x)[ 1. Ahora
a
1
(x) es continua en x = 0, entonces [a
1
(0)[ 1. Esto implica que el resultado
se cumple para el caso n = 1.
Ahora su ongase que el resultado se cumple para n = k, y considere la funci on
f(x) = a
1
(x)senx +a
2
(x)sen2x +.... +a
k+1
(x)sen(k + 1)x,
donde [f(x)[ < [senx[ con las a
i
(x) diferenciables. Como antes, esta puede ser
reescrita en la forma equivalente
f(x) = [a
1
(x) a
k+1
(x)coskx] senx +a
2
(x)sen2x +.....
+a
k1
(x)(k 1)x + [a
k
(x) +a
k+1
(x)cosx] senkx
Podemos ahora aplicar la hip otesis inductiva, y cocluir que
[[a
1
(0) +a
k+1
(0)] + 2a
2
(0) +..... + (k 1)a
k1
(0) +k [a
k
(0) +a
k+1
(0)][ 1
pero esto es lo mismo que
[a
1
(0) + 2a
2
(0) +..... +ka
k
(0) + (k + 1)a
k+1
(0)[ 1
que es la forma deseada (una prueba no inductiva es dada en 6.3.2)
Problemas
2.4.5 Suponga que S denota un retculo cuadrado nn, n 3. Mostrar que es
posible dibujar un camino poligonal consistente de 2n 2 segmentos los cuales
pasan a traves de todos los n puntos reticulares de S.
2.4.6 Sea f
0
(x) =
1
(1 x)
, y dena f
n+1
(x) = xf

n
(x). Probar que f
n+1
(x) >
0 para 0 < x < 1.
2.5. RECURRENCIA 87
2.5. Recurrencia
En la segunda soluci on de 1.1.1, suponga que A
n
denota el n umero de subcon-
juntos de un conjunto con n elementos. Hemos mostrado que A
n+1
= 2A
n
,
A
0
= 1. Este es un ejemplo de relaci on recurrente. Aunque no tengamos una
f ormula explcita para A
n
(como el metodo de inducci on requiere), la relaci on de
recurrencia dene una espiral o algoritmo el cual nos muestra como calcular
A
n+1
. En esta secci on veremos acerca de problemas que pueden ser reducidos a
problemas equivalentes con par ametros peque nos, la idea es aplicar la reducci on
con argumentos recursivos hasta que los par ametros alcanzan valores para los
cuales el problema puede ser resuelto.
2.5.1 (Problema de la Torre de Hanoi). Su ongase n anillos, con diferentes
di ametros (en su circunferencia exterior), son pasados sobre una estaca verti-
cal, el m as grande en el fondo, en forma de piramide (gura 2.3). Otras dos
estacas verticales son colocadas sucientemente lejos. Deseamos transferir to-
dos los anillos, uno en una vez, a la segunda estaca para formar una piramide
identica. Durante las transferencias, no estamos permitidos a colocar un anillo
sobre otro menor (esto hace necesario usar el tercer anillo). Cual es el menor
n umero de movimientos necesarios para completar la transferencia?
Soluci on. Suponga que M
n
denota el mnimo n umero de movimientos para
una pila de n anillos. Claramente M
1
= 1, ahora suponga n > 1. En orden
hay que conseguir pasar el anillo m as grande al fondo de la segunda estaca,
para ello hay que mover los restantes n 1 anillos a la tercera estaca. Esto
nos toma un mnimo de M
n1
movimientos (seg un nuestra notacion elegida).
Un movimiento es necesario para transferir el mayor anillo a la segunda estaca,
nalmente M
n1
movimientos son necesarios para transferir los restantes n 1
anillos a la segunda estaca. De esta manera
Figura 2.3.
88CAP

ITULO2. DOS IMPORTANTES PRINCIPIOS: EL DE INDUCCCI

ONYEL DEL PALOMAR.


M
n
= 2M
n1
+ 1 M
1
= 1.
Un proceso de induccion facil, basado en esta recurrencia, muestra que M
n
=
2
n
1,
(M
n+1
= 2M
n
+ 1 = 2 [2
n
1] + 1 = 2
n+1
1).
Suponga que a
1
, a
2
, ....., a
n
es una permutaci on de 1, 2, ...., n. Podemos inter-
pretar esta permutaci on geometrica en la siguiente forma. Tome un tablero de
ajedrez nn, ahora para cada i, coloque una torre en la i-esima columna (par-
tiendo de la izquierda) y la a
i
-esima la (partiendo de abajo). Por ejemplo, la
permutaci on 3, 2, 5, 4, 1 esta representada en la gura 2.4. En esta manera ve-
mos que una permutaci on de 1, 2, ...., n corresponde a una coleccion de n torres
inofensivas en un tablero de ajedrez n n. Esta correspondencia permite a
uno pensar las permutaciones geometricas y utilizar el lenguaje e imagenes de
torres inofensivas en un tablero de ajedrez.
Figura 2.4.
2.5.2 Suponga que Q
n
denota el n umero de maneras de colocar n torres in-
ofensivas en un tablero de n n tal que que el arreglo es simetrico respecto a
la diagonal que va de la esquina inferior izquierda a la esquina supeior derecha.
Mostrar que
Q
n
= Q
n1
+ (n 1)Q
n2
.
Soluci on. Una torre en la primera columna puede o no ocupar el cuadro en
la esquina inferior izquierda del tablero. Si ello ocurre, hay Q
n1
maneras de
2.5. RECURRENCIA 89
colocar las restantes n 1 torres. Si ello no ocurre, una torre puede ocupar
cualquiera de los n1 cuadros en la primera columna. Una vez que es colocada,
se determina de manera unica la colocacion de una torre puesta simetricamente
(simetrica con respecto a la diagonal dada) en la primera la. Las restantes n2
torres pueden ser colocadas en Q
n2
maneras. Poniendo esas ideas juntas da el
resultado.
2.5.3 Una moneda es lanzada n veces. Cu al es la probabilidad de que dos caras
aparezcan una tras otra en alg un momento en la secuencia de tiros?
Soluci on. Suponga que P
n
denota la probabilidad de que no aparezcan dos
caras consecutivas en n lanzamientos. Claramente P
1
= 1, P
2
=
3
4
. Si n > 2,
all hay dos casos.
Si el primer lanzamiento es cruz, entonces dos caras consecutivas no apareceran
en los restantes n 1 lanzamientos con probabilidad P
n1
(de acuerdo a nues-
tra notacion). Si el primer lanzamiento es cara, el segundo lanzamiento debe
ser cruz para evitar dos caras consecutivas, entonces dos caras consecutivas no
apareceran en los restantes n 2 con probabilidad P
n2
. As de esa manera,
P
n
=
1
2
P
n1
+
1
4
P
n2
, n > 2.
Esta recurrencia puede ser transformada a una forma m as familiar multiplicando
cada lado por 2
n
:
2
n
P
n
= 2
n1
P
n1
+ 2
n2
P
n2
y poniendo S
n
= 2
n
P
n
para cada n:
S
n
= S
n1
+S
n2
.
Esta es la recurrencia para la susecion de Fibonacci (observe que S
n
= F
n+2
).
As pues, la probabilidad que buscamos es Q
n
= 1 P
n
= 1 F
n+2
/2
n
.
Problemas
2.5.4 Suponga que P
n
denota el n umero de regiones formadas cuando n lneas
son dibujadas en el plano Euclidiano de tal manera que no hay tres que son
concurrentes ni dos que son paralelas. Mostrar que P
n+1
= P
n
+ (n + 1).
90CAP

ITULO2. DOS IMPORTANTES PRINCIPIOS: EL DE INDUCCCI

ONYEL DEL PALOMAR.


2.5.5 .
Suponga que E
n
denota el determinante de una matriz n n teniendo 1s
abajo de la diagonal principal (que va de la izquierda arriba a la derecha abajo)
y 1s en y sobre la diagonal principal. Mostrar que E
1
= 1 y que E
2
= 2E
n1
para n > 1.
Suponga que D
n
denota el determinante de otra matriz n n cuyo (i, j)-esimo
elemento (el elemento en la i-esima la y la j-esima columna) es el valor abso-
luto de la diferencia de i y j. Mostrar que D
n
= (1)
n1
(n 1)2
n2
.
Suponga que F
n
denota el determinante de otra matriz nn con a en la diagonal
principal, b en la superdiagonal (la diagonal inmediata abajo de la diagonal
principal teniendo n1 elementos). Mostrar que F
n
= aF
n1
bcF
n2
, n > 2.
Que pasa cuando a = b = 1 y c = 1?
Eval ue el determinante A
n
n n, cuyo (i, j)-esimo elemento es a
|ij|
hallando
una relaci on recursiva entre A
n
y A
n1
.
2.5.6 .
Suponga que a
1
, a
2
, ...., a
n
son n umeros reales positivos y A
n
= (a
1
+....+a
n
)/n.
Mostrar que A
n
A
(n1)/n
n1
a
1
n
n
con la igualdad solo si A
n1
= a
n
.(Sugerencia:
aplique la desigualdad de 2.1.5)
La desigualdad entre la media geometrica y la media aritmetica. Usando la parte
(a), mostrar que
a
1
+...... +a
n
n
(a
1
....a
n
)
1/n
con la igualdad solo si a
1
= a
2
= .....a
n
.
2.5.7 Dos jugadores de ping-pong, A y B, acuerdan varios juegos. Los ju-
gadores son igualmente capaces, su ongase, que quien quiera que juegos. Los ju-
gadores son igualmente capaces, su ongase, que quien quiera puede ser el jugador
A en un juego, o el jugador B en otro). Supongase que A sirve primero en el
primer juego, pero despues el perdedor sirve primero. Suponga que P
n
denota la
probabilidad de que A gane el n-esimo juego. Mostrar que
P
n+1
= P
n
(1 P) + (1 P
n
)

P.
2.5. RECURRENCIA 91
2.5.8 Un estudiante juega a lanzar una moneda equitativa y alcanza un punto
para cada cara que aparece y dos puntos para cada cruz. Probar que la proba-
bilidad de que el estudiante alcance exactamente n puntos en alg un momento
en una suseci on de n lanzamientos es
1
3
_
2 +
_

1
2
_
n

. (Sugerencia: suponga que


P
n
denota la probabilidad de alcanzar exactamente n puntos el alg un momento.
Exprese P
n
en terminos de P
n1
y P
n2
. Use esta relaci on recurrente para dar
una prueba inductiva).
2.5.9 (Problema de Josephus). Arregle los n umeros 1, 2, ..., n consecutivamente
(en el sentido de las agujas del reloj) sobre la circunferencia de un crculo.
Ahora, remueva el n umero 2 y proceda en el sentido de las agujas del reloj
removiendo cualquier otro n umero, entre aquellos que quedan, hasta que solo un
n umero queda. (as, para n = 5, los n umeros son removidos en el orden 2, 4, 1, 5
quedando solo el 3). Suponga que f(n) denota el n umero nal que queda. Mostrar
que
f(2n) = 2f(n) 1
f(2n + 1) = 2f(n) + 1
este problema continua en (3.4.5).
2.5.10 .
(a) Suponga que R
n
denota el n umero de maneras de colocar n torres inofen-
sivas en el tablero de ajedrez n n, tambien que el arreglo es simetrico
con respecto a una rotaci on (en el sentido de la agujas del reloj) de 90
0
de el tablero alrededor de el centro. Mostrar que
R
4n
= (4n 2)R
4n4
,
R
4n+1
= R
4n
,
R
4n+2
= 0 = R
4n+3
.
(b) Suponga que S
n
denota el n umero de maneras de colocar n torres inofen-
sivas en un tablero de ajedrez n n, tambien que el arreglo es simetrico
con respecto a el centro del tablero. Mostrar que
S
2n
= 2nS
2n2
S
2n+1
= S
2n
.
(c) Suponga que T
n
denota el n umero de caminos de colocar n torres inofen-
sivas en un tablero de ajedrez de nn, tambien que el arreglo es simetrico
con respecto a las dos diagonales. Mostrar que
T
n
= 2,
T
2n+1
= T
2n
,
T
2n
= 2T
2n2
+ (2n 2)T
2n4
.
92CAP

ITULO2. DOS IMPORTANTES PRINCIPIOS: EL DE INDUCCCI

ONYEL DEL PALOMAR.


2.5.11 Un (n + 2)- agono regulares es inscrito en un crculo. Suponga que T
n
denota el n umero de formas posibles para juntar sus vertices en pares tal que los
segmentos resultantes no se intersectan uno con otro. Si establecemos T
0
= 1,
mostrar que
T
n
= T
0
T
n1
+T
1
T
n2
+T
2
T
n3
+...... +T
n1
T
0
(para una continuaci on de este problema, ver 5.4.10)
2.5.12 Sean a
1
, a
2
, ....., a
n
una permutaci on de el conjunto S
n
= 1, 2, ...., n.
Un elemento i en S
n
es llamado un punto jo de esta permutaci on si a
i
= i.
a. Un desarreglo de S
n
es una permutaci on de S
n
sin puntos jos. Sea g
n
el
n umero de desarreglos de S
n
. Mostar que
g
1
= 0, g
2
= 1,
g
n
= (n 1)(g
n1
+g
n2
), Para n > 2.
(Sugerencia: Un desarreglo, o intercambia el primer elemento con otro o
no lo es)
b. Sea f
n
el n umero de permutaciones de S
n
con exactamente un punto jo.
Mostar que [f
n
g
n
[ = 1.
2.5.13 Su ongase que n hombres entregan sus sombreros al llegar a un ban-
quete, como los dejan, los sombreros les son de vueltos tomandolos al azar.
Cu al es la probabilidad de que un hombre no consiga en esta devoluci on su pro-
pio sombrero? (Sugerencia: Suponga que P
n
denota esta probabilidad. Entonces
P
n
= g
n
/n!, donde g
n
es como en 2.5.13. Sea C
n
= P
n
P
n1
. Use la relaci on de
recurrencia hallando en 2.5.13 (a) para mostrar que C
2
= 1/2, C
n
= C
n1
/n.
Use esto para mostrar que P
n
= 1/2! 1/3! +.... + (1)
n
/n!. Entonces para n
grande P
n
1/e)
2.5.14 .
(a) Sea I
n
=
_
/2
0
sen
n
xdx. Hallar una relaci on de recurrencia para I
n
.
(b) Mostrar que
I
2n
=
1 3 5 ..... (2n 1)
2 4 6 ....... 2n

2
.
(c) Mostrar que
I
2n+1
=
2 4 6 ..... (2n 2)
1 3 5 ....... (2n 1)
.
2.6. PRINCIPIO DEL PALOMAR 93
Ejemplos adicionales
1.1.1(Soluci on 2), 4.3.9, 5.3.5, 5.3.14, 5.3.15, 5.4.8, 5.4.9, 5.4.24, 5.4.25, 5.4.26.
Estrechamente relacionadas la induccion y la recursi on son argumentos basados
sobre argumentos repetidos. Ejemplos de que eso ha signicado aqu son 4.4.4,
4.4.17, la prueba de el teorema del valor medio en 6.1, 6.1.5, 6.1.6, 6.3.6, 6.8.10,
y la heurstica para el signicado geometrico de una desigualdad arimetica dada
en la seccion 7.2
2.6. Principio del Palomar
Cuando una coleccion sucientemente grande de objetos dividida en un n umero
sucientemente peque no de clases, una de las clases contendr a un ciertamente
mnimo n umero de objetos. Este hecho se preciso m as en la siguiente proposici on
evidente:
Principio del palomar. Si kn + 1 objetos (k 1) son distribuidos entre n cajas
una de las cajas va a contener al menos k + 1 objetos
Este principio, incluso cuando k = 1, es una muy poderosa herramienta para
probar teoremas de existencia.
2.6.1 Dado un conjunto de n+1 enteros positivos, ninguno de los cuales excede
a 2n, mostrar que al menos un miembro de el conjunto debe dividir a otro
miembro de el conjunto.
Soluci on. Este es el mismo que el de 2.2.7, donde estaba planteado por induc-
cion sobre n. Sin embargo, el problema es realmente un problema de existencia
para un n dado, y puede ser llevado muy atinadamente hacia el principio del
palomar, como veremos en seguida.
Suponga que los n umeros elegidos se denota por x
1
, x
2
, ....., x
n+1
, ahora para
cada i escribimos x
i
= 2
ni
y
i
, donde n
i
es entero no negativo y y
i
es impar. Sea
T = yi : 1, 2, ....., n + 1. Entonces T es una coleccion de n+1 enteros impares,
cada uno de ellos menor que 2n. De aqu que all hay solo n n umeros impares
menores que 2n, el principio del palomar implica que dos n uemros en T son
iguales, digamos y
i
= y
j
, i < j. Entonces
x
i
= 2
ni
y
i
y x
j
= 2
nj
y
i
.
Si n
i
n
j
, entonces x
i
divide a x
j
; si n
i
> n
j
, entonces x
j
divide a x
i
. Esto
94CAP

ITULO2. DOS IMPORTANTES PRINCIPIOS: EL DE INDUCCCI

ONYEL DEL PALOMAR.


completa la prueba.
2.6.2 Considere cinco puntos cualesquiera P
1
, P
2
, P
3,
P
4
, P
5
en el interior de
un cuadrado S de lado con longitud 1. Denote por d
ij
la distancia entre los
puntos P
i
y P
j
. Probar que al menos una de las distancias d
ij
es menor que

2/2.
Figura 2.5.
Soluci on. Divida S en cuatro cuadrados congruentes como se muestra en la
gura 2.5. Por el principio de palomar, dos puntos pertenecen a uno de esos
cuadrados (un punto en la frontera de dos cuadrados peque nos puede ser con-
siderado de ambos cuadrados). La distancia entre esos puntos es menor que

2/2.
Figura 2.6.
2.6.3 Sup ongase que cada cuadrado de un tablero de ajedrez 4 7, como se
muestra en la gura, es coloreado en blanco o negro. Probar que en una tal
coloraci on, el tablero debe contener un rect angulo (formado por las lneas hor-
izontales y verticales de el tablero), tal como el trazado en la gura 2.6, cuyos
cuadrados de las distintas esquinas son todos del mismo color.
Soluci on. Un tal rectangulo existe incluso en un tablero 3 7. El color de
las conguraciones de cada columna debe de ser de uno de los tipos mostrados
2.6. PRINCIPIO DEL PALOMAR 95
Figura 2.7.
en la gura 2.7. Suponga que una de las columnas es del tipo 1. Habremos
terminado si una de las 6 restantes es del tipo1, 2, 3 o 4. Suponga pues que
cada una de las seis columnas restantes es del tipo 5, 6, 7 u 8. Entonces por el
principio del palomar , dos de esas seis columnas deben ser del mismo tipo y
habremos terminado.
El mismo razonamiento se hace si una de las columnas es del tipo 8.
Ahora suponga que ninguna de las columnas es del tipo 1 o del tipo 8. Entonces
tenemos siete columnas pero solo seis tipos. Por el principio del palomar, dos
columnas deben tener el mismo tipo y la prueba es terminada.
2.6.4 Probar que existen tres enteros a, b, c no todos cero y cada uno con valor
absoluto menor que un millon, tales que

a +b

2 +c

< 10
11
.
Soluci on. Sea S el conjunto de 10
18
n umeros reales r + s

2 + t

3 con r, s, t
cada uno en
_
0, 1, 2, ...., 10
6
1
_
, ahora sea d = (1 +

2 +

3)10
6
. Entonces
cada x en S esta en el intervalo 0 x < d. Dividiendo este intervalo en 10
18
1
subintervalos iguales, cada uno de longitud e = d/(10
18
1). Por el principio del
palomar, dos de los 10
18
n umeros de S deben estar en el mismo subintervalo. Su
diferencia, a +b

2 +c

3, da los deseados a, b, c. ya que e < 10


7
/10
18
= 10
11
.
2.6.5 Dado un conjunto de diez n umeros naturales entre 1 y 99 inclusive (no-
taci on decimal), probar que all existen dos subconjuntos no vacios y disjuntos,
del conjunto con sumas iguales de sus elementos.
Soluci on. Con la elecci on del conjunto de diez n umeros, podemos formar 2
10

96CAP

ITULO2. DOS IMPORTANTES PRINCIPIOS: EL DE INDUCCCI

ONYEL DEL PALOMAR.


1 = 1023 (diferentes) subconjuntos no vacios. Cada uno de esos subcojuntos
tiene una suma menor que 1000, ya que incluso 90 + 91 + . . . + 99 < 1000.
Por tanto por el principio del palomar, dos subconjuntos A y B deben tener
la misma suma. As por extraccion de los elementos que pertenescan a ambos
conjuntos obtenemos dos conjuntos disjuntos x = AA B y y = B A B,
con la misma suma. (Ni x ni y son vacios, porque esto signicara que: A B
o B A, lo cual es imposible, ya que la suma de sus elementos da el mismo
n umero.)
Problemas
2.6.6 Sea A conjunto de 20 enteros distintos elegidos de la progresi on ar-
itmetica 1, 4, 7, . . . , 100. Probar que existen dos enteros distintos en A cuya suma
es 104.
2.6.7 .
a. Sea S una regi on cuadrada (en el plano) de lado con longitud 2 pulgadas.
Mostrar que entre cualesquiera nueve puntos en S, hay tres los cuales son los
vertices de un tri angulo de area 1/2 de una pulgada cuadrada.
b. Diecinueve dardos son lanzados hacia un blanco el cual tiene la forma de un
hex agono regular con lado de longitud un pie. Mostrar que dos dardos est an a
una distancia de a lo m as

3/3 pies uno de otro.


2.6.8 Mostrar que si hay n personas en una esta, entonces dos de ellas cono-
cen el mismo n umero de personas (entre aquellas presentes).
2.6.9 Quince sillas son equitativamente colocadas alrededor de una mesa cir-
cular en la cual estan asignadas chas para quince invitados. Los invitados igno-
ran la informaci on de estas chas hasta despues que se han sentado, y resulta
que ninguno est a sentado frente a su propia cha. Probar que la mesa puede
ser rotada de manera que al menos dos de los invitados esten simultaneamente
sentados correctamente.
2.6.10 Sea X un n umero real. Probar que entre los n umeros
x, 2x, . . . , (n 1) x
hay uno que diere de un entero por a lo mas 1/n.
2.6. PRINCIPIO DEL PALOMAR 97
2.6.11 .
(a.) Probar que en un grupo de seis personas hay tres que, o bien son mutua-
mente conocidas o bien son mutuamente desconocidas. (Sugerencia: Represente
las personas con los vertices de un hex agono regular. Una dos vertices con un
segmento de lnea rojo si el par representado por esos vertices es de conocidos
de lo contrario una esos vertices con un segmento de lnea azul. Considere uno
de los vertices, llamemosle A. Al menos tres segmentos de lnea parten de A
teniendo el mismo color. Aqui hay dos casos a considerar.)
(b.) Diecisiete personas mantienen correspondencia entre si (cada uno con todos
los dem as) por correo. En sus cartas solo tres temas son discutidos. Cada par
de correspondencias trata solo uno de los temas. Probar que hay al menos tres
personas que se escriben unos a otros sobre el mismo tema.
2.6.12 Probar que no existe un conjunto de siete enteros positivos, no mayores
de 24, en el cual sus subconjuntos tienen todos sumas diferentes.
Ejemplos adicionales
1.10.1, 3.2.1, 3.2.5, 3.2.19, 3.2.20, 3.3.24, 4.4.10.
98CAP

ITULO2. DOS IMPORTANTES PRINCIPIOS: EL DE INDUCCCI

ONYEL DEL PALOMAR.


Captulo 3
Aritmetica
En este captulo consideraremos metodos que son importantes en la soluci on de
problemas de Aritmetica. Muchas de las tecnicas estan basadas en las demostra-
ciones de los teoremas fundamentales de la Aritmetica, como el hecho de que
cada n umero entero se puede escribir como un producto de n umeros primos. La
estructura te orica necesaria para la prueba de este y otros teoremas requiere
de una discusion de la nocion de divisibilidad. As que , veremos en la primera
seccion de este captulo algunos resultados y ejemplos acerca de temas como
m aximo com un divisor y mnimo com un m ultiplo. Para lo cual es importante
entender el algoritmo de la divisi on y el algoritmo de Euclides.
En la segunda seccion introduciremos la tecnica de la aritmetica modular, y
veremos la eciencia de metodos para resolver muchos pro- blemas concernientes
a la relacion entre enteros. En las ultimas dos secciones haremos enfasis acerca
de la importancia de la notacion para resolver problemas, y consideraremos
teoremas relacionados con la repre- sentaci on de n umeros as como de la notacion
posicional para enteros.
3.1. Maximo Com un Divisor
Dados dos enteros a y b, decimos que a divide a b, y lo escribimos a [ b, si existe
un entero q tal que b = aq. De esta denicion basica se desprende el siguiente
resultado: Si n divide a dos de los terminos de la expresi on a = b +c, entonces n
divide a los tres terminos.(Nota: En este captulo, a menos que se indique otra
cosa, las variables son variables enteras.)
99
100 CAP

ITULO 3. ARITM

ETICA
Si a
1
, a
2
, ..., a
n
son enteros dados, denotamos el m aximo com un divisorde ellos
por MCD(a
1
, a
2
, ..., a
n
) y su mnimo com un m ultiplo por mcm(a
1
, a
2
, ..., a
n
).
3.1.1 Encontrar todas las funciones f que satisfacen las tres condiciones
(i) f(x, x) = x
(ii) f(x, y) = f(y, x)
(iii) f(x, y) = f(x, x +y).
suponiendo que las variables y los valores de la funci on son enteros positivos.
Soluci on. Un mirada a casos especiales permite sospechar que f(x, y) = mcd(x, y).
Probaremos esto usando induccion sobre la suma x +y.
El valor m as peque no para x +y es 2, y esto ocurre cuando x = y = 1. Por (i),
f(1, 1) = 1, y tambien mcd(1, 1) = 1, luego la suposici on es verdadera para este
caso.
Supongase que x e y son enteros positivos tales que x+y = k > 2, y supongamos
que la conjetura se cumple para todas las sumas menores. Por (i) y (ii), sin
perdida de generalidad se puede suponer que x < y. Por (iii) f(x, y) = f(x, x +
(yx)) = f(x, yx). Pero por la hip otesis de induccion, f(x, yx) = mcd(x, y
x). La prueba se completa si logramos probar que mcd(x, y x) = mcd(x, y).
Si c [ x y c [ y, entonces c [ y x. Se sigue que mcd(x, y) mcd(x, y x).
Similarmente, si c [ x y c [ y x, entonces c [ x y c [ y, luego mcd(x, y
x) mcd(x, y). Tomando estas desigualdades, encontramos que mcd(x, y) =
mcd(x, y x).
El siguiente resultado es de relevante importancia en teora de n umeros.
Algoritmo de la Divisi on. Si a y b son enteros arbitrarios, b > 0, existen q
y r tales que
a = bq +r, 0 r < b.
Mediante el uso repetido del algoritmo de la divisi on podemos calcular el m aximo
com un divisor de dos enteros. Veamos esto, sup ongase que son enteros positivos,
con b
1
< b
2
. Por el algoritmo de la divisi on existen enteros q y b
3
tales que
b
1
= qb
2
+b
3
, 0 b
3
< b
2
.
Es facil checar, usando esta ecuaci on, que mcd(b
1
, b
2
) = mcd(b
2
, b
3
).
3.1. M

AXIMO COM

UN DIVISOR 101
Si b
3
= 0, entonces mcd(b
1
, b
2
) = b
2
. Si b
3
> 0, repetimos el procedimiento,
usando b
2
y b
3
en vez de b
1
y b
2
, esto produce un entero b
4
tal que mcd(b
2
, b
3
) =
mcd(b
3
, b
4
), b
3
> b
4
0.
Continuando de esta manera, generamos una sucesion decreciente de n umeros
enteros no negativos
b
1
> b
2
> b
3
>
tales que mcd(b
1
, b
2
) = mcd(b
2
, b
3
) = mcd(b
3
, b
4
) = ... = mcd(b
i
, b
i+1
), i =
1, 2, 3, ....
Como tal sucesion no puede decrecer innitamente, existe un primer n tal que
b
n+1
= 0. En este punto mcd(b
1
, b
2
) = mcd(b
n
, b
n+1
) = b
n
.
Este procedimiento para encontrar mcd(b
1
, b
2
) recibe el nombre de Algoritmo
de Euclides.
En adelante daremos unos ejemplos de este algoritmo, y haremos una prueba
del mayor resultado de esta seccion.
3.1.2 Dados enteros positivos a y b, existen enteros s y t tales que
sa +tb = mcd(a, b).
Soluci on. Probaremos este resultado haciendo induccion sobre el n umero de
pasos requeridos por el algoritmo de Euclidespara encontrar el m aximo com un
divisor de a y b. (Otra prueba esta dada en 3.1.9.)
Supongase que a > b. si s olo requerimos un paso, existe un entero q tal que
a = bq, y en este caso mcd(a, b) = b. Tambien, en este caso, mcd(a, b) = b =
a + (1 q)b, luego s = 1 y t = 1 q.
Asumamos ahora que el resultado ha sido probado para todos los pares de
enteros positivos que necesitan menos de k pasos, y asumamos que a y b son
enteros que requieren k pasos, k > 1. Por el algoritmo de la divisi on, existen
enteros q y r tales que
a = bq +r, 0 r < b.
El m aximo com un divisor de b y r puede ser encontrado por el algoritmo de
Euclides en k 1 pasos, luego por la hip otesis de induccion, existen enteros c y
d tales que
cb +dr = mcd(b, r)
102 CAP

ITULO 3. ARITM

ETICA
De estas dos ecuaciones, se sigue que
mcd(a, b) = mcd(b, r)
= cb +dr
= cb +d(a bq)
= da + (c dq)b
y la prueba se completa tomando s = d y t = c dq.
Los pasos de la prueba pueden ser claricados con un ejemplo
3.1.3 Encontrar los enteros x e y tales que
754x + 221y = mcd(754, 221).
Soluci on. Apliquemos primero el algoritmo de Euclides para encontrar el m axi-
mo com un divisor entre 754 y 221. Tenemos que
754 = 3 221 + 91
221 = 2 91 + 39
91 = 2 39 + 13
39 = 3 13.
Esto prueba que mcd(754, 221) = 13.
Para encontrar los enteros x e y, procederemos trabajando hacia atr as a traves
de los pasos del algoritmo de Euclides (esto es en escencia el paso inductivo de
la prueba anterior):
13 = 91 2 39
= 91 2(221 2 91)
= 5 91 2 221
= 5(754 3 221) 2 221
= 5 754 17 221
Luego la soluci on es x = 5 e y = 17.
El siguiente resultado es presentado usualmente.
3.1.4 La ecuaci on ax+by = c, a, b, c enteros, tiene una soluci on en los enteros
x e y si y s olo si mcd(ab) divide a c. M as aun, si (x
0
, y
0
) son una soluci on
entera, entonces para cada entero k, los valores
3.1. M

AXIMO COM

UN DIVISOR 103
x

= x
0
+ bk/d
d = mcd(a, b)
y

= y
0
ak/d
Soluci on. Para la primera parte, es claro que mcd(a, b) debe dividir a c, ya que
mcd(a, b) divide a ax+by. De aqu, mcd(a, b) [ c es una condici on necesaria para
la existencia de uan soluci on. Por otro lado, si c es un m ultiplo de mcd(a, b),
es decir c = mcd(a, b)q, podemos encontrar una soluci on entera de la siguiente
manera. Sabemos que existe enteros s y t tales que sa +tb = mcd(a, b). Luego
tomando x = sq y y = tq. Entonces ax + by = asq + btq = q(as + bt) =
mcd(a, b)q = c.
Figura 3.1.
Un c alculo directo prueba que (x

, y

), como los dados, son una soluci on, proban-


do que (x
0
, y
0
) es una soluci on. La prueba de que todas las soluciones enteras
tienen la forma descrita la haremos con un argumento geometrico como sigue
(Figura 3.1.).
Note que el problema de resolver ax +by = c en los enteros x e y es equivalente
al problema de encontrar los puntos lattice que pertenecen a la linea recta
ax+by = c. Supongase que (x
0
, y
0
) es un punto lattice sobre la recta ax+by = c;
esto es, sup ongase que
ax
0
+by
0
= c.
El resultado es facil de probar si b = 0, sup ongase que b ,= 0. Si (x

, y

) es
cualquier punto lattice en el plano, entonces (x

, y

) sera un punto sobre la recta


ax +by = c si y s olo si
y

y
0
x

x
0
=
a
b
=
a
d
b
d
, donde d = mcd(a, b).
Como a/d y b/d son primos relativos, esta ecuaci on es cierta si y s olo si existe
104 CAP

ITULO 3. ARITM

ETICA
un entero k tal que
y

y
0
= (a/d)k
x

x
0
= (b/d)k
Se sigue que toda soluci on entera de ax+by = c estan dados por las ecuaciones
y

= y
0
(a/d)k
x

= x
0
+ (b/d)k
k un entero y d = mcd(a, b).
3.1.5 Probar que la fracci on (21n + 4)/(14n + 3) es irreducible para cualquier
n umero natural n.
Soluci on. Necesitamos probar que 14n+3 y 21n+4 son primos relativos para
toda n. De la discusi on precedente deberemos ver que es posible probar que
existen enteros s y t tales que
s(21n + 4) +t(14n + 3) = 1,
o equivalentemente,
7n(3s + 2t) + (4s + 3t) = 1.
Esta ecuaci on deber a ser cierta para todo n si podemos encontrar los enteros s
y t que satisfagan
3s + 2t = 0,
4s + 3t = 1.
Directamente podemos ver que estas ecuaciones son satisfechas para s = 2 y
t = 3, y esto completa la prueba.
3.1.6 La medida de un angulo dado es 180
o
/n, donde n es un entero positivo
no divisible por 3. Probar que el angulo puede ser trisecado con herramienta
euclidiana (regla y comp as).
Soluci on. Desde luego no esperabamos que este problema pueda ser resuelto
con n umeros, sin embargo, cual es el signicado de la condici on que n no es
divisible por 3? Este es que 3 y n son primos relativos, luego existen enteros s
y t tales que
ns + 3t = 1.
Construyamos ahora el angulo 60
o
/n. Cuando multiplicamos ambos lados de
esta ultima ecuaci on por 60
o
/n, tenemos que
60
o
s + (180
o
/n)t = 60
o
/n.
3.1. M

AXIMO COM

UN DIVISOR 105
Ahora observemos que el lado izquierdo de la ecuaci on describe la construcci on
de 60
o
/n. Esto se debe a que podemos construir el angulo de 60
o
, el angulo
180
o
/n es dado, los enteros s y t pueden ser encontrados , y podemos construir
60
o
s + (180
o
/n)t.
Problemas
3.1.7 Si mcd(a, b) = 1, probar que
(a) mcd(a b, a +b) 2,
(b) mcd(a b, a +b, ab) = 1
(c) mcd(a
2
ab +b
2
, a +b) 3.
3.1.8 La suma algebraica de cualquier n umero de fracci ones irreducibles cuyos
denominadores son primos relativos, uno con cada uno de los otros, no puede
ser un entero. Esto es, si mcd(a
i
, b
i
) = 1, i = 1, 2, ..., n, y mcd(b
i
, b
j
) = 1 para
i ,= j, probar que
a
1
b
1
+
a
2
b
2
+... +
a
n
b
n
no es un entero.
3.1.9 Si S es un conjunto de enteros no vacio tal que
(i) la diferencia x y est a en S cuando x e y est an en S, y
(ii) todos los m ultiplos de x est an en S cuando x est a en S.
(a) Probar que existe un entero d en S tal que S consiste de todos los m ultiplos
de d. (Sugerencia: Considere el menor entero positivo en S).
(b) Probar que la parte (a) se aplica al conjunto ma + nb : m y n son enteros
positivos, y probar que la d resultante es mcd(a, b).
3.1.10 .
(a) Probar que cualesquiera dos n umeros sucesivos de Fibonacci F
n
, F
n+1
, n > 2,
son primos relativos.
106 CAP

ITULO 3. ARITM

ETICA
(b) Dado que T
1
= 2, y T
n+1
= T
2
n
T
n
+ 1, n > 0, probar que T
n
y T
m
son
primos relativos cuando n ,= m.
3.1.11 Para enteros positivos a
1
, ..., a
n
, probar que existen enteros k
1
, ..., k
n
tales que k
1
a
1
+... +k
n
a
n
= mcd(a
1
, ..., a
n
).
3.1.12 Probar que (a +b)/(c +d) es irreducible si ad bc = 1.
3.1.13 Probar que mcd(a
1
, ..., a
m
)mcd(b
1
, ..., b
n
) = mcd(a
1
b
1
, ..., a
m
b
n
), cuan-
do el parentesis del lado derecho incluye todos los mn productos a
i
b
j
, i =
1, ..., m, j = 1, ..., n.
3.1.14 Cuando el Sr. Smith cambia un cheque de x dolares y y centavos, recibe
y d olares y x centavos, y se encuentra que recibi o dos centavos m as del doble
que lo anotado. Por cu anto estaba escrito el cheque?.
3.1.15 Encontrar el entero positivo m as peque no a para el cual
1001x + 770y = 1000000 +a
es posible, y probar que entonces hay 100 soluciones en los enteros positivos.
3.1.16 Un hombre cuenta con un contenedor de 9 litros y uno de 16 litros.
Probar que puede conseguir un litro de agua en el contenedor de 16 litros.(Sugerencia:
Encontrar los enteros s y t tale que 9s + 16t = 1).
3.1.17 Existe mas de un entero mayor que 1 para el cual, cuando lo dividimos
por cualquier entero k tal que 2 k 11, se tienen un residuo igual a 1. Cu al
es la diferencia entre los dos enteros m as peque nos con esa propiedad?.
3.1.18 Si b es un entero mayor que 1. Probar que para cualquier entero
no negativo N, existe un unico entero no negativo n y unicos enteros a
i
, i =
0, 1, ..., n, 0 a
i
< b, tales que a
n
,= 0 y
N = a
n
b
n
+a
n1
b
n1
+... +a
2
b
2
+a
1
b +a
0
.
(El resultado es inmediato para N < b, asumamos que N b. Use inducci on).
Ejemplos adicionales
3.2.4, 3.2.21, 3.3.11, 3.3.19, 3.3.28, 4.1.9, 4.2.1, 4.2.2, 4.2.4, corolario (iii) del
teorema de Lagrangeen la seccion 4.4, 4.4.5, 4.4.6, 4.4.8.
3.2. ARITM

ETICA MODULAR 107


3.2. Aritmetica Modular
La paridad de un n umero entero es expresada o relativa al n umero 2. Especca-
mente, un n umero es par o impar de acuerdo a si el residuo que deja el n umero
al ser dividido por 2 es cero o uno respectivamente. Esta formulacion de paridad
permite generalizar de manera natural la idea como sigue.
Dado un entero n 2, dividimos el conjunto de los enteros en clases de congru-
encia de acuerdo a su residuo al ser dividido por n; es decir que dos n umeros
pertenecen a la misma clase de congruencia si tienen el mismo residuo al ser
divididos por n. Por ejemplo, para n = 4, los enteros son divididos en cuatro
conjuntos identicados con los posibles residuos 0, 1, 2, 3. Para un n umero n 2,
existen n clases de congruencia, a saber 0, 1, 2, , n 1.
Dos enteros x y y se dice son congruentes m odulo n, se escribe
x y (mod n),
si cada uno de ellos deja el mismo residuo al dividirse por n (o equivalentemente,
y en la practica m as conveniente, si x y es divisible por n).
Es facil probar que
(i) x x(mod n),
(ii) x y(mod n) implica y x(mod n), y
(iii) [x y(mod n) y y z(mod n)] implica x z(mod n).
Estas propiedades indican que la congruencia tiene las mismas caractersticas
que la igualdad, y obtenemos el concepto de congruencia como un simil de la
igualdad (en efecto podemos algunas veces leer x y(mod n) como x es igual
a y m odulo n).
3.2.1 Probar que en cualquier subconjunto de 55 n umeros tomados del conjunto
1, 2, 3, 4, ..., 100 podemos encontrar dos n umeros que dieran en 9.
Soluci on. Existen nueve clases de congruencia m odulo 9; 0, 1, 2, 3, 4, 5, 6, 7, 8.
Por el principio de las casillas (generalizado), siete n umeros de los 55 dados
pertenecen a la misma clase de congruencia (en cada clase hay seis o menos,
esto permite acomodar 54 de los 55 elementos). Sean a
1
, ..., a
7
esos n umeros, y
supongamos que estan ordenados a
1
< a
2
< a
3
< ... < a
7
. Como a
i+1
a
i
(mod 9), a
i+1
a
i
9, 18, .... Luego tenemos que a
i+1
a
i
= 9 para alg un
i. Porque si no, entonces para cada i, a
i+1
a
i
18, y esto nos lleva a que
108 CAP

ITULO 3. ARITM

ETICA
a
7
a
1
6 18 = 108. Pero esto es imposible, ya que a
7
a
1
< 100. Luego
dos de los elementos (entre a
1
, ..., a
7
) dieren en 9.
La real potencia de la congruencia es una consecuencia de la siguiente propiedad
facil de probar.
Aritmetica Modular. Si x y(mod n) y u v (mod n), entonces
x +u y +v (mod n),
x u y v (mod n).
Este resultado permite hacer aritmetica trabajando unicamente con los resid-
uos m odulo n. Por ejemplo, como
17 5 (mod 12) y 40 4 (mod 12),
sabemos que
17 + 40 5 + 4 = 9 (mod 12)
y
17 40 5 4 8 (mod 12)
Sea n un entero positivo, n > 1, y Z
n
= 1, 2, 3, ..., n 1. Observe que si x y y
son elementos de Z
n
, existen unicos elementos r, s, t en Z
n
tales que
x y r (mod n)
x +y s (mod n)
x y t (mod n)
El conjunto Z
n
con estas operaciones de suma resta y multiplicaci on es llamado
el conjunto de los enteros m odulo n. En este sistema, las operaciones son real-
izadas de la misma forma que en los enteros excepto el resultado que se reduce
(m odulo n) a un n umero equivalente en el conjunto Z
n
.
3.2.2 Sea N = 22 31 + 11 17 + 13 19. Determinar (a) la paridad de N;
(b) el dgito de las unidades de N; (c) el residuo de N al ser dividido por 7. (De
hecho, la idea de determinar estos datos se puede hacer calculando directamente
N.)
Soluci on. Para la parte (a), 22 31 es par, ya que 22 es par, 11 17 es impar,
y 1319 es impar, luego la suma es par + impar +impar, y esto es par. Notese
que este razonamiento es equivalente a computar m odulo 2:
22 31 + 11 17 + 13 19 0 1 + 1 1 + 1 1 1 + 1 0(mod 2).
Para la parte (b), necesitamos solamente estimar el dgito de las unidades: 2231
tiene como dgito de la unidades al 2, 11 17 tiene a l 7 como dgito de las
3.2. ARITM

ETICA MODULAR 109


unidades y13 19 tambien al 7. De donde el dgito de las unidades de N es
el mismo que el de 2 + 7 + 7 o sea 6. Aqu tambien, el analisis es equivalente
computando N m odulo 10 :
22 31 + 11 17 + 13 19 2 1 + 1 7 + 3 9(mod 10)
2 + 7 + 7 6(mod 10)
A partir de las partes (a) y (b) podemos asumir que el trabajo en aritmetica
modular es conable, pero aparentemente no es aplicable de inmediato para
la parte (c). El punto en este ejemplo es que la parte (c) tambien puede ser
obtenida trabajando de igual forma como una extensi on en los casos previos.
Trabajemos m odulo 7 :
22 31 + 11 17 + 13 19 1 3 + 4 3 + (1) 5(mod 7)
3 + 5 5 3(mod 7)
Luego N es mayor en 3 que un m ultiplo de 7. (Como un chequeo: N = 1116 =
159 7 + 3.)
3.2.3 Cuales son los ultimos dos dgitos de 3
1234
?
Soluci on. Trabajemos m odulo 100. Podemos hacerlo con 3
1234
. Por ejemplo,
3
2
9(mod 100), 3
4
81(mod 100), 3
8
8181 61(mod 100), 3
10
619
49(mod 100), 3
20
49 49 1(mod 100). Como 1234 = 20 61 + 14, tenemos
3
1234
= (3
20
)
61
(3)
14
3
14
3
4
3
10
81 49 69(mod 100). Los dos ultimos
son pues 69.
3.2.4 Probar que algunos m ultiplos de 21 tiene a 241 como sus tres ultimos
dgitos.
Soluci on. Debemos probar que existe un entero positivo n tal que
21n 241(mod 1000).
Como 21 y 1000 son primos relativos, existen enteros s y t tales que
21s + 1000t = 1.
Multiplicando ambos lados de esta ecuaci on por 241, y rearreglando en la forma
21(241s) 241 = 241 1000t.
En notacion de congruencias, la ultima ecuaci on dice que
21 241s 241(mod 1000).
110 CAP

ITULO 3. ARITM

ETICA
Si s es positivo, habremos terminado, tomando n = 241s. Si s es negativo,
hacemos n = 241s+1000k, donde k es un entero lo sucientemente grande para
que n sea positivo (podemos tomar k de manera apropiada, podemos tambien
asumir que n esta entre 0 y 1000). Se sigue que
21n 21(241s + 1000k) 21 241s 241(mod 1000).
3.2.5 Probar que para cualquier conjunto de n enteros, existe un subconjunto
de ellos cuya suma es divisible por n.
Soluci on. Si x
1
, x
2
, ..., x
n
denotan los enteros dados y si
y
1
= x
1
,
y
2
= x
1
+x
2
,
.
.
.
y
n
= x
1
+x
2
+... +x
n
.
Si y
i
0(mod n) para alg un i, habremos terminado, as que supongamos que
este no es el caso. Entonces tenemos n n umeros y
1
, y
2
, ..., y
n
, y n 1 clases
de congruencia m odulo n (a saber, 1, 2, ..., n 1), luego por el principio de
las casillas, dos de los y

i
s seran congruentes entre si m odulo n. Supongase
y
i
y
j
(mod n), con i < j. Entonces
x
i+1
+x
i+2
+... +x
j
y
i
y
j
0(mod n).
y la prueba esta completa.
En el ejemplo precedente, hemos usadoen hecho de que n divide a a si y s olo
si a 0(mod n). Por esta correspondencia, los problemas concernientes a di-
visibilidad pueden ser trasladados directamente en el lenguaje de aritmetica
modular.
3.2.6 Probar que si 2n + 1 y 3n + 1 son cuadrados perfectos, entonces n es
divisible por 40.
Soluci on Se requiere probar que n es divisible por 5 y por 8. Esto es equivalente
a probar que n 0(mod 5) y que n 0(mod 8).
Consideremos m odulo 5. La tabla siguiente prueba que un cuadrado es 0, 1 o
1 m odulo 5.
Luego, 2n + 1 y 3n + 1 seran 0, 1, o 1 m odulo 5. Esto ofrece nueve casos a
considerar: 2n+1 puede ser 0, 1, o 1 m odulo 5 y 3n+1 puede ser 0, 1, o 1. Sin
3.2. ARITM

ETICA MODULAR 111


embargo podemos reducir el n umero de casos a s olamente dos, como veremos.
Supongase que 2n + 1 a(mod 5) y 3n + 1 b(mod 5), a, b 1, 0, 1.
Caso 1. a ,= b. En este caso, sumamos las ultimas dos ecuaciones para obtener
2 a +b(mod 5).
Pero esta ecuaci on no puede ser resuelta para los valores dadoe de a y b, de
donde este caso no puede ocurrir.
Caso 2. a = b. En este caso, restamos la primera ecuaci on de la segunda
n 0(mod5).
En este caso n es divisible por 5. (que es parte de lo que deseamos probar).
Ahora consideremos m odulo 8. Es este caso, la tabla muestra que los cuadrados
son 0, 1, o 4 m odulo 8 : Tambien, aqui se presentan 9 casos, dependiendo de
los valores de 2n + 1 y 3n + 1 m odulo 8. Estos nueve casos pueden reducirse
ex actamente a dos como en el caso de m odulo 5, y el argumento en cada caso
es ex actamente el mismo. Concluyendo que n es divisible por 8, y la prueba se
completa.
112 CAP

ITULO 3. ARITM

ETICA
En aritmetica de congruencias, las operaciones de adicion, sustracci on, y mul-
tiplicacion se realizan como en aritmetica ordinaria (excepto que son tomadas
con respecto al m odulo en consideraci on). Que acerca de la divisi on?
Decimos que a divide b m odulo n si existe un entero c tal que ac b(mod n).
Si existe un entero c tal que ac 1(mod n), entonces c es llamado el inverso
(multiplicativo) de a, algunas veces denotado por a
1
. Notar que si a tienen un
inverso, la ecuaci on ax b(mod n) puede ser resuelta simplemente multiplican-
do cada lado de la ecuaci on por a
1
; x a
1
b (mod n).
Un importante hecho te orico es que un entero a tiene un inverso multiplicativo
con respecto al m odulo n aritmeticamente si y s olo si a y n son primos relativos
(ver 3.2.21).
Como un caso especial del resultadodel paragrafo previo, considerar el caso en
que el m odulo n es un n umero primo, digamos p. En este caso, cada uno de
1, 2, ..., p 1 es primo relativo con p, luego todos ellos tienen inverso multiplica-
tivo. En efecto, los n umeros de Z
p
= 0, 1, 2, ..., p 1 pueden ser sumados,
sustraidos, multiplicados y divididos (por elementos distintos de cero), y ellos
forman un campo (ver seccion 4.4).
3.2.7 Probar que las expresiones
2x + 3y, 9x + 5y
son divisibles por 17 para el mismo conjuntos de valores de x y y.
Soluci on. Es suciente probar que
2x + 3y 0(mod 17) si y s olo si 9x + 5y 0(mod 17).
El plan es m ultiplicar cada lado de la congruencia de la izquierda por una con-
stante adecuada que la transforme en la congruencia de la derecha. Preguntemos:
existir a una constante c tal que c(2x + 3y) 9x + 5y(mod 17)? Para que esto
sea posible es necesario que 2c 9(mod 17). Como 2 es primo relativo con 17,
tiene un inverso. De hecho 2
1
= 9, y de aqu, c = 9 9 = 81 13(mod 17).
De donde 2x + 3y 0(mod 17) implica
13(2x + 3y) 0(mod 17)
26x + 39y 0(mod 17)
9x + 5y 0(mod 17)
Inversamente, multiplicando cada lado de 9x + 5y 0(mod 17) por 4 tenemos
que 2x + 3y 0(mod 17).
El siguiente ejemplo es un resultado te orico que no solamente es interesante
desde un punto de vista conceptual, sino tambien por que tiene m ultiples apli-
caciones a traves de las matematicas.
3.2. ARITM

ETICA MODULAR 113


3.2.8 (Teorema Chino del Residuo) Si m y n son enteros primos relativos
mayores que uno, y a y b son enteros arbitrarios, existe un entero x tal que
x a(mod m),
x b(mod n).
M as generalmente, si m
1
, m
2
, ..., m
k
son (por pares) primos relativos mayores
que uno, y a
1
, a
2
, ..., a
k
son enteros arbitrarios, existe un entero x tal que
x a
i
(mod m
i
), i = 1, 2, ...k.
Soluci on. Considerar los n n umeros a, a + m, a + 2m, ..., a + (n 1)m. Cada
uno de ellos es congruente con a m odulo m. Ademas, culesquiera dos de ellos no
son congruentes m odulo n. Ya que, si a + im a +jm(mod n), 0 i < j < n,
entonces (i j)m 0(mod n). Pero m y n son primos relativos, luego esta
ultima congruencia es cierta solamente si n divide a i j. Por otro lado, i j no
puede ser un m ultiplo de n debido a las restricciones sobre i y j. Luego, i = j.
Se sigue que los n umeros a, a + m, a + 2m, ..., a + (n 1)m. son congruentes
en alg un orden a los n umeros 0, 1, 2, ..., n 1 m odulo n. De donde, para alg un
i, a + im 0(mod n). La prueba de la primera parte se termina tomando
x = a +im.
La propiedad m as general puede ser probada de manera similar, usando induc-
cion sobre k. (Sea c = m
1
m
k1
, y considerar a, a+c, a+2c, ..., a+(m
k
1)c,
donde a es el obtenido por la hip otesis inductiva a a
i
(mod m
i
), i = 1, 2, ..., k
1. Entonces a + ic a
i
(mod m
i
), i = 1, 2, ..., k 1, y cualesquiera dos de ellos
no son congruentes m odulo m
k
, etc.)
3.2.9 Existe 1,000,000 de enteros consecutivos cada uno de los cuales contiene
un factor primo repetido?
Soluci on. Si p
1
, p
2
, ..., p
1,000,000
denotan distintos n umeros primos. Entonces
p
2
i
y p
2
j
son primos relativos si i ,= j, luego por el teorema chino del residuo,
existe un n umero x tal que
x k(mod p
2
k
), k = 1, 2, ..., 10
6
.
Se sigue que x + k es divisible por p
2
k
(es decir, x + k tiene un factor primo
repetido), y la respuesta a la pregunta es si: tomense los enteros consecutivos
x + 1, x + 2, x + 3, ...x + 1, 000, 000.
3.2.10 Un punto laticce en (x, y) Z
2
es visible si mcd(x, y) = 1. mProbar o
refutar: Dado un entero positivo n, existe un punto laticce (a, b) cuya distancia
desde cada punto visible es n.
114 CAP

ITULO 3. ARITM

ETICA
Soluci on. Veamos primero un caso especial, pues el patr on para el caso general
es una generalizaci on simple como veremos claramente. Tomando nueve primos
distintos p
1
, p
2
, ..., p
9
. ahora veamos para un punto laticce (a, b) tal que
a 1 0(mod p
1
p
2
p
3
)
a 0(mod p
4
p
5
p
6
)
a + 1 0(mod p
7
p
8
p
9
)
y
b + 1 0(mod p
1
p
4
p
7
)
b 0(mod p
8
p
5
p
8
)
b 1 0(mod p
3
p
6
p
9
)
Geometricamente, (a, b) es un punto caracterizado por el siguiente diagrama
Como p
1
p
2
p
3
, p
4
p
5
p
6
, p
7
p
8
p
9
son primos relativos, el teorema chino del residuo
asegura que existe un entero a que satisface las ecuaciones (1). Similarmente,
como p
1
p
4
p
7
, p
2
p
5
p
8
, p
3
p
6
p
9
son primos relativos, existe un entero b que satisface
las ecuaciones (2). Por ello tomando a y b, es claro que los 8 puntos latices
cercanos a (a, b) son invisibles. Al tomar, por ejemplo, el punto (a + 1, b),que
tiene la forma (k
1
p
4
p
5
p
6
, k
2
p
1
p
4
p
7
) para algunos enteros k
1
y k
2
. Como p
4
es
factor com un de las coordenadas, este punto es invisible. Un argumento similar
se aplica para los otros siete puntos latices cercanos.
El caso general puede ser abordado de la misma manera, y se pide la de-
mostracion en el problema 3.2.26.
3.2. ARITM

ETICA MODULAR 115


Problemas
3.2.11 Probar que en cualquier subconjunto de 55 n umeros tomados del con-
junto 1, 2, 3, ..., 100 debe contener n umeros que dieran por 10, 12 y 13 pero
no necesariamente un par que diera por 11.
3.2.12 Los elementos de un determinante son enteros arbitrarios. Determi-
nar la probabilidad de que el valor del determinante sea impar. (Sugerencia:
Trabajarm odulo 2).
3.2.13 .
(a) Determinar si la siguiente matriz es o no singular
_
_
_
_
54401 57668 15982 103790
33223 26563 23165 71489
36799 37189 16596 46152
21689 55538 79922 51237
_
_
_
_
(Sugerencia: Una matriz A es no singular si det A ,= 0. Examinar la pari-
dad del determinante de la matriz dada; esto es, computar el determinante
m odulo 2).
(b) Determinar si la siguiente matriz es o no singular
_
_
_
_
64809 91185 42391 44350
61372 26563 23165 71489
82561 39189 16596 46152
39177 55538 79922 51237
_
_
_
_
3.2.14 .
(a) Probar que 2
2x+1
+ 1 es divisible por 3
(b) Probar o refutar: 2
x
2
y
(mod n) si x y (mod n)
(c) Probar que 4
3x+1
+ 2
3x+1
+ 1 es divisible por 7.
(d) Si n > 0, probar que 12 divide a n
4
4n
3
+ 5n
2
2n.
(e) Probar que (2903)
n
(803)
n
(464)
n
+ (261)
n
es divisible por 1897.
116 CAP

ITULO 3. ARITM

ETICA
3.2.15 .
(a) Probar que ningun primo mayor en tres que un m ultiplo de 4 es suma de dos
cuadrados. (Sugerencia: Trabajar m odulo 4).
(b) Probar que la sucesi on (en notaci on base 10)
11, 111, 1111, 11111, ...
no contiene cuadrados.
(c) Probar que la diferencia entre los cuadrados de cualesquiera dos n umeros
impares es divisible por 8.
(d) Probar que 2
70
+ 3
70
es divisible por 13.
(e) Probar que la suma de dos cuadrados impares no puede ser un cuadrado.
(f ) Determinar todas las soluciones enteras de a
2
+b
2
+c
2
= a
2
b
2
. (Sugerencia:
Analice m odulo 4)
3.2.16 .
(a) Si x
3
+ y
3
= z
3
tiene una soluci on en los enteros x, y, z, probar que uno de
los tres debe ser m ultiplo de 7.
(b) Si n es un entero mayor que 1 tal que 2
n
+n
2
es primo, probar que n 3(mod
6).
(c) Sea x un entero 1 menor que un m ultiplo de 24. Probar que si a y b son
enteros positivos tales que ab = x, entonces a +b es m ultiplo de 24.
(d) Probar que si n
2
+m y n
2
m son cuadrados perfectos, entonces m es divisible
por 24.
3.2.17 Sea S un conjunto de n umeros primos tales que a, b S (a y b no
necesariamente distintos) implica que ab + 4 S. Probar que S debe ser vaco.
(Sugerencia: Se puede aprovechar el trabajo m odulo 7).
3.2.18 Probar que no existen enteros x e y para los cuales
x
2
+ 3xy 2y
2
= 122.
3.2. ARITM

ETICA MODULAR 117


(Sugerencia: Usar la ecuaci on cuadr atica y resolverla para x; entonces vemos el
discriminante m odulo 17. Puede este ser un cuadrado perfecto?).
3.2.19 Dado un entero n, probar que podemos encontrar un entero escrito
s olamente con los dgitos 0 y 1 (en notaci on base 10) y que es divisible por n.
3.2.20 Probar que si n divide a un n umero de Fibonacci, entonces dividir a a
una innidad de n umeros de Fibonacci.
3.2.21 Supongamos que a y n son enteros, n > 1. Probar que la ecuaci on
ax 1 (mod n) tiene una soluci on si y s olo si a y n son primos relativos.
3.2.22 Sean a, b, c, d enteros jos donde d no es divisible por 5. Asumir que
m es un entero para el cual
am
3
+bm
2
+cm+d
es divisible por 5. probar que existe un entero n para el cual
dn
3
+cn
2
+bn +a
es tambien divisible por 5.
3.2.23 Probar que
21n 3
4
y
15n + 2
4
no pueden ambos ser enteros para el
mismo entero positivo n.
3.2.24 .
(a) Existen n enteros consecutivos para los cuales el jesimo entero 1 j n,
tiene un divisor que no divide a cualquier otro termino de la sucesi on?
(b) Existen n enteros consecutivos para los cuales el jesimo entero 1 j n,
tiene al menos j divisores, ninguno de los cuales divide a cualquier otro termino
de la sucesi on?.
3.2.25 Sean m
0
, m
1
, ..., m
r
enteros positivos que son primos relativos por pare-
jas. Probar que existen r +1 enteros consecutivos s, s +1, ..., s +r tales que m
i
,
divide a s +i para i = 0, 1, ..., r.
118 CAP

ITULO 3. ARITM

ETICA
3.2.26 Completar la prueba de 3.2.10.
Ejemplos adicionales
3.3.11, 3.4.3, 3.4.9, 4.1.3, 4.2.4, 4.3.4, 4.3.5, 4.4.6, 4.4.7, 4.4.8, 4.4.9, 4.4.19,
4.4.20, 4.4.21, 4.4.22, 4.4.23, 4.4.24, 4.4.29, 4.4.30, 4.4.31.
3.3. Factorizacion

Unica
Uno de los resultados m as usado y con alto potencial en el centro de la teora
de n umeros elemental es el hecho de que cada n umero natural mayor que uno
puede ser factorizado de manera unica (sin importar el orden de los factores)
en un producto de n umeros primos. Mas precisamente, cada n umero natural n
puede ser representado en una y s olo una manera en la forma
n = p
a1
1
p
a2
2
... p
a
k
k
donde p
1
, p
2
, ..., p
k
son n umeros primos diferentes y a
1
, a
2
, ..., a
k
son enteros
positivos. Aqu probaremos algunas faciles, pero muy usadas, consecuencias.
3.3.1 Todos los divisores de
n = p
a1
1
p
a2
2
... p
a
k
k
son de la forma
m = p
b1
1
p
b2
2
... p
b
k
k
, 0 b
i
a
i
, i = 1, ..., k,
y cada uno de tales n umeros es divisor de n. Se sigue entonces que n tiene
exactamente (a
1
+ 1)(a
2
+ 1) ... (a
k
+ 1) divisores distintos.
3.3.2 Un entero n = p
a1
1
p
a2
2
... p
a
k
k
es un cuadrado perfecto si y s olo si
a
i
es par para cada i, un cubo perfecto si y s olo si a
i
es m ultiplo de tres, y
as sucesivamente.
3.3.3 Sea a, b, ..., g un n umero nito de enteros positivos. Sup ongase que sus
factorizaciones unicas son
a = p
a1
1
p
a2
2
... p
a
k
k
, b = p
b1
1
p
b2
2
... p
b
k
k
, ..., g = p
g1
1
p
g2
2
... p
g
k
k
,
donde a
1
, a
2
, ..., a
k
, b
1
, b
2
, ..., b
k
, ..., g
1
, g
2
, ..., g
k
son enteros no negativos (algunos
pueden ser cero). Entonces
mcd(a, b, ..., g) = p
m1
1
p
m2
2
... p
m
k
k
,
3.3. FACTORIZACI

ON

UNICA 119
y
mcm(a, b, ..., g) = p
M1
1
p
M2
2
... p
M
k
k
,
donde m
i
= mna
i
, b
i
, ..., g
i
y M
i
= m axa
i
, b
i
, ..., g
i
para cada i = 1, 2, ..., k.
De aqu es f acil probar que
mcd(a, b, )mcm(a, b, ) = ab.
3.3.4 Usar factorizaci on unica para probar que

2 es irracional.
Soluci on. Supongase que existen enteros r y s tales que

2 = r/s. entonces
2s
2
= r
2
. Pero esta ecuaci on no tiene soluci on (por factorizaci on unica), ya que
en el lado izquierdo, el primo 2 esta elevado a una potencia impar, y en lado
derecho a una potencia par (el 2 aparece un n umero par de veces (puede ser
cero) en s
2
y r
2
). Esta contradiccion implica que

2 es irracional.
3.3.5 Encontrar el menor entero positivo n tal que n/2 es un cuadrado perfecto,
n/3 es un cubo perfecto, y n/5 es una potencia quinta perfecta.
Soluci on Como n es divisible por 2, 3, y 5, podemos asumir que tiene la forma
n = 2
a
3
b
5
c
. Entonces n/2 = 2
a1
3
b
5
c
, n/3 = 2
a
3
b1
5
c
, n/5 = 2
a
3
b
5
c1
. Las
condiciones son tales que a 1 debe ser par, y a debe ser m ultiplo de ambos
3 y 5. El menor de tales a es a = 15. Similarmente, los m as peque nos valores
de b y c son b = 10 y c = 6. Luego n = 2
15
3
10
5
6
es el menor de tales enteros
positivos.
3.3.6 Probar que existe uno y s olo un n umero natural n tal que 2
8
+ 2
11
+ 2
n
es un cuadrado perfecto.
Soluci on Supongase que m
2
= 2
8
+ 2
11
+ 2
n
. Entonces
2
n
= m
2
2
8
2
11
= m
2
2
8
(1 + 2
3
)
= m
2
(3 2
4
)
2
= (m+ 48)(m48).
Debido a la factorizaci on unica, existen enteros no negativos s y t tales que
m48 = 2
s
, m+ 48 = 2
t
, s +t = n.
Luego m = 2
s
+ 48, m = 2
t
48, de aqu
2
s
+ 48 = 2
t
48,
2
t
2
s
= 96,
2
s
(2
ts
1) = 2
5
3.
120 CAP

ITULO 3. ARITM

ETICA
Como 2
ts
1 es impar, la factorizaci on unica implica que 2
ts
1 = 3. Se
sigue que s = 5, t = 7, y n = 12.
3.3.7 Sea n un entero positivo dado. Cu antas soluciones existen en los poares
ordenados (x, y) de la ecuaci on
xy
x +y
= n?
Soluci on. Escribamos la ecuaci on en la forma
xy = n(x +y)
xy nx ny = 0
(x n)(y n) = n
2
.
Como queremos soluciones enteras positivas, debemos ver los casos en que x > n
y y > n (0 < x < n y 0 < y < n implican (x n)(y n) < n
2
).
Supongase que la factorizaci on prima de n es p
a1
1
p
a2
2
... p
a
k
k
. Entonces n
2
=
p
2a1
1
p
2a2
2
... p
2a
k
k
. Cada divisor de n
2
determina una soluci on, y de aqu el
n umero de tales soluciones es (2a
1
+ 1)(2a
2
+ 1) ... (2a
k
+ 1)
3.3.8 Sean r y t enteros positivos. Derive una f ormula para el n umero de
cu adruplas (a, b, c, d) de enteros positivos tales que
3
r
7
s
= mcm(a, b, c) = mcm(a, b, d) = mcm(a, c, d) = mcm(b, c, d).
Soluci on. En vista de los resultados de 3.3.3, se desprende que cada uno
de a, b, c, d debe tener la forma 3
m
7
n
con m en 0, 1, ..., r y n en 0, 1, ..., s.
Tambien, m debe ser r para al menos dos de los 4 n umeros, y n debe ser s para al
menos dos de los 4 n umeros. Existen
_
4
2
_
r
2
maneras viables de tomar las m

s en
donde exactamente dos m

s son iguales a r; existen


_
4
3
_
r maneras permitidas en
las cuales exactamente tres m

s sean iguales a r; existen


_
4
4
_
maneras permitidas
en las cuales todas las m

s sean iguales a r. Por todo esto, existen


_
4
4
_
+
_
4
3
_
r +
_
4
2
_
r
2
formas de tomar las m

s. Similarmente, existen
_
4
4
_
+
_
4
3
_
s +
_
4
2
_
s
2
formas de tomar las n

s. El n umero es por tanto (1 + 4r + 6r


2
)(1 + 4s + 6s
2
).
3.3. FACTORIZACI

ON

UNICA 121
3.3.9 Dados enteros positivos x, y, z probar que
(x, y)(x, z)(y, z)[x, y, z]
2
= [x, y][x, z][y, z](x, y, z)
2
,
donde (a, b, ..., g) y [a, b, ..., g] denotan mcd(a, b, ..., g) y mcm(a, b, ..., g) respec-
tivamente.
Soluci on. Debido a la factorizaci on unica, es suciente probar que para cada
primo p, la potencia de p en el lado izquierdo (en su factorizaci on prima) es
igual a la potencia de p en el lado derecho. Supongamos pues que x = p
a
r,
y = p
b
s, y z = p
c
t, para enteros r, s, t cada uno primo relativo con p. Podemos
asumir (a casusa de la simetra del problema, y renombrando si es necesario)
que a b c. Entonces la potencia de p en la factorizaci on unica de [x, y, z]
2
es 2c; las potencias de p en (x, y), (x, z), (y, z) son a, a y b respectivamente. De
aqu la potencia de p en el lado izquierdo es 2a +b + 2c.
De la misma manera, las potencias del lado derecho son b+c+c+2a = 2a+b+2c.
Luego, por lo remarcado anteriormente, la prueba se completa.
3.3.10 Probar que 1000! tiene 249 al nal.
Soluci on. Escribamos 1000! = 2
a
5
b
r, donde r es un entero primo relativo con
10. Claramente, a b, y el n umero de ceros al nal de 1000! debe ser igual a b.
Luego, debemos encontrar b.
Cada quinto entero de la sucesion 1, 2, 3, 4, 5, 6, ..., 1000 es divisible por 5; existen
__
10005

= 200 m ultiplos de 5 en la sucesion. Cada 25avo entero de la sucesion


es divisible por 25, cada uno de ellos contribuye con un factor adicional; existen
__
1000
25

= 40 de estos. Cada 125avo entero en la sucesion es divisible por 125,


y cada uno de ellos contribuye con un factor adicional ; existen
__
1000
125

= 8
de estos. Cada 625avo entero de la sucesion tambien contribuye con un factor
adicional; existen
__
1000
625

= 1 de estos.
Luego b =
__
1000
5

+
__
1000
25

+
__
1000
125

+
__
1000
625

= 200 + 40 + 8 + 1 = 249.
Exactamente de la misma manera, la potencia de p en n! esta dada por la suma
nita
__
n
p
__
+
__
n
p
2
__
+
__
n
p
3
__
+....
3.3.11 Probar que existe un n umero innito de primos de la forma 6n 1.
Soluci on. Primero, notese que si p es un n umero primo mayor que 3, entonces
p 1 (mod 6) o p 1 (mod 6). [Si p 2 (mod 6), por ejemplo, entonces
p = 6k + 2 para alg un k, lo cual implica que p es par, una contradiccion. Un
argumento similar funciona para p 3 (mod 6) o p 4 (mod 6)].
122 CAP

ITULO 3. ARITM

ETICA
Ahora sup ongase que existe un n umero nito de primos de la forma p = 6n1.
Considere el n umero N = p! 1, donde p es el mayor primo de la forma 6n1.
Escribamos N como un producto de primos, es decir
N = p! 1 = p
1
p
2
... p
m
, (1)
Observese que cada uno de los primos p
k
es mayor que p . Ya que, si p
k
p
entonces la ecuaci on (1) prueba que p
k
divide a 1, un imposible. Como p es el
mayor primo congruente con 1 m odulo 6, se sigue que p
k
1 (mod 6) para
cada k.
Si ahora consideramos la ecuaci on (1) m odulo 6 encontramos que
p! 1 1 (mod 6),
o equivalentemente,
p! 2 (mod 6).
Pero esto es claramente imposible, ya que p! 0 (mod 6). De donde existe un
n umero innito de primos de la forma 6n 1.
Problemas
3.3.12 En un cierto colegio, el n umero de estudiantes es menor que 5000, una
tercera parte de los estudiantes son ni nos, dos septimos son adolecentes, un
quinto j ovenes y el resto adultos. El departamento de historia ofrece un curso
popular en el cual se registran un cuarto de todos los ni nos del colegio, un sexto
de los adolecentes, y un noveno de todos los j ovenes, el resto que es la tercera
parte de la clase de historia son todos adultos. Cu antos estudiantes est an en la
clase de historia?
3.3.13 Encontrar el menor n umero con 28 divisores.
3.3.14 Dados enteros distintos a, b, c, d tales que
(x a)(x b)(x c)(x d) 4 = 0
tiene una raiz entera r, probar que 4r = a +b +c +d.
3.3.15 .
(a) Probar que
3

72 es irracional.
3.3. FACTORIZACI

ON

UNICA 123
(b) Probar que no existe un conjunto de enteros m, n, p excepto 0, 0, 0 para el
cual m+n

2 +p

3 = 0.
3.3.16 Dados enteros positivos a, b, c, d tales que a
3
= b
2
, c
3
= d
2
, y ca = 25,
determinar a, b, c y d.
3.3.17 Probar que si ab, ac y bc son cubos perfectos para algunos enteros pos-
itivos a, b, c, entonces a, b y c deber an ser cubos perfectos.
3.3.18 Un vestidor tiene n lockers numerados de 1 a n, y todos est an cerrados.
Una lnea de n clientes P
1
, P
2
, ..., P
n
pasan por el vestidor en orden. Cada cliente
P
k
cambia la condici on de los lockers (y s olamente de ellos) cuyo n umero sea
divisible por k : si tal locker est a abierto, P
k
lo cierra; si est a cerrado, P
k
lo
abre. Que lockers est an abiertos despues de que hubieron pasado los n clientes
por el vestidor? Cu al ser a la situaci on si los clientes hubieran hecho la misma
operaci on, pero pasando en otro orden?.
3.3.19 La geometra de los n umeros escritos en lnea establece que es claro
que en cualquier conjunto de n enteros consecutivos, uno de ellos es divisible
por n. Este hecho es frecuentemente usado, como por ejemplo en los siguientes
problemas.
(a) Probar que si uno de los n umeros 2
n
1 y 2
n
+ 1 es primo, n > 2, entonces
el otro n umero es compuesto.
(b) Cu al es el mayor entero N para el cual n
5
5n
3
+4n es divisible por N para
cada entero n?
(c) Probar que cada entero positivo tiene un m ultiplo cuya representaci on decimal
involucra los diez dgitos.
3.3.20 Para cada entero positivo n sea H
n
= 1+1/2+...+1/n. Probar que para
n > 1, H
n
no es entero. (Sugerencia: Sup ongase que H
n
es entero. Multiplique
cada lado de la igualdad por mcm(1, 2, ..., n), y pruebe que el lado izquierdo de
la identidad es par mientras que el lado derecho es impar.)
3.3.21 Si mcd(a, b) = 1 entonces prueba que
(i) mcd((a +b)
m
, (a b)
m
) 2
m
, y
124 CAP

ITULO 3. ARITM

ETICA
(ii) mcd(a
m
+b
m
, a
m
b
m
) 2.
3.3.22 Para eteros positivos a, b, ..., g si (a, b, ..., g) y [a, b, ..., g] denotan mcd (a, b, ..., g)
y mcm(a, b, ..., g) respectivamente. Probar que
(a) xyz = (xy, xz, yz) [x, y, z] ,
(b) (x, [y, z]) = [(x, y) , (x, z)] ,
(c) [x, (y, z)] = ([x, y] , [x, z]) ,
(d) ([x, y] , [x, z] [y, z]) = [(x, y) , (x, z) , (y, z)] ,
(e) [x, y, z] (x, y) , (x, z) , (y, z) = xyz (x, y, z) ,
(f ) (x, y) = (x +y, [x, y]) .
3.3.23 Sea m divisible por 1, 2, ..., n. Probar que los n umeros 1 + m(1 + i),
i = 0, 1, 2, ..., n son primos relativos por parejas.
3.3.24 La factorizaci on prima de r +1 enteros positivos (r 1) involucra so-
lamente r primos. Probar que existe un subconjunto de esto conjunto de enteros
cuyo producto es un cuadrado perfecto.
3.3.25 .
(a) Determinar todas las soluciones racionales positivas de x
y
= y
x
.
(b) Determinar todas las soluciones racionales positivas de x
x+y
= (x +y)
y
.
3.3.26 Sup ongase que a
2
+ b
2
= c
2
, a, b, c enteros. Asumir que mcd(a, b) =
mcd(a, c) = mcd(b, c) = 1. Probar que existen enteros u y v tales que c b =
2u
2
, c +b = 2v
2
, mcd(u, v) = 1. Concluir que a = 2uv, b = v
2
u
2
, c = u
2
+v
2
(Sugerencia:Examinando m odulo 4, no es posible el caso de que a y b sean
ambos impares; necesariamente uno de ellos o ambos es par. Luego, sin perdida
de generalidad, a es par y b es impar.)
Inversamente, probar que si u y v son dados, entonces existen n umeros a, b, c
dados por las f ormulas anteriores tales que a
2
+b
2
= c
2
.
3.4. NOTACI

ON POSICIONAL 125
3.3.27 Encontrar todos los conjuntos de tres cuadrados perfectos en progresi on
aritmetica. (Sugerencia: Sup ongase a < b < c y b
2
a
2
= c
2
b
2
, o equivalen-
temente c
2
+ a
2
= 2b
2
. Si s = (c + a)/2, t = (c a)/2. Probar que s
2
+ t
2
= b
2
ahora aplique el resultado de 3.3.26.)
3.3.28 .
(a) Sup ongase que existe solamente un n umero nito de primos de la forma
6n 1; llamemosle p
1
, p
2
, ..., p
k
. Arribe a una contradicci on considerando N =
(p
1
p
2
... p
k
)
2
1.
(b) Probar que existe una innidad de primos de la forma 4n 1.
Ejemplos adicionales:
1.10.9, 1.10.10, 2.6.1, 3.1.4, 3.4.8, 4.1.3, 4.2.3, 4.2.16b, 4.4.9, 5.2.1, 5.2.4, 5.2.6,
5.2.9, 5.2.14, 5.2.15, 5.2.16, 5.2.17.
3.4. Notacion Posicional
Asumimos cierta familiaridad con la notacion posicional del sistema de los
n umeros reales. En otras palabras, si b es un entero mayor que 1 (le llamamos
base), cada n umero real x puede ser expresado (de manera unica) el la forma
posicional
x = A
n
A
n1
...A
1
A
0
.a
1
a
2
a
3
...
donde los A
0
, A
1
, ..., A
n1
, A
n
.a
1
, a
2
, a
3
, ...(llamados dgitos) son enteros, 0
A
i
< b, 0 .a
i
< b, y no existe un entero m tal que a
k
= b 1 para todo k > m.
Esta representaci on es usada para denotar la suma de la serie
A
n
b
n
+A
n1
b
n1
+... +A
1
b +A
0
+a
1
b
1
+a
2
b
2
+....
3.4.1 Si C denota la clase de los enteros positivos tales que, cuando son es-
critos en base 3, no requieren al dgito 2. Probar que no existen tres enteros en
C que esten en progresi on aritmetica.
Soluci on. Si d denota la diferencia com un para cualquier progresion aritmetica
arbitraria de tres enteros positivos, y sup ongase que cuando d es escrita en
notacion base 3 tiene su primer dgito distinto de cero, contando a partir del
extremo derecho, en la k-esima pisicion. Ahora sea a un entero arbitrario , y
escrib amoslo en notacion base 3. La siguiente tabla proporciona el k-esimo dgito
126 CAP

ITULO 3. ARITM

ETICA
de cada uno de los enteros a, a +d, a +2d, dependiendo desde luego del k-esimo
dgito de d y a :
En cada caso uno de a, a +d, a + 2d tiene al 2 en el k-esimo dgito, por lo cual
el n umero correspondiente no pertenece a C.
3.4.2 Tiene la ecuaci on [[x]] +[[2x]] +[[4x]] +[[8x]] +[[16x]] +[[32x]] = 12345
una soluci on?
Soluci on. Supongase que x es tal n umero. Es facil probar que 195 < x < 196
(ya que 63 195 = 12285 < 12345 < 12348 = 63 196). Ahora, escribamos la
parte racional de x en notacion base 2 (los n umeros a, b, c, ... son 0 o 1):
x = 195.abcdef....
Entonces
2x = 2 195 +a.bcdef...,
4x = 4 195 +ab.cdef...,
8x = 8 195 +abc.def...,
16x = 16 195 +abcd.ef...,
32x = 32 195 +abcde.f...,
3.4. NOTACI

ON POSICIONAL 127
En esta forma vemos que
[[x]] = 195,
[[2x]] = 2 195 +a
[[4x]] = 4 195 + 2a +b
[[8x]] = 8 195 + 4a + 2b +c
[[16x]] = 16 195 + 8a + 4b + 2c +d
[[32x]] = 32 195 + 16a + 8b + 4c + 2d +e
Sumando, encontramos que [[x]] +[[2x]] +[[4x]] +[[8x]] +[[16x]] +[[32x]] = 63
195+31a+15b +7c+ 3d+e. El problema se reduce ahora a encontrar a, b, c, d, e,
cada uno 0 o 1, tales que 31a + 15b +7c+ 3d + e = 60. Pero esta ecuaci on no
puede ser resuleta con las restricciones sobre a, b, c, d, e ya que 31a + 15b +7c+
3d +e = 57 < 60. De aqu, no puede existir tal x.
Cuando un entero es escrito en notacion decimal (base 10), es posible determinar
muy facilmente si es divisible por 2 o 5. Existen otras pruebas de divisibilidad
que son facilmente aplicables. Por ejemplo; un entero N es divisible por 4 si y
s olo si sus ultimos dos dgitos son divisibles por 4. Veamos esto, escriba N en
notacion base 10:
N = a
n
10
n
+a
n1
10
n1
+... +a
2
10
2
+ (a
1
10 +a
0
)
y note que a
n
10
n
+ a
n1
10
n1
+ ... + a
2
10
2
es siempre divisible por 4. Luego,
4 [ N si y s olo si 4 [ (a
1
10 +a
0
).
Una de las m as y usadas pruebas de divisibilidad es que un entero es divisible
por 9 si y s olo si la suma de sus dgitos (en notacion decimal) es divisible por
9. Veamos esto, notece que 10 1(mod 9), y de aqu, por las propiedades de
aritmetica modular, 10
2
1(mod 9), 10
3
1(mod 9), y as sucesivamente. Se
sigue que
N = a
n
10
n
+a
n1
10
n1
+... +a
1
10 +a
0
a
n
+a
n1
+... +a
1
+a
0
(mod 9).
Una prueba similar demuestra que un entero es divisible por 3 si y solamente si
la suma de sus dgitos es divisible por 3. Como una aplicaci on de esta prueba,
supongamos que queremos encontrar: para que dgitos x es 4324x98765 223 es
divisible por 3? Simplemente necesitamos sumar los dgitos m odulo 3. en este
caso, la suma de los dgitos es 1 +x m odulo 3, luego el n umero es divisible por
3 si y s olo si x = 2, 5 u 8.
3.4.3 Cuando 4444
4444
es escrito en notaci on decimal, la suma de sus dgitos
es A. Si B es la suma de los dgitos de A. Encontrar la suma de los dgitos de
B. (A y B son escritos en notaci on decimal).
Soluci on. Sea N = 4444
4444
. Entonces N < (10
5
)
4444
= 10
22220
, esto signica
que cuando N es escrito en notacion decimal, tiene menos que 22220 dgitos.
128 CAP

ITULO 3. ARITM

ETICA
Como cada uno de los dgitos es menor o igual que 9, estamos seguros que
A < 9 22220 = 199980.
En una forma similar, A tiene a lo m as 6 dgitos, la suma de los dgitos de A es
menor que 54(= 6 9); Es decir, B < 54.
De los enteros positivos menores que 54, el n umero con mayor suma de sus
dgitos es 49, y su suma es 13. Si C denota la suma de los dgitos de B. Hemos
visto justamente que C 13.
Del razonamiento precedente al problema, sabemos que
N A B C (mod 9),
calculemos la clase de congruencia de C calculando la clase de congruencia de N.
Primero 4444 = 9493+7, y de aqu 4444 7(mod9). Tambien 7
3
1(mod9).
Como 4444 = 3 1481 + 1, tenemos
4444
4444
7
4444
(mod 9)
7
31481
7(mod 9)
7(mod 9).
Luego, C 7(mod9) y C 13. El unico n umero que puede satisfacer ambos
requerimientos es C = 7, y el problema esta resuelto.
3.4.4 Una tripleta (ordenada) (x
1
, x
2
, x
3
) de n umeros irracionales positivos
con x
1
+ x
2
+ x
3
= 1 es llamada balanceada si cada x
i
< 1/2. si una tripleta
es no balanceada, es decir si x
j
> 1/2, el siguiente procedimiento acto de
balanceo:
B(x
1
, x
2
, x
3
) = (x

1
, x

2
, x

3
) ,
donde x

i
= 2x
i
si i ,= j y x

j
= 2x
j
1. Si la nueva tripleta es no balanceada,
continuamos con el procedimiento de acto de balanceo. Es posible continuar este
procedimiento para obtener una tripleta balanceada aplicando un n umero nito
de veces el acto de balanceo?
Soluci on. Escribamos x
1
, x
2
, x
3
en notacion base 2 de la manera descrita al
comienzo de esta seccion, veamos
x
1
= .a
1
a
2
a
3
...,
x
2
= .b
1
b
2
b
3
...,
x
3
= .c
1
c
2
c
3
...,
donde a
i
, b
i
, c
i
son 0 o 1.
Vea que cada x
i
< 1/2 implica que a
1
, b
1
,y c
1
son todos iguales a cero. Notese
que el acto de balanceo consiste en recorreo el punto decimal un lugar y
3.4. NOTACI

ON POSICIONAL 129
entonces quitarle la parte entera. Luego si x
1
, x
2
, x
3
es no balanceada entonces
la representaci on (base 2) de x

1
, x

2
, x

3
esta dada por
x

1
= .a
2
a
3
a
4
...,
x

2
= .b
2
b
3
b
4
...,
x

3
= .c
2
c
3
c
4
....
Se pueden dar muchos ejemplos de para probar que el proceso no necesariamente
termina en una tripleta balanceada. Por ejemplo, dena x
1
, x
2
, x
3
(usando la
notacion anterior) por
a
i
=
_
1 si i es cuadrado perfecto,
0 en otros casos,
b
i
=
_
1 si i es uno m as que un cuadrado perfecto,
0 en otros casos,
c
i
=
_
1 a
i
+b
i
= 0,
0 en otros casos,
esto es,
x
1
= ,100100001000000100...,
x
2
= ,010010000100000010...,
x
3
= ,001001110011111001...,
Cada uno de los x
1
, x
2
, y x
3
es irracional (los n umeros racionales son aquellos a
los que les corresponde una representaci on decimal periodica), y su suma es
1 (ya que x
1
+x
2
+x
3
=
1
2
+
1
4
+
1
8
+...+ = 1). Repitiendo las aplicaciones del
acto de balanceo nunca podemos transformar la tripleta x
1
+ x
2
+ x
3
en una
tripleta balanceada (debido a que, en cada caso, uno de los a
i
, b
i
, c
i
es igual a
1).
3.4.5 (Continuaci on del 2.5.10). Sup ongase que f es una funci on sobre
los enteros positivos que satisface
f(2k) = 2f(k) 1
f(2k + 1) = 2f(k) + 1
Sea a un entero positivo arbitrario cuya representaci on binaria est a dada por
a = a
n
a
n1
...a
2
a
1
a
0
(= a
n
2
n
+a
n1
2
n1
+... +a
2
2
2
+a
1
2 +a
0
)
probar que f(a) = b
n
2
n
+b
n1
2
n1
+... +b
2
2
2
+b
1
2 +b
0
donde
130 CAP

ITULO 3. ARITM

ETICA
b
i
=
_
1 si a
i
= 1,
1 si a
i
= 0,
(La idea es reemplazar cada uno de los 0s en la suma binaria para a con 1s
; por ejemplo, para n = 10, f(1010
2
) = 1111
2
(los 1s representan a los 1s) =
8 4 + 2 1 = 5.)
Soluci on. Hagamos induccion sobre el n umero de dgitos en la representaci on
binaria de a.
El resultado es cierto para a = 1, sup ongase que el resultado es verdadero
cuando a tiene menos que k + 1 dgitos. Ahora consideremos un entero a con
k + 1 dgitos (en base 2), es decir
a = a
k
a
k1
...a
2
a
1
a
0
.
Si a
0
= 0, entonces a = 2(a
k
a
k1
...a
2
a
1
), f(a) = 2f(a = a
k
a
k1
...a
2
a
1
) 1 =
2(b
k
2
k1
+b
k1
2
k2
+...+ b
2
2+b
1
) 1 = b
k
2
k
+b
k1
2
k1
+... +b
2
2
2
+b
1
2+b
0
,
y el resultado es verdadero. Si a
0
= 1, entonces a = 2(a
k
a
k1
...a
2
a
1
) + 1,
f(a) = 2f(a = a
k
a
k1
...a
2
a
1
) +1 = 2(b
k
2
k1
+b
k1
2
k2
+... +b
2
2 +b
1
) +1 =
b
k
2
k
+b
k1
2
k1
+... +b
2
2
2
+b
1
2 +b
0
, y el resultado tambien es verdadero.
Esta es una na aplicaci on de la representaci on numerica. Notese la simpleza
de los c alculos: f(25) = f(11001
2
) = 11111
2
= 16 + 8 4 2 + 1 = 19.
En el siguiente ejemplo, usamos una representaci on especial que es producto de
la investigaci on y de la comprensi on de la una parte esencial de los n umeros
reales en el analisis avanzado.
3.4.6 Si K denota el subconjunto de [0, 1] que consiste de todos los n umeros
que teniendo una expansi on ternaria

n=1
a
n
3
n
en la cual a
n
= 0 o 2. Este es llamado el conjunto de Cantor. Probar que K
es el complemento de la uni on de intervalos abiertos disjuntos I
n
, n = 1, 2, 3, ...,
cuya suma de longitudes es 1.
Soluci on. Primero observece que ninguno de los n umeros del intervalo I
1
=
(
1
3
,
2
3
) esta en K. Esto es debido a que las representaciones ternarias de los
n umeros de este intervalo son de la forma
(,1a
2
a
3
a
4
...)
3
.
3.4. NOTACI

ON POSICIONAL 131
Similarmente, ninguno de los n umeros en el intervalo I
2
= (
1
9
,
2
9
) esta en K,
debido a que las representaciones ternarias de los n umeros de este intervalo son
de la forma
(,01a
3
a
4
a
5
...)
3
.
Figura 3.2.
Tambien, los n umeros en el intervalo I
3
= (
7
9
,
8
9
) tienen representaciones ternar-
ias de la forma
(,21a
3
a
4
a
5
...)
3
,
luego no estan en K. De la misma manera, ninguno de los intervalos I
4
=
(
1
27
,
2
27
), I
5
= (
19
27
,
20
27
), I
6
= (
7
27
,
8
27
), I
7
= (
25
27
,
26
27
) contienen elementos de K.
Esto representa que este proceso de extraer puede ser repetido sistematicamente.
La gura 3.2 y la tabla 3.1 ayudan a tener una idea precisa.
Para encontrar I
n
(es decir, X
n
y Y
n
) para un entero positivo arbitrario n,
escribimos n en notacion base dos:
n = (a
k
a
k1
...a
2
a
1
)
2
(es decir, n = 2
k1
a
k
+...+2a
2
+a
1
, a
i
= 0 o 1, a
k
,= 0), sea b
i
= 2a
i
, i = 1, 2, ..., k
y sea I
n
= (X
n
, Y
n
) donde
X
n
=
b
1
3
+
b
2
3
2
+... +
b
k1
3
k1
+
1
3
k
= (.b
1
b
2
...b
k
1)
3
,
Y
n
=
b
1
3
+
b
2
3
2
+... +
b
k1
3
k1
+
2
3
k
= (.b
1
b
2
...b
k
2)
3
.
Es facil ver X
n
y Y
n
son elementos de K para cada n (Notese que X
n
=
b1
3
+
b2
3
2
+ ... +
b
k1
3
k1
+

i=0
(
2
3
k+i
),) y que ninguno de los elementos de I
n
estan en
K (El k

esimo dgito de cada elemento de I


n
= (X
n
, Y
n
) es 1) De este hecho se
sigue que los I
n
s son disjuntos.
132 CAP

ITULO 3. ARITM

ETICA
Tabla 3.1. I
n
= (X
n
, Y
n
)
n n X
n
Y
n
I
n
(base 10) (base 2) (base 3) (base 3) (forma fraccional)
1 1 0,1 0,2 (
1
3
,
2
3
)
2 10 0,01 0,02 (
1
9
,
2
9
)
3 11 0,21 0,22 (
7
9
,
8
9
)
4 100 0,001 0,002 (
1
27
,
2
27
)
5 101 0,201 0,202 (
19
27
,
20
27
)
6 110 0,021 0,022 (
7
27
,
8
27
)
7 111 0,221 0,222 (
25
27
,
26
27
)
8 1000 0,0001 0,0002 (
1
81
,
2
81
)
9 1001 0,2001 0,2001 (
55
81
,
56
81
)
Tambien la longitud de los I
n
es 1/3
k
donde k = [[log
2
n]] , y de aqu

n=1
I
n
=

n=1
1
3
[[log
2
n]]+1
=

m=0
_
_
2
m+1
1

n=2
m
1
3
[[log
2
n]]+1
_
_
=

m=0
2
m
_
1
3
m+1
_
=
1
3

m=0
_
2
3
_
m
= 1.
Otra construcci on de los I
n
es clara a partir de que K es el resto despues de haber
removido los intervalos I
n
del intervalo [0, 1] , y el resultado ha sido probado.
Problemas
3.4.7 Probar que no existe un entero que es su doble cuando el dgito inicial
es pasado al nal.
3.4.8 Encontrar el n umero natural m as peque no n que tiene las siguientes
propiedades:
(i) su representaci on decimal tiene al 6 como ultimo dgito, y
(ii) si el ultimo dgito 6 es borrado y pasado al frente de los dem as dgitos, el
resultado es un n umero cuatro veces el original n.
3.4.9 .
3.4. NOTACI

ON POSICIONAL 133
(a) Resolver la siguiente ecuaci on para enteros positivos x y y :
(360 + 3x)
2
= 492y04.
(b) Derivar una prueba de divisibilidad para reconocer cuando un n umero es
divisible por 11. (Sugerencia: 10 1(mod11).)
(c) Si 62ab478 es un m ultiplo de 99, encontrar a y b.
(d) Encontrar la probabilidad de que si los dgitos 0, 1, 2, ..., 9 son colocados aleato-
riamente en los espacios vacos de 5 383 8 2 936 5 8 203 9 3 76, el resultado sea
un n umero divisible por 396.
3.4.10 Dada una balanza de dos platillos y un sistema de pesas de 1, 3, 3
2
, 3
3
, 3
4
, ...gramos,
probar que se puede pesar cualquier cantidad entera de gramos (el peso puede ser
colocado en cualquiera de los dos platillos). (Sugerencia: Probar que cualquier
entero positivo puede ser representado como sumas y restas de potencias de 3).
3.4.11 .
(a) El n umero 0,1234567891011121314..., que se obtiene de escribir sucesiva-
mente todos los n umeros enteros, representa un n umero racional?
(b) El n umero 0,011010100010100..., donde a
i
= 1 si n es primo, 0 en otros
casos, representa un n umero racional?
3.4.12 Sea S = a
0
a
1
a
2
..., donde a
n
= 0 si existe un n umero par de 1s en
la expresi on de n en base 2 y a
n
= 1 si existe un n umero impar de 1s. Luego
S = 01101001100.... Denir T = b
1
b
2
b
3
..., donde los b
i
es el n umero de unos
entre la ocurrencia del iesimo y el (i+1)esimo 0 en S. Probar que T contiene
solamente los tres smbolos 0,1,2.
3.4.13 Probar que existe una correspondencia uno-a-uno entre los puntos del
intervalo cerrado [0, 1] y los puntos del intervalo abierto (0, 1) . Dar una descrip-
ci on explcita de una tal correspondencia.
Ejemplos adicionales
1.1.1 (soluci on 5), 4.4.8, 5.2.5, 6.1.1, 6.1.4, 6.1.8, 6.2.13, 7.6.6.
134 CAP

ITULO 3. ARITM

ETICA
3.5. Aritmetica de los N umeros Complejos
Recordemos que un n umero complejo z puede escribirse de varias formas difer-
entes:
forma rectangular z = a +bi
forma polar z = r(cos +isen)
forma exponencial z = re
i
,
donde a, b, r y estan relacionados como en la Figura 3.3, y e
i
= cos +isen.
el angulo es el argumento de z (determinado como lo que sobra de un m ultiplo
de 2) y r es la magnitud (valor absoluto) de z; estos son denotados por arg z
y [z[ respectivamente.Los n umeros a y b son llamados la parte real y la parte
imaginaria de z respectivamente, y son denotados por Re(z) e Im(z).
Si z = a + bi y w = c + di, entonces z + w = a + c + (b + d)i correspondiendo
geometricamente a la diagonal del paralelogramo teniendo z y w como los lados
adyacentes (ver Figura 3.4).
Si z = re
i
y w = se
i
, entonces zw = rsre
i(+)
. Notese que [zw[ = rs =
[z[ [w[ y arg(zw) = + = arg z + arg w; es decir, para la multiplicaci on, la
multiplicaci on de los valores absolutos y la suma de los argumentos.
Figura 3.3.
3.5.1 Si a, b y n son enteros positivos, probar que existen enteros x y y tales
que
(a
2
+b
2
)
n
= x
2
+y
2
.
Soluci on. Si z = a +bi . Entonces (a
2
+b
2
)
n
= (

z
2

)
n
= [z[
2n
= ([z[
n
)
2
. Pero
z
n
= x + yi para algunos enteros x y y (debido a que a y b son enteros), luego
([z
n
[)
2
= [x +yi[
2
= x
2
+y
2
, y la prueba esta completa.
3.5. ARITM

ETICA DE LOS N

UMEROS COMPLEJOS 135


Figura 3.4.
3.5.2 Sea n un entero 3, y sean , , n umeros complejos tales que
n
=

n
=
n
= 1, + + = 0. Probar que n es un m ultiplo de 3.
Soluci on. Podemos asumir sin perdida de generalidad que = 1 (ya que si no,
al dividir cada lado de + + = 0 por se tiene 1 +

= 0, y entonces
tomamos
_
1,
1
=

,
1
=

_
satisface la condiciones del problema). Tambien
asumimos que 0 arg < arg < 2.
Ahora, y son de magnitud 1 (ya que
n
=
n
= 1), luego estan sobre el
crculo unitario (centro en (0, 0) y radio 1). De la ecuaci on + = 1, podemos
igualar las partes imaginarias, a saber Im( + ) = Im() + Im() = 0, o
equivalentemente, Im() = Im() (Figura 3.5). Igualmente en cuanto a las
partes reales tenemos Re()+Re() = 1 como tambien hemos establecido que
[[ = [[ = 1, se debe dar el caso de que Re() = Re() =
1
2
luego = e
i2/3
,
= e
i4/3
y el hecho de que
n
= 1 implica e
i2n/3
= 1, este caso solamente es
cierto cuando n es m ultiplo de 3.
Figura 3.5.
El siguiente resultado es muy usado; se puede probar por induccion.
136 CAP

ITULO 3. ARITM

ETICA
Teorema de Moivre. Para cada entero n,
(cos +isen)
n
= cos n +isenn.
(En notacion exponencial (e
i
)
n
= e
in
.)
3.5.3 Expresar cos 5 en terminos de cos .
Soluci on. Una manera eciente para esto es reconocer a cos 5 como la parte
real de e
i5
. Entonces podemos aplicar el teorema de Moivre:
cos 5 +isen5 = (cos +isen)
5
= cos
5
+ 5 cos
4
isen + 10 cos
3
i
2
sen
2

+ 10 cos
2
i
3
sen
3
+ 5 cos i
4
sen
4
+i
5
sen
5

= cos
5
10 cos
3
sen
2
+ 5 cos sen
4

+ i(sen
5
10sen
3
cos
2
+ 5sen cos
4
)
Igualando las partes real e imaginaria, tenemos
cos 5 = cos
5
10 cos
3
sen
2
+ 5 cos sen
4

sen5 = sen
5
10sen
3
cos
2
+ 5sen cos
4

Para el caso de cos 5,


cos 5 = cos
5
10 cos
3
(1 cos
2
) + 5 cos (1 cos
2
)
2
= 16 cos
5
20 cos
3
+ 5 cos .
3.5.4 Encontrar constantes a
0
, a
1
, ..., a
6
tales que
cos
6
= a
6
cos 6 +a
5
cos 5 +... +a
1
cos 2 +a
0
.
Soluci on. Como en el ultimo problema, podemos hacer esto vericando de
manera natural explorando las relaciones entre las funciones trigonometricas
(especialmente seno y coseno) y las variables complejas. En este caso, escribimos
cos en la forma
cos =
e
i
+e
i
2
3.5. ARITM

ETICA DE LOS N

UMEROS COMPLEJOS 137


y aplicamos el teorema del binomio para obtener
cos
6
=
_
e
i
+e
i
2
_
6
=
1
2
6
[
_
e
i
_
6
+ 6
_
e
i
_
5
_
e
i
_
+ 15
_
e
i
_
4
_
e
i
_
2
+ 20
_
e
i
_
3
_
e
i
_
3
+ 15
_
e
i
_
2
_
e
i
_
4
+ 6
_
e
i
_ _
e
i
_
5
+
_
e
i
_
6
]
=
1
2
6
[
_
e
6i
+e
6i
_
+ 6
_
e
4i
+e
4i
_
+ 15
_
e
2i
+e
2i
_
+ 20]
=
1
2
6
[2 cos 6 + 6 2 cos 4 + 15 2 cos 2 + 20]
=
1
32
[cos 6 + 6 cos 4 + 15 cos 2 + 10].
3.5.5 Sea G
n
= x
n
sennA + y
n
sennB + z
n
sennC, donde x, y, z, A, B, C son
n umeros reales y A + B + C es un m ultiplo entero de . Probar que si G
1
=
G
2
= 0, entonces G
n
= 0 para todo entero positivo n.
Soluci on. Un procedimiento estandar (similara al de 3.5.3) es reconocer que
G
n
es la parte imaginaria de la expresi on
H
n
= x
n
e
inA
+y
n
e
inB
+z
n
e
inC
.
Supongase que H
n
es real para n = 0, 1, ..., k y considerar H
k+1
. Tenemos que
H
1
H
k
= H
k+1
+H,
donde
H = xe
iA
y
k
e
ikB
+xe
iA
z
k
e
ikC
+ye
iB
x
k
e
ikA
+ ye
iB
z
k
e
ikC
+ze
iC
x
k
e
ikA
+ze
iC
y
k
e
ikB
= xye
i(A+B)
_
y
k1
e
i(k1)B
+x
k1
e
i(k1)A
_
+ xze
i(A+C)
_
z
k1
e
i(k1)C
+x
k1
e
i(k1)A
_
+ yze
i(B+C)
_
y
k1
e
i(k1)B
+z
k1
e
i(k1)C
_
= H
k1
_
xye
i(A+B)
+xze
i(A+C)
+yze
i(B+C)
_
xyze
i(A+B+C)
H
k2
= H
k1
k xyze
i(A+B+C)
H
k2
,
donde K = xye
i(A+B)
+xze
i(A+C)
+yze
i(B+C)
.
138 CAP

ITULO 3. ARITM

ETICA
Observese que H
2
= H
2
1
+ 2K, y como H
1
y H
2
, son reales, por hip otesis, se
tiene que K debe ser real. Tambien, por el supuesto inductivo H
k1
y H
k2
son
reales. Debido a que A+B+C es un m ultiplo de , xyze
i(A+B+C)
es real. Ahora
como H
k
es real, por la hip otesis inductiva, y como H
1
H
k
= H
k+1
+H, se sigue
que H
k+1
es real. Luego, el resultado se sigue por induccion matematica.
Problemas
3.5.6 .
(a) Dado que 13 = 2
2
+ 3
2
y 74 = 5
2
+ 7
2
, expresar 13 74 = 962 como una
suma de dos cuadrados. (Sugerencia: Sea z = 2 + 3i y w = 5 + 7i, y usar que

z
2

w
2

= [zw[
2
.)
(b) Probar que 4 arctan
1
5
arctan
1
239
=
1
4
. (Sugerencia: Considerar (5i)
4
(1+
i).)
3.5.7 Sup ongase que A es un n umero complejo y n un entero positivo tal que
A
n
= 1 y (A+ 1)
n
= 1. Probar que n es divisible por 6 y que A
3
= 1.
3.5.8 Probar que
_
n
1
_

_
n
3
_
+
_
n
5
_

_
n
7
_
+... = 2
n
2
cos
n
4
y
_
n
0
_

_
n
2
_
+
_
n
4
_

_
n
6
_
+... = 2
n
2
sen
n
4
(Sugerencia: considerar (1 +i)
n
.)
3.5.9 Considerando todas las posibles magnitudes y argumentos
(a) encontrar todos los valores de
3

i;
(b) encontrar los valores de (3 4i)
3
8
que est an sobre el eje imaginario.
3.5.10 .
(a) Probar que si z = e
i
, entonces z z
1
= 2isen y z
n
z
n
= 2isenn.
3.5. ARITM

ETICA DE LOS N

UMEROS COMPLEJOS 139


(b) Usando la parte (a), expresar sen
2n
como una suma de senos cuyos angulos
son m ultiplos de .
3.5.11 Probar que
tan n =
_
n
1
_
tan
_
n
3
_
tan
3
+...
_
n
0
_

_
n
2
_
tan
2
+...
.
3.5.12 .
(a) Probar que

k=0
(1)
k+1
_
n
k
_
cos k
cos
k

=
_
0 Si n es impar,
(1)
1+n/2
tan
n
Si n es par
(Sugerencia: Considerar i tan = 1 + (cos +isen)/ cos .)
(b) Probar que

k=0
(1)
k1
_
n
k
_
cos
k
cos k =
_
(1)
(n+1)/2
sen
n
senn Si n es impar,
(1)
1+n/2
sen
n
cos n Si n es par.
(Sugerencia: Considerar 1 + cos [cos +isen] = isen [cos v +isen] .)
3.5.13 Probar que
cos n
cos
n

= 1
_
n
2
_
tan
2
+
_
n
4
_
tan
4
....
3.5.14 Probar que si e
i
satisface la ecuaci on z
n
+a
n1
z
n1
+...+a
1
z+a
0
= 0,
donde los a
i
son reales, entonces a
n1
sen +a
n2
sen2 +... +a
1
sen(n1) +
a
0
senn = 0.
Ejemplos adicionales
1.3.2, 4.2.10, 4.2.11, 4.2.13, 4.2.15, 4.2.17, 4.2.20, 4.2.22, 4.3.18, 5.2.2, 5.2.3,
5.2.11, 5.3.4, 5.3.10, 5.4.11, 5.4.28, 5.4.29. Tambien, ver la Seccion 8.4 (Ge-
ometra en los N umeros Complejos).
140 CAP

ITULO 3. ARITM

ETICA
Captulo 4
Algebra
El

Algebra es uno de los temas m as antiguos de las matematicas, adem as es una
de las areas que se encuentra m as activa en la ense nanza de la matematica.
En el

Algebra de secundaria aprendemos a manipular ecuaciones y formulas
en forma de equivalencias que son m as faciles de entender e interpretar. Una
gran porci on de problemas de este libro hacen uso de estos conceptos basicos.
Una de las manipulaciones algebraicas m as importantes es la factorizaci on de
expresiones algebraicas. En la primera seccion de este captulo veremos algunas
factorizaciones elementales.
Las siguientes dos secciones estar an dedicadas al algebra clasica: llamese, el es-
tudio de polinomios. Mucho de este material es desrrollado en una parte del
algebra llamada teora de ecuaciones. Los rudimentos de estos conceptos se en-
cuentran dispersos en la currcula de secundaria y bachillerato. En esta seccion
daremos algunas ideas de como usar estos conocimientos para resolver proble-
mas.
En la seccion nal introduciremos los topicos en los que piensa un matematico
profesional cuando escucha hablar de algebra. Aqu enfatizamos sobre los sis-
temas formales y el pensamiento formal. Los conocimientos contienen un mundo
nuevo de conceptos que generalizan las ideas clasicas y metodos. Introducimos
las estructuras m as fundamentales que forman el conocimiento de esta materia:
grupos, anillos, y campos.
141
142 CAP

ITULO 4. ALGEBRA
4.1. Identidades Algebraicas
En esta seccion veremos la aplicaci on de algunas de las fomulas basicas de
factorizaci on, en las cuales incluimos las siguientes:
a
2
b
2
= (a b)(a +b)
a
2
+ 2ab +b
2
= (a +b)
2
a
2
+b
2
+c
2
+ 2ab + 2ac + 2bc = (a +b +c)
2
a
n
b
n
= (a b)(a
n1
+a
n2
b +a
n3
b
2
+... +ab
n2
+b
n1
Si n es entero positivo impar, cuando reemplazamos b por b en la ultima formu-
la obtenemos una formula para la factorizaci on de una suma de dos potencias
n-esimas perfectas:
a
n
+b
n
= (a +b)(a
n1
a
n2
b +a
n3
b
2
... ab
n2
+b
n1
, n impar
4.1.1 Probar que n
4
20n
2
+4 es un n umero compuesto cuando n es cualquier
entero.
Soluci on . La idea de la demostracion de esta armaci on es factorizar la expre-
sion. Procedamos n
4
20n
2
+ 4 = (n
4
20n
2
+ 100) 96 = (n
2
10)
2
96, lo
cual, debido a que 96 no es un cuadrado perfecto, no se puede factorizar, as que,
n
4
20n
2
+4 = (n
4
4n
2
+4)16n
2
= (n
2
2)
2
(4n)
2
= (n
2
24n)(n
2
2+4n).
Si logramos probar que ninguno de estos dos factores es 1 o -1 habremos con-
cluido la prueba.
Supongamos que n
2
2 4n = 1; o equivalentemente, n
2
4n 3 = 0. Luego
por formula general para la soluci on de ecuaciones de segundo grado tenemos
que, n = 2

7, que no es entero. Luego, si n es un entero, n


2
2 4n no es
igual a 1. Usando argumentos similares se prueban los otros tres casos.
4.1.2 Determinar todas las soluciones de n umeros reales x, y, z, w del sistema
x +y +z = w,
1
x
+
1
y
+
1
z
=
1
w
.
Soluci on. Una conjetura inicial es que las soluciones son aquellos n umeros
reales x, y, z para los cuales uno de ellos es igual a w y los otros dos son inversos
4.1. IDENTIDADES ALGEBRAICAS 143
aditivos entre s. Ciertamente, estas son las soluciones, pero ahora debemos
probar que no hay otras.
De la segunda ecuaci on,
1
x
+
1
y
+
1
z
=
yz +xz +xy
xyz
=
1
w
,
y esto, al multiplicarlo por la primera ecuaci on tenemos
(yz +xz +xy)(x +y +z) = xyz,
Esta se expande a
x
2
y +x
2
z +xy
2
+z
2
x +z
2
y +y
2
z + 2xyz = 0.
cuya factorizaci on es
(y +z)(x +y)(x +z) = 0
De donde tenemos que la conjetura inicial es verdadera (es decir, uno de y +
z, x +y, x +z es igual a cero, veamos z = y y luego x = x +y +z = w).
4.1.3 .
(a) Encontrar todas las parejas (m, n) de enteros positivos tales que [ 3
m

2
n
[= 1
(b) Encontrar todos los pares (m, n) de enteros mayores que 1 tales que [
p
m
q
n
[= 1 donde p y q son primos.
Soluci on. (a) Cuando m = 1 o 2 tenemos que
(m, n) = (1, 1), (1, 2), (2, 3)
Probemos que no existen otras.
Supongamos que (m, n) es soluci on de [ 3
m
2
n
[, cuando m > 2 (y por supuesto
n > 3). Entonces 3
m
2
n
= 1 o 3
m
2
n
= 1.
Caso 1. Supongamos que 3
m
2
n
= 1, n > 3. Entonces 3
m
1 (mod 8).
Pero esta congruencia no puede darse, pues 3
m
1 o 3 (mod 8), dependiendo
de si m es par o impar (3 3 (mod 8), 3
2
1 (mod 8), 3
3
3 (mod 8), 3
4
1
(mod 8), ...).
Caso2. Supongamos 3
m
2
n
= 1, n > 3. Entonces 3
m
1 (mod 8). de donde
m es par, es decir, m = 2k, k > 1. Entonces 2
n
= 3
2k
1 = (3
k
1)(3
k
+ 1).
Por factorizaci on unica, 2
r
= 3
k
+ 1 para alg un r > 3. Pero por caso 1 esto no
144 CAP

ITULO 4. ALGEBRA
puede ser. De donde hemos completado la prueba de (a). (b) Es inmediato que
si ambos p y q son impares, entonces p
m
q
n
es par. Supongamos pues que q = 2
Probaremos, usando solamente las identidades algebraicas de esta seccion, que
la unica soluci on es la encontrada en la seccion (a), a saber [ 3
2
2
3
[= 1.
Supongamos que m y n son mayores que 1, y que [p
m
2
n
[= 1. Es imposible
el que m y n sean ambos pares, pues m = 2r y n = 2s implica que
1 =[ p
m
2
n
[=[ p
2r
2
2s
[=[ p
r
2
s
[[ p
r
+ 2
s
[,
lo cual es una contradiccion ( [ p
r
+ 2
s
[> 1.)
Supongamos pues que m es impar. Entonces
2
n
= p
m
1 = (p 1)(p
m1
p
m2
+... p + 1)
y esto es imposible ya que el ultimo factor del lado derecho de la ecuaci on es un
n umero impar mayor que uno.
Luego, solo queda el caso en que n sea impar y m par.
Supongamos m = 2
r
k , donde k es impar y supongamos k > 1. Entonces
2
n
= p
m
1 = (p
2
r
)
k
1 = (p
2
r
1)((p
2
r
)
k1
... (p
2
r
) + 1)
y dado que el factor del lado derecho es impar tenemos una contradiccion.
De aqu m = 2
r
para alg un entero positivo r y n es impar, y la ecuaci on tiene
la forma [ p
m
2
n
[= 1. Consideremos p
2
r
2
n
= 1, o p
2
r
2
n
= 1.
Caso 1 Si p
2
r
2
n
= 1, entonces
p
2
r
= 2
n
1 = (2 1)(2
n1
+ 2
n2
+... + 2 + 1) 3 (mod 4),
lo cual es imposible , ya que x
2
1(mod4).
Caso 2 Si p
2
r
2
n
= 1, entonces 2
n
= p
2
r
1 = (p
2
r1
+1)(p
2
r1
1)1. Solamente
es cierto cuando p
2
r
+1 y p
2
r
1 son potencias consecutivas de 2 luego p
2
r
1 = 2
y p
2
r
+ 1 = 4 de donde p
2
r
= 3, y esto implica p = 3, r = 1, m = 2 y n = 3, Lo
cual completa la prueba.
4.1.4 Probar que no existen primos en la sucesi on innita de n umeros enteros
10001, 100010001, 1000100010001, ....
4.1. IDENTIDADES ALGEBRAICAS 145
Soluci on. Los terminos de la sucesion pueden ser escritos como
1 + 10
4
, 1 + 10
4
+ 10
8
, ..., 1 + 10
4
+... + 10
4n
, ....
consideremos, mas generalmente, entonces, la sucesion
1 +x
4
, 1 +x
4
+x
8
, ..., 1 +x
4
+... +x
4n
, ....
para cualquier entero arbitrario x, x > 1.
Si n es impar, tomamos n = 2m+ 1,
1 +x
4
+x
8
... +x
4(2m+1)
=
= (1 +x
4
) +x
8
(1 +x
4
) +... +x
8m
(1 +x
4
)
= (1 +x
4
)(1 +x
8
+... +x
8m
).
Luego si m > 0, el n umero es compuesto. Para m = 0 y x = 10, tambien
tenemos un n umero compuesto, como 10001 = 73 137.
Supongamos que n es par, luego n = 2m. Entonces
1 +x
4
+x
8
... +x
4(2m)
=
1 (x
4
)
2m+1
1 x
4
= (
1 (x
2m+1
)
2
1 x
2
)(
1 + (x
2m+1
)
2
1 +x
2
)
= (1 +x
2
+... + (x
2
)
2m
)
(1 x
2
+... + (x
2
)
2m
).
Esta factorizaci on prueba que el n umero es compuesto.
Problemas
4.1.5 .
(a) Si a y b son enteros consecutivos, probar que a
2
+b
2
+(ab)
2
es un cuadrado
perfecto.
146 CAP

ITULO 4. ALGEBRA
(b) Si 2a es la media arm onica de b y c (es decir, 2a =
2
1
b
+
1
c
), probar que la
suma de los cuadrados de los tres n umeros, a, b y c es el cuadrado de un
n umero racional.
(c) Si N es un n umero entre dos cuadrados sucesivos y sus diferencias con
ellos las denotamos por x y y respectivamente, probar que N xy es un
cuadrado.
4.1.6 Probar que existe una innidad de n umeros naturales a con la siguiente
propiedad: El n umero n
4
+a no es primo para cualquier valor del n umero natural
n.
4.1.7 Suponiendo que un entero n es la suma de dos n umeros triangulares,
n =
a
2
+a
2
+
b
2
+b
2
,
escribir 4n + 1 como la suma de dos cuadrados, 4n + 1 = x
2
+y
2
, y probar que
x y y pueden ser expresados en terminos de a y b. Probar que, inversamente, si
4n + 1 = x
2
+y
2
, entonces n es la suma de dos n umeros triangulares.
4.1.8 Sea N el n umero que expresado en notaci on decimal consistente de 91
unos:
N = 111..,11.
Probar que N es un n umero compuesto.
4.1.9 Probar que cualesquiera dos n umeros de la siguiente sucesi on son primos
relativos:
2 + 1, 2
2
+ 1, 2
4
+ 1, 2
8
+ 1, ..., 2
2
n
+ 1, ....
Probar que este resultado prueba que existe una innidad de n umeros primos.
4.1.10 Determinar todas las ternas de enteros (x, y, z) que satisfacen la ecuaci on
x
3
+y
3
+z
3
= (x +y +z)
3
.
4.2. FACTORIZACI

ON UNICA DE POLINOMIOS 147


Ejemplos adicionales
1.8.4, 1.12.7, 3.3.6, 6.2.5, 5.2.15, 5.3.7, 7.1.11. Tambien, ver Seccion 5.2 (Series
Geometricas).
4.2. Factorizacion Unica de Polinomios
Un polinomio de grado n (n es un entero no negativo) en una variable x es una
expresi on de la forma
a
n
x
n
+a
n1
x
n1
+... +a
1
x +a
0
,
donde a
0
, a
1
, ..., a
n
son constantes (llamados coecientes), y a
n
,=?0. Un poli-
nomio en donde todos los coecientes son cero se llama polinomio cero: a este
polinomio no se le asigna grado. El coeciente a
n
es llamado el coeciente prin-
cipal ; si es igual a 1 decimos que el polinomio es un polinomio m onico. Dos
polinomios son llamados (identicamente) iguales si sus coecientes son iguales
termino a termino, es decir, sus coecientes para las mismas potencias de las
variables son iguales.
Si los coecientes del polinomio P(x) son enteros, decimos que P(x) es un poli-
nomio sobre los enteros; similarmente si los coecientes son racionales decimos
que es un polinomio sobre los racionales, y as sucesivamente.
En muchos aspectos los polinomios se comportan como los enteros. Pueden
ser sumados, restados, y multiplicados; adem as, justamente como en el caso
de los enteros, cuando dividimos un polinomio entre otro el resultado es un
polinomio cociente adem as de un polinomio residuo. Un polinomio F divide a
un polinomio G (exactamente) si existe un polinomio Q tal que G = FQ (es
decir, G es m ultiplo de F). Un polinomio H es el m aximo com un divisor de los
polinomios F y G si y solamente si (1) H divide a F y G y (2) si K es cualquier
otro polinomio que divide a F y G, entonces K divide a H. Se puede probar
que H es unico salvo m ultiplos constantes.
Tambien como en el caso de los enteros existe el algoritmo de la divisi on.
Algoritmo de la Divisi on para Polinomios. Si F(x) y G(x) son poli-
nomios sobre el campo K (por ejemplo K puede ser el conjunto de los n umeros
racionales, los reales, los complejos, los enteros m odulo p para p primo), existen
polinomios unicos Q(x) y R(x) sobre el campo K tales que
148 CAP

ITULO 4. ALGEBRA
F(x) = G(x)Q(x) +R(x)
donde R(x) 0 o grad(R(x)) < grad(G(x)). (grad denota el grado)
Adem as, si K es un dominio entero (tal como el conjunto de los enteros), el
mismo resultado es probado cuando G(x) es un polinomio m onico.
Como un ejemplo del algoritmo de la divisi on para polinomios, sea F(x) =
3x
5
+ 2x
2
5 y G(x) = 2x
3
+ 6x + 3. Entonces
F(x) = 3x
5
+ 2x
2
5 = (G(x) = 2x
3
+ 6x + 3)(
3
2
x
2

9
2
) +
1
2
x
2
+ 27x
1
2
En este caso, Q(x) =
3
2
x
2

9
2
, y R(x) =
1
2
x
2
+ 27x
1
2
. (Este ejemplo ilustra
claramente como se trabaja el algoritmo de la divisi on en casos generales sola-
mente tomando los coecientes en un campo; adem as es claro que en un dominio
entero es suciente trabajar con ellos cuando el polinomio es m onico.) Como en
el caso de los enteros, el algoritmo de la divisi on sirve para encontrar el m aximo
com un divisor entre dos polinomios. Mas aun, como en el caso de los enteros, si
F y G son polinomios (sobre el campo K), existen polinomios S y T tales que
mcd(F, G) = SF +TG,
donde mcd(F, G) denota el m aximo com un divisor de F y G.
4.2.1 Encontrar un polinomio P(x) tal que P(x) es divisible por x
2
+ 1 y
P(x) + 1 es divisible por x
3
+x
2
+ 1.
Soluci on . Por las condiciones del problema, existen polinomios S(x) y T(x)
tales que
P(x) = (x
2
+ 1)S(x),
P(x) + 1 = (x
3
+x
2
+ 1)T(x).
Se sigue que (x
2
+ 1)S(x) = (x
3
+x
2
+ 1)T(x) 1, o equivalentemente
(x
3
+x
2
+ 1)T(x) (x
2
+ 1)S(x) = 1.
Por la observaci on al ejemplo precedente, (x
3
+x
2
+ 1) y (x
2
+ 1) son primos
relativos y podemos encontrar usando el algoritmo de Euclides los polinomios
S(x) y T(x). Luego, tenemos
(x
3
+x
2
+ 1) = (x + 1)(x
2
+ 1) + (x),
x
2
+ 1 = x(x) + 1,
4.2. FACTORIZACI

ON UNICA DE POLINOMIOS 149


y trabajando hacia atr as, tenemos
1 = (x
2
+ 1) +x(x)
= (x
2
+ 1) +x[(x
3
+x
2
+ 1) (x + 1)(x
2
+ 1)]
= (x
2
+ 1)[1 x(x + 1)] +x[x
3
+x
2
+ 1]
= [x
3
+x
2
+ 1]x (x
2
+ 1)(x
2
+x 1).
De esta forma, encontramos que tomando los polinomios S(x) = (x
2
+x 1) y
T(x) = x. Se sigue que
P(x) = (x
2
+ 1)(x
2
+x 1).
4.2.2 Probar que la fracci on (n
3
+2n)/(n
4
+3n
2
+1) es irreducible para cada
n umero natural n.
Soluci on .Tenemos
(n
4
+ 3n
2
+ 1) = n(n
3
+ 2n) + (n
2
+ 1),
(n
3
+ 2n) = n(n
2
+ 1) +n,
(n
2
+ 1) = n(n) + 1,
n = n(1).
Se sigue que mcd(n
3
+ 2n, n
4
+ 3n
2
+ 1) = 1, y la prueba esta completa. Sea
F(x) un polinomio sobre un dominio entero D , y consideremos la ecuaci on
polinomial F(x) = 0. Si un elemento a de D es tal que F(a) = 0, decimos que a
es una raiz de F(x) = 0 , o que a es un cero de F(x). El siguiente es un teorema
muy usado y es una aplicaci on facil del algoritmo de la divisi on.
Teorema del Factor. Si F(x) es un polinomio sobre un dominio entero D, un
elemento a de D es una raiz de F(x) = 0 si y solamente si x a es un factor
de F(x).
Mediante la aplicaci on repetida del teorema del factor, podemos probar que
existe un unico entero no negativo m y un unico polinomio G(x) sobre D tales
que
150 CAP

ITULO 4. ALGEBRA
F(x) = (x a)
m
G(x),
donde G(a) ,= 0 . En este caso decimos que a es un cero de multiplicidad m.
El siguiente es un ejemplo ilustrativo del teorema del factor.
4.2.3 Dado el polinomio F(x) = x
n
+ a
n1
x
n1
+ ... + a
1
x + a
0
con coe-
cientes enteros a
0
, a
1
, ..., a
n1
, y dado tambien que existen cua- tro distintos
enteros a, b, c, d tales que F(a) = F(b) = F(c) = F(d) = 5, probar que no existe
un entero k tal que F(k) = 8.
Soluci on . Sea G(x) = F(x) 5. Por el teorema del factor, x a, x b, x c,
y x d son factores de G(x), y podemos escribir
G(x) = (x a)(x b)(x c)(x d)H(x),
donde H(x) es un polinomio de coecientes enteros. Si k es un entero tal que
F(k) = 8, entonces G(k) = F(k) 5 = 8 5 = 3, o equivalentemente,
(k a)(k b)(k c)(k d)H(k) = 3.
El lado izquierdo representa el producto de 5 enteros, y cada uno de los enteros
k a, k b, k c, k d deben ser distintos, pues a, b, c, d son distintos. Pero
esto es imposible, ya que a lo m as uno de los n umeros k a, k b, k c, k d
puede ser igual a 3 y los otros tres deber an ser 1. Luego, tal k no puede ser
encontrada.
4.2.4 Probar que si F(x) es un polinomio con coecientes enteros, y existe un
entero k tal que ninguno de los enteros F(1), F(2), ..., F(k) es divisible por k,
entonces F(x) no tiene ceros enteros.
Soluci on . Probar lo anterior es equivalente a probar que si F(x) tiene un cero
entero, digamos r, entonces para cualquier k entero positivo, al menos uno de
los F(1), F(2), ..., F(k) es divisible por k. Supongamos pues que F(r) = 0. Por
el teorema del factor, podemos escribir
F(x) = (x r)G(x),
4.2. FACTORIZACI

ON UNICA DE POLINOMIOS 151


donde G(x) es un polinomio con coecientes enteros. Del algoritmo de la divisi on
para enteros, existen enteros q y s tales que r = qk + s, 0 < s k (note las
desigualdades para s). Sustituyendo s = r qk en la ecuaci on anterior, tenemos
F(s) = (s r)G(s) = qkG(s).
Esta ecuaci on prueba que k divide a F(s) (G(s) es un entero), lo cual completa
la prueba.
Una prueba simple de este teorema esta basada en la aritmetica modular,
observese que si a b (mod k) entonces F(a) F(b) (mod k). El resulta-
do se sigue directamente del hecho de que para cualquier entero dado a, F(a)
es congruente con alguno de los F(1), F(2), ..., F(k) m odulo k, y por hip otesis
ninguno de estos es divisible por k.
El teorema de factorizaci on unica para los enteros establece que cada n umero
entero puede escribirse como un producto unico de n umeros primos. Existe un
teorema similar para polinomios: cada polinomio sobre un campo puede ser
escrito de manera unica como un producto de polinomios irreducibles (es decir,
factores primos). En el caso de los n umeros complejos, los factores irreducibles
son polinomios de primer grado (lineales). En el caso de los n umeros reales, los
polinomios irreducibles son los polinomios de primer grado y los de segundo
grado con discriminante negativo (esto es, son de la forma ax
2
+bx +c, donde
b
2
4ac < 0).
Como en el caso de los enteros, el teorema de factorizaci on unica proporciona
una forma de representar los polinomios. Los siguientes dos ejemplos ilustran la
idea.
4.2.5 Probar que cada polinomio sobre los n umeros complejos tiene un poli-
nomio m ultiplo no cero cuyos exponentes son todos divisibles por 1, 000, 000.
Soluci on . Sea el polinomio dado representado por factorizaci on unica
P(x) = A(x s
1
)
m1
(x s
2
)
m2
(x s
k
)
m
k
donde A es una constante, s
1
, s
2
, ..., s
k
son las raices de P(x) de multiplicidades
m
1
, m
2
, ..., m
k
respectivamente. Para cualquier entero positivo a (por ejemplo
a = 1, 000, 000),(x
a
s
a
i
)/(x s
i
) es un polinomio sobre los n umeros complejos
(ver seccion 4.1). Tomemos
Q(x) = x
a
(
x
a
s
a
1
x s
1
)
m1
(
x
a
s
a
k
x s
k
)
m
k
.
152 CAP

ITULO 4. ALGEBRA
Entonces Q(x) es un polinomio sobre los n umeros complejos, y
P(x)Q(x) = A(x s
1
)
m1
(x s
k
)
m
k
x
a
(
(x
a
s
a
1
)
m
1
(xs1)
m
1
(
(x
a
s
a
k
)
m
k
(xs
k
)
m
k
))
= Ax
a
(x
a
s
a
1
)
m1
(x
a
s
a
k
)
m
k
es un polinomio donde todos sus exponentes son divisibles por a.
4.2.6 Sea f un polinomio con coecientes reales. Entonces todos los ceros de f
son reales si y solamente si f
2
no puede escribirse como la suma de cuadrados
f
2
= g
2
+h
2
donde g y h son polinomios con coecientes reales y grad(g) ,= grad(h).
Soluci on . Supongamos que f
2
= g
2
+ h
2
, donde g y h son polinomios con
coecientes reales y grad(g) ,= grad(h), y supongamos que todos los ceros de f
son reales. Escribiendo f en forma factorizada:
f(x) = A(x a
1
)
m1
(x a
2
)
m2
... (x a
k
)
m
k
,
donde A es un n umero real distinto de cero.
De la ecuaci on
A
2
(x a
1
)
2m1
(x a
2
)
2m2
... (x a
k
)
2m
k
= (g(x))
2
+ (h(x))
2
se sigue que para cada i = 1, 2, ..., k,
0 = (g(a
i
))
2
+ (h(a
i
))
2
.
Como g(a
i
) y h(a
i
) son ambos n umeros reales, se tiene que g(a
i
) = 0 = h(a
i
).
En particular se sigue que cada uno de los a
i
es un cero de multiplicidad al
menos m
i
. Luego el teorema del factor implica la existencia de polinomios g
1
(x)
y h
1
(x) con coecientes reales tales que g(x) = f(x)g
1
(x) y h(x) = f(x)h
1
(x).
Se sigue que
4.2. FACTORIZACI

ON UNICA DE POLINOMIOS 153


1 = (g
1
(x))
2
+ (h
1
(x))
2
.
Pero esta ecuaci on es imposible, ya que grad(g
1
) ,= grad(h
1
) (esto es, g
1
y h
1
no son ambos constantes). Esta contradiccion implica que f tiene al menos un
cero no real.
Ahora supongamos que no todos los ceros de f son n umeros reales, y escribamos
f en forma factorizada:
f(x) = A(x a
1
)
m1
(x a
2
)
m2
... (x a
k
)
mr
(x
2
+b
1
x +c
1
)
n1
... (x
2
+b
s
x +c
s
)
ns
,
donde A es un n umero real, m
1
, m
2
, ..., m
r
son enteros no negativos, s es un
entero positivo y n
1
, n
2
, ..., n
s
son enteros positivos, a
i
, b
j
, c
j
son n umeros reales,
y b
2
j
4c
j
< 0 para j = 1, ..., s. Tenemos
x
2
+b
j
x +c
j
= (x
2
+b
j
x +
1
4
b
2
j
) + (c
j

1
4
b
j
)
= (x +
1
2
b
j
)
2
+ (
1
2
_
4c
j
b
j
)
2
esto ultimo prueba que cada factor cuadratico de f es una suma de cuadra-
dos. Sustituyendo cada factor cuadratico en la factorizaci on unica de f
2
por su
representaci on como suma de cuadrados. Obteniendo una ecuaci on de la forma
f
2
(x) = A
2
(x a
1
)
2m1
(x a
2
)
2m2
... (x a
k
)
2mr

((g
1
(x))
2
+ (h
1
(x))
2
)
n1
... ((g
s
(x))
2
+ (h
s
(x))
2
)
ns
donde g
1
, ..., g
s
, h
1
, ..., h
s
son polinomios, con grad(g
i
) = 1, y grad(h
i
) = 0.
El resultado se sigue usando repetidamente el hecho de que el producto de una
suma de dos cuadrados con otra suma de dos cuadrados es una suma de dos
cuadrados:
(f
2
+g
2
)(h
2
+k
2
) = (fh gk)
2
+ (fk +gh)
2
.
Tambien, en esta identidad, si grad(f) > grad(g) y grad(h) > grad(k), entonces
154 CAP

ITULO 4. ALGEBRA
grad(fh gk) > grad(fk + gh). Luego, vemos que existen polinomios g(x) y
h(x) con coecientes reales, grad(g) ,= grad(h) tales que f
2
= g
2
+h
2
.
Problemas
4.2.7 Encontrar los polinomios F(x) y G(x) tales que
(x
8
1)F(x) + (x
5
1)G(x) = x 1.
4.2.8 Cu al es el m aximo com un divisor de x
n
1 y x
m
1?.
4.2.9 Sea f(x) un polinomio que tiene residuo A cuando se divide por xa y
residuo B cuando se divide por x b, a ,= b. Encontrar el residuo cuando f(x)
se divide por (x a)(x b).
4.2.10 Probar que x
4a
+ x
4b+1
+ x
4c+2
+ x
4d+3
, a, b, c, d enteros positivos, es
divisible por x
3
+x
2
+x + 1. (Sugerencia: x
3
+x
2
+x + 1 = (x
2
+ 1)(x + 1).)
4.2.11 Probar que cada polinomio (cos+xsen)
n
cosnxsenn es divisible
por x
2
+ 1.
4.2.12 Para cada n el polinomio 1 + x
2
+ x
4
+ ... + x
2n2
es divisible por el
polinomio 1 +x +x
2
+... +x
n1
?
4.2.13 Un n umero real es llamado algebraico si es un cero de un polinomio
con coecientes enteros.
(a) Probar que

2 +

3 es algebraico.
(b) Probar que cos

2n
es albebraico para cada entero positivo n. (Sugerencia:
Usar el teorema de Moivre al expresar cosnx como un polinomio en cosx.)
4.2. FACTORIZACI

ON UNICA DE POLINOMIOS 155


4.2.14 Si P(x) es un polinomio m onico con coecientes enteros y k es cualquier
entero, probar que existe un entero m para el cual existen al menos k divisores
primos distintos de P(m) (Sugerencia: Primero prueba, por inducci on, que ex-
isten k primos distintos q
1
, q
2
, ..., q
k
y k enteros n
1
, n
2
, ..., n
k
tales que q
i
divide
a P(n
i
) para i = 1, 2, ..., k. Entonces prueba que un primo q divide a P(n) si y
solamente si q divide a P(n +sq) para todo entero s. Una respuesta armativa
se sigue de este hecho adem as de una aplicaci on del teorema chino del residuo.)
4.2.15 .
(a) Factorizar x
8
+x
4
+1 en factores irreducibles (i) sobre los racionales, (ii)
sobre los reales, (iii) sobre los complejos.
(b) Factorizar x
n
1 sobre los n umeros complejos.
(c) Factorizar x
4
2x
3
+ 6x
2
+ 22x + 13 sobre los n umeros complejos, dado
que 2 + 3i es un cero.
4.2.16 Enseguida dos resultados que son usados en factorizaci on de poli-
nomios con coecientes enteros e irreducibles.
Teorema de las raices racionales. Si P(x) = a
n
x
n
+a
n1
x
n1
+...+a
1
x+a
0
es un polinomio con coecientes enteros, y si el n umero racional r/s (r y s son
primos relativos) es una raiz de P(x) = 0, entonces r divide a a
0
y s divide a
a
n
.
Lema de Gauss Sea P(x) un polinomio con coecientes enteros . Si P(x) puede
ser factorizado en un producto de dos polinomios con coecientes racionales,
entonces P(x) puede ser factorizado como un producto de dos polinomios con
coecientes enteros.
(a) Sea f(x) = a
n
x
n
+a
n1
x
n1
+... +a
1
x+a
0
un polinomio de grado n con
coecientes enteros. Si a
n
, a
0
y f(1) son impares, probar que f(x) = 0 no
tiene raices racionales.
(b) Para que enteros a x
2
x +a divide a x
13
+x + 90?
4.2.17 (a) Sup ongase que f(x) es un polinomio sobre los n umeros reales y
que g(x) es un divisor de f(x) y de f

(x) que es irreducible con la segunda


derivada. Probar que (g(x))
2
divide a f(x).(Este hecho puede ser usado
para checar las raices m ultiples de f(x).)
(b) Usar la idea de la parte (a) para factorizar x
6
+x
4
+ 3x
2
+ 2x + 2 en un
producto de polinomios irreducibles sobre los n umeros complejos.
156 CAP

ITULO 4. ALGEBRA
4.2.18 Determinar todos los pares de enteros positivos (m, n) tales que 1 +
x
n
+x
2n
+... +x
mn
es divisible por 1 +x +x
2
+... +x
m
.
4.2.19 .
(a) Sea F(x) un polinomio sobre los n umeros reales. Probar que a es un cero
de multiplicidad m si y solamente si F(a) = F

(a) = ... = F
(m)
(a) = 0 y
F
(m+1)
(a) ,= 0.
(b) La ecuaci on f(x) = x
n
nx +n 1 = 0, n > 1, es satisfecha para x = 1.
Cu al es la multiplicidad de esta raiz?
4.2.20 Si n > 1, probar que (x +1)
n
x
n
1 = 0 tiene una raiz m ultiple si y
solamente si n 1 es divisible por 6.
4.2.21 Sea P(x) un polinomio con coecientes reales, y asumamos que P(x)
0 para toda x. Probar que P(x) puede ser expresado en la forma (Q
1
(x))
2
+
(Q
2
(x))
2
+ ... + (Q
n
(x))
2
donde (Q
1
(x)), (Q
2
(x)), ..., (Q
n
(x)) son polinomios
con coecientes reales.
4.2.22 .
(a) Sea = cos(2/n) +isen(2/n). Probar que
x
n1
+x
n2s
+... +x + 1 = (x )(x
2
) (x
n1
)
(b) Sea x = 1 y tomar el valor absoluto de cada lado para probar que
sen

n
sen
2
n
sen
(n 1)
n
=
n
2
n1
.
Ejemplos adicionales
1.12.2, 1.12.5, 6.5.13, 6.9.3.
4.3. El Teorema de Identidad
Sea P un polinomio no identicamente cero de grado n sobre un dominio entero
D. Recordando el teorema del factor, si a es una raiz de P(x) = 0, existe un
4.3. EL TEOREMA DE IDENTIDAD 157
polinomio Q de grado n1 tal que P(x) = (x a)Q(x). Usando este hecho, un
argumento sencillo de induccion prueba que P tiene a lo m as n ceros.
La observaci on anterior tiene un importante corolario. Supongamos que F y
G son polinomios sobre un dominio D con grados menores o iguales que n, y
supongamos que F y G son iguales para n+1 distintos valores. Entonces F G
es un polinomio de grado menor que n + 1 con n + 1 ceros. Si F G no es el
polinomio identicamente cero, tenemos una contradiccion con el razonamiento
del parrafo anterior. De donde, F G es el polinomio identicamente cero, es
decir F = G (coeciente a coeciente). (Para otra prueba, ver 6.5.10.)
Teorema de Identidad Supongamos que dos polinomios en x sobre un do-
minio entero innito cada uno de grado n. Si ambos polinomios tienen valores
iguales para mas de n valores distintos de x, entonces los dos polinomios son
identicos.
4.3.1 Determinar todos los polinomios P(x) tales que P(x
2
+1) = (P(x))
2
+1
y P(0) = 0.
Soluci on. Veamos para algunos casos:
P(1) = P(0
2
+ 1) = (P(0))
2
+ 1 = 1,
P(2) = P(1
2
+ 1) = (P(1))
2
+ 1 = 1 + 1 = 2,
P(3) = P(2
2
+ 1) = (P(2))
2
+ 1 = 4 + 1 = 5,
P(4) = P(3
2
+ 1) = (P(3))
2
+ 1 = 5
2
+ 1 = 26.
En general, denir x
0
= 0, y para n > 0 denir x
n
= x
2
n1
+ 1. entonces un
argumento facil de induccion prueba que P(x
n
) = x
n
. Luego el polinomio P(x)
y el polinomio x son iguales para una innidad de valores de n umeros enteros,
y por tanto, por el teorema de identidad, P(x) x. Esto es, existe solamente
un polinomio con las propiedades establecidas, a saber, P(x) = x.
4.3.2 Probar que si m y n son enteros positivos y 1 k n, entonces
k

r=0
_
m
k r
__
n
r
_
=
_
m+n
k
_
.
Soluci on. Hemos probado esta identidad en el captulo 1 (ver 1.3.4) usando
argumentos de conteo. Aqu haremos otra prueba usando el teorema de identi-
dad. La tecnica es estandard: los polinomios (1 +x)
m
(1 +x)
n
y (1 +x)
m+n
son
iguales para todos los valores de x. Luego, por el teorema de identidad, sus coe-
cientes son iguales; esto es, para cada k, el coeciente de x
k
en (1+x)
m
(1+x)
n
es igual al coeciente de x
k
en (1 +x)
m+n
. Se sigue que
158 CAP

ITULO 4. ALGEBRA
k

r=0
_
m
k r
__
n
r
_
=
_
m+n
k
_
4.3.3 Para cada entero positivo n, probar que la identidad
n

k=0
_
n
k
_
x
k
y
nk
= (x +y)
n
., x , y enteros positivos
implica la identidad
n

k=0
_
n
k
_
x
k
y
nk
= (x +y)
n
., x , y n umeros reales
Soluci on. Sea y
0
un entero positivo arbitrario jo, y sean
P(x) = (x +y
0
)
n
y Q(x) =
n

k=0
_
n
k
_
x
k
y
nk
0
.
P(x) y Q(x) son polinomios en x, y son iguales cuando x es un entero positivo.
Luego, por el teorema de identidad, son iguales para todos los n umeros reales
x.
Ahora, sea x
0
un n umero real jo, y sea
S(y) = (x
0
+y)
n
y T(y) =
n

k=0
_
n
k
_
x
k
0
y
nk
.
S(y) y T(y) son polinomios en y, y como son iguales cuando y es un entero pos-
itivo, se sigue que S(y) T(y) para todos los n umeros reales y. Esto completa
la prueba.
4.3. EL TEOREMA DE IDENTIDAD 159
(Incidentalmente, la identidad
n

k=0
_
n
k
_
x
k
y
nk
= (x +y)
n
., x , y n umeros reales
puede ser probada diestramente como sigue. Sea S = 1, 2, ..., n; sea A un
conjunto con x elementos y B un conjunto, disjunto con A, con y elementos.
Ahora, contemos, de dos diferentes maneras, el n umero de funciones de S en
A

B. Esto, adem as de proporcionar una soluci on al precedente, constituye


otra prueba del teorema del binomio.)
4.3.4 Es x
5
x
2
+ 1 irreducible sobre los racionales?
Soluci on. Por el teorema de raices racionales (ver 4.2.16), solamente son
posibles raices racionales 1, y ninguno de estos es cero. Ademas, si el polinomio
es reducible, necesariamente sera un producto de un polinomio cuadratico y un
c ubico. Supongase
x
5
x
2
+ 1 = (x
2
+ax +b)(x
3
+cx
2
+dx +e).
Por el lema de Gauss (ver 4.2.16), podemos asumir que a, b, c, d, e son enteros.
Como estos polinomios son iguales para toda x, sus coecientes son iguales;
luego, trabajando con los coecientes, tenemos las siguientes ecuaciones:
a +c = 0,
b +ac +d = 0,
bc +ad +c = 1,
bd +ae = 0,
be = 1.
No es dicil probar que estas ecuaciones no pueden ser resueltas simultanea-
mente. Por ejemplo, la ultima ecuaci on muestra que b y e son ambos impares.
Tambien, la cuarta ecuaci on muestra que a y d tienen la misma paridad. Similar-
mente, la primera ecuaci on muestra que a y c tienen la misma paridad. Luego,
a, c y d tienen la misma paridad. Pero entonces ac + d es par, y la segunda
ecuaci on no puede ser verdad (b es impar). Luego x
5
x
2
+ 1 no es reducible
sobre los racionales.
Otra forma de proceder con el problema esta basada en la siguiente observaci on.
Si f, g, y h son polinomios sobre los enteros y f = gh, entonces

f g

h (mod
160 CAP

ITULO 4. ALGEBRA
n), donde

f, g, y

h son son los polinomios formados por f, g, y h pero tomando


los coecientes m odulo n. Si f es reducible sobre los enteros, entonces

f es
reducible sobre los enteros tomando m odulo n. En tal caso, el polinomio x
5

x
2
+ 1 se transforma en x
5
+ x
2
+ 1 (mod 2). El unico polinomio cuadratico
irreducible sobre Z
2
= 0, 1 es x
2
+ x + 1 (los otros polinomios cuadraticos y
sus factorizaciones m odulo 2 son x
2
= xx, x
2
+1 = (x+1)
2
, y x
2
+x = x(x+1))).
Pero x
2
+x+1 no divide a x
5
+x
2
+1 en Z
2
(x
5
+x
2
+1 = (x
3
+x
2
)(x
2
+x+1)+1
(mod 2)), y luego, x
5
+x
2
+ 1 es irreducible sobre Z
2
, se sigue que x
5
+x
2
+ 1
es irreducible sobre los enteros, los racionales.
En la discusion precedente, hemos usado el hecho de que los polinomios sobre
Z
n
pueden ser sumados, restados y multiplicados de mane- ra usual excepto que
la aritmetica (sobre los coecientes) es tomada en Z
n
(es decir, m odulo n). Si
n es un n umero primo, digamos n = p, entonces Z
p
es un campo, luego todos
los resultados a polinomios sobre campos (por ejemplo, el teorema del factor, el
teorema de identidad) continuan siendo verdaderos. Este no es el caso si n no es
primo. Por ejemplo, 2x
3
2x, como un polinomio sobre Z
4
, tiene cuatro ceros
distintos sobre Z
4
, a saber, 0, 1, 2, y 3, cuando solamente poda tener a lo m as
tres si el trabajo aritmetico fuera sobre un campo.
Sea p un primo, y consideremos el teorema del binomio m odulo p
(1 +x)
p

k=0
_
p
k
_
x
k
(modp),
donde cada lado es considerado como un polinomio sobre Z
p
. Para 1 k p1,
teniendo (
p
k
) 0 (mod p) cuando ninguno de los factores de k!(p k)! divide al
factor de p en p!. Luego, como polinomio sobre Z
p
,
(1 +x)
p
1 +x
p
(modp).
Mas generalmente, para cada entero positivo n,
(1 +x)
p
n
1 +x
p
n
(modp).
El argumento es por induccion. Si es verdad para n = 1, y asumiendo es verdad
para k, tenemos
4.3. EL TEOREMA DE IDENTIDAD 161
(1 +x)
p
k+1
(1 +x)
p
k
(1 +x)
p
k
(1 +x)
p
k
(modp)
(1 +x
p
k
)(1 +x
p
k
) (1 +x
p
k
)(modp)
(1 +x
p
k
)
p
(modp)
1 + (x
p
k
)
p
(modp)
1 + (x
p
k+1
)(modp).
Pero igualando los coecientes de x
i
en cada lado, encontramos que
_
p
n
i
_
0(modp), 1 i < p
n
.
4.3.5 Probar que el n umero de coecientes binomiales impares en cualquier
expresi on binomial nita es una potencia de 2.
Soluci on. Una conjetura, basada por examinar varios casos especiales (ver
1.1.9), es que el n umero de coecientes impares en (1 +x)
n
es 2
k
, donde k es el
n umero de dgitos distintos de cero cuando n es expresado en notacion binaria.
Un ejemplo puede aclarar este hecho para despues hacer la prueba en el caso
general. Considerar n = 3. En notacion binaria 13 = 1101
2
= 8+4+1. Ademas,
(1 +x)
13
= (1 +x)
8+4+1
= (1 +x)
8
(1 +x)
4
(1 +x)
(1 +x
8
)(1 +x
4
)(1 +x)(mod2),
haciendo uso del resultado previamente establecido. De esto podemos ver que
existen ocho coecientes binomiales impares en (1 + x)
13
. Esto se debe a que
cuando el lado derecho de la ecuaci on precedente es expandido, (1 +x
4
)(1 +x)
tiene cuatro terminos, y (1 +x
8
)(1 +x
4
+x +x
5
) tendr a ocho terminos.(En el
caso general, si 1 + x
n
es multiplicado por un polinomio P(x) de grado menor
que n, el resultado sera un polinomio que tiene el doble de coecientes distintos
de cero que el n umero correspondiente en P(x).)
Considerando la ecuaci on polinomial x
2
+ ax + b = 0, y suponiendo que sus
raices son r
1
y r
2
. Entonces podemos escribir
162 CAP

ITULO 4. ALGEBRA
x
2
+ax +b = (x r
1
)(x r
2
)
= x
2
(r
1
+r
2
)x +r
1
r
2
.
De esto, usando el teorema de identidad, se sigue que
r
1
+r
2
= a
r
1
r
2
= b.
Similarmente, six
3
+ax
2
+bx +c = 0, tiene raices r
1
, r
2
, r
3
tenemos
x
3
+ax
2
+bx +c = (x r
1
)(x r
2
)(x r
3
)
= x
3
(r
1
+r
2
+r
3
)x
2
+(r
1
r
2
+r
1
r
3
+r
2
r
3
)x r
1
r
2
r
3
.
En este caso,
r
1
+r
2
+r
3
= a,
r
1
r
2
+r
1
r
3
+r
2
r
3
= b,
r
1
r
2
r
3
= c.
En cada caso, tenemos expresados los coecientes de las ecuaciones polinomiales
en terminos de sus raices (representando un patr on). Un argumento de induccion
prueba que esto es verdad en general: especcamente,
Si x
n
+a
n1
x
n1
+... +a
1
x +a
0
tiene raices r
1
, r
2
, ..., r
n
entonces
S
1
= r
1
+r
2
+... +r
n
= a
n1
,
S
2
= r
1
r
2
+r
1
r
3
+... +r
1
r
n
+r
2
r
3
+... +r
2
r
n
+... +r
n1
r
n
= a
n2
,
S
3
= r
1
r
2
r
3
+... +r
1
r
2
r
4
+... +r
2
r
3
r
4
+... +r
n2
r
n1
r
n
= a
n3
,
.
.
.
S
n
= r
1
r
2
... r
n
= (1)
n
a
0
.
4.3. EL TEOREMA DE IDENTIDAD 163
donde S
i
es la suma de todos los productos de las raices tomando i al mismo
tiempo.
4.3.6 Considerar todas las lineas rectas que intersectan a la gr aca
y = 2x
4
+ 7x
3
+ 3x 5
en cuatro distintos puntos, digamos (x
i
, y
i
), i = 1, 2, 3, 4. Probar que
x
1
+x
2
+x
3
+x
4
4
es independiente de la linea, y encontrar este valor.
Soluci on . Sea y = mx+b que intersecta a la curva en cuatro puntos (x
i
, y
i
), i =
1, 2, 3, 4. Entonces x
1
, x
2
, x
3
, x
4
son raices de la ecuaci on
mx +b = 2x
4
+ 7x
3
+ 3x 5,
o equivalentemente, de
x
4
+
7
2
x
3
+
3 m
2
x
5 +b
2
= 0.
Se sigue de esto mediante observaciones sencillas que (x
1
+ x
2
+ x
3
+ x
4
)/4 =
(
7
2
)/4 =
7
8
, y esto es independiente de m y b.
4.3.7 Sea P un punto sobre la gr aca de f(x) = ax
3
+ bx, y sea la tangente
en P que intersecta a la curva y = f(x) tambien en Q. Sea x
0
la absisa de P.
Probar que la absisa de Q es 2x
0
.
Soluci on. Podemos aprovechar el trabajo hacia adelante al escribir la ecuaci on
de la tangente a la curva y = f(x) en P, digamos y = T(x), y resolver y = T(x)
y y = f(x) simultaneamente para encontrar Q.
Otro argumento que podemos aprovechar es como sigue. Recordemos que re-
solver simultaneamente y = T(x) y y = f(x) es lo mismo que encontrar las
raices de f(x) T(x) = 0. Ahora x
0
es una raiz doble (es decir, de multiplicidad
164 CAP

ITULO 4. ALGEBRA
2) de esta ecuaci on, ya que T(x) es tangente a f(x) en x
0
. As que debemos
buscar la tercera raiz, denotada por x
1
. Sabemos que la suma de las raices,
2x
0
+x
1
es igual a k/a, donde k es el coeciente de x
2
. Pero el coeciente de
x
2
es 0, se sigue que x
1
= 2x
0
.
4.3.8 Sean x
1
y x
2
las raices de la ecuaci on
x
2
(a +d)x + (ad bc) = 0
Probar que x
3
1
y x
3
2
son las raices de
y
2
(a
3
+d
3
+ 3abc + 3bcd)y + (ad bc)
3
= 0
Soluci on . Sabemos que
x
1
+x
2
= a +d
x
1
x
2
= ad bc
Como (x
1
+x
2
)
3
= x
3
1
+ 3x
2
1
x
2
+ 3x
1
x
2
2
+x
3
2
, tenemos
x
3
1
+x
3
2
= (x
1
+x
2
)
3
3x
2
1
x
2
3x
1
x
2
2
= (a +d)
3
3x
1
x
2
(x
1
+x
2
)
= (a +d)
3
3(ad bc)(a +d)
= (a +d)(a
2
+ 2ad +d
2
3ad + 3bc)
= (a +d)(a
2
ad +d
2
+ 3bc)
= a
3
+d
3
+ 3abc + 3bcd.
Ademas,
x
3
1
x
3
2
= (ad bc)
3
,
y la prueba esta completa.
4.3. EL TEOREMA DE IDENTIDAD 165
4.3.9 Sean a, b, c n umeros reales tales que a +b +c = 0. Probar que
a
5
+b
5
+c
5
5
= (
a
3
+b
3
+c
3
3
)(
a
2
+b
2
+c
2
2
).
Soluci on. Aqu es muy clara la soluci on basada en las ideas de esta seccion.
Sea A = ab +ac +bc y B = abc. Entonces a, b, c son las raices de la ecuaci on
x
3
+Ax +B = 0.
Para cada entero positivo n, sea T
n
= a
n
+b
n
+c
n
. Entonces
T
0
= 3,
T
1
= 0,
T
2
= (a +b +c)
2
2(ab +ac +bc) = 2A.
Para n 0, T
n+3
= AT
n+1
+ BT
n
(sustituyendo a, b, c en x
n+3
= Ax
n+1
+
Bx
n
y sumando), y esto da
T
3
= AT
1
+BT
0
= 3B,
T
4
= AT
2
+BT
1
= 2A
2
,
T
5
= AT
3
+BT
2
= 5AB.
Se sigue que
T
5
5
= AB =
T
3
3
T
2
2
.
4.3.10 Probar que la ecuaci on polinomial con coecientes reales
P(x) = a
n
x
n
+a
n1
x
n1
+... +a
3
x
3
+x
2
+x + 1 = 0
166 CAP

ITULO 4. ALGEBRA
no puede tener todas sus raices reales.
Soluci on. Sean r
1
, r
2
, ..., r
n
las raices de P(x) = 0. Ninguna de r
1
, r
2
, ..., r
n
es
cero. Dividamos cada lado de P(x) = 0 por x
n
y sea y =
1
x
, tomando
Q(y) y
n
+y
n1
+y
n2
+a
3
y
n3
+... +a
n1
y +a
n
= 0.
Notese que r es una raiz de P(x) = 0 si y solamente si
1
r
es raiz de Q(y) = 0.
Ademas, las raices de Q(y) = 0 son s
1
, s
2
, ..., s
n
donde s
i
=
1
ri
, i = 1, 2, ..., n. Se
sigue que

n
i=1
s
i
= 1,

i<j
s
i
s
j
= 1,
y de donde,
n

i=1
s
2
i
= (
n

i=1
s
i
)
2
2

i<j
s
i
s
j
= 1 2 = 1.
Esta ecuaci on implica que no todas las s

i
s son reales; equivalentemente, no todas
las r

i
s son reales.
Problemas
4.3.11 Sea K un entero positivo. Encontrar todos los polinomios
P(x) = a
n
x
n
+a
n1
x
n1
+... +a
1
x +a
0
,
donde las a
i
son reales, que satisfacen la ecuaci on
P(P(x)) = [P(x)]
k
.
4.3.12 .
4.3. EL TEOREMA DE IDENTIDAD 167
(a) Probar que logx no puede ser expresado en la forma
f(x)
g(x)
donde f(x) y
g(x) son polinomios con coecientes reales.
(b) Probar que e
x
no puede ser expresado en la forma
f(x)
g(x)
donde f(x) y g(x)
son polinomios con coecientes reales.
4.3.13 Probar que
(1 +x)
n
x(1 +x)
n
+x
2
(1 +x)
n
+... x
k
(1 +x)
n
= (1 +x)
n1
(1 (x)
k+1
).
y usar la identidad para probar que
_
n 1
k
_
=
_
n
k
_

_
n
k 1
_
+...
_
n
0
_
.
4.3.14 .
(a) Derivando cada lado de la identidad
(1 +x)
n
=
n

k=0
_
n
k
_
x
k
.
Comparando los coecientes de x
k1
en el resultado de la identidad, probar
que
n
_
n 1
k 1
_
= k
_
n
k
_
(b) Usando el resultado de la parte (a) probar que
n1

i=0
(1)
i
_
n 1
i
_
1
i + 1
=
1
n
.
4.3.15 Sea x
(n)
= x(x 1) (x n + 1) para n un entero positivo, y sea
x
(0)
= 1. Probar que para cualesquiera n umeros reales x y y
168 CAP

ITULO 4. ALGEBRA
(x +y)
(n)
=
n

k=0
_
n
k
_
x
(k)
y
(nk)
.
(Sugerencia: Esto puede ser tratado por inducci on, pero considerando una prueba
similar a 4.3.3 podemos establecer prime- ro el resultado para enteros positivos
x y y. Para esto, contar de dos maneras diferentes, el n umero de funciones uno
a uno de 1, 2, ..., n sobre AB, donde A es un conjunto con x elementos y B
es un conjunto, disjunto de A, con y elementos. Probar la identidad para todos
los n umeros reales haciendo uso del teorema de identidad.)
4.3.16 Es x
4
+ 3x
3
+ 3x
2
5 reducible sobre los enteros?
4.3.17 Sea p un n umero primo. Probar que
(a)
_
p 1
k
_
(1)
k
(mod p), 0 k p 1,
(b)
_
p + 1
k
_
0 (mod p), 2 k p 1,
(c)
_
pa
pb
_

_
a
b
_
(mod p), a b 0,
(d)
_
2p
p
_
2 (mod p).
4.3.18 Sea = cos(2/n) +isen(2/n).
(a) Probar que 1, ,
2
, ...,
n1
son raices de x
n
1 = 0.
(b) Probar que (1 )(1
2
) (1
n1
) = n.
(c) Probar que +
2
+ +
n1
= 1.
4.3.19 .
(a) Resolver la ecuaci on x
3
3x
2
+ 4 = 0, dado que dos de sus raices son
iguales.
(b) Resolver la ecuaci on x
3
9x
2
+ 23x 15 = 0, dado que sus raices est an
en progresi on aritmetica.
4.3. EL TEOREMA DE IDENTIDAD 169
4.3.20 Dado que r, s, t son raices de x
3
+ax
2
+bx +c = 0.
(a) Evaluar 1/r
2
+ 1/s
2
+ 1/t
2
, con la condici on de que c ,= 0.
(b) Encontrar un polinomio cuyas raices sean r
2
, s
2
, t
2
.
4.3.21 Dados los n umeros x, y, z tales que
x +y +z = 3,
x
2
+y
2
+z
2
= 5,
x
3
+y
3
+z
3
= 7,
encontrar x
4
+y
4
+z
4
= 7. (Sugerencia: Usar un argumento similar al de 4.3.9.)
Terminamos esta secci on con tres problemas que dirigen la atenci on a algunos
resultados adicionales acerca de polinomios que son muy usa- dos en ciertos
problemas.
4.3.22 (Teorema) Si x
1
, x
2
, ..., x
n
son distintos n umeros, y y
1
, y
2
, ..., y
n
son
n umeros cualesquiera, no todos cero, existe un unico polinomio f(x) de grado no
mayor que n1 con la propiedad de que f(x
1
) = y
1
, f(x
2
) = y
2
, ..., f(x
n
) = y
n
.
En torno a la prueba:
(a) Sea g(x) = (x x
1
)(x x
2
) ... (x x
n
). Probar que
g(x)
(x x
1
)g

(x
1
)
=
(x x
2
)(x x
3
) ... (x x
n
)
(x
1
x
2
)(x
1
x
3
) ... (x
1
x
n
)
es un polinomio de grado n 1 con ceros en x
2
, x
3
, ..., x
n
y que es igual a
1 en x = x
1
.
(b) F ormula de interpolaci on de Lagrange. Probar que
f(x) =
g(x)
(x x
1
)g

(x
1
)
y
1
+
g(x)
(x x
2
)g

(x
2
)
y
2
+... +
g(x)
(x x
n
)g

(x
n
)
y
n
toma tales valores y
1
, y
2
, ..., y
n
en los puntos x
1
, x
2
, ..., x
n
respectivamente.
(c) Aplicaci on. Supongamos que P(x) es un polinomio que cuando es dividido
por x 1, x 2, x 3 deja residuos 3, 5, 2 respectivamente. Determinar el
residuo de P(x) cuando es dividido por (x1)(x2)(x3). (Sugerencia:
Escribir P(x) = Q(x)(x1)(x2)(x3) +R(x), donde R(x) es de grado
menor que 3. Encontrar R(x) por la f ormula de interpolaci on de Lagrange,
con R(1) = 3, R(2) = 5, R(3) = 2.)
170 CAP

ITULO 4. ALGEBRA
4.3.23 (Fracciones Parciales)
(a) Probar que si f(x) es un polinomio cuyo grado es menor que n entonces
la fracci on
f(x)
(x x
1
)(x x
2
) ... (x x
n
)
,
donde x
1
, x
2
, ..., x
n
son n n umeros distintos, puede ser representada como
una suma de fracciones parciales
A
1
(x x
1
)
+
A
2
(x x
2
)
+... +
A
n
(x x
n
)
,
donde A
1
, A
2
, ..., A
n
son constantes (independientes de x). (Suge- rencia:
Usar la f ormula de interpolaci on de Lagrange: dividir cada lado por g(x),
etc.)
(b) Aplicaci on. Sea f(x) un polinomio m onico de grado n con ceros distintos
x
1
, x
2
, ..., x
n
. Sea g(x) cualquier polinomio m onico de grado n1. Probar
que
n

j=1
g(x
j
)
f

(x
j
)
= 1.
(Sugerencia: Escribir
g(xj)
f(xj)
como una suma de fracciones parciales.)
4.3.24 Una sucesi on de n umeros u
0
, u
1
, u
2
, ... es llamada una sucesi on de
kesimo orden si existe un polinomio de grado k,
P(x) = a
k
x
k
+a
k1
x
k1
+... +a
1
x +a
0
tal que u
i
= P(i) para i = 0, 1, 2, ....
La sucesi on primera derivada de la sucesi on u
0
, u
1
, u
2
, ... es la sucesi on u
(1)
0
, u
(1)
1
, u
(1)
2
, ...
denida por
u
(1)
n
= u
n+1
u
n
, n = 0, 1, 2, 3, ....
(a) Probar que si u
0
, u
1
, u
2
, ... es una sucesi on de orden k, entonces la sucesi on
primera derivada es una sucesi on de orden k 1. Deniendo la sucesi on
segunda derivada de la sucesi on u
0
, u
1
, u
2
, ... como la primera derivada de
la sucesi on primera derivada, esto es, la sucesi on u
(2)
0
, u
(2)
1
, u
(2)
2
, ... denida
por
u
(2)
n
= u
(1)
n+1
u
(1)
n
= u
n+2
2u
n+1
+u
n
, n = 0, 1, 2, ....
De la parte (a) se sigue que u
(2)
0
, u
(2)
1
, u
(2)
2
, ... es una sucesi on de orden
k 2. Similarmente deniendo la sucesi on tercera derivada, la sucesi on
4.3. EL TEOREMA DE IDENTIDAD 171
cuarta derivada, y as sucesivamente. Repitiendo la aplicaci on de la parte
(a) probamos que si u
0
, u
1
, u
2
, ... es una sucesi on de orden k, la sucesi on
k +1esima derivada es identicamente cero. Conjeturemos lo contrario: si
las deriva- das sucesivas de una sucesi on arbitraria u
0
, u
1
, u
2
, ... eventual-
mente se hacen identicamente cero, entonces los terminos de la sucesi on
original son valores sucesivos de una expresi on polinomial; es decir, existe
un polinomio P(x) tal que u
n
= P(n), n = 0, 1, 2, ... .
(b) Usar inducci on para probar que
u
n
=
_
n
0
_
u
0
+
_
n
1
_
u
(1)
0
+
_
n
2
_
u
(2)
0
+ +
_
n
n
_
u
(n)
0
.
(c) Suponer que la sucesi on original es descrita por la funci on F(x). Es decir,
suponer que F(n) = u
n
, n = 0, 1, 2, .... Para k = 0, 1, 2, ..., sea
k
F(0) =
u
(k)
0
, y para un n umero real x y un entero positivo i, sea x
(i)
= x(x
1)(x 2) (x i + 1). Probar que el resultado de la parte (b) puede
escribirse en la forma
F(n) =

k=0

k
F(0)
k!
n
(k)
.
Note la similitud con la expansi on de Taylor de F(x):
F(x) =

k=0
F
(k)
(0)
k!
x
(k)
.
(d) Probar que si la (k +1)

esima derivada de una sucesi on es identicamente


cero, entonces la sucesi on original est a dada por
P(n) =
k

i=0

k
F(0)
i!
n
(i)
.
(e) Usar el resultado de la parte (d) para encontrar una f ormula abreviada
para la suma de la serie 1
4
+ 2
4
+ + n
4
. (Sugerencia: N otese que
la sucesi on primera derivada est a dada por un polinomio de grado 4, y
adem as la suma de ellos es un polinomio de grado 5.)
Ejemplos adicionales
4.4.30, 4.4.31, 7.2.10, 8.2.2, 8.2.3, 8.2.10, 8.4.11.
172 CAP

ITULO 4. ALGEBRA
4.4.

Algebra Abstracta
Un grupo es un conjunto G con una operacion binaria sobre G tal que
(i) Propiedad Asociativa. Para cualesquier elementos a, b, c en G
(a b) c = a (b c).
(ii) Identidad. Existe un unico elemento e en G (llamado la identidad de G) tal
que para cada elemento a en G,
a e = a = e a.
(iii) Inverso. Para cada elemento a en G, existe un unico elemento a
1
a
1
a = e = a a
1
.
Cuando trabajamos con grupos, recordamos la operacion como multipli-
cacion, y en este caso podemos suprimir escribiendo productos. Luego, a b
es escrita como ab, y a (b c) es escrita como a(bc), o abc, y as sucesivamente.
Ademas cuando pensamos en como producto, denotamos al elemento identidad
como 1. En suma, usamos notacion exponencial para simplicar expresiones;
por ejemplo, a
4
= aaaa, etc. No es dicil probar las leyes de exponentes en un
grupo, a saber,
a
n
a
m
= a
n+m
, (a
n
)
m
= a
nm
, n, m enteros
La operacion del grupo no necesariamente es conmutativa; es decir, puede no
ser el caso de que ab = ba para todos los elementos a, b de G. Un ejemplo de un
tal grupo es el conjunto las matrices no-singulares de n por n sobre los n umeros
reales.
En cualquier grupo G, se da el caso que
(ab)
1
= b
1
a
1
, a, b G.
Esta identidad es fundamental y puede ser probada de la siguiente manera.
Observese que (ab)(b
1
a
1
) = a(b(b
1
a
1
)) = a(bb
1
)a
1
) = a(ea
1
) = aa
1
=
e y (b
1
a
1
)(ab) = b
1
(a
1
(ab)) = b
1
(a
1
a)b) = b
1
(eb) = b
1
b = e. De
donde b
1
a
1
es un inverso para ab. Pero ab tiene un unico inverso, denotado
por (ab)
1
. Se sigue que (ab)
1
= b
1
a
1
. Si un grupo G es conmutativo (es
decir, si ab = ba para todos a, b G), es facil probar que
(ab)
n
= a
n
b
n
, ab G n entero.
4.4.

ALGEBRA ABSTRACTA 173
4.4.1 Supongamos que G es un conjunto y una operaci on binaria sobre G tal
que:
(i) Propiedad asociativa. Para todos a, b, c en G, a (b c) = (a b) c;
(ii) Identidad derecha. Existe un elemento e en G tal que para cada elemento
a en G, a e = a; y
(iii) Inversa derecha. Para cada elemento a en G, existe un elemento a
1
en
G tal que a a
1
= e.
Probar que G es un grupo.
Soluci on . Debemos probar que la identidad derecha e es tambien identidad
izquierda, y que el inverso derecho a
1
es tambien inverso izquierdo. Entonces
probaremos que e y a
1
son unicos.
Observese que a
1
es un elemento de G, y adem as por (ii), existe un elemento
(a
1
)
1
en G tal que (a
1
) (a
1
)
1
= e. Ahora calculemos
(a
1
)a = (a
1
a)e = (a
1
a)(a
1
(a
1
)
1
)
= a
1
_
a(a
1
(a
1
)
1
)

= a
1
_
(aa
1
)(a
1
)
1

= a
1
_
e(a
1
)
1

= (a
1
e)(a
1
)
1
= a
1
(a
1
)
1
= e.
Esto prueba que a
1
es un inverso (inverso izquierdo e inverso derecho).
Tambien, ea = (aa
1
)a = a(a
1
a) = ae = a, y por tanto e es un identico para
G (es decir, para cada a, ea = a = ae).
Supongase que e

es tambien una identidad de G. Entonces e = e e

(debido
a que e

es una identidad) = e

(debido a que e es una identidad). Esto prueba


que el elemento identidad de G es unico.
Supongase que (a
1
)

es tambien un inverso de a. Entonces


(a
1
)

= (a
1
)

e = (a
1
)

(aa
1
) =
_
(a
1
)

a
1
= ea
1
= a
1
. Esto prueba
que el inverso de a es unico.
Se sigue que G es un grupo.
174 CAP

ITULO 4. ALGEBRA
4.4.2 Sea G un grupo.
(a) Propiedad de cancelaci on. Para todo a, b, c en G, probar que
ab = ac implica b = c
ba = ca implica b = c.
(b) Sea a un elemento de G, y considerando la sucesi on
1, a, a
2
, a
3
, ....
Probar que todos los elementos en la sucesi on son diferentes o existe el menor en-
tero positivo n tal que a
n
= 1 y 1, a, a
2
, ..., a
n1
son distintos. En esta situaci on,
n es llamado el orden de a, denotado por ord(a); en el caso enterior decimos
que a tiene orden nito.
Soluci on . (a) Esto se sigue inmediatamente multiplicando cada lado de la
igualdad por la izquierda (derecha respectivamente) por a
1
.
(b) Supongase que no todos los elementos de la sucesion son dife- rentes, y sea
n el entero m as peque no tal que a
n
es una repeticion de un elemento previo de
la sucesion. Entonces a
n
= 1, ya que si a
n
= a
i
, 0 < i < n, entonces por la ley
de cancelacion, a
n1
= a
i1
, y esto contradice lo supuesto para n.
4.4.3 Sean a y b dos elementos en un grupo tal que aba = ba
2
b, a
3
= e, y
b
2n1
= e para alg un entero positivo n. Probar que b = e.
Soluci on . Notar que si ab = ba, entonces aba = ba
2
b es lo mismo que a
2
b =
a
2
b
2
, y la propiedad de cancelacion implica b = e. A pesar de que el grupo no sea
conmutativo, probaremos que este particular conjunto de ecuaciones implican
que ab = ba.
Notese que ab = ba es lo mismo que ab
2n
= b
2n
a ya que hemos asumido que
b
2n
= b. Para probar que ab
2n
= b
2n
a es suciente probar que ab
2
= b
2
, ye que
ab
2n
= a(b
2
)
n
= (b
2
)
n
a (repitiendo la aplicaci on ab
2
= b
2
) = b
2n
a.
Luego, la prueba se completa despues de observar que ab
2
= (aba)
(a
1
b) = (ba
2
b)(a
1
b) = (ba
2
)(ba
1
b) = (ba
2
)(ba
2
b) = (ba
2
)(aba) = ba
3
ba =
b
2
a = (ya que a
3
= e).
4.4.

ALGEBRA ABSTRACTA 175
Sea G un grupo decimos que H es un subgrupo de G si H es un subconjunto de
G que a la vez es un grupo (con la operacion de G). El orden de H esta denido
por el n umero de elementos de H, y este n umero es denotado por ord(H).
Una clase importante de subgrupos son los siguientes. Sea a G, y sea
a) = a
n
: n es un entero.
Es facil checar que a) es un subgrupo de G; este es llamado el subgrupo cclico
generado por a. Note que ord(a) = ord(a)).
El siguiente teorema constituye uno de los m as importantes resultados en la
teora de grupos nitos.
Teorema de Lagrange.Si H es un subgrupo de un grupo nito G, entonces el
orden de H divide al orden de G.
Aqu tres colorarios importantes.
(i) Si G es un grupo de orden n y a G, entonces a
n
= 1.
(ii) Si G es un grupo de orden p, donde p es un primo, entonces G es un grupo
cclico (es decir, G = a) para alg un a G).
(iii) Si G es un grupo y a
n
= 1, entonces el orden de a divide a n.
Veremos la prueba del teorema de Lagrange como un problema (ver seccion
4.4.18); adem as veamos unos argumentos instructivos para los corolarios.
Prueba de (i). Sea a G y sea m = ord(a)). Por el teorema de Lagrange m
divide a n, supongamos que n = mq para alg un entero q. Entonces a
n
= a
mq
=
(a
m
)
q
= 1
q
= 1.
Prueba de (ii).Sea a un elemento de G diferente de la identidad. Entonces a)
es un subgrupo de G con m as de un elemento (estan, 1 y a). Por el teorema de
Lagrange, el orden de a) divide a p, pero p es un n umero primo, se tiene el caso
que a) es de orden p; esto es,a) = G.
Prueba de (iii). Sea m = Ord(a). Por el algoritmo de la divisi on existen enteros
q y r tales que n = mq +r, 0 r < m. 1 = a
n
= a
mq+r
= (a
m
)
q
a
r
= a
r
. Ya que
1, a, ..., a
m1
son distintos, se sigue de esto que r = 0, y por tanto que m dividde
a n (esta es una aplicaci on tpica del algoritmo de la divisi on para enteros).
176 CAP

ITULO 4. ALGEBRA
4.4.4 Si en el grupo G tenemos a
5
= 1, aba
1
= b
2
para algunos a, b G,
encontrar el orden de b.
Soluci on . Si a
5
= 1, el orden de a es 1 o 5. Si ord(a) = 1, entonces a = 1 y se
sigue que b = b
2
, o b = 1, y luego ord(b) = 1.
Supongase que ord(a) = 5. Tenemos (aba
1
)(aba
1
) = (b
2
)
2
, o equivalente-
mente, ab
2
a
1
= b
4
. Sustituyendo aba
1
por b
2
en el lado izquierdo de esta
ecuaci on obtenemos a
2
ba
2
= b
4
, Elevando al cuadrado, se tiene (a
2
ba
2
)(a
2
ba
2
) =
(b
4
)
2
, o equivalentemente, a
2
b
2
a
2
= b
8
. Nuevamente, sustituyendo aba
1
por
b
2
a la izquierda se tiene a
3
ba
3
= b
8
. Elevando al cuadrado tenemos a
3
b
2
a
3
=
b
16
, y sustituyendo, a
4
ba
4
= b
16
. Una vez mas: a
4
b
2
a
4
= b
32
, o equivalen-
temente, a
5
ba
5
= b
32
. Pero a
5
= a
5
= 1, luego b = b
32
, y por la ley de
cancelacion, tenemos b
31
= 1. Ya que 31 es un n umero primo, el orden de b es
1 (si b es la identidad) o 31.
4.4.5 Si G es un grupo nito y m es un entero positivo primo relativo con el
orden de G, entonces para cada a en G existe un unico b en G tal que b
m
= a.
Soluci on . Sea T : G G denida por T(x) = x
m
. Probemos que T es
una funci on uno-a-uno. Supongamos que T(x) = T(y) para algunos elementos
x y y de G. Entonces x
m
= y
m
. Sea n = ord(G). Como n y m son primos
relativos, existen enteros s y t tales que sn + tm = 1. De aqu x = x
sn+tm
=
(x
n
)
s
(x
m
)
t
= (x
m
)
t
(ya que x
n
= 1) = (y
m
)
t
(ya que x
m
= y
m
) = (y
n
)
s
(y
m
)
t
(ya que y
n
= 1) = y
ns+mt
= y.
Luego T es una funci on uno-a-uno, y como G es un conjunto nito, T es sobre en
G. Es decir, existe un unico elemento b en G tal que T(b) = a (equivalentemente
b
m
= a).
El primer corolario del teorema de Lagrange establece que a
ord(G)
= 1 para
cada elemento en el grupo nito G. Esto tiene una cantidad interesante de im-
portantes consecuencias cuando es aplicado a grupos particulares. Por ejemplo,
sea V
n
el conjunto de todos los enteros positivos menores que n que son primos
relativos con n. Los elementos de V
n
forman un grupo con la multiplicaci on
m odulo n. Sea (n) = ord(V
n
). (La funci on es llamada la -funci on de Euler.)
Entonces el teorema de Lagrange implica el siguiente.
Teorema de Euler. Si a es un entero primo relativo con n, entonces
a
(n)
1 (mod n).
Cuando n es un n umero primo, digamos n = p, tenemos que (p) = p1, luego
a
p1
1 (mod p) cuando a no es m ultiplo de p. Si multiplicamos cada lado por
4.4.

ALGEBRA ABSTRACTA 177
a, tenemos que a
p
a (mod p). Esta congruencia siempre es cierta aun cuando
a sea un m ultiplo de p, y luego tenemos el siguiente resultado.
El peque no teorema de Fermat. Si a es un entero y p es un primo, entonces
a
p
a (mod p).
4.4.6 Probar que cada divisor primo de 2
p
1, donde p es un n umero primo,
es mayor que p. (Un corolario de este es que el n umero de primos es innito.)
Soluci on . El resultado es verdadero para p = 2, luego podemos asumir que p es
impar. Supongamos que q es un primo que divide a 2
p
1. Entonces q es impar y
2
p
1 (mod q). Por el peque no teorema de Fermat, 2
q1
1 (mod q). Si q = p
tenemos que 2 = 2 1 2 2
q1
2
q
= 2
p
1 (mod q), una contradiccion. Si
q < p, entonces q 1 y p son primos relativos, luego existen enteros s y t tales
que sp + t(q 1) = 1. Se sigue que 2 = 2
sp+t(q1)
= (2
p
)
s
(2
q1
)
t
1 (mod q),
una contradiccion. Luego q es mayor que p.
4.4.7 Probar que si n es un entero mayor que 1, entonces n no divide a 2
n
1.
Soluci on . Supongase que n divide a 2
n
1: es decir, 2
n
1 (mod n). Clara-
mente, n es un n umero impar, ya que 2
n
1 es un n umero impar. Supongase
que p es un divisor primo de n. Entonces 2
n
1 (mod p). Ahora, pensando a
2 como un elemento del grupo V
p
. Sabemos que 2
p1
1 (mod p) (El peque no
teorema de Fermat, ya que mcd(2, p) = 1). Por el tercer corolario del teorema
de Lagrange, p 1 divide a n. Que es una contradiccion. Ademas, supongamos
que p esta dado como el menor primo que divide a n. Entonces las mismas
conclusiones son ciertas, pero ahora, el hecho de que ord(2) divide a n y ord(2)
divide a p 1 producen una contradiccion a lo dicho de p. De donde, n nunca
puede dividir a 2
n
1.
4.4.8 Probar que para cualquier entero positivo n existe una potencia de 2 con
una sucesi on de m as de n ceros sucesivos (en su representaci on decimal).
Soluci on . Para cualquier entero positivo s existe un entero positivo t tal que
2
t
1 (mod 5
s
) (por ejemplo, tomar t = (5
s
)). Sea s = 2n. existen enteros
positivos q y s tales que 2
r
1 = q 5
2n
. Multiplicando ambos lados por 2
2n
,
reescribimos como
2
r+2n
= 2
2n
+q 10
2n
,
y notese que 2
r+2n
tiene al menos n ceros consecutivos en su representaci on
decimal, ya que 2
2n
< 10
n
.
178 CAP

ITULO 4. ALGEBRA
4.4.9 Dados enteros positivos a y b, probar que existe un entero positivo c tal
que una innidad de n umeros de la forma an +b (n un entero positivo) tienen
todos sus factores primos c.
Soluci on . El resultado es obviamente verdadero cuando a = 1, suponga- mos
pues que a > 1. Primero, consideremos el caso en que mcd(a, b) = 1. Debemos
probar que existe una innidad de terminos de la sucesion aritmetica an +b en
los terminos de la sucesion (a +b)
k
, k = 1, 2, 3, ... .
Del teorema de Euler, b
(a)
1 (mod a), ya que b es primo relativo con a. Se
sigue que para cada entero positivo s,
(a +b)
s(a)+1
b
s(a)+1
(b
(a)
)
s
b
s
b (mod a).
Esto signica que para cada entero positivo s existe un entero q
s
tal que
(a +b)
s(a)+1
= q
s
a +b.
Se sigue que cada uno de los terminos de q
s
a + b, s = 1, 2, 3, ... tiene solamente
los factores primos que aparecen en a +b.
Ahora consideremos el caso en que mcd(a, b) = d > 1. Entonces mcd(a/d, b/d) =
1, luego a partir del argumento anterior, existe c tal que una innidad de termi-
nos de la sucesion (a/d)n + b tienen todos sus factores primos c. De esto se
sigue que una innidad de miembros de la forma an+b tienen todos sus factores
primos dc. Esto completa la prueba.
Un Anillo es un conjunto R con dos operaciones binarias, + y , tales que
(i) R es un grupo conmutativo con respecto a la operacion +;
(ii) Para todos a, b, c en R, a(bc) = (ab)c (` u ` u es suprimido);
(iii) Para todos a, b, c en R,
a(b +c) = ab +ac,
(b +c)a = ba +ca.
R no necesariamente tiene un identico multiplicativo: si lo tiene, R es un anillo
con identidad. La multiplicaci on en R no necesariamente es conmutativa: si lo
es, decimos que R es un anillo conmutativo.
4.4.

ALGEBRA ABSTRACTA 179
4.4.10 Sean a y b elementos de un anillo nito tales que ab
2
= b. Probar que
bab = b.
Soluci on . Obviamente, si el anillo es conmutativo el resultado es inmediato,
pero debemos probar que el resultado es verdadero aun cuando el anillo no sea
conmutativo. Adicionalmente, no podemos asumir que el anillo tenga identico
multiplicativo.
Supongase que b = b
2
. Entonces bab = bab
2
= b
2
= b, y hemos terminado.
Supongase que b = b
m
para alg un entero m > 2. Entonces bab = bab
m
=
b(ab
2
)b
m2
= b
2
b
m2
= b
m
= b, y hemos terminado. Luego es suciente probar
que b
m
= b para alg un entero m 2.
Supongase que el anillo tiene n elementos. Por el principio de las casillas, al
menos dos elementos en la sucesion b, b
2
, ..., b
n
, b
n+1
son iguales. Sea i el menor
entero para el cual b
i
es igual a alguna potencia subsecuente de b en la sucesion
precedente; es decir, b
i
= b
i+j
, para alg un j > 0. Supongase que i > 1. Entonces
multiplicando cada lado de ab
2
= b por la derecha por b
i+j2
se tiene ab
i+j
=
b
i+j1
. Pero como b
i
= b
i+j
, tenemos ab
i
= b
i+j1
. De aqu existen dos casos a
considerar.
Supongase que i = 2. Entonces b = ab
2
= b
j+1
(de la ultima ecuaci on), y esto
contradice lo dicho de i. Luego, sup ongase que i > 2. Entonces b
i1
= b b
i2
=
(ab
2
) b
i2
= ab
i
= b
i+j1
, que tambien contradice lo dicho de i. De donde,
i = 1; es decir, b = b
j
para alg un j. Por el argumento del primer paragrafo, la
prueba esta completa.
Un dominio entero D es un anillo conmutativo con unidad en el cual para a, b
en D, ab = 0 implica que a = 0 o b = 0. la propiedad de cancelacion es vedadera
en un dominio entero. Pues, sup ongase ab = ac y a ,= 0. Entonces a(b c) = 0,
luego b c = 0, o equivalentemente, b = c. Similarmente ba = ca, a ,= 0, implica
b = c.
Un campo es un anillo conmutativo en donde cada elemento distinto de cero
tiene un inverso multiplicativo.
4.4.11 Probar que cada dominio entero nito (un dominio entero con s olo un
n umero nito de elementos) es un campo.
Soluci on . Debemos probar que cada elemento distinto de cero de un dominio
entero tiene un inverso multiplicativo. Luego, sea D

= a
1
, a
2
, ..., a
n
los ele-
mentos no cero del dominio entero, y considerar un elemento arbitrario a de D

.
Denir T : D

por T(a
i
) = aa
i
. Si T(a
i
) = T(a
j
) entonces aa
i
= aa
j
,
luego por la propiedad de cancelacion, a
i
= a
j
. Luego, vemos que T es una
funci on uno-a-uno. Como D

es nito, el mapeo T es sobre en D

. Pero uno
180 CAP

ITULO 4. ALGEBRA
de los elementos en D

es el identico multiplicativo, denotado por 1. De donde


T(a
k
) = 1 para alg un a
k
D

; es decir aa
k
= 1. Esto prueba que a tiene un
inverso multiplicativo.
Problemas
4.4.12 Sea G un conjunto, y una operaci on binaria sobre G que es asociativa
y tal que para todos a, b en G, a
2
b = b = ba
2
(suprimiendo ). Probar que G es
un grupo conmutativo.
4.4.13 A es un subconjunto de un grupo nito G, y A contiene m as de la
mitad de los elementos de G. probar que cada elemento de G es el producto de
dos elementos de A.
4.4.14 Sea H un subgrupo con h elementos de un grupo G. Sup ongase que G
tiene un elemento a tal que para todo x en H, (xa)
3
= 1, la identidad. En G, sea
P el conjunto de todos los productos x
1
ax
2
a x
n
a, con n un entero positivo y
los x
i
en H. Probar que P no tiene m as que 3h
2
elementos.
4.4.15 Si a
1
ba = b 1 y b
1
ab = a
1
para los elementos a, b de un grupo,
probar que a
4
= b
4
= 1.
4.4.16 Sean a y b elementos de un grupo nito G.
(a) Probar que ord(a) = ord(a
1
).
(b) Probar que ord(ab) = ord(ba).
(c) Si ba = a
4
b
3
, probar que ord(a
4
b) = ord(a
2
b
3
).
4.4.17 Sean a y b elementos de un grupo. Si b
1
ab = a
k
, probar que b
r
a
s
b
r
=
a
sk
r
para todos enteros positivos r y s.
4.4.18 (Secuencia de la prueba del teorema de Lagrange). Sea G un grupo ni-
to y H un subgrupo con m elementos distintos, digamos H = 1, h
2
, h
3
, ..., h
m
.
Para cada a G, sea Ha = a, h
2
a, h
3
a, ..., h
m
a.
(a) Probar que Ha contiene m elementos distintos.
(b) Probar que Hh
i
= H.
4.4.

ALGEBRA ABSTRACTA 181
(c) Si b Ha, probar que Ha y Hb son conjuntos disjuntos.
(d) Probar que existen elementos a
1
, a
2
, , ..., a
k
en G tales que G = Ha
1

Ha
2

...

Ha
k
y Ha
i

Ha
j
= si i ,= j.
(e) Usar el resultado previo para formular una prueba del teorema de La-
grange.
4.4.19 Encontrar el entero positivo m as peque no n tal que 2
n
1 es divisible
por 47.
4.4.20 Probar que si p es un primo, p > 3, entonces ab
p
ba
p
es divisible por
6p.
4.4.21 Sean a y b enteros primos relativos. probar que existen enteros m y n
tales que a
m
+b
n
= 1 (mod ab).
4.4.22 Si a, b, c, d son enteros positivo, probar que 30 divide a a
4b+d
a
4c+d
.
4.4.23 Sea T
n
= 2
n
+1 para cada entero positivo. Sea la -funci on de Euler,
y se k cualquier entero positivo y m = n + k(T
n
). Probar que T
m
es divisible
por T
n
.
4.4.24 Probar que existe un entero positivo k tal que k2
n
+ 1 es compuesto
para cada entero n. (Sugerencia: Considerar las clases de congruencia m odulo
24 y aplicar el Teorema Chino del Residuo.)
4.4.25 Un Anillo Booleano es un anillo para el cual a
2
= a para cada elemento
a del anillo. Un elemento a de un anillo es Nilpotente si a
n
= 0 para alg un
entero positivo n. Probar que un anillo R es Booleano si y solamente si R es
conmutativo, R contiene un elemento no cero nilpotente, y ab(a + b) = 0 para
todos a, b en R. (Sugerencia: Probar que a
4
a
5
= 0, y considerar (x
2
x
3
)
2
.)
4.4.26 Sea R un anillo con identidad, y sea a R. Sup ongase que existe un
unico elemento a

tal que aa

= 1. Probar que a

a = 1.
4.4.27 Sea R un anillo con identidad, y sea a un elemento nilpotentede R (ver
4.4.25). Probar que 1 a es invertible (es decir, probar que existe un elemento
b en R tal que b(1 a) = 1 = (1 a)b).
182 CAP

ITULO 4. ALGEBRA
4.4.28 Sea R un anillo, y sea C = x R : xy = yx para todo y en R
. Probar que si x
2
x C para todo x en R, entonces R es conmutativo.
(Sugerencia: probar que xy +yx C considerando x +y, y entonces probar que
x
2
C.)
4.4.29 Sea p un n umero primo. Sea J el conjunto de todas las matrices de
dos-por-dos
_
a b
c d
_
donde las entradas est an tomadas de 0, 1, 2, ..., p1 y que satisfagan la condi-
ci on a+d 1 (mod p), adbc 0 (mod p). Determinar el n umero de terminos
que tiene J.
4.4.30 Sea p un n umero primo, y sea Z
p
= 0, 1, 2, ..., p 1.Z
p
es un campo
con las operaciones de suma y producto m odulo p.
(a) Probar que 0, 1, ..., p1 son los ceros de x
p
x (Conside- rado como polinomio
sobre Z
p
). Concluir que x
p
x = x(x 1)(x 2) (x (p 1)) (mod p).
(b) Teorema de Wilson. De la parte (a) probar que
(p 1)! 1(modp)
(c) Considerar el determinante [ a
ij
[ de orden 100 con a
ij
= i j. Probar
que el valor absoluto de cada uno de los 100! terminos de la expansi on de este
determinante es congruente con 1 m odulo 101.
4.4.31 Sea F un campo nito que tiene un n umero impar m de elementos.
Sea p(x) un polinomio irreducible sobre F de la forma x
2
+bx+c, b, c F. Para
cuantos elementos k en F es P(x) +k irreducible sobre F?
Ejemplos adicionales
1.1.5, 1.1.12.
Captulo 5
Calculo de Series
En este captulo pondremos nuestra atencion en algunas de las formulas basicas
de sumatorias. La lista es corta (es decir, teorema del binomio, formulas para se-
ries aritmeticas y geometricas, formulas elementales de series de potencias) pero
veremos el uso de tecnicas estandar (como son, series telescopicas, diferenciacion
e integraci on) haremos esto extremadamente versatil y potente.
5.1. Coeciente Binomial
Aqu estan algunas identidades basicas; asumimos que n y k son enteros positivos
n k 0.
Representaci on factorial:
_
n
k
_
=
n!
k!(n k)!
(1)
Condici on de simetra:
_
n
k
_
=
_
n
n k
_
(2)
F ormula de entrada y salida:
_
n
k
_
=
n
k
_
n 1
k 1
_
(3)
F ormula de adici on:
_
n
k
_
=
_
n 1
k
_
+
_
n 1
k 1
_
(4)
183
184 CAP

ITULO 5. C

ALCULO DE SERIES
Las siguientes formulas son obtenidas mediante la aplicaci on repetida de la
formula de adicion. F ormula de sumatoria:
_
n
0
_
+
_
n + 1
1
_
+... +
_
n +k
k
_
=
_
n +k + 1
k
_
(5)
Suma de productos :
_
r
0
__
s
n
_
+
_
r
1
__
s
n 1
_
+... +
_
r
n
__
s
0
_
=
_
r +s
n
_
(6)
Teorema del binomio (ver 2.1.1, 2.1.11, 4.3.3):
n

k=0
_
n
k
_
x
k
y
nk
= (x +y)
n
(7)
5.1.1 Usar la f ormula de sumatoria para probar que
(a) 1 + 2 + 3 +... +n =
n(n+1)
2
(b) 1
2
+ 2
2
+ 3
2
+... +n
2
=
n(n+1)(2n+1)
6
Soluci on. (a) Tenemos
1 + 2 + 3 +... +n =
_
1
1
_
+
_
2
1
_
+
_
3
1
_
+... +
_
n
1
_
=
_
1
0
_
+
_
2
1
_
+
_
3
2
_
+... +
_
n
n 1
_
=
_
n + 1
n 1
_
=
_
n + 1
2
_
=
n(n + 1)
2
.
(b) Primero veamos para que constantes a y b tales que
k
2
= a
_
k
2
_
+b
_
k
1
_
= a
k(k 1)
2
+bk
para k = 1, 2, ..., n. Sabiendo que cada lado de la ecuaci on es un polinomio en
k de grado 2. La identidad es cierta si y s olamente si los coecientes de las
respectivas potencias de k son iguales: es decir si y s olo si
1 = a/2
0 = a/2 +b.
5.1. COEFICIENTE BINOMIAL 185
Esto es cierto para a = 2 y b = 1. Se sigue que
1
2
+ 2
2
+ 3
2
+... +n
2
=
_
2
_
1
2
_
+
_
1
1
__
+
_
2
_
2
2
_
+
_
2
1
__
+... +
_
2
_
n
2
_
+
_
n
1
__
= 2
__
2
2
_
+
_
3
2
_
+... +
_
n
2
__
+
__
1
1
_
+
_
2
1
_
+... +
_
n
1
__
= 2
__
2
0
_
+
_
3
1
_
+... +
_
n
n 2
__
+
__
1
0
_
+
_
2
1
_
+... +
_
n
n 1
__
= 2
_
n + 1
n 2
_
+
_
n + 1
n 1
_
= 2
_
n + 1
3
_
+
_
n + 1
2
_
= 2
n(n + 1)(n 1)
6
+
n(n + 1)
2
=
n(n + 1)(2n + 1)
6
.
(Otra prueba de la parte (b) esta dada en 5.3.11.)
Es deseable memorizar las formulas precedentes pero de alguna manera es posi-
ble dar con ellas facilmente. Una manera de recordar la primera formula es
mostrada en la Figura 5.1 (para n = 5).
El diagrama tambien proporciona el siguiente argumento para el caso general.
Si S denota la suma de los primeros n enteros positivos. Entonces
S = 1 + 2 +... +n
S = n + (n 1) +... + 1.
Sumando, tenemos
2S = (n + 1) + (n + 1) +... + (n + 1)
= n(n + 1),
y se sigue que
S =
n(n + 1)
2
.
La tecnica de calcular una suma mediante el rearreglo de los terminos es muy
com un. En particular , cuando los terminos estan representados en una doble
sumatoria, esto es obtener ventajas al cambiar el orden de la sumatoria. El
siguiente ejemplo es una ilustraci on de esta idea.
5.1.2 Sumar
n

j=0
n

i=j
_
n
i
__
i
j
_
186 CAP

ITULO 5. C

ALCULO DE SERIES
Figura 5.1.
Soluci on Los terminos de esta suma estan idexados por parejas ordenadas
(i, j), donde (i, j) vara sobre los elementos del siguiente arreglo triangular:
En la suma dada, los elementos son sumados primero siguiendo las columnas.
cuando intercambiamos el orden de la sumatoria, tal que los terminos sean ahora
sumados siguiendo las las, la suma es expresada en la forma
n

i=0
i

j=0
_
n
i
__
i
j
_
,
5.1. COEFICIENTE BINOMIAL 187
o, equivalentemente
n

i=0
_
n
i
_
i

j=0
_
i
j
_
.
Esto es facilmente evaluado de la siguiente manera. de acuerdo al teorema del
binomio,
(1 +x)
i
=
i

j=0
_
i
j
_
x
j
.
cuando x = 1, tenemos
i

j=0
_
i
j
_
= 2
i
La suma original es ahora
n

i=0
_
n
i
_
2
i
que por el teorema del binomio es (1 + 2)
n
= 3
n
.
5.1.3 Sumar las siguientes:
(a)
_
n
1
_
+ 2
_
n
2
_
+ 3
_
n
3
_
+... +n
_
n
n
_
,
(b) 1 +
1
2
_
n
1
_
+
1
3
_
n
2
_
+... +
1
n+1
_
n
n
_
.
Soluci on La primera suma es
n

i=1
i
_
n
i
_
.
Procedamos, en sumatorias de este tipo, es usada la formula de entrada y sal-
ida para despues ver como inside el ndice de la sumatoria en los coecientes
binomiales. Como
_
n
i
_
=
n
i
_
n 1
i 1
_
,
188 CAP

ITULO 5. C

ALCULO DE SERIES
se sigue que
i
_
n
i
_
= n
_
n 1
i 1
_
,
y de aqu
n

i=1
i
_
n
i
_
=
n

i=1
n
_
n 1
i 1
_
= n
n

i=1
_
n 1
i 1
_
= n
n1

i=0
_
n 1
i
_
= n 2
n1
.
La segunda suma es
n

i=0
1
i + 1
_
n
i
_
.
Pero
_
n + 1
i + 1
_
=
n + 1
i + 1
_
n
i
_
,
y por lo tanto
n

i=0
1
i + 1
_
n
i
_
=
n

i=0
1
n + 1
_
n + 1
i + 1
_
=
1
n + 1
n

i=0
_
n + 1
i + 1
_
=
1
n + 1
n+1

i=1
_
n + 1
i
_
=
1
n + 1
_
n+1

i=0
_
n + 1
i
_

_
n + 1
0
_
_
=
1
n + 1
_
2
n+1
1

.
Existen otras formas instructivas de evaluar estas sumas, basadas en la diferen-
ciaci on e integraci on de cada lado de
(1 +x)
n
=
n

i=0
_
n
i
_
x
i
.
5.1. COEFICIENTE BINOMIAL 189
Para la parte (a), derivando tenemos
n

i=0
i
_
n
i
_
x
i1
= n(1 +x)
n1
,
y, con x = 1, tenemos
n

i=0
i
_
n
i
_
= n 2
n1
.
Para la parte (b), integrando tenemos
n

i=0
_
n
i
_
x
i+1
i + 1
=
(1 +x)
n+1
n + 1
+C.
Cuando x = 0, el lado izquierdo de esta ecuaci on es 0, y esto implica que
C = 1/(n + 1) . Luego, cuando x = 1, tenemos (como antes)
n

i=0
1
i + 1
_
n
i
_
=
1
n + 1
_
2
n+1
1

.
5.1.4 Probar que
_
n
1
_

1
2
_
n
2
_
+
1
3
_
n
3
_
... + (1)
n+1
1
n
_
n
n
_
= 1 +
1
2
+... +
1
n
.
Soluci on El lado izquierdo de la identidad puede verse como la integral denida
de una serie binomial, y esta idea proviene del siguiente argumento:
(1 x)
n
=
_
n
0
_

_
n
1
_
x
1
+
_
n
2
_
x
2
...,
1 (1 x)
n
=
_
n
1
_
x
_
n
2
_
x
2
+
_
n
3
_
x
3
...,
1 (1 x)
n
x
=
_
n
1
_

_
n
2
_
x +
_
n
3
_
x
2
....
Ahora podemos integrar cada lado de 0 a 1, y obtenemos
_
1
0
1 (1 x)
n
x
dx =
_
n
1
_

1
2
_
n
2
_
+
1
3
_
n
3
_
....
190 CAP

ITULO 5. C

ALCULO DE SERIES
Para terminar el problema, debemos probar que la integral del lado izquierdo
es igual a 1 +
1
2
+
1
3
+... +
1
n
. Sea y = 1 x. Entonces
_
1
0
1 (1 x)
n
x
dx =
_
1
0
1 y
n
1 y
dy
=
_
1
0
_
1 +y +y
2
+... +y
n1

dy
= y +
1
2
y
2
+... +
1
n
y
n

1
0
= 1 +
1
2
+... +
1
n
.
El problema puede no ser abordado con herramientas de c alculo, usando identi-
dades basicas de esta seccion, pero es tecnicamente espinoso. De todas formas,
como esto es intructivo, chequemos la idea.
Primero, usando repetidamente la formula de adicion y la formula de entrada y
salida, tenemos, para n i 1,
1
i
_
n
i
_
=
1
i
__
n 1
i
_
+
_
n 1
i 1
__
=
1
i
_
n 1
i
_
+
1
n
_
n
i
_
=
1
i
__
n 2
i
_
+
_
n 2
i 1
__
+
1
n
_
n
i
_
=
1
i
_
n 2
i
_
+
1
n 1
_
n 1
i
_
+
1
n
_
n
i
_
,
y continuando de esta manera, se tiene
1
i
_
n
i
_
=
1
n
_
n
i
_
+
1
n 1
_
n 1
i
_
+... +
1
i
_
i
i
_
.
De aqu,
n

i=0
(1)
i+1
1
i
_
n
i
_
=
n

i=0
(1)
i+1
_
_
ni

j=0
1
n j
_
n j
i
_
_
_
,
Cuando cambiamos el orden de la sumatoria obtenemos
n1

j=0
_
nj

i=1
(1)
i+1
_
n j
i
_
1
n j
_
.
Si k = n j, el lado derecho es
n

k=1
_
k

i=1
(1)
i+1
_
k
i
_
1
k
_
=
n

k=1
_
1
k
k

i=1
(1)
i+1
_
k
i
_
_
=
n

k=1
1
k
(ver 5.1.9(a)).
5.1. COEFICIENTE BINOMIAL 191
5.1.5 Sumar
n1

i=0
n+1

j=i+1
_
n + 1
j
__
n
i
_
.
Soluci on Esta puede ser evaluada usando identidades basicas de esta seccion;
sin embargo queremos ilustrar otra tecnica. Aunque este acercamiento parecera
articial y no muy gustado, el hecho es que qal pensarlo de esta manera no es
tan inusual como se podra creer a primera vista. la idea es interpretr la suma
en en terminos de probabilidad, de la siguienta manera. Multiplicando la suma
por 1/2
2n+1
y escribiendolo en la forma
n1

i=0
_
_
_
n
i
__
1
2
_
n n+1

j=i+1
_
n + 1
j
__
1
2
_
n+1
_
_
.
Ahora consideremos la siguiente mec anica de juego entre dos jugadores A y B. El
jugador A lanza n+1 monedas y se queda con las n en las que obtiene el mayor
n mero de caras, el jugador B lanza n monedas. Gana el que haya obtenido
el mayor n mero de caras o B. Observese que la suma anterior representa la
probabilidad de que gane el jugador A. Vamos a calcular esta probabilidad
de otra manera. El juego es equivalente a lo siguiente: Sean A y B cada uno
lanzando n monedas, gana el jugador que obtenga m as caras. Si cada uno tiene
el msmo n umero de caras, pero no todas caras, A lanza la n + 1 moneda,
gana si cae cara, pierde sic ae cruz. Visto de esta manera A y B tienen la
misma probabilidad de ganar. En el caso en que A y B obtuviesen puras caras.
B ganara independientemente del

ltimo lanzamiento de A. Luego, B gana en
ex actamente dos casos m as que A. Esto es, el total de lanzamientos a realizarse
2
2n+1
B gana en los dos ultimos casos descritos y en exactamente la mitad de
los casos restantes(1/2(2
2n1
)). Luego la probabilidad de que A gane es
1 Pr(B gana) = 1
2 +
1
2
_
2
2n+1
2

2
2n+1
=
2
2n+1
2 2
2n
+ 1
2
2n+1
=
2
2n
1
2
2n+1
Se sigue que la suma original es 2
2n
1.
Problemas
192 CAP

ITULO 5. C

ALCULO DE SERIES
5.1.6 .
(a) Sumar todos los n umeros entre 0 y 1000 que son m ultiplos de 7 o de 11.
(b) Sumar todos los n umeros entre 0 y 1000 que son m ultiplos de 7, 11 o de
13.
5.1.7 .
(a) Probar que para cualquier entero k > 1 y cuaquier entero positivo n, n
k
es
la suma de n impares consecutivos.
(b) Sea n un entero positivo y m cualquier entero de la misma paridad que n.
Probar que el producto mn es igual a la suma de n impares consecutivos.
5.1.8 Usar la f ormula (5) de sumatoria para sumar (a)

n
k=1
k
3
; (b)

n
k=1
k
4
.
5.1.9 Sumar cada una de las siguientes:
(a) 1
_
n
1
_
+
_
n
2
_

_
n
3
_
+... + (1)
n
_
n
n
_
.
(b) 1 2
_
n
2
_
+ 2 3
_
n
3
_
+... + (n 1)n
_
n
n
_
.
(c)
_
n
1
_
+ 2
2
_
n
2
_
+ 3
2
_
n
3
_
+... +n
2
_
n
n
_
.
(d)
_
n
1
_
2
2
_
n
2
_
+ 3
2
_
n
3
_
... + (1)
n+1
n
2
_
n
n
_
.
(e)
_
n
0
_

1
2
_
n
1
_
+
1
3
_
n
2
_
... + (1)
n 1
n+1
_
n
n
_
.
(f )

j1
_
(1)
j
_
n
j 1
_
/

1kj
k
_
.
5.1.10 .
(a) Cu al es la probabilidad de que un n umero impar de seises aparezcan aleato-
riamente en n lanzamientos dado? (Al evaluar la suma, considerar
1
2
[(x +y)
n
(x y)
n
] .)
5.1. COEFICIENTE BINOMIAL 193
(b) Probar que si n es un m ultiplo positivo de 6,
_
n
1
_
3
_
n
3
_
+ 3
2
_
n
5
_
... = 0,
_
n
1
_

1
3
_
n
3
_
+
1
3
2
_
n
5
_
... = 0, .
5.1.11 Probar las siguientes identidades
(a)
_
n
1
_
1 2

_
n
2
_
2 3
+
_
n
3
_
3 4
... + (1)
n
_
n
n
_
n (n + 1)
=
1
2
+
1
3
+... +
1
n + 1
,
(b)
_
n
0
_
1
2

_
n
1
_
2
2
+
_
n
2
_
3
2
... + (1)
n
_
n
n
_
(n + 1)
2
=
1
n + 1
_
1 +
1
2
+... +
1
n
_
,
5.1.12 Probar que
(a)
_
r
0
__
s
n
_
+
_
r
1
__
s
n + 1
_
+
_
r
2
__
s
n + 2
_
+...
_
r
n
__
s
n +n
_
=
_
r +s
s n
_
.
(b)
_
n
0
_
2
+
_
n
1
_
2
+
_
n
2
_
2
+... +
_
n
n
_
2
=
_
2n
n
_
.
5.1.13 Usar las identidades de esta secci on para probar que
n

k=0
_
n 2k
n
_
n
k
__
2
=
2
n
_
2n 2
n 1
_
.
5.1.14 Sumar
2n1

i=n
_
i 1
n 1
_
2
1i
.
(Sugerencia: Para i = n, n + 1, ..., 2n 1, computar P(E
i
), la probabilidad de i
de una moneda legal son requeridos para obtener n caras o n cruces.)
5.1.15 Un cierto estudiante, juega a un juego de suma muy particular, comen-
zando con escribir en linea en un papel x
1
y
2
. Despues de algunos lineas el
194 CAP

ITULO 5. C

ALCULO DE SERIES
estudiante ha escrito:
x
1
y
2
(1) x
1
y
2
y
3
x
4
(2) x
1
y
2
y
3
x
4
y
5
x
6
x
7
y
8
(3) x
1
y
2
y
3
x
4
y
5
x
6
x
7
y
8
y
9
x
10
x
11
y
12
x
13
y
14
y
15
x
16
Sobre cada lnea, el estudiante ha copiado exactamente la lnea anterior, y en-
tonces hace una copia de ello, cambiando las x

s a y

s y las y

s a xs, las
subindisa en orden. El estudiante nota que la suma de los subndices de las
x

s es igual a la suma de los subndices de las y

s en la lnea (1). En la lnea (2)


se satisface la misma ecuaci on, similarmente para las sumas de los cuadrados
de los subndices de las x

s y las y

s, es decir, 1
2
+4
2
+6
2
+7
2
= 2
2
+3
2
+5
2
+8
2
.
El estudiante inmediatamente hace la congetura inductiva para la lnea (n), la
suma de las k-esimas potencias de los subndices de las x

s debe ser igual a la


suma de las k-esimas potencias de los subndices de las y

s para k = 1, 2, ..., n.
Probar esto para toda n > 0.
Ejemplos adicionales
1.3.4, 1.3.15, 1.11.4, 2.1.1, 2.1.2, 4.3.5, 4.3.13, 4.3.,14, 4.3.15, 4.3.24, 5.4.8, 6.8.3,
7.2.9, 7.3.8. Aplicaciones del teorema del binomio: 1.1.1 (Solucion 4), 1.1.2, 1.3.8,
1.6.6(b), 1.12.4, 3.5.8, 3.5.12, 3.5.13, 4.2.13, 4.3.5, 4.4.9, 5.1.2, 5.1.15, 5.2.13,
6.8.3, 7.1.5, 7.1.15.
5.2. Serie Geometrica
La serie geometrica aparece naturalmente en muchos problemas, y es adem as
conocida su suma de forma imperativa:
n

i=0
x
i
=
1 x
n+1
1 x
, x ,= 1

i=0
x
i
= lim
n
n

i=0
x
i
= lim
n
1 x
n+1
1 x
=
1
1 x
, [x[ < 1
5.2.1 Para un entero positivo n, encontrar la f ormula para (n), la suma de
los divisores de n.
Soluci on Claramente si (1) = 1. Si p es un primo, sus unicos divisores son 1
y p, luego (p) = 1 +p.
5.2. SERIE GEOM

ETRICA 195
Sin es una potencia de un primo, digamos n = p
m
, los divisores de n son
1, p, p
2
, p
3
, ..., p
m
, luego (n) = 1 +p +p
2
+p
3
+ +p
m
=
1p
m+1
1p
.
Supongamos que n = ab donde a y b son primos relativos, cada uno mayor
que uno. Supongamos que los divisores de a son a
1
, a
2
, ..., a
s
y los de b son
b
1
, b
2
, ..., b
s
. Entonces los divisores de n son de la forma a
i
, b
j
, i = 1, 2, ...s, j =
1, 2, ...r, y la suma de ellos es
a
1
, b
1
+a
1
, b
2
+ a
1
, b
r
+a
2
, b
1
+a
2
, b
2
+ a
2
, b
r
+ a
s
, b
1
+a
s
, b
2
+ a
s
, b
r
o equivalentemente
(a
1
+a
2
+ +a
s
)(b
1
+b
2
+ +b
r
)
Luego, (n) = (a)(b)
Consideremos ahora un entero arbitrario n y supongamos que su factorizaci on
unica es
n = p
e1
1
p
e2
2
p
e
k
k
. Del trabajo precedente, encontramos que
(n) =
_
1 p
e1+1
1
1 p
1
__
1 p
e2+1
2
1 p
2
_

_
1 p
e
k
+1
k
1 p
k
_
5.2.2 Sea n = 2m donde m es un entero impar mayor que 1. Sea = e
2i/n
.
Expresar (1 )
1
explcitamente como un polinomio en ,
a
k

k
+a
k1

k1
+ +a
1
+a
0
con coeciente enteros a
i
.
Soluci on Notese que es una raiz n-esima de la unidad y que
m
=
_
e
2i/2m
_
m
=
e
i
= 1. Luego,
1 + +
2
+ +
m1
=
1
m
1
=
2
1
(5.1)
Tambi n, como m es impar, tenemos
1 +
2
+
m1
=
1 ()
m
1 ()
= 0 (5.2)
Ahora, sumando las ecuaciones (1) y (2), tenemos
2 + 2
2
+ + 2
m1
=
2
1
,
196 CAP

ITULO 5. C

ALCULO DE SERIES
o equivalentemente
1
1
= 1 +
2
+ +
m1
.
5.2.3 Sumar la serie nita cos +cos2 + +cosn.
Soluci on La serie que debemos evaluar es la parte real de la serie geometrica
e
i
+e
2i
+e
3i
+ +e
ni
,
esta suma es
e
i(n+1)
1
e
i
1
1 =
e
i(n+1)
1 (e
i
1)
e
i
1
=
e
i(n+1)
e
i
e
i
1
=
e
i(n+
1
2
)
e
i(
1
2
)
e
i(
1
2
)
e
i(
1
2
)
=
1
2i
_
e
i(n+
1
2
)
e
i(
1
2
)
sen
1
2

_
=
1
2i
1
sen
1
2

_
cos(n +
1
2
) cos
1
2
+i(sen(n +
1
2
) sen
1
2
)
_
=
1
2sen
1
2

_
sen(n +
1
2
) sen
1
2
+i(cos(n +
1
2
) cos
1
2
)
_
.
Igualando las partes reales, tenemos
cos +cos2 + +cosn =
1
2sen
1
2

_
sen(n +
1
2
) sen
1
2

_
=
sen(n +
1
2
)
2sen
1
2

1
2
.
5.2.4 Probar que la fracci on
1 3 5 (2n 1)
2 4 6 2n
cuando es reducida en terminos menores, es de la forma a/2
m
donde a es impar
y w < 2n.
Soluci on Podemos escribir la fracci on en la forma
(2n)!
2
2n
n!n!
=
1
2
2n
_
2n
n
_
.
Ahora,
_
2n
n
_
es entero, luego lo unico que queda por probar es que w < 2n. La
mayor potencia de 2 en (2n)! es
__
2n
2
__
+
__
2n
4
__
+
__
2n
8
__
+ +
__
2n
2
k
__
+
5.2. SERIE GEOM

ETRICA 197
ver (3.3.10), y en n! es
__
n
2
__
+
__
n
4
__
+
__
n
8
__
+ +
__
n
2
k
__
+
Se sigue que
w = 2n + 2

k=1
__
n
2
k
__

k=1
__
2n
2
k
__
Pero

k=1
__
2n
2
k
__
=

k=1
__
n
2
k1
__
=

k=0
__
n
2
k
__
= n +

k=1
__
n
2
k
__
,
y de aqu
w = 2n + 2

k=1
__
n
2
k
__
n

k=1
__
n
2
k
__
= n +

k=1
__
n
2
k
__
< n +

k=1
n/2
k
= 2n.
(Alternativamente,
_
2n
n
_
=
2n
n
_
2n1
n1
_
=2
_
2n1
n1
_
, luego w < 2n.)
5.2.5 Para x 0, evaluar en forma cerrada

n=1
(1)
[[2
n
x]]
2
n
.
Soluci on Escribamos x en la forma
x = [[x]] +

n=1
a
n
2
n
donde a
n
es 0 o 1, si x tiene la forma m/2
n
(m un entero impar), tomamos a
k
para k sucientremente grande. Para cada n, [[2
n
x]] es para si y solo si a
n
es 0.
198 CAP

ITULO 5. C

ALCULO DE SERIES
Se sigue que para cada n, (1)
[[2
n
x]]
= 1 2a
n
. de donde

n=1
(1)
[[2
n
x]]
2
n
=

n=1
1 2a
n
2
n
=

n=1
1
2
n
2

n=1
a
n
2
n
= 1 2(x [[x]]).
5.2.6 Evaluar en forma cerrada

(p,q)=1
1
x
p+q
1
, [x[ > 1,
donde la suma se extiende sobre todos los enteros positivos p y q tales que p y q
son primos relativos.
Soluci on

(p,q)=1
1
x
p+q
1
=

(p,q)=1
1
x
p+q
_
1
1 1/x
p+q
_
=

(p,q)=1
1
x
p+q
_

n=0
_
1
x
p+q
_
n
_
=

(p,q)=1

n=1
_
1
x
n(p+q)
_
.
Como p, q y n varian sobre el mismo conjunto de ndices en esta suma, las poten-
cias de 1/x varian sobre todos los posibles pares ordenados de enteros positivos
(i, j). Como la serie es absolutamente convergente (1/ [x[ < 1), podemos rear-
5.2. SERIE GEOM

ETRICA 199
reglar las series en la forma

(p,q)=1

n=1
_
1
x
n(p+q)
_
=

i=1

j=1
frac1x
i+j
=

i=1
1
x
i

j=1
frac1x
j
=
_

i=1
1
x
i
_
2
=
_
1
1 1/x
1
_
2
=
_
x
x 1
1
_
2
=
_
1
x 1
_
2
.
Problemas
5.2.7 Sea n = 2
p1
(2
p
1) , y supongamos que 2
p
1 es un n umero primo.
Probar que la suma de todos los divisores (positivos) de n, no incluyendo a
n, es ex actamente n. (Un n umero que tiene esta propiedad es llamado n umero
perfecto)
5.2.8 Sumar la serie 1 + 22 + 333 + +n(11..,1)
. .
n
5.2.9 Si E(n) denota el mayor entero k tal que 5
k
es un divisor del producto
1
1
2
2
3
3
n
n
. Encontrar una f ormula cerrada para E(5
m
), m un entero positivo.
Que pasar a con E(5
m
)/5
2m
cuando m ?
5.2.10 Una sucesi on est a denida por a
1
= 2 y a
n
= 3a
n1
+ 1. Encontrar
a
1
+a
2
+ +a
n
.
5.2.11 Vericar las siguientes f ormulas
(a)

n
1
sen(2k 1) =
sen
2
n
sen
(b)

n
1
sen
2
(2k 1) =
1
2
n
sen4n
4sen2
.
200 CAP

ITULO 5. C

ALCULO DE SERIES
5.2.12 .
(a)
(b)
5.2.13 Si a, b y c son races de la ecuaci on x
3
x
2
x 1 = 0
(a) probar que a, b y c son distintas.
(b) probar que
a
1000
b
1000
a b
+
b
1000
c
1000
b c
+
c
1000
a
1000
c a
es un entero.
5.2.14 .
(a)
(b)
5.2.15 Probar que

n=0
(1 +x
2
n
) =

n=0
x
n
= 1/(1 x), [x[ < 1.
5.2.16 Evaluar en forma cerrada:

n=0
(x
2
n
/(1 x
2
n+1
)), [x[ < 1.
5.2.17 .
Ejemplos adicionales
1.12.1, 4.1.4, 4.1.8, 4.1.9, 4.25, 4.2.8, 4.2.12, 4.2.18, 4.3.13, 4.3.18(c), 5.1.4, 5.1.11,
5.4.1, 5.4.7, 5.4.9, 7.6.6.
5.3. SERIES TELESC

OPICAS 201
5.3. Series Telescopicas
Innitas series e innitos productos pueden ser evaluados algunas veces por
medio de telescopa. Los ejemplos son facilmente explicativos
5.3.1 Sumar la serie

i=1
1
(3i 2)(3i + 1)
.
Soluci on El truco consiste en dividir la suma como una suma de fracciones
parciales, con el resultado de los terminos en las sumas parciales podemos sumar
cancelando. Busquemos los n umeros A y B tale que
1
(3i 2)(3i + 1)
=
A
(3i 2)
+
B
(3i + 1)
.
Esto es
1 = A(3i + 1) +B(3i 2)
e igualando coecientes, tenemos
3A+ 2B = 0,
A2B = 1.
Se sigue que A =
1
3
y B =
1
3
. Luego
S
n
=
n

i=1
_
1
3
(3i 2)

1
3
(3i + 1)
_
=
__
1
1
4
_
+
_
1
4

1
7
_
+
_
1
7

1
10
_
+... +
1
3n 2

1
3n + 1
_
.
En esta suma tenemos la propiedad de telescopa : el segundo termino de cada
par se cancela con el primer termino en el par sucesivo, resultando que
S
n
=
1
3
_
1
1
3n + 1
_
.
Se sigue que la serie innita es lm
n
S
n
=
1
3
.
5.3.2 Sumar la serie innita
3
1 2 3
+
5
2 3 4
+
7
3 4 5
+
9
4 5 6
+....
202 CAP

ITULO 5. C

ALCULO DE SERIES
Soluci on Nuevamente, por fracciones parciales, busquemos los n umeros A, B, C
tales que
2n + 1
n(n + 1)(n + 2)
=
A
n
+
B
n + 1
+
C
n + 2
.
Tenemos que
2n + 1 = A(n + 1)(n 1) +Bn(n + 2) +Cn(n + 1).
Tomando n = 0 se tiene A =
1
2
; tomando n = 1 se tiene B = 1; tomando
n = 2 se tiene C =
3
2
. Luego, la nesima suma parcial es
S
n
=
_
1
2
1
+
1
2

3
2
3
_
+
_
1
2
2
+
1
3

3
2
4
_
+
_
1
2
3
+
1
4

3
2
5
_
+... +
_
1
2
n 2
+
1
n 1

3
2
n
_
+
_
1
2
n 1
+
1
n

3
2
n + 1
_
+
_
1
2
n
+
1
n + 1

3
2
n + 2
_
.
En este caso la telescopa la tomamos cruzando los lugares por grupos: el ultimo
termino de una tripleta se cancela con la suma del termino medio del siguiente
grupo y el primer termino del tercer grupo ya que:
1
2
n
+
1
n

3
2
n
= 0.
El resultado de la suma entonces es igual a
S
n
=
_
1
2
1
+
1
2
_
+
_
1
2
2
_
+
_

3
2
n + 1
_
+
_
1
n + 1

3
2
n + 2
_
=
5
4

1
2
n + 1

3
2
n + 2
.
Se sigue que la suma innita es lm
n
S
n
=
5
4
.
5.3.3 Expresar

n=1

m=1
1
m
2
n +mn
2
+ 2mn
como un n umero racional.
5.3. SERIES TELESC

OPICAS 203
Soluci on Si S es la suma en cuestion. Entonces, omitiendo los detalles de la
descomposici on en fracciones parciales, tenemos
S =

n=1

m=1
1
mn(m+n + 2)
=

n=1
1
n

m=1
1
m(m+n + 2)
=

n=1
1
n

m=1
1
n+2
m

1
n+2
m+n + 2
=

n=1
1
n(n + 2)
__
1
1
n + 3
_
+
_
2
1
n + 4
_
+
_
1
1
n + 3
_
+...
_
=

n=1
1
n(n + 2)
_
1 +
1
2
+
1
3
+... +
1
+n + 2
_
=

n=1
_
1
2
n

1
2
n + 2
__
1 +
1
2
+
1
3
+... +
1
+n + 2
_
=
1
2
__
1
1
3
__
1 +
1
2
+
1
3
_
+
_
1
2

1
3
__
1 +
1
2
+
1
3
+
1
4
_
+
_
1
3

1
5
__
1 +
1
2
+
1
3
+
1
4
+
1
5
_
+...
_
=
1
2
__
1 +
1
2
+
1
3
_
+
1
2
_
1 +
1
2
+
1
3
+
1
4
_
+
1
3
_
1
4
+
1
5
_
+
1
4
_
1
5
+
1
6
_
+
1
5
_
1
6
+
1
7
_
+...
_
=
1
2
_
11
6
+
1
2

25
12
+
_
1
3 4
+
1
4 5
+
1
5 6
+...
_
+
_
1
3 5
+
1
4 6
+
1
5 7
+...
__
=
1
2
_
11
6
+
25
24
+
__
1
3

1
4
_
+
_
1
4

1
5
_
+...
_
+
1
2
__
1
3

1
5
_
+
_
1
4

1
6
_
+
_
1
5

1
7
_
...
__
=
1
2
_
11
6
+
25
24
+
1
3
+
1
6
+
1
8
_
=
1
2
_
44 + 25 + 8 + 7
24
_
=
84
48
=
7
4
.
204 CAP

ITULO 5. C

ALCULO DE SERIES
5.3.4 Sumar la serie

n=1
3
n1
sen
3
_
x
3
n
_
.
Soluci on Usando el teorema de Moivre, tenemos
sen3 = (Im)(e
3i
) = (Im)
_
_
e
i
_
3
_
= (Im) [cos +isen]
3
= (Im)
_
cos
3
+ 3 cos
2
isen + 3 cos i
2
sen
2
+i
3
sen
3

= 3 cos
2
sen sen
3

= 3
__
1 sen
2

_
sen

sen
3

= 3sen 4sen
3
.
Se sigue que
sen
3
=
3
4
sen
1
4
sen3.
Luego
S
k
=
k

n=1
3
n1
sen
3
_
x
3
n
_
=
k

n=1
3
n1
_
3
4
sen
_
x
3
n
_

1
4
sen
_
x
3
n1
_
_
=
_
3
4
sen
_
x
3
_

1
4
senx
_
+
_
3
2
4
sen
_
x
3
2
_

3
4
sen
_
x
3
_
_
+
_
3
3
4
sen
_
x
3
3
_

3
2
4
sen
_
x
3
2
_
_
+...
+
_
3
k
4
sen
_
x
3
k
_

3
k1
4
sen
_
x
3
k1
_
_
=
3
k
4
sen
_
x
3
k
_

1
4
senx.
De donde, la suma de la serie es
lm
k
S
k
= lm
k
_
3
k
4
sen
_
x
3
k
_

1
4
senx
_
= lm
k
_
x
4
sen
_
x
3
k
_
_
x
3
k
_
1
4
senx
_
=
x senx
4
.
La idea de la telescopa es particularmente usada en la resoluci on de relaciones
de recurrencia. Aqu esta u ejemplo; otros ejemplos estan expuestos en la sigu-
iente seccion.
5.3. SERIES TELESC

OPICAS 205
5.3.5 Una sucesi on de n umeros satisface la recursi on
x
0
= 0, nx
n
= (n 2)x
n1
+ 1 para n > 0.
Encontrar una expresi on en forma cerrada para x
n
.
Soluci on Vemos que x
0
= 0, x
1
= 1, x
2
=
1
2
, x
3
=
1
2
, y consecuentemente
x
n
=
1
2
para toda n 2. pero encontrar un patron para una recurrencia no es
tan simple, y es m as instructivo considerar el problema de la siguiente manera.
Para n 2, multiplique cada lado de la recursi on por n 1, y para cada n, sea
y
n
= n(n 1)x
n
. La recursi on en terminos de las y
n
s es
y
n
= y
n1
+ (n 1), y
1
= 0.
Se sigue que
y
2
y
1
= 1,
y
3
y
2
= 2,
y
4
y
3
= 3,
.
.
.
y
n
y
n1
= n 1.
Sumando (notese la telescopa), tenemos
y
n
= 1 + 2 +... + (n 1) =
(n 1)n
2
,
y de aqu,
x
n
=
1
2
, n 2.
Problemas
5.3.6 Sumar la siguientes :
(a)
1
2!
+
2
3!
+
3
4!
+... +
n1
n!
,
(b) 1 1! + 2 2! + 3 3! +... +n n!,
(c)
2
123
+
4
234
+
6
345
+... +
2n
n(n+1)(n+2)
.
5.3.7 Evaluar los siguientes productos innitos:
206 CAP

ITULO 5. C

ALCULO DE SERIES
(a)

n=1
_
1
1
n
2
_
,
(b)

n=1
n
3
1
n
3
+1
.
(c) Probar que un producto innito puede transformarse en una serie innita
mediante el uso de la identidad P = e
log p
. Trabajar la parte (a) de esta
forma para encontrar la serie innita

n=1
log
_
1
1
n
2
_
.
5.3.8 Probar que para cada entero positivo m,
m
(m+ 1)(2m+ 1)
<
2m

r=m+1
1
r
2
<
m
(m+ 1)(2m+ 1)
+
3m+ 1
4m(m+ 1)(2m+ 1)
(Sugerencia: Note que
1
r(r+1)
<
1
r
2
<
1
(r+1)(r1)
.)
5.3.9 Sea F
1
, F
2
, ... la serie de Fibonacci. Usar la propiedad telesc opica para
probar las siguientes identidades:
(a) F
1
+F
2
+... +F
n
= F
n+2
1. (Sugerencia: F
n2
= F
n
F
n1
.)
(b) F
1
+F
3
+... +F
2n1
= F
2n
.
(c) F
2
1
+ F
2
2
+ ... + F
2
n
= F
n
F
n+1
. (Sugerencia: F
2
n
= F
n
(F
n+1
F
n1
) =
F
n
F
n+1
F
n
F
n1
.)
(d)

n=2
1
Fn1Fn+1
= 1. (Probar que
1
Fn1Fn+1
=
1
Fn1Fn

1
FnFn+1
.)
(e)

n=2
Fn
Fn1Fn+1
= 1.
5.3.10 Sumar las siguientes series innitas:
(a) sen
3
x +
1
3
sen
3
3x +
1
3
2
sen
3
3
2
x +...,
(b) cos
3
x +
1
3
cos
3
3x +
1
3
2
cos
3
3
2
x +....
5.3.11 .
(a) Usar la identidad (k + 1)
3
k
3
= 3k
2
+ 3k + 1 para evaluar la suma
de los primeros n cuadrados. (Sugerencia: Varie k desde 1 hasta n en la
identidad dada, y considerar la suma, sobre el lado izquierdo y sobre el
lado derecho, resultantes de las n ecuaciones.)
5.3. SERIES TELESC

OPICAS 207
(b) Usar la idea de telescopa, como en la parte (a), para evaluar la suma de
los primeros n cubos.
(c) Encontrar
lm
n
_
n

h=1
n

k=1
_
5h
4
18h
2
k
2
+ 5k
4
_
_
.
5.3.12 Probar que el recproco de cada entero mayor que 1 es la suma de un
n umero nito de terminos consecutivos de la serie innita

n=1
1
n(n+1)
.
5.3.13 Si m > 1 es un entero y x es real, denir
f(x) =

k=0
m=1

i=1
__
x +im
k
m
k+1
__
.
Probar que
f(x) =
_
[[x]] si x 0,
[[x + 1]] si x < 0.
(Sugerencia: Ver 1.2.3.)
5.3.14 Resolver las siguienters relaciones de recurrencia (por los metodos de
esta secci on).
(a) x
0
= 1, x
n
= 2x
n1
+ 1 para n > 0. (Sugerencia: Divida cada lado por
2
n
).
(b) x
0
= 0, nx
n
= (n + 2)x
n1
+ 1 para n > 0.
(c) x
0
= 1, x
1
= 1, x
2
= 2, x
n+3
= x
n
+ 3 para n 0.
5.3.15 Probar que un plano queda dividido por n rectas, de modo que cua-
lesquiera dos no sean paralelas ni haya tres concurrentes, en
1
2
_
n
2
+n + 2
_
regiones.
5.3.16 Sea
d
n
=
1
n + 1
+
1
n + 2
+... +
1
2n
.
Probar que
d
n
= 1
1
2
+
1
3

1
4
+... +
1
2n 1

1
2n
.
208 CAP

ITULO 5. C

ALCULO DE SERIES
(Sugerencia: Consoderar la serie telesc opica

n1
i=1
(d
i+1
d
i
). ) Concluir que
d
n
log 2 cuando n . (Para otra prueba de la primera parte, considerar
la diferencia de cada expresi on con la suma arm onica 1 +
1
2
+ ... +
1
2(n1)
.).
Tambien ver la secci on 6.8.
Ejemplos adicionales
6.6.6, 7.1.8, 7.2.2.
5.4. Series de Potencias
Una serie de potencias es una expresi on del tipo
a
0
+a
1
x
1
+a
2
x
2
+... +a
n
x
n
+...
donde a
0
, a
1
, a
2
, ... son n umeros reales.
Dada una serie de potencias, podemos denir una funci on f(x) cuyo dominio es
el conjunto de los n umeros reales x en donde la serie de potencias es una serie
innita convergente, y cuyo valor esta dado por
f(c) = a
0
+a
1
c
1
+a
2
c
2
+... +a
n
c
n
+...
para cualquier c donde el lado derecho converge.
Dada una serie de potencias

i=0
a
i
x
i
, podemos probar que exactamente una
de las siguietntes es verdad.
(i) La serie es convergente para todo n umero real x.
(ii) La serie es convergente s olo para x = 0.
(iii) Existe un n umero real r tal que la serei es convergente para [x[ < r y
divergente para [x[ > r.
Denimos el radio de convergencia que sera + si (i) es verdadera, 0 si (ii) es
verdadera, y r si (iii) es verdadera.
podemos hacernos la siguiente pregunta: Dada una funci on f, podemos repre-
sentar f como una serie de potencias? Uno de los resultados en ese sentido el el
teorema de Taylor (que dice): Si f puede ser derivable un n umero considerable
de veces sobre un intervalo abierto alrededor del cero, entonces para cada x en
ese intervalo abierto,
f(x) = f(0) +
f

(0)
1!
x +
f

(0)
2!
x
2
+... +
f
(n)
(0)
n!
x
n
+R
n
(x),
5.4. SERIES DE POTENCIAS 209
donde R
n
(x) =
f
(n+1)
(c)
(n+1)!
x
n+1
para alg un punto c entre 0 y x. La parte impor-
tante aqu es que si R
n
(x) se comporta bien, esto es R
n
(x) 0 como n ,
entonces
f(x) =

n=0
f
(n)
(0)
n!
x
n
.
Esto da un metodo para encontrar una serie de potencias para una funci on dada
f(x).
Usando esta idea, uno puede encontrar la expansion en serie de potencias para
las funciones comunes elementales. Las siguientes series pueden ser obtenidas y
facilmente memorizadas:
e
z
= 1 +x +
x
2
2!
+
x
3
3!
+... +
x
n
n!
+...,
senx = x
x
3
3!
+
x
5
5!
... + (1)
n1
x
2n1
(2n 1)!
+...,
cos x = 1
x
2
2!
+
x
4
4!
... + (1)
n
x
2n
(2n)!
+...
1
1 x
= 1 +x +x
2
+x
3
+... +x
n
+..., [x[ < 1,
log(1 +x) = x
x
2
2
+
x
3
3
... + (1)
n
x
n
n
+..., [x[ < 1,
(1 +x)
r
= 1 +
_
r
1
_
x +
_
r
2
_
x
2
+... +
_
r
n
_
x
n
+..., r real, [x[ < 1,
donde
_
r
n
_
=
r(r1)(r2)...(rn+1)
n!
.
5.4.1 Probar que e es un n umero irracional.
Soluci on. Supongase que e = h/k, donde h y k son enteros. Usando la expan-
sion en serie de potencias para e
x
y tomando x = 1, tenemos
e
z
= 1 +
1
1!
+
1
2!
+
1
3!
+... +
1
k!
+
_
1
(k + 1)!
+
1
(k + 2)!
+...
_
.
Multiplicando cada lado por k!, y escribiendo en la forma
k!
_
h
k
1
1
1!

1
2!

1
3!
...
1
k!
_
=
1
(k + 1)
+
1
(k + 1)(k + 2)
+...
210 CAP

ITULO 5. C

ALCULO DE SERIES
Notese que el lado derecho de esta ecuaci on es positivo y que el lado izquierdo
es un entero. Luego, el ledo izquierdo es un entero positivo. Ademas en el lado
derecho,
1
(k + 1)
+
1
(k + 1)(k + 2)
+
1
(k + 1)(k + 2)(k + 3)
+...
=
1
(k + 1)
_
1 +
1
(k + 2)
+
1
(k + 2)(k + 3)
+...
_
<
1
(k + 1)
_
1 +
1
(k + 1)
+
_
1
(k + 1)
_
2
+...
_
=
1
(k + 1)
_
1
1
1
(k+1)
_
=
1
(k + 1)
_
(k + 1)
k
_
=
1
k
< 1.
Luego, el lado derecho no es un entero positivo, y tenemos una contradicci on.
De donde se deduce que e es irracional.
5.4.2 Probar que la representaci on en serie de potencias para la serie innita

n=0
x
n
(x1)
2n
/n! no puede tener tres coecientes consecutivos iguales a cero.
Soluci on La serie es la suma de f(x) = e
x(x1)
2
. Para encontrar la rere-
sentaci on en serie de potencias, necesitamos calcular f
(n)
(0) para n = 1, 2, 3, ...
.Tenemos
f

(x) = e
x(x1)
2 _
3x
2
4x + 1
_
que es de la forma
f

(x) = f(x)g(x)
donde g(x) es un polinomio dse grado 2. Se sigue que
f

(x) = f

(x)g(x) +f(x)g

(x)
f

(x) = f

(x)g(x) + 2f

(x)g

(x) +f(x)g

(x)
f
iv
(x) = f

(x)g(x) + 3f

(x)g

(x) + 3f

(x)g

(x)
(observaci on: g

(x) = 0). Un argumento inductivo prueba que para n = 3, 4, 5, ...


f
n+1
(x) = f
(n)
(x)g(x) +a
n
f
(n1)
(x)g

(x) +b
n
f
(n2)
(x)g

(x)
para algunos enteros a
n
y b
n
.
Supongamos que existen tres terminos sucesivos en la serie de potencias para
f(x) que son cero, sean f
(n1)
(0) = f
(n)
(0) = f
(n+1)
(0) = 0. entonces la
formula de recurrencia anterior implica que f
(k)
(0) = 0 para todo k > n, y
esto prueba que f(x) es un polinomio, una contradiccion. De donde concluimos
que la serie de potencias para f(x) no puede tener tres coecientes consecutivos
iguales a cero.
5.4. SERIES DE POTENCIAS 211
5.4.3 Evaluar lm
x
_
(e/2) x +x
2
[(1 + 1/x)
x
e]

.
Soluci on Encontrando los primeros terminos de la serie de Taylor para (1 + 1/x)
x
en potencias de 1/x. Tenemos
(1 + 1/x)
x
= e
x log(1+1/x)
= exp
_
x
_
(1/x)
(1/x)
2
2
+
(1/x)
3
3
+...
__
= exp
_
1
1
2
_
1
x
_
+
1
3
_
1
x
_
2

1
4
_
1
x
_
3
+...
_
= ee

1
2
(
1
x
)
e
1
3
(
1
x
)
2
e

1
4
(
1
x
)
3
...
= e
_
1
1
2x
+
1
2!
_
1
2x
_
2
...
_

_
_
1 +
1
3
_
1
x
_
2
+
1
2!
_
1
3
_
1
x
_
2
_
2
+...
_
_
...
= e
_
1
1
2x
+
11
24
_
1
x
_
2
+ potencias mayores de
1
x
_
.
Se sigue que
lm
x
_
e
2
x +x
2
__
1 +
1
x
_
x
e
__
= lm
x
_
e
2
x +x
2
_
e
e
2x
+
11e
24
_
1
x
_
2
+ ... e
__
= lm
x
_
11e
24
+ potencias mayores de
1
x
_
=
11e
24
.
Un hecho frecuentemente usado de las series de potencias es que pueden ser
derivadas o integradas termino a termino en el intervalo de convergencia. Por
esto decimos que si

a
n
x
n
tiene radio de convergencia r y si f(x) =

a
n
x
n
entonces
f

(x) =

na
n
x
n1
y
_
x
0
f(t)dt =

a
n
x
n+1
n + 1
,
y ambas series resultantes tienen radio de convergencia r.
Una consecuencia del resultado precedente es que la representacon en serie de
potencias de una funci on f es unica; Es decir, si f(x) =

a
n
x
n
=

b
n
x
n
,
212 CAP

ITULO 5. C

ALCULO DE SERIES
entonces a
n
= b
n
para todo n. En efecto, a
n
= b
n
= f
(n)
(0) /n!. Para ver esto,
simplemente derivamos en x = 0. Por ejemplo, a
0
= f (0) ; f

(x) =

na
n
x
n1
,
luego f

(0) = a
1
; f

(x) =

n(n 1)a
n
x
n2
, luego f

(0) = 2!a
2
, o equivalen-
temente a
2
= f

(0)/2!, y as sucesivamente.
5.4.4 Sumar la serie innita
1
2
0!
+
2
2
1!
+
3
2
2!
+
4
2
3!
+....
Soluci on. Comenzando con la serie
e
x
=

n=0
x
n
n!
.
Multiplicando cada lado por x :
xe
x
=

n=0
x
n+1
n!
y derivando cada lado:
(1 +x) e
x
=

n=0
(n + 1) x
n
n!
.
Multiplicando nuevamente cada lado por x :
(1 +x) xe
x
=

n=0
(n + 1) x
n+1
n!
y volviendo a derivar
_
1 + 3x +x
2
_
e
x
=

n=0
(n + 1)
2
x
n
n!
.
Ahora tomando x = 1, y se encuentra que

n=0
(n + 1)
2
n!
= 5e.
El siguiente teorema se obtiene facilmente.
Teorema del Lmite de Abel. Sea r > 0, y sup ongase que

a
n
r
n
converge.
Entonces

a
n
x
n
converge absolutamente para [x[ < r, y
lm
xr

n=0
a
n
x
n
=

n=0
a
n
r
n
.
5.4. SERIES DE POTENCIAS 213
5.4.5 Sumar la serie innita
1
1
4
+
1
7

1
10
+....
Soluci on Sabemos que
1
1 +x
3
= 1 x
3
+x
6
x
9
+..., [x[ < 1,
y de aqu
_
x
0
dx
1 +x
3
= x
x
4
4
+
x
7
7

x
10
10
+..., [x[ < 1,
Ahora, la serie del lado derecho converge para [x[ < 1 (por la prueba de la serie
alternante), y de aqu, por el teorema del lmite de Abel,
1
1
4
+
1
7

1
10
+... = lm
x1
_
x
0
dx
1 +x
3
.
Esta integral puede ser trabajada por fracciones parciales (los detalles no son
de interes en este tema), y tenemos
_
x
0
dx
1 +x
3
=
1
3
_
log
1 +x

1 x +x
2
+

3
_
arctan
2x 1

3
arctan
1

3
__
.
Luego la suma de la serie es
1
3
_
log 2 +

3
_
.
5.4.6 Sea S
n
=

n
i=1
(1)
i+1
/i y S = lm
n
S
n
. Probar que

n=1
(S
n
S) =
log 2
1
2
.
Soluci on. Procedemos a evaluar la doble serie

n=1

i=1
(1)
n+i
n +i
.
Para este prop osito considerar la funci on
F(x) =

j=1

i=1
(x)
i+j
i +j
, [x[ < 1.
214 CAP

ITULO 5. C

ALCULO DE SERIES
Entonces
F

(x) =

j=1

i=1
(x)
i+j1
(1)
=

j=1
(x)
j
(1)

i=1
(x)
i1
=

j=1
(x)
j
(1)
1
1 +x
=
x
1 +x

j=0
(x)
j
=
x
(1 +x)
2
=
1
1 +x

1
(1 +x)
2
.
Se sigue que
_
x
0
F

(x)dx =
_
x
0
1
1 +x
dx
_
x
0
1
(1 +x)
2
dx,
F(x) F(0) = log (1 +x)[
x
0
+
1
1 +x

x
0
,
y encontramos
F(x) = log(1 +x) +
1
1 +x
1.
La serie para F(x) es convergente para x = 1, luego por el teorema del lmite
de Abel,
F(1) = log 2 +
1
2
1 = log 2
1
2
.
5.4.7 Dada la serie de potencias a
0
+ a
1
x + a
2
x
2
+ ... con a
n
=
_
n
2
+ 1
_
3
n
,
probar que existe una relaci on de la forma
a
n
+pa
n+1
+qa
n+2
+ra
n+3
= 0,
donde p, q, r son independientes de n. Encontrar estas constantes y la suma de
la serie.
Soluci on. Sustituyendo los valores dados de a
n
en la recurrencia, encontramos
_
n
2
+ 1
_
3
n
+p
_
n
2
+ 2n + 2
_
3
n+1
+q
_
n
2
+ 4n + 5
_
3
n+2
+r
_
n
2
+ 6n + 10
_
3
n+3
= 0.
Ahora dividimos cada lado por 3
n
. Entonces igualando los coecientes, encon-
tramos que p, q, r deben satisfacer
3p + 9q + 27r = 1,
2p + 12q + 54r = 0,
6p + 45q + 270r = 1.
5.4. SERIES DE POTENCIAS 215
Estas ecuaciones tienen una soluci on: p = 1, q =
1
3
, r =
1
27
.
Para la segunda parte, hacemos la suma de la serie

n=0
_
n
2
+ 1
_
3
n
x
n
,
dividiendola en dos partes
4

n=0
n
2
(3x)
n
+

n=0
(3x)
n
.
Sea S =

n=0
(3x)
n
. Si [x[ <
1
3
, S = 1/ (1 3x) . Luego, de aqu

n=0
(3x)
n
=
1
1 3x
, [x[ <
1
3
,
se sigue que

n=0
n(3x)
n1
3 =
3
(1 3x)
2
,

n=0
n(3x)
n
=
3x
(1 3x)
2

n=0
n
2
(3x)
n1
3 =
d
dx
_
3x
(1 3x)
2
_
=
9x + 3
(1 3x)
3
,

n=0
n
2
(3x)
n
=
3x(3x + 1)
(1 3x)
3
.
Combinando, tenemos

n=0
_
n
2
+ 1
_
(3x)
n
=
3x(3x + 1)
(1 3x)
3
+
1
1 3x
=
18x
2
3x + 1
(1 3x)
3
.
5.4.8 Evaluar en forma cerrada:
S
n
=
n

k=0
(4)
k
_
n +k
2k
_
.
216 CAP

ITULO 5. C

ALCULO DE SERIES
Soluci on. Podemos calcular los primeros terminos:
S
0
= 1,
S
1
=
_
1
0
_
4
_
2
2
_
= 1 4 = 3,
S
2
=
_
2
0
_
4
_
3
2
_
+ 16
_
4
4
_
= 1 12 + 16 = 5,
S
3
=
_
3
0
_
4
_
4
2
_
+ 16
_
5
4
_
64
_
6
6
_
= 1 24 + 80 64 = 7.
De este patr on, tenemos la espectativa de que S
n
= (1)
n
(2n + 1) .
Si queremos probar la conjetura por induccion matematica, veamos una relaci on
de recurrencia. Esta se obtiene del siguiente razonamiento:
_
n +k
2k
_
=
_
n +k 1
2k 1
_
+
_
n +k 1
2k
_
=
__
n +k 2
2k 2
_
+
_
n +k 2
2k 1
__
+
_
n +k 1
2k
_
=
_
n +k 2
2k 2
_
+
__
n +k 1
2k
_

_
n +k 2
2k
__
+
_
n +k 1
2k
_
=
_
n +k 2
2k 2
_
+ 2
_
n +k 1
2k
_

_
n +k 2
2k
_
.
5.4. SERIES DE POTENCIAS 217
Luego,
S
n
=
n

k=0
(4)
k
_
n +k
2k
_
=
n

k=0
(4)
k
_
n +k 2
2k 2
_
+2
n

k=0
(4)
k
_
n +k 1
2k
_

k=0
(4)
k
_
n +k 2
2k
_
=
n

k=0
(4)
k
_
n +k 2
2k 2
_
+2
n1

k=0
(4)
k
_
n 1 +k
2k
_

n2

k=0
(4)
k
_
n 2 +k
2k
_
= 4
n

k=0
(4)
k1
_
n +k 2
2k 2
_
+ 2S
n1
S
n2
= 4
n1

k=0
(4)
k
_
n +k 1
2k 2
_
+ 2S
n1
S
n2
= 4S
n1
+ 2S
n1
S
n2
= 2S
n1
S
n2
.
Usando esta relacion de recurrencia, podemos usar induccion matematica para
establecer la veracidad de S
n
= (1)
n
(2n + 1) .
Considerando la relacon de recurrencia
S
n
= 2S
n1
S
n2
. S
0
= 1, S
1
= 3,
y, por la sake de la ilustraci on, si suponemos que la formula descubierta para
S
n
por la consideraci on de los primeros casos no es cierta. Daremos aqu una
tecnica para este descubrimiento. El metodo que usaremos es el de generar la
funci on F(x), de la manera siguiente.
Sea F(x) la serie de potencias cuyos coecientes son S
0
, S
1
, S
2
, ..., es decir,
F(x) = S
0
+S
1
x +S
2
x
2
+... +S
n
x
n
+....
We will act as though la serie converge en x a la funci on F(x). Entonces
2xF(x) = 2S
0
x + 2S
1
x
2
+ 2S
2
x
3
+... + 2S
n1
x
n
+....
x
2
F(x) = S
0
x
2
+S
1
x
3
+... +S
n2
x
n
+....
218 CAP

ITULO 5. C

ALCULO DE SERIES
Sumando y haciendo uso del hecho de que S
n
+2S
n1
+2S
n2
= 0, Encontramos
que
_
1 2x +x
2
_
F(x) = S
0
+ (S
1
+ 2S
0
) x,
o, equivalentemente,
F(x) =
1 x
(1 +x)
2
.
Ahorta expresemos el lado derecho de esta ecuaci on en serie de potencias. Me-
diante, la derivada de cada lado de
1
1 +x
=

n=0
(1)
n
x
n
obteniendo
,1
(1 +x)
2
=

n=0
(1)
n
nx
n1
Entonces multiplicando cada lado por x 1 tenemos
F(x) =

n=0
(1)
n
n(x 1) x
n1
=

n=0
(1)
n
nx
n

n=0
(1)
n
nx
n1
=

n=0
(1)
n
nx
n
+

n=0
(1)
n
(n + 1)x
n
=

n=0
(1)
n
(2n + 1)x
n
.
De aqu, encontramos que S
n
, el coeciente de x
n
, es (1)
n
(2n + 1).
5.4.9 Sumar la serie nita a
0
+a
1
+... +a
n
donde a
0
= 2, a
1
= 5, para n > 1,
a
n
= 5a
n1
6a
n2
.
Soluci on Los primeros terminos de la sucesion a
i
son
2, 5, 13, 35, 97, 275, 393, ...
Aqu, una formula general para el enesimo termino no es aparente, as que
turnemos el trabajo a la generacion de funciones. Considerar
F(x) = a
0
+a
1
x +a
2
x
2
+... +a
n
x
n
+...
5.4. SERIES DE POTENCIAS 219
Tenemos
5xF(x) = 5a
0
x 5a
1
x
2
5a
2
x
3
... 5a
n1
x
n
+...,
6x
2
F(x) = 6a
0
x
2
+ 6a
1
x
3
+ 6a
2
x
4
+... + 6a
n2
x
n
+...,
Sumando y usando la recurrencia a
n
5a
n1
+ 6a
n2
= 0, tenemos
1 5x + 6x
2
F(x) = a
0
+ (a
1
5a
01
) x,
luego que
F(x) =
2 5x
(1 2x) (1 3x)
.
Si se escribe esta como una suma de fracciones parciales, y hacemos uso de la
serie geometrica, encontramos que
F(x) =
1
1 2x
+
1
1 3x
=

i=0
(2x)
i
+

i=0
(3x)
i
=

i=0
_
2
i
+ 3
i
_
x.
Luego, a
i
= 2
i
+ 3
i
, para i = 0, 1, 2, 3, ....(Como una prueba, podemos vericar
esta ormula por induccion. Note que a
0
= 2
0
+ 3
0
= 2, a
1
= 2
1
+ 3
1
= 5,
y que, para i 2, a
i
= 5a
i1
6a
i2
= 5
_
2
i1
+ 3
i1
_
6
_
2
i2
+ 3
i2
_
=
5
_
2
i1
+ 3
i1
_
3 2
i1
2 3
i1
= 2
i
+ 3
i
).
Hacemos ahora facilmente el c aculo de la suma:
a
0
+a
1
+... +a
n
=
n

i=0
_
2
i
+ 3
i
_
=
n

i=0
2
i
+
n

i=0
3
i
=
2
n+1
1
2 1
+
3
n+1
1
3 1
= 2
n+1
1 +
3
n+1
1
2
=
2
n+2
+ 3
n+1
3
2
.
5.4.10 Encontrar una expresi on en forma cerrada para T
n
, si T
0
= 1 y para
n 1
T
n
= T
0
T
n1
+T
1
T
n2
+... +T
n1
T
0
.
220 CAP

ITULO 5. C

ALCULO DE SERIES
Soluci` on. Esta relaci on de recurrencia fue inducida en 2.5.12 Resolv amosla,
sea
f(x) = T
n
= T
0
+T
1
x +T
2
x
2
+... +T
n
x
n
+...
y hagamos
F(x) = xf(x) = T
0
x +T
1
x
2
+T
2
x
3
+... +T
n
x
n+1
+....
La raz on de este paso es que
(F(x))
2
= T
2
0
x
2
+ (T
0
T
1
+T
1
T
0
) x
3
+...
+(T
0
T
n1
+T
1
T
n2
+... +T
n1
T
0
) x
n1
+...,
en vista de la relaci on de recurrencia tenemos
(F(x))
2
= T
1
x
2
+T
2
x
3
+... +T
n
x
n+1
+...
= F(x) T
0
x.
Us ando la f ormula cuadr atica encontramos que
F(x) =
1

1 4x
2
.
(Desechamos el signo negativo debido a que F(0) = 0; con el signo positivo
F(0) = 1.)
Ahora por la expansi on en serie de potencias,

1 4x = 1 +
_
1
2
1
_
(4x) +
_
1
2
2
_
(4x)
2
+...
+
_
1
2
n + 1
_
(4x)
n+1
+....
Se tiene entonces que el coeciente de x
n+1
en F(x) es
T
n
=
1
2
_
1
2
n + 1
_
(4)
n+1
=
1
2
_
1
2
_ _

1
2
_ _

3
2
_

_

2n1
2
_
(n + 1)!
(1)
n+1
4
n+1
=
1
2
1 3 5 (2n 1)
(n + 1)!
(1)
n
2
n+1
(1)
n+1
4
n+1
=
1
2
1 2 3 4 2n
(n + 1)!2
n
n!
4
n+1
2
n+1
=
1
n + 1

(2n)!
n!n!
=
1
n + 1
_
2n
n
_
.
5.4. SERIES DE POTENCIAS 221
DE manera an aloga como en el caso de los n umeros reales, podemos introducir
la noci on de una serie de potencias con variable compleja

n=0
a
n
z
n
,
donde los vcoecientes son n umeros complejos y z es una variable compleja. Los
valores de z para los cuales la serie converge dene una funci on
f(z) =

n=0
a
n
z
n
.
Podemos probar que las series de potencias (i) a (vi) dadas para funciones el-
ementales en el comienzo de esta secci on continuan siendo v alidas cuando la
variable real x es reemplazada por la variable compleja z.
Un hecho a destacar de las series de potencias complejas es que si f(z) =

n=0
a
n
z
n
entonces (Re) (f(z)) =

n=0
(Re) (a
n
z
n
) y (Im) (f(z)) =

n=0
(Im) (a
n
z
n
) .
Como un ejemplo, podemos justicar el uso de la formula e
i
= cos + isen
introducido en la seccion 3.5. Tenemos
(Re)e
i
= (Re)

n=0
(i)
n
n!
=

n=0
(Re)
(i)
n
n!
=

k=0
(1)
k
()
2k
(2k)!
= cos
y
(Im)e
i
= (Im)

n=0
(i)
n
n!
=

n=0
(Im)
(i)
n
n!
=

k=0
(1)
k
()
2k+1
(2k + 1)!
= sen
Se sigue que e
i
= (Re)e
i
+i(Im)e
i
= cos +isen.
5.4.11 Suma la serie innita
S = r cos +
r
2
2
cos 2 +
r
3
3
cos 3 +..., 0 < r < 1, 0 < < .
Soluci` on. Considerar la serie innita
log (1 z) = z +
z
2
2
+
z
3
3
+... +
z
n
3
+..., [z[ < 1,
222 CAP

ITULO 5. C

ALCULO DE SERIES
Un hecho frecuentemente usado de las series de potencias es que pueden ser
derivadas o integradas termino a termino en el intervalo de convergencia. Por
esto decimos que si

a
n
x
n
tiene radio de convergencia r y si f(x) =

a
n
x
n
entonces
f

(x) =

na
n
x
n1
y
_
x
0
f(t)dt =

a
n
x
n+1
n + 1
,
y ambas series resultantes tienen radio de convergencia r.
Una consecuencia del resultado precedente es que la representac on en serie de
potencias de una funci on f es unica; Es decir, si f(x) =

a
n
x
n
=

b
n
x
n
,
entonces a
n
= b
n
para todo n. En efecto, a
n
= b
n
= f
(n)
(0) /n!. Para ver esto,
simplemente derivamos en x = 0. Por ejemplo, a
0
= f (0) ; f

(x) =

na
n
x
n1
,
luego f

(0) = a
1
; f

(x) =

n(n 1)a
n
x
n2
, luego f

(0) = 2!a
2
, o equivalen-
temente a
2
= f

(0)/2!, y as sucesivamente.
Problemas
5.4.12 Sean p y q n umeros reales con 1/p 1/q = 1, 0 < p
1
2
. Probar que
p +
1
2
p
2
+
1
3
p
3
+... = q
1
2
q
2
+
1
3
q
3
....
5.4.13 Encontrar la expansi on en series de potencias de cada una de las sigu-
ientes:
(a)
1
x
2
+5x+6
.
(b)
1+x
(1+x
2
)(1x)
2
.
(c) arcsenx
(d) arctanx (use esta para encontrar la serie de n umeros raciones que con-
verge a ).
5.4.14 Sumar la siguiente serie innita:
(a)

n=0
_
4
2
r
2
_
n
(2n + 1)!
, r es un entero distinto de cero.
5.4. SERIES DE POTENCIAS 223
(b) 1 +
1
3
+
13
2!3
2
+
135
3!3
3
+....
(c)
2
9
+
2
2!
_
2
9
_
2
+
25
3!
_
2
9
_
3
+
258
4!
_
2
9
_
4
+....
(d)
1
2
1!
x +
1
2
+2
2
2!
x
2
+
1
2
+2
2
+3
2
3!
x
3
+
1
2
+2
2
+3
2
+4
2
4!
x
4
+....
5.4.15 Sea f
0
(x) = e
x
y f
n+1
(x) = xf

n
(x) para n = 0, 1, 2, .... Probar que

n=0
f
n
(1) =
n!
= e
e
.
(Sugerencia: Considerar g(x) = e
e
x
).
5.4.16 Probar que el valor de la nesima derivada de
x
3
x
2
1
para x = 0 es cero
si n es par y n! si n es impar y mayor que 1.
5.4.17 Probar que la ecuaci on funcional
f
_
2x
x
2
+ 1
_
=
_
x
2
+ 1
_
f(x)
es satisfecha por
f(x) = 1 +
1
3
x
2
+
1
5
x
4
+
1
7
x
6
+..., [x[ < 1.
5.4.18 Usando series de potencias, probar que sen(x + y) = senxcos y +
seny cos x.
5.4.19 Probar que
senx
1 x
= x +x
2
+ (1
1
3!
)x
3
+ (1
1
3!
)x
4
+(1
1
3!
+
1
5!
)x
5
+ (1
1
3!
+
1
5!
)x
6
+(1
1
3!
+
1
5!

1
7!
)x
7
+ (1
1
3!
+
1
5!

1
7!
)x
8
+....
224 CAP

ITULO 5. C

ALCULO DE SERIES
5.4.20 Sea B(n) el n umero de unos de la expresi on en base 2 para el en-
tero positivo n. Por ejemplo, B(6) = B(110
2
) = 2 y B(15) = B(1111
2
) = 4.
Determinar la validez de

n=1
B(n)
n(n + 1)
es un n umero racional.
5.4.21 Para cualquier n umero real a encontrar la sucesi on denida por la
condici on inicial u
0
= a y la recursi on u
n+1
= 2u
n
n
2
tomando u
n
> 0 para
todo n > 0.
5.4.22 Probar que
_
1 +x
1 x
_
3
= 1 + 6x + 18x
2
+... + (4n
2
+ 2)x
n
+..., [x[ < 1.
5.4.23 Sea T
n
=

n
i=1
(1)
i+1
(2i 1) , T = lm
n
T
n
. Probar que
n

i=1
(T
n
T) =

8

1
4
.
5.4.24 Resolver la relacion de recurrencia a
0
= 1, a
1
= 0, a
2
= 5 y para
n 3
a
n
= 4a
n1
5a
n2
+ 2a
n3
.
5.4.25 Usar la tecnica de generaci on de funciones para probar que el n-esimo
n umero de Fibonacci es igual a
F
n
=
_
1 +

5
2
_
n
+
_
1

5
2
_
n

5
.
5.4.26 Sumar la serie nita a
0
+a
1
+a
2
+... +a
n
, donde a
0
= 2, a
1
= 17, y
para i > 1, a
i
= 7a
i1
12a
i2
.
5.4. SERIES DE POTENCIAS 225
5.4.27 Probar que la serie de potencias para la funci on e
ax
cos bx, a > 0, b > 0
en potencias de x no tiene coecientes cero o tiene una innidad de coecientes
iguales a cero.
5.4.28 Sumar la serie innita
S = 1 2r cos + 3r
2
cos 2 4r
3
cos +..., [r[ < 1.
5.4.29 Probar que

n
i=1
(senn) /n! = sen(sen)e
cos
.
5.4.30 Usar series innitas para evaluar lim
x
_
3

x3 5x2 + 1 x

.
Ejemplos adicionales
1.12.1, 5.3.16, 6.8.1, 7.6.7(c). Tambien, ver seccion 5.2 (Seres geometricas) y
Seccion 7.5 (Desigualdades por Series).
226 CAP

ITULO 5. C

ALCULO DE SERIES
Captulo 6
Analisis Real Intermedio
En este captulo revisaremos, a traves de algunos problemas, el sistema de deni-
ciones y resultados concernientes a la continuidad, difernciabilidad e integraci on
de funciones. Encontraremos lo que los lectores entienden de lmite y revisaremos
las deniciones m as importantes (continuidad en la seccion 6.1, diferenciabilidad
en la seccion 6.3 e intergrabilidad en la seccion 6.8). Taqmbien pondremos aten-
cion a las propiedades m as importantes de estas clases de funciones. Usaremos
lo conocido, por ejemplo, que si un problema involucra una funci on continua,
entonces nos permitiremos aplicar el teorema del valor intermedio o el teorema
del valor extremo; o ambos, si el problema involucra una funcion diferenciable,
espectativamente aplicaremos el teorema del valor medio. Ejemplos de estas
aplicaciones son incluidas en este captulo, tambien veremos aplicaciones de la
regla de LHopital y del teroema fundamental del c alculo.
A traves de este captulo, R denota el conjunto de los n umeros reales.
6.1. Funciones Continuas
Una funcion real de valores reales es continua en a si f(x) f(x) cuando
x a, o m as precisamente, si
(i) f(a) esta denida,
(ii) lm
xa
f(x) existe, y
(iii) lm
xa
f(x) = f(a).
227
228 CAP

ITULO 6. AN

ALISIS REAL INTERMEDIO


(Si a es un punto frontera en el dominio de f, se entiende que las x de ii)
eswtan rstringidas al dominio de f. Asumiremos que el lector esta familiarizado
con estas contingencias).
Una funci on f es continua en un dominio D si es continua en cada punto de D.
Sin ninguna dicultad probaremos que f es continua en a si y s olo si para cadas
sucesion x
n
que converge a a la sucesion f(x
n
) converge a f(a).
La forma secuencial de la denicion de continuidad de f es usada m as bien para
obtener un caso para probar que una funci on es discontinua en un punto. Por
ejemplo, la funci on f denida por
f(x) =
_
sen
1
x
si x ,= 0
0 si x = 0
es discontinua en 0, ya que, la sucesion x
n
=
2
(4n+1)
converge a 0, mientras que
la sucesion
_
f(x
n
) = sen(2n +

2
)
_
converge a 1 (recuerde que f(0) = 0).
6.1.1 Dena f : [0, 1] [0, 1] de la siguiente manera : f(1) = 1, y si a =
.a
1
a
2
a
3
a
4
... es la representaci on decimal de a (escrito de manera terminal cuan-
do sea posible, por ejemplo, .09999... ser a reemplazado por .1), denimos f(a) =
,0a
1
0a
2
0a
3
0a
4
... . Discutir la continuidad de f.
Soluci on. Observese que f es una funci on monotona creciente. Probaremos
que f es discontinua en cada n umero decimal terminal (es decir, en cada punto
de la forma
N
10
n
, N es un entero, 1 N < 10
n
).
Consideremos, como un ejemplo, el punto a = ,413. Por denicion, f(a) =
,040103. Ahora denamos laq sucesion x
n
por
x
1
= ,4129
x
2
= ,41299
x
3
= ,412999
.
.
.
x
n
= ,412999..,9.
La sucesion x
n
converge a a; adem as
f(x
n
) = ,0401020909..,09,
y podemos ver que f(x
n
) no converge a f(a). Luego f no es continua en a.
6.1. FUNCIONES CONTINUAS 229
Una construcci on similar puedde ser hecha para probar que f es discontinua en
cada n umero decimal terminal. El argumento esta basado en el hecho de que los
n umeros decimales terminales tienen dos representaciones decimales a saber
a = .a
1
a
2
a
3
a
4
....a
n-1
a
n
, a
n
,= 0,
a = .a
1
a
2
a
3
a
4
....a
n-1
(a
n
1)999... .
Ahora supongamos que a (0, 1) no es un n umero decimal terminal. Probemos
que f es una funci on continua en a. Escribimos a en su unica forma decimal:
a = .a
1
a
2
a
3
a
4
....
Debido a que el n umero a no es un n umero terminal decimal, existe un entero
arbitrariamente grande n tal que a
n
,= 0 y a
n
,= 9. Para cada n denimos X
n
y Y
n
por
X
n
= .a
1
a
2
a
3
...a
n
_
=
n

1
a
i
10
i
_
Y
n
= .a
1
a
2
a
3
...a
n
(a
n+1
+ 1) = X
n
+
a
n+1
+ 1
10
n+1
Entonces a (X
n
, Y
n
) . Ademas los primeros n dgitos de cada uno de los
n umeros en (X
n
, Y
n
) son los mismos que de X
n
y Y
n
. consecuentemente todos
los n umeros en (X
n
, Y
n
) son mapeados al intervalo (f(X
n
), f(Y
n
)) .
Es claro que las sucesiones X
n
y Y
n
convergen a a; adem as las sucesiones
f(X
n
) y f(Y
n
) convergen a f(a). Como cualquier sucesion x
n
que con-
verja a a eventualmente cae en el interior de (X
n
, Y
n
) para cualquier n, se tiene
pues que f(x
n
) converge a f(a). Se sigue que f es continua en a.
El ejemplo anterior es difcil de visualizar geometricamente, y una prueba com-
pletamente entendible requiere de un claro entendimiento de la continuidad. El
siguiente ejemplo demanda un preciso usao de la denicion: una funci on f es
continua en a si para cada > 0 existe un n umero > 0 tal que [x a[ <
implica [f(x) f(a)[ < .
6.1.2 Supongamos que f : R R es una funci on continua uno a uno con un
punto jo x
0
(esto es, f(x
0
) = x
0
) tal que f(2xf(x)) = x para toda x. Probar
que f(x) x.
230 CAP

ITULO 6. AN

ALISIS REAL INTERMEDIO


Soluci on Sea S = x [ f(x) = x . Debido a que f es continua, el conjunto S
es un subconjunto cerrado de R (es decir, si x
n
S y x
n
x, entonces x S;
ya que x = lm
n
x
n
= lm
n
f(x
n
) = f(lm
n
x
n
) = f(x).)
Ahora supongamos que S ,= R. Si x
0
es un punto frontera de S (cada vecindad
de x
0
contiene puntos que no estan en S; notar que x
0
S, ya que SS es
cerrado).
Si y es un punto fuera de S, existe un n umero real distinto de cero tal que
f(y) = y +r. El hecho de que f es uno a uno y satisface que f(2x f(x)) = x
implica que
f(y +nr) = (yn +r) +r
para cada entero n (esto es el contenido de 2.1.12). Esta identidad es crucial
en el argumento que sigue. Aqu esta la idea: Supongase que x no esta en S; es
Figura 6.1.
decir, f(x) ,= x. Tomando y en R S cercano a x
0
y f(y) cercano a y (esto
puede hacerse debido a que f es continua en x
0
y f(x
0
) = x
0
). entonces si r es
tal que f(y) = y+r, y si r es sucientemente peque na, el hecho de que f(y+nr)
= (y +nr) +r lleva a una contradiccion con la continuidad de f en x (ver gura
6.1).
Una prueba formal es como sigue. Supongamos, como antes, que x
0
es un punto
frontera de S y que x es tal que f(x) ,= x. Sea = [f(x) x[ . Debido a que f
es continua en x, existe > 0, y podemos asumir que

4
, tal que [z x[ <
implica que [f(z) f(x)[ <

4
. Y como f es continua en x
0
, existe > 0, <
, tal que [w x
0
[ < implica que [f(w) f(x
0
)[ < .
Ahora tomando y (x
0
w, x
0
+w) tal que f(y) ,= y (una tal y existe debido
a que x
0
es punto frontera de S.) Entonces
6.1. FUNCIONES CONTINUAS 231
0 < [f(y) y[ [f(y) f(x
0
)[ +[f(x
0
) y[
= [f(y) f(x
0
)[ +[x
0
y[
< +
< 2
Sea r = f(y) y (nota: r puede ser negativo) Como 0 < [r[ < 2, existe un
entero n talque y +nr (x , x +) . Pero como f(y +nr) = (y +nr) +r. Se
sigue
= [f(x) x[
[f(x) f(y +nr)[ +[f(y +nr) x[


4
+[(y +nr) +r x[


4
+[(y +nr) x[ +[r[
<

4
+ + 2
<

4
+

4
+

2
=
Esta es una contradiccion y por tanto S = R y la prueba esta completa.
Los dos m as importantes hechos acerca de funciones continuas sobre intervalos
cerrados [a, b] son que obtienen sus valores m aximo y mnimo sobre el intervalo
y que toman cada valor entre estos dos. Este es el contenido de los siguientes
dos teoremas.
[Teorema del valor extremo] Si f es una funcion continua sobre [a, b] , entonces
existen n umeros c y d en [a, b] tales que f(c) f(x) f(d) para todo x en [a, b]
(decimos que, f(d) es el valor maximo de f sobre [a, b] , y f(c) el valor mnimo).
[Teorema del valor intermedio] Si f es una funcion continua sobre [a, b] y si
f(a) < y < f(b) (o f(b) < y < f(a)), entonces existe un n umero c en [a, b] tal que
f(c) = y.
Estos resultados pueden ser probados de distintas maneras: haremos una prueba
del teorema del valor intermedio haciendo uso de una metodologa (bisecci on
repetida) que es aplicable en otros problemas (por ejemplo, ver 6.3.6).
Supongamos que f es una funci on continua sobre el intervalo cerrado [a, b] y
supongamos que f(a) < f(b) (una prueba similar se puede hacer si f(b) < f(a)).
232 CAP

ITULO 6. AN

ALISIS REAL INTERMEDIO


Sea y [f(a), f(b)] . Encontremos un elemento c en [a, b] tal que f(c) = y. El
proceso es como sigue (un diagrama puede ayudar). Sean a
0
= a, b
0
= b, y
x
1
el punto medio de [a, b] (la primera bisecci on). Si f(x
1
) < y, denimos
a
1
= x
1
, b
1
= b, de otra forma f(x
1
) > y, denimos a
1
= a, b
1
= x
1
. En
cualquier caso f(a
1
) < y < f(b
1
),y la longitud de [a
1
, b
1
] es la mitad de la
longitud de [a, b] .
Ahora, sea x
2
el punto medio de [a
1
, b
1
] (la segunda bisecci on). Si f(x
2
) < y,
denimos a
2
= x
2
, b
2
= b
1
, y si f(x
2
) > y, denimos a
2
= a
1
, b
2
= x
2
. En
cualquier caso f(a
2
) < y < f(b
2
),y b
2
a
2
= (b a)/4 .
Continuando de esta manera. El resultado es una sucesion innita de intervalos
cerrados encajados
[a
0
, b
0
] [a
1
, b
1
] [a
2
, b
2
] ... ...
cuyas longitudes convergen a cero (en efecto b
i
a
i
= (b a) /2
i
). Estas condi-
ciones implican que a
i
y b
i
convergen al mismo n umero real en [a, b] ;
llamemos este n umero c.
Por la continuidad de f, lm
i
f(a
i
) = f(c) y lm
i
f(b
i
) = f(c). Ademas,
para cada i, f(a
i
) < f(c) < f(b
i
), y por tanto (por el principio del emparedado,
que sera tratado en la seccion 7.6),
f(c) = lm
i
f(a
i
) y lm
i
f(b
i
) = f(c)
se sigue que f(c) = y, y el teorema queda probado.
La prueba del teorema del valor extremo puede hacerse de manera similar y es
propuesto como un problema (6.1.5).
Problemas
6.1.3 Sup ongase que f es acotada para a x b y, para cada par de valores
x
1
, x
2
con a x
1
x
2
b,
f(
1
2
(x
1
+x
2
))
1
2
(f(x
1
) +f(x
2
)).
Probar que f es continua para a x b (Sugerencia: Probar que f(x + )
f(x)
1
2
[f(x + 2) f(x)] ... (1/2
n
) [f(x + 2
n
) f(x)], a < x+2
n
< b.
Tome 0).
6.1. FUNCIONES CONTINUAS 233
6.1.4 Una funci on continua de valores reales satisface que para todos valores
reales x e y la ecuaci on funcional
f
_
_
x
2
+y
2
_
= f(x)f(y).
Probar que f(x) = [f(1)]
x
2
(Sugerencia: primero pruebe el teorema para todos
los n umeros de la forma 2
n/2
donde n es un entero. Pruebe entonces la prueba
del teorema para todos los n umeros de la forma m/2
n
, m entero, n entero no
negativo.)
6.1.5 Usar el metodo de biyecci on sucesiva para probar el teorema del valor
extremo.
6.1.6 Si f(0) > 0, f(1) < 1. Probar que f(x) = 0 para alg un x bajo la suposi-
ci on de que existe una funci on continua g tal que f +g es no decreciente. (Sug-
erencia: Usar la bisecci on sucesiva -tomando el lado derecho del intervalo si ex-
iste un punto x tal que f(x) 0, de otra forma tomar el lado izquierdo. Esto ase-
gura necesariamente una sucesi on de intervalos encajados [a
1
, b
1
] [a
1
, b
1
] ...
que converge a un punto c. Note que para cada n existe un punto y
n
en el
intervalo [a
n
, c] tal que f(y
n
) 0. Probar que f(c) = 0.)
6.1.7 Si f est a denida en el intervalo [0, 1] por
f(x) =
_
0 si x es racional
1/q si x = p/q (reducidos)
(a) Probar que f es discontinua en cada n umero racional en [0, 1]
(b) Probar que f es continua en cada n umero irracional en [0, 1]
6.1.8 Si x es un elemento en el conjunto de Cantor K (ver 3.4.6), puedde ser
expresado en forma unica por
x =

n=1
2b
n
3
n
donde b
n
= 0 o 1. Denir g : K [0, 1] por
g(x) =

n=1
b
n
2
n
234 CAP

ITULO 6. AN

ALISIS REAL INTERMEDIO


Ahora extienda g a [0, 1] de la siguiente manera. Si x [0, 1] no es un elemento
del conjunto de Cantor, entonces, usando la notaci on de 3.4.6, existe un unico
entero n tal que x I
n
, donde I
n
= (X
n
, Y
n
) , X
n
y Y
n
est an en K. Denir
g(x) = g(Y
n
). (Note que para todo n, g(X
n
) = g(Y
n
), y luego le hemos he-
cho constante a g en el intervalo cerrado [X
n
, Y
n
].) Probar que g es continua.
(Tambien vea 6.2.13.)
Ejemplos adicionales
6.3.1, 6.3.5, 6.3.6, 6.4.3, 6.7.2, 6.7.7, 6.8.9, 6.8.10, 6.9.5. La continuidad es una
propiedad que asumimos en la mayora de los ejemplos en el Captulo 6; en
particular, ver la seccion 6.2 (teorema del valor intermedio).
6.2. Teorema del Valor Intermedio
El teorema del valor intermedio establece que si f es una funci on continua sobre
el intervalo cerrado [a, b] y si d esta entre f(a) y f(b), entonces existe un n umero
c entre a y b tal que f(c) = d. El poder del teorema estriba en el hecho de que
provee de una forma de conocer acerca de la existencia de algo sin que se requiera
sea explcitamente encontrado.
Como un ejemplo, probar que 2x
5
+ 4x = 1 tiene una soluci on en el intervalo
(0, 1) . Considerar f(x) = 2x
5
+ 4x 1, y tomar dos puntos extremos: f(0)
como el m as peque no, y f(1) el m as grande. Luego, por el teorema del valor
intermedio, existe un n umero en (0, 1) que satisface la condici on.
6.2.1 Un corredor a campo travieza recorre seis millas en 30 minutos. Probar
que en alg un momento en el transcurso de la carrera recorri o una milla en
exactamente 5 minutos.
Soluci on. Si x denota la distancia recorrida, medida en millas desde el punto
de partida. Para cada x en [0, 5] , si f(x) denota el tiempo que emplea para la
milla desde el punto x hasta el punto x +1. La funci on f es continua. Sabemos
que f(0) + f(1) + f(2) + f(3) + f(4) + f(5) = 30. Se sigue que no todos los
f(0), ..., f(5) son menores que 5, y similarmente, no todos los f(0), ..., f(5) son
mayores que 5. De donde, existen puntos a y b en [0, 5] tales que f(a) 5 f(b).
Luego, por el teorema del valor intermedio existe c entre a y b tal que f(c) = 5;
es decir, la milla desde c hasta c + 1 se recorri o en ex actamente 5 minutos.
6.2.2 Sup omgase que f : [a, b] R es una funci on continua.
(a) El teorema del valor medio para integrales. Probar que existe un n umero
6.2. TEOREMA DEL VALOR INTERMEDIO 235
Figura 6.2.
c en [a, b] tal que
_
b
a
f(t)dt = f(c)(b a).
(b) Probar que existe un n umero c en [a, b] tal que
_
c
a
f(t)dt =
_
b
c
f(t)dt.
(Nota: para esto, es suciente con que f sea integrable sobre [a, b] .)
Soluci on. (a) Sean M y m los valores m aximo y mnimo de la funci on sobre
[a, b] respectivamente (esto esta garantizado por el teorema del valor extremo), y
sea A =
_
b
a
f(t)dt. La intuici on para el argumento que sigue es mostrada (para
el caso de una funci on positiva f) en la Figura 6.2. Como la lnea y = h se
mueve continuamente desde y = m hasta y = M, el area A(h) en el rectangulo
acotado por y = h, y = 0, x = a, x = b se mueve desde un valor m as peque no
que A (en A(m)) hasta uno mayor que A (en A(M)). Algebraicamente (es
cierto independientemente de la interpretacion de area), A(m) = m(b a)
_
b
a
f(t)dt M(b a) = A(M). Como A(h) = h(b a) es una funci on continua
de h, se sigue del teorema del valor intermedio que existe un punto d tal que
A(d) = A; o equivalentemente, d(b a) = A. Pero d esta entre m y M, luego
por el teorema del valor intermedio, ya que f es una funci on continua, existe c
en [a, b] tal que f(c) = d, se sigue que
_
b
a
f(t)dt = f(c)(b a).
(b) Ahora, la intuici on mostrada en la Figura 6.3 (para el caso de una funci on
positiva). Si A =
_
b
a
f(t)dt, y si A(h) =
_
h
a
f(t)dt. En la gura, para a < h < b,
A(h) representa el area acotada por y = f(x), y = 0, x = a, x = h (sombreada).
El problema el problema consiste en encontrar un punto c tal que A(c) =
1
2
A.
Es claro como la lnea vertical x = h se mueve continuamente a la derecha desde
x = a hasta x = b, la correspondiente integral (area) se mueve desde 0 hasta
A, y por lo tanto pasa por
1
2
A en alg un punto. El precedente argumento es
perfectamente valido una vez que probemos que A(h) es una funci on continua
236 CAP

ITULO 6. AN

ALISIS REAL INTERMEDIO


Figura 6.3.
de h. Veamos esto, note que
A(h +x) A(h) =
_
h+x
h
f(t)dt.
De la parte (a), sabemos que existe un punto c
x
, entre h y h +x tal que
_
h+x
h
f(t)dt = f(c
x
)x.
De aqu,
lm
x0
[A(h +x) A(h)] = lm
x0
f(c
x
)x = 0.
(Nota: f(c
x
) es acotado debido a que f es integrable.) Luego A(h +x) A(h)
cuando x 0, y se tiene que A(h) es continua en h.
6.2.3 Si A es un conjunto de 2n puntos en el plano, ninguna terna de ellos
colineales. Supongamos que n de ellos son coloreados de rojo y el resto de azul.
Probar o refutar: existen n segmentos de recta, ning un par de ellos con puntos
comunes, tales que los extremos de cada segmento son puntos de A de diferente
color.
Soluci on. Este problema ha sido considerado en 1.11.2, pero aqu tenemos una
prueba instructiva basada en las propiedades del teorema del valor intermedio.
Haremos la prueba de que el resultado es cierto por induccion sobre n. Cierta-
mente para n = 1 la propiedad es verdadera. supongamos que el resultado es
cierto cuando n = 1, 2, ..., k, y consideremos un conjunto A de 2(k +1) puntos,
ninguna terna de ellos colineales, tales que k+1 estan coloreados de rojo y k+1
coloreados de azul.
6.2. TEOREMA DEL VALOR INTERMEDIO 237
Supongamos que dos vertices de la c ascara convexa de A tienen diferente color.
Entonces, existen dos vertices consecutivos en el permetro de la c ascara convexa
de A, digamos P y Q, que tienen diferente color. Por la hip otesis de induccion,
el conjunto de puntos A P, Q puede ser conectado de la manera descrita.
Ninguno de los segmentos sera intersectado por el segmento PQ debido a la
manera en que P y Q fueron tomados, y por tanto, el resultado es cierto para
el conjunto A.
Ahora consideremos el caso en que todos los vertices de la c ascara convexa tienen
el mismo color, digamos rojo. Si L es cualquier linea no horizontal en el plano, si
B(L) denota el n umero de puntos azules de A a la izquierda de L, R(L) denota
el n umero de puntos rojos de A a la izquierda de L, y si D(L) = B(L) R(L).
Ahora tomemos una recta no horizontal L que deje a la izquierda todos los
puntos de A y que no sea paralela a alg un segmento de los que puedan ser
formados tomando como extremos los puntos de A. En esta posici on D(L) = 0.
Como L se mueve continuamente a la izquierda deber a encontrarse los puntos
de A uno a la vez, y pasando un tal punto, D(L) cambiara +1 si el punto es
azul y 1 si es rojo. Como L se mueve a al izquierda, su primer valor distinto
de cero sera 1 (obtenido justamente despues de pasar el primer punto de A).
Como el ultimo punto de A encontrado por L tambien es rojo, se tiene que el
ultimo valor no cero sera positivo (obtenido justamente antes de pasar el ultimo
punto de A).
Se sigue de estas observaciones que D(L) sera igual a cero en alg un momento
entre el primero y ultimo puntos de A (note que D(L) es una funci on de valores
enteros). Cuando L esta en tal posici on, la hip otesis de induccion puede ser
aplicada a los puntos a la izquierda de L y tambien a los puntos a la derecha
de L. como ning un de los segmentos resultantes se intersectan, el resultado se
sigue para el conjunto A, y por induccion la prueba se completa.
Problemas
6.2.4 Sup ongase que f : [0, 1] [0, 1] es continua. Probar que existe un punto
c en [0, 1] tal que f(c) = c.
6.2.5 Un monta nista comienza a escalar una monta na a las 7:00 A.M. del
s abado y llega a la cima a las 5:00 P.M. Acampa en la cima y comienza a
descender el domingo, exactamente a las 7:00 A.M., siguiento la misma ruta,
llegando al punto de partida a las 5:00 P.M.. Probar que en alg un momento del
da domingo estuvo en el mismo lugar a la misma hora que el s abado.
6.2.6 Probar que una funci on continua puede tomar un valor m as de una vez
pero toma alg un valor exactamente una vez.
238 CAP

ITULO 6. AN

ALISIS REAL INTERMEDIO


6.2.7 Probar que el polinomio trigonometrico
a
0
+a
1
cos x +... +a
n
cos nx,
donde los coecientes son todos n umeros reales y [a
0
[ +[a
1
[ +...+ [a
n1
[ a
n
,
tiene al menos 2n ceros en el intervalo [0, 2) .
6.2.8 Establecer las condiciones necesarias y sucientes sobre la constante k
para la existencia de una funci on continua de valores reales f(x) que satisface
f(f(x)) = kx
9
para todo valor real x.
6.2.9 .
(a) Sup ongase que f : [a, b] R es continua y g : [a, b] R es integrable tal
que g(x) 0 para todo x [a, b] . Probar que existe un n umero c en [a, b]
tal que
_
b
a
f(x)g(x)dx = f(c)
_
b
a
g(x)dx.
(b) Sup ongase que f : [a, b] R es creciente (y adem as integrable), y g :
[a, b] R es integrable y tal que g(x) 0 para todo x [a, b] . Probar que
existe un n umero c en [a, b] tal que
_
b
a
f(x)g(x)dx = f(a)
_
c
a
g(x)dx +f(b)
_
b
c
g(x)dx
6.2.10 Si f : [a, b] R es continua y f(0) = f(1). Probar que para cada
entero positivo n existe un x en
_
0, 1
1
n

tal que f(x) = f(x +


1
n
).
6.2.11 Un polinomio P(t) de grado no mayor que 3 describe la temperatura
de un cierto cuerpo en el tiempo t. Probar que la tempertura media del cuerpo
entre las 9:00 A.M. y las 3:00 P.M. puede ser encontrada tomando el promedio
de la temperatura en dos momentos jos, que son independientes de cualquier
polinomio. Tambien, probar que estos dos momentos son 10:16 A.M. y la 1:44
P.M. (Sugerencia: Use el teorema del valor medio para integrales; ver 6.2.2(a).)
6.2.12 Para cualquier par de tri angulos, probar que existe una recta que los
biseca simultaneamente.
6.2.13 Dar un ejemplo de una funci on continua de valores reales f de [0, 1] en
[0, 1] que toma para cada valor en [0, 1] un n umero innito de veces. (Sugerencia:
Para esto basta modicar la funci on continua denida en 6.1.8.)
6.3. LA DERIVADA 239
Ejemplos adicionales
6.1.6, 6.5.2, 6.5.3, 6.5.4, 6.5.13, 6.6.4, 6.6.5, 6.6.6, 6.6.9, 7.6.13.
6.3. La Derivada
La derivada de f : [a, b] R en un punto x en (a, b) esta denida por
f

(x) = lm
h0
f(x +h) f(x)
h
,
si este lmite existe. Notemos que si f tiene derivada en x, entonces f es continua
en x, debido a
lm
h0
[f(x +h) f(x)] = lm
h0
__
f(x +h) f(x)
h
_
h
_
= lm
h0
__
f(x +h) f(x)
h
__
lm
h0
h
= f

(x) lm
h0
h
= 0
6.3.1 Si la funci on xf(x) tiene derivada en un punto dado x
0
,= 0, y si f es
continua all, probar que f tiene una derivada all.
Soluci on. Sea
L = lm
xx0
xf(x) x
0
f(x
0
)
x x
0
.
El lmite de la derecha existe, ya que representa la derivada de xf(x) en el punto
x
0
(en la denicion de derivada dada anteriormente basta sustituir h por xx
0
).
Para x sucientemente cercana a, pero diferente de, x
0
(y adem as diferente de
240 CAP

ITULO 6. AN

ALISIS REAL INTERMEDIO


cero),
f(x) f(x
0
)
x x
0
=
xf(x)
x

x0f(x0)
x0
x x
0
=
x
0
xf(x) xx
0
f(x
0
)
xx
0
(x x
0
)
=
x
0
xf(x) x
2
f(x) xx
0
f(x
0
) +x
2
f(x)
xx
0
(x x
0
)
=
xf(x)(x
0
x) +x(xf(x) x
0
f(x
0
))
xx
0
(x x
0
)
=
xf(x) x
0
f(x
0
)
x
0
(x x
0
)

f(x)
x
0
Se sigue de aqu que
f

(x
0
) = lm
xx0
f(x) f(x
0
)
x x
0
= lm
xx0
_
xf(x) x
0
f(x
0
)
x
0
(x x
0
)

f(x)
x
0
_
= lm
xx0
xf(x) x
0
f(x
0
)
x
0
(x x
0
)
lm
xx0
f(x)
x
0
=
1
x
0
[L f(x
0
)]
El hecho de que lm
xx0
f(x) = f(x
0
) se sigue de la hip otesis de que f es
continua en x
0
. Sin embargo, esta suposici on no es necesaria, debido a que
f(x) f(x
0
)
x x
0
=
x
0
xf(x) xx
0
f(x
0
)
xx
0
(x x
0
)
=
x
0
xf(x) x
2
0
f(x) xx
0
f(x
0
) +x
2
0
f(x)
xx
0
(x x
0
)
xf(x) x
0
f(x
0
)
x(x x
0
)

f(x
0
)
x
Usando esto tenemos que
f

(x
0
) = lm
xx0
f(x) f(x
0
)
x x
0
= lm
xx0
_
xf(x) x
0
f(x
0
)
x(x x
0
)

f(x
0
)
x
_
=
1
x
0
[L f(x
0
)] .
6.3. LA DERIVADA 241
6.3.2 Si f(x) = a
1
senx + a
2
sen2x + ... + a
n
sennx, donde a
1
, a
2
...a
n
son
n umeros reales y n es entero positivo. Dado que [f(x)[ [senx[ para todo real
x, probar que [a
1
+ 2a
2
+... +na
n
[ 1.
Soluci on Damos una soluci on de este problema por induccion en 2.4.4; sin
embargo, una soluci on m as natural esta basada notando que f

(x) = a
1
cos x +
2a
2
cos 2x + ... + na
n
cos nx, de aqu tenemos que f

(0) = a
1
+ 2a
2
+ ... + na
n
(que es el lado izquierdo de la desigualdad que deseamos probar).

Esto requiere
de revisar lo siguiente
[f

(0)[ = lm
xx0

f(x) f(0)
x 0

= lm
xx0

f(x)
x

lm
xx0

senx
x

= 1.
y se completa la prueba.
6.3.3 Si f es diferenciable en x = a, y f(a) ,= 0. Evaluar
lm
n
_
f(a +
1
n
)
f(a)
_n
.
Soluci on. Es suciente evaluar
lm
x0
_
f(a +x)
f(a)
_1
x
.
Para x sucientemente peque no, f(a + x) y f(a) tienen el mismo signo, y se
sigue que
log
_
lm
x0
_
f(a +x)
f(a)
_1
x
_
= lm
x0
_
log
_
f(a +x)
f(a)
_1
x
_
= lm
x0
log [f(a +x)[ log [f(a)[
x
.
La ultima expresi on de la derecha es la denicion de la derivada de log [f(x)[ en
x = a, que conocemos en c alculo como f(a)/f(a). Luego,
lm
x0
_
f(a +x)
f(a)
_1
x
= e
f(a)/f(a)
.
Problemas
242 CAP

ITULO 6. AN

ALISIS REAL INTERMEDIO


6.3.4 .
(a) Supongamos que de la denici on usual de derivada, la denotamos por
Df(x), denimos una nueva derivada D

f(x) por la f ormula


D

f(x) = lm
h0
f
2
(x +h) f
2
(x)
h
.
Expresar D

f(x) en terminos de Df(x).


(b) Si f es diferenciable en x, calcular
lm
h0
_
f(x +ah) f(x +bh)
h
_
.
(c) Sup ongase que f es diferenciable en x = 0 y que satisface la ecuaci on fun-
cional f(x+y) = f(x)+f(y) para todos x e y. Probar que f es diferenciable
en cada n umero real x.
6.3.5 Denir f por
f(x) =
_
x
2
sen
1
x
si x ,= 0,
0 si x = 0.
(a) probar que f(x) existe para todo x pero f no es continua en x = 0. (La
derivada para x ,= 0 es 2xsen
1
x
cos
1
x
; cu al es la derivada en 0?.)
(b) Si g(x) = x + 2f(x). Probar que g(0) 0 pero que g no es mon otona en
cualquier intervalo alrededor del 0.
6.3.6 Si f : [0, 1] R es una funci on diferenciable. Sup ongase que no existe
x en [0, 1] tal que f(x) = 0 = f(x). Probar que f tiene s olamente un n umero
nito de ceros en [0, 1] . (Sup ongase que existe un n umero innito de ceros. Luego
_
0,
1
2

o
_
1
2
, 1

contiene un n umero innito de ceros (o ambos). tomar uno de


ellos, y continuar por bisecci on repetida. De esta forma, construir una sucesi on
convergente de ceros distintos. Usar este hecho para llegar a una contradicci on).
6.3.7 Probar que si f es diferenciable sobre (a, b) y tiene un extremo (esto es,
un m aximo o un mnimo) en un punto c en (a, b) , entonces f(c) = 0 (Para
aplicaciones de este resultado, ver 6.4.1, 6.4.2, 6.4.5, 6.4.6, 6.4.7, 6.6.4, 7.4.1.)
Ejemplos adicionales 6.6.2, 6.7.2, 6.9.1, 7.6.2.
6.4. EL TEOREMA DEL VALOR EXTREMO 243
6.4. El Teorema del Valor Extremo
Un teorema de existencia es un teorema que establece alguna existencia (por
ejemplo, un punto en el dominio de una funci on que tiene alguna propiedad
establecida). Sin obntener el objeto especial en el que ocurre alguna posici on
extrema.

Esta es la forma en la que uno puede hacer uso del teorema del valor
extremo: Si f es una funci on continua sobre un intervalo cerrado [a, b] , existen
puntos c y d en [a, b] tales que f(c) f(x) f(d) para todo x en [a, b] .
6.4.1 Supongamos que f : [a, b] R es una funci on diferenciable. Probar
que f satisface la conclusi on del teorema del valor intermedio (es decir, si d es
cualquier n umero entre f(a) y f(b), entonces existe un n umero c en el intervalo
(a, b) tal que f(c) = d). Soluci on. Si f fuera una funci on continua el resultado
Figura 6.4.
se sigue de una aplicaci on directa del teorema del valor intermedio (aplicado a
f). Sin embargo, f no necesariamente es continua (por ejemplo, ver 6.3.5 (a)),
c omo debemos proceder?.
Para generar ideas, considerar la Figura 6.4. En esta gura, una recta L de pen-
diente d es dibujada por el punto (a, f(a)) , donde f(b) < d < f(a). Para cada
punto x en [a, b] , sea g(x) la distancia desde el punto (x, f(x)) a la recta L (la
longitud de AB en la gura). La intuici on es que el punto que buscamos es aquel
en que la funci on g alcanza su m aximo. Probemos que este es necesariamente el
caso, pero para simplicar veamos una funci on diferente.
Para cada x en [a, b] , sea h(x) la longitud del segmento vertical desde (x, f(x))
a la recta L (la longitud de BC en la gura). Observemos que el punto en el
que h alcanza su m aximo sobre [a, b] es el mismo en el que lo alcanza g sobre
[a, b] . (Esto es debido a que g(x) = h(x) cos , donde es la inclinacion de L).
La ventaja de considerar la funci on h es que la podemos expresar en terminos
de f y de la ecuaci on de la recta L.
244 CAP

ITULO 6. AN

ALISIS REAL INTERMEDIO


Retornemos al problema planteado, y consideremos la funci on
h(x) = f(x) [f(a) +d(x a)] .
Veamos que
h(x) = f(x) d.
Como f(b) < d < f(a), tenemos que h(b) < 0 < h(a). Esta desigualdad
implica que ni h(a) ni h(b) son valores m aximos de h sobre [a, b] (esta es una
consecuencia de la denicion de la derivada). De donde, como h es una funci on
continua sobre [a, b] , el teorema del valor extremo establece que h toma su valor
m aximo en un punto c de (a, b) . En este punto, por 6.3.7, h(c) = 0, por lo cual
f(c) = d.
Un argumento similar puede hacerse si f(a) < d < f(b). En este caso, h toma
un valor mnimo para alg un punto c de (a, b) , y en este punto f(c) = d.
6.4.2 P es un punto interior del angulo cuyos lados son los rayos OA y OB.
Localizar el punto X sobre OA y Y sobre OB tales que el segmento de recta
XY contiene a P y el producto (PX)(PY ) es un mnimo.
Soluci on. La situaci on esta ilustrada en la Figura 6.5.
El problema es un problema tpico de m aximos y mnimos encontrado en los
cursos de c alculo: aqu la pregunta no es existe el valor mnimo?, sino mas bien
donde ocurre el valor mnimo?. La tecnica consiste en aplicar el resultado de
6.3.7: si el mnimo es un punto interior del intervalo abierto, se tiene que en ese
punto la derivada es cero. Luego, necesitamos expresar (PX)(PY ) como una
funci on de una sola variable, y encontrar el punto dode la derivada es cero.
Para cada n umero positivo x, existe un unico punto X sobre OA tal que x =
[OX[ , y este punto determina a su vez un unico punto Y sobre OB tal que X, P
y Y son colineales. Luego (PX)(PY ) es una funci on de x. Ahora, una expresi on
explcita para esta funci on no es muy facil de encontrar; sin embargo existe una
forma. Notar que (PX)(PY ) queda determinado de forma unica por el angulo
(ver Figura 6.5). Para obtener uan forma explcita para (PX)(PY ), primero
usamos la ley de senos en los tri angulos OXP y OPY teniendo
sen
PX
=
sen
OP
y
sen
PY
=
sen( )
OP
.
Entonces se sigue que
F() = (PX)(PY ) =
sen
sen
(OP)
_
sen
sen( )
_
(OP)
= C(csc )(csc ( )), 0 < <
donde C = sensen(OP)
2
es una constante.
6.4. EL TEOREMA DEL VALOR EXTREMO 245
Figura 6.5.
La funci on F es continua y dertivable sobre (0, ) , y F() cuando 0
+
y cuando

, y por tanto F tiene un mnimo en un punto interior de (0, ) .


Un punto donde F() = 0; esto es
0 = (csc ) csc ( ) [cot cot ( )] .
Como nunca csc ni csc ( ) pueden ser cero en (0, ) , entonces el
mnimo ocurre cuando cot = cot ( ) . Pero debido a que 0 < <
y que 0 < < , solamente sucede cuando = . Luego,
el mnimo ocurre cuando OXY es un tri angulo isosceles; es decir, cuando
OX = OY. (Para otra prueba, ver 8.1.3.)
Problemas
6.4.3 .
(a) Sea f [a, b] R una funci on continua tal que f(x) > 0 para toda x en
[a, b] . Probar que existe una constf

ante positiva c tal que f(x) c para


toda x en [a, b] .
(b) Probar que no existe una funci on continua que mande el intervalo cerrado
[0, 1] en el abierto (0, 1) .
6.4.4 Si f [a, b] R es diferenciable en cada punto de [a, b] , y sup ongase que
f

(a) = f

(b). Probar que existe al menos un punto c en (a, b) tal que


f

(c) =
f(c) f(a)
c a
.
246 CAP

ITULO 6. AN

ALISIS REAL INTERMEDIO


6.4.5 .
(a) Teorema de Rolle. Sup ongase que f [a, b] R es continua sobre [a, b] y
difeenciable sobre (a, b) . Entonces existe un n umero real c en (a, b) tal que
f

(c) = 0.
(b) Teorema del valor medio. Si f [a, b] R es continua sobre [a, b] y difer-
enciable sobre (a, b) , entonces existe un n umero c en (a, b) tal que
f(b) f(a)
b a
= f

(c).
6.4.6 Si A, B, y C son las medidas de los angulos de un tri angulo, probar que
2 sen3A+sen3B +sen3C
3
2

2,
y determinar en que casos se d a la igualdad.
6.4.7 Dado un crculo de radio r y una recta tangente L al crculo que pasa
por el punto P en el crculo. Dado un punto variable R sobre el crculo, trazar
la perpendicular RQ a L con Q en L. Determinar el area m axima del tri angulo
PQR.
Ejemplos Adicionales
1.11.5, 6.6.1, 6.6.4, 6.6.5.
6.5. Teorema de Rolle
Una de las propiedades m as importantes de las funciones diferenciales es el
siguiente teorema de existencia
Teorema de Rolle. Supongase que f : [a, b] R es continua sobre [a, b] y difer-
erenciable sobre (a, b). Si f(a) = f(b), entonces existe un n umero c en (a, b) tal que
f

(c) = 0. Este resultado es consecuencia directa de 6.3.7: Para un punto c dado en


(a, b) tal que f(c) es un extremo (tal punto c existe por el teorema del valor extremo).
Entondes por 6.3.7. f

(c) = 0. El teorema de Rolle es importante desde el punto de


vista teorico (podemos eventualmente probar que el teorema del valor medio y sus col-
orarios son faciles consecuencias del teorema de Rolle), pero es tambien un importante
metodo en la resolucion de problemas.
6.5.1 Probar que 4ax
3
+3bx
2
+2cx = a +b +c tiene al menos una raiz entre cero y
uno.
6.5. TEOREMA DE ROLLE 247
Soluci on. Cualquier intento de aplicar el teorema del valor intermedio (de manera
similar a la solucion de problemas de la seccion 6.2) acarreara complicaciones, debido
a a que carecemos de informacion acerca de los valores de a, b, c. Pero considetrando
la funcion f(x) = ax
4
+bx
3
+cx
2
(a +b +c)x notemos que f(0) = 0 = f(1). Por el
teorema de Rolle, existe un punto d en (0, 1) tal que f

(d) = 0; es decir que, d es una


raiz de 4ax
3
+ 3bx
2
+ 2cx = a +b +c, y la prueba esta completa.
6.5.2 Probar que si las funciones diferenciables f y g que satisfacen f

(x)g(x) =
g

(x)f(x) para toda x, entonces entre cualesquiera dos races de f(x) = 0 existe una
raiz de g(x) = 0
Soluci on. Sean a y b dos races de f, a < b. La condicion implica que ni a ni b son
races de g(x) = 0. Supongase que g no tiene races entre a y b. Entonces como una
consecuencia del teorema del valor intermedio, el signo de g sobre [a, b] es siempre el
mismo (esto es, g(x) > 0 para todo x en [a, b] o g(x) < 0 para todo x en [a, b]). Ahora
consideremos la funcion F(x) = f(x)/g(x). Esta funcion es continua y diferenciable
sobre [a, b] y F(a) = 0 = F(b). Luego, por el teorema de Rolle, existe un punto c tal
que F

(c) = 0. Pero esto es una contradiccion, ya que


F

(x) =
f

(x)g(x) g

(x)f(x)
g(x)
2
Y, por hipotesis f

(x)g(x) g

(x)f(x) = 0. Esta contradiccion implica que g debe


tener un cero entre a y b, y la prueba esta completa.
Un corolario del teorema de Rolle es que si f es una funcion continua y diferenciable,
digamos sobre el intervalo [a, b], y si x1 y x2 son ceros de f, a < x1 < x2 < b, entonces
f tiene un cero entre x1 y x2. Mas generalmente, si f tiene n ceros distintos entonces
f

tiene n 1 ceros distintos (estos intercalados con los ceros de f), f

tiene n 2
ceros (asumiendo que f

es continua y diferenciable sobre [a, b]), y as sucesivamente.


6.5.3 Probar que x
2
= xsenx +cosx para exactamente dos valores reales de x
Soluci on. Considerar la funcion f(x) = x
2
xsenx cosx. Entonces f(/2) > 0,
f(0) < 0, luego el teorema del valor intermedio implica que f tiene al menos dos ceros.
Si f tiene tres o mas ceros, entonces, por la observacion anterior, f

tiene al menos
dos ceros. Pero
f

(x) = 2x senx xcosx +senx


= x[2 cosx]
tiene solamente un cero De donde, f tiene exactamente dos ceros y se cumple lo
requerido.
6.5.4 Sea P(x) un polinomio con coecientes reales y la forma polinomial
Q(x) = (x
2
+ 1)P(x)P

(x) +x
_
(P(x))
2
+ (P

(x))
2

.
248 CAP

ITULO 6. AN

ALISIS REAL INTERMEDIO


Dado que la ecuacion P(x) = 0 tiene n races reales distintas mayores que 1, probar o
refutar que la ecuacion Q(x) = 0 tiene al menos 2n + 1 races reales distintas.
Soluci on. Sean a1, a2, ..., an n races reales distintasde P(x) = 0 con 1 < a1 < a2 <
< an, y escribamos Q(x) en la forma
Q(x) = (x 1)
2
P(x)P

(x) +x
_
(P(x)) + (P

(x))

2
Supongase que P(x) no tiene ceros en ls intervalos abiertos (ai < ai+1), i = 1, 2, ..., n
1. (Esto sin perdida de generalidad, ya que si hay mas, digamos m, m > n renombramos
las a

i
s y las incluimos, continuando la prueba de que Q(x) tiene al menos 2m 1
races reales distintas.) Por teorema de Rolle existe un punto bi en (ai < ai+1) tal que
P

(bi) = 0. Como P es un polinomio P

(x) = 0 tiene solamente un n umero nito de


races en (ai < ai+1) para cada i podemos asumir que bi es el mayor cero de P

en
(ai < ai+1). Supongamos que P(x) es positiva para toda x en (ai, ai+1) (ver Figura
Figura 6.6.
6.6)y considerar la funcion F(x) = P(x) +P

(x). La idea es encontrar un punto ci en


(bi, ai+1) donde F(ci) < 0. Entonces, como F(bi) > 0, el teorema del valor intermedio
implica que existe un punto di en (bi, ci) tal que F(di) = 0 y consecuentemente,
Q(bi) = bi(F(bi))
2
> 0
Q(di) = (di 1)
2
p(di)P

(di) < 0
(note que P

(x) < 0 para toda x en (bi, ai+1)), y


Q(ai+1) = ai+1(F(ai+1))
2
> 0.
Ademas, por el teorema del valor intermedio, existen puntos xi en (bi, di) y yi en
(di, ai+1] tal que Q(xi) = 0 = Q(yi) Por el argumento anterior trabajaremos para
probar que existe un punto ci en (bi, ai+1) donde F(ci) < 0. Si ai+1 es una raiz de
multiplicidad uno, entonces F(ai+1) = P

(ai+1) < 0, y dicho ci puede ser encontrado


en una vecindad sucientemente pequea de ai+1. Si ai+1 es una raiz de multiplicidad
mayor que 1, entonces P(ai+1) = 0 = P

(ai+1) y existe un intervalo (ai+1 , ai+1)


para > 0 sucientemente pequea, donde P

(x) > 0 (ver Figura 6.7). Para tal x se


tiene que
P

(x) <
P(x) P(ai+1)
x ai+1
=
P(x)
x ai+1
6.5. TEOREMA DE ROLLE 249
Figura 6.7.
y de aqu
F(x) = P(x) +P

(x)
< P(x)
_
1 +
1
x ai+1
_
= P(x)
_
x ai+1 + 1
x ai+1
_
Luego, sea ci = x, donde x esta sucientemente cerca de ai+1 tal que el denominador
de la ultima expresion es positivo y el denominador negativo. Entonces para una tal
ci, F(ci) < 0, bi < ci < ai+1. Esto completa el argumento: Q(x) = 0 tiene dos races
en (bi, ai+1)
El argumento anterior esta basado sobre la suposicion de que P(x) > 0 para x en
(ai, ai+1). Para el caso en que P(x) < 0 para toda x en (ai, ai+1), un argumento
exactamente analogo nos llevara a la misma conclusion. Luego, hemos probado que Q
tiene al menos 2n2 ceros (dos en cada uno de los intervalos (ai, ai+1), i = 1, 2, ..., n
1). La solucion debe completarse si probamos que Q tiene un cero en , a1). Existen
varios casos a considerar:
Supongase que P

(x) = 0 tiene una raiz en el intervalo (0, a1). Entonces, haciendo caso
omiso de algunos detalles, el mismo argumento prueba que Q tiene un cero en (b0, a1)
donde b0 es tomado como el cero de P

en (0, a1)
Ahora consideremos el caso en que P

(x) = 0 no tiene una raiz en el intervalo (0, a1).


Si P(x) > 0 para toda x en (0, a1), entonce P

(x) < 0 para toda x en (0, a1) y de aqu,


Q(0) < 0 y Q(a1) > 0. Por el teorema del valor intermedio, Q(x) = 0 tiene una raz
en el intervalo (0, a1). similarmente si P(x) < 0 para toda x en (0, a1), tendremos que
Q(0) < 0 y Q(a1) > 0, etc. Luego, en todos los caso Q(x) = 0 tiene al menos 2n 1
races distintas.
El analisis anterior, parece tedioso y complicado, esta basado enteramente en los prici-
pios basicos: teorema de Rolle y teorema del valor intermedio. con estas dos ideas los
aspectos conceptuales de la prueba resultan naturales y faciles de entender. Existe otra
250 CAP

ITULO 6. AN

ALISIS REAL INTERMEDIO


solucion que es mucho mas facil, despues de un habilidoso, pero no muy com un, paso
clave (por ejemplo, ver 6.5.11 y 6.9.4). Como esto es instructivo, lo consideraremos
tambien. Primero, notese que Q puede ser escrito como un producto en la siguiente
manera:
Q(x) = (x
2
+ 1)P(x)P

(x) +x
_
(P(x))
2
+ (P

(x))
2

<
_
P

(x) +xP(x)
_
xP

(x) +P(x)

Sea F(x) = P

(x) + xP(x) y G(x) = xP

(x) + P(x). El paso clave lo podemos ver,


notando que F(x) = e
x
2
/2
_
e
x
2
/2
P(x)
_

y G(x) = [xP(x)]

Asumiendo que P(x) tienen exactamente m ceros distintos ai mayores que 1, con 1 <
a1 < a2 < < am, m > n. Entonces e
x
2
/2
P(x) tambien tiene ceros en a1, a2, , am,
luego por el teorema de Rolle,
_
e
x
2
/2
P(x)
_

, y en consecuencia F(x), tienen al menos


m 1 ceros bi con ai < bi < ai+1. Similarmente por el teorema de Rolle tuiene al
menos m ceros c0, c1, , cm1, 0 < c0 < a1, ai < ci < ai+1, i = 1, 2, ..., m1. Como
vemos solo falta probar que bi = ci para i = 1, 2, ..., m1.
Asumamos que bi = ci para alg un i, i sea r su valor com un. A partir de que F(r) = 0,
encontramos que P

(r) = rP(r). Sustituyendo esto en G(r) = 0, tenemos que


r[rP(r)] + P(r) = 0, o equivalentemente, (r
2
1)P(r) = 0. Como r > 1, la ulti-
ma ecuacion implica que P(r) = 0. Pero como ai < r < ai+1, entonces tenemos una
contradiccion al haber hecho la suposicion respectiva a las races de P(x) = 0 (re-
cuerdese, hemos asumido que ai y ai+1 son rices consecutivas de P;es decir, todas las
races de P mayores que 1 estn incluidas en las a

i
s. Se sigue entonces que las b

i
s y las
c

i
s son diferentes, y ademas Q(x) = 0 tiene al menos 2m1 ( 2n 1) races reales
distintas.
Problemas
6.5.5 .
(a) Probar que 5x
4
4x + 1 tiene una raz entre 0 y 1.
(b) Si a0, a1, , an son n umeros reales que satisfacen
a0
1
+
a1
2
+ +
an
n + 1
= 0,
probar que la ecuacion a0 +a1x + +anx
n
= 0 tiene al menos una raz real.
6.5.6 .
6.5. TEOREMA DE ROLLE 251
(a) Supongase que f : [0, 1]
mathbfR es diferenciable, f(0) = 0 y f(x) > 0 para x en (0, 1). Probar que
existe un n umero c en (0, 1). tal que
2f

(c)
f(c)
=
f

(1 c)
f(1 c)
.
(sugerencia. Considerar f
2
(x)f(1 x).)
(b) Existe un n umero d en (0, 1). tal que
3f

(d)
f(c)
=
f

(1 c)
f(1 c)
?.
6.5.7 .
(a) Teorema del valor medio de Cauchy. Si f y g son funciones continuas sobre
[a, b] y diferenciables sobre (a, b), entonces existe un n umero c tal que
[f(b) f(a)] g

(c) = [g(b) g(a)] f

(c)
(b) Probar que el teorema del valor medio (6.5.4 (b)) es un caso especial de (a).
6.5.8 .
(a) Probar que x
3
3x +b puede no tener mas que un cero en [1, 1], independien-
temente de los valores de b.
(b) Sea f(x) = (x
2
1)e
cx
. Probar que f

(x) = 0 para exactamente un valor de x


en el intervalo (1, 1) y que esta x tiene el mismo signo que el parametro c.
6.5.9 Cuantos ceros tiene la funcion f(x) = 2
x
1 x
2
sobre la recta real?
6.5.10 Sea f(x) = a0 +a
l
x +... +anx
n
un polinomio con coecientes reales tal que
f tiene n + 1 ceros reales distintos. Usar el teorema de Rolle para probar que a
k
= 0
para 0 k n.
6.5.11 Si f : R R es una funcion diferenciable, probar que existe una raz de
f(x) af(x) = 0 entre cualesquiera dos races de f(x) = 0.
6.5.12 Supongase que n es un entero no negativo y
c0e
r
0
x
+c1e
r
1
x
+ +cne
rnx
donde ci y ri son n umeros reales. Probar que si f tiene mas de n ceros en R entonces
f(x) = 0. (Sugerencia: Hacer induccion sobre n.)
252 CAP

ITULO 6. AN

ALISIS REAL INTERMEDIO


6.5.13 El n

esimo polinomio de Legendre esta denido por


Pn(x) =
1
2
n
n!
D
n
__
x
2
1
_
Donde D
n
denota la n

esima derivada con respecto a x . Probar que Pn(x) tiene


eaxatamente n races rales distintas y todas en el intervalo (1, 1). (Sugerencia:(x
2

1)
n
= (x l)
n
(x + 1)
n
. Probar por un argumento inductivo, que la k

esima derivada
de (xl)
n
(x+1)
n
tiene al 1 como un cero de multiplicidad nk, al 1 como un cero
de multiplicidad n k, y tiene al menos k ceros distintos entre 1 y 1.)
6.6. Teorema del Valor Medio
Supngase que f : [a.h] R es continua sobre [a, b] y diferenciable sobre (a, b). De
manera similar a lo que usamos en la solucion de 6.4.1,considerar la funcion
F(x) = f(x) L(x).
(ver la gura 6.8), donde y = L(x) es la ecuacion de la recta desde (a, f(a)) a (b, f(b)).
Geometricamente, F(x) representa la distancia a lo largo del segmento vertical desde
(x, f(x)) a la recta y = L(x). Como f(a) = 0 = f(b). Sabemos por el teorema de Rolle
que existe un punto c en (a.b) tal que F

(c) = O. En tal punto f

(c) L

(c) = O, o
equivalentemente
f

(c) = L

(c) = (la pendiente de L) =


f(b) f(a)
b a
Tenemos probado justamente el siguiente.
Teorema del Valor Medio. Si f : [a.b] R es continua en [a.b] y diferenciable en
(a, b). Entonces existe un n umero c en (a, b) tal que
f

(c) =
f(b) f(a)
b a
Si f(a) = f(b), es justamente lo establecido en el teorema de Rolle. De otra forma,
diremos que existe un punto entre a y b donde la pndiente de la curva es igual a la
pendiente de la recta que pasa por (a, f(a)) y (b, f(b)).
6.6.1 Sea g(x) una funcion que tiene una primera derivada continua g

(x) para todos


los valores de x. Supongase que la siguientes condiciones son verdaderas
(i) g(0) = 0,
(ii) |g

(x)| |g(x)| para todo x.


Probar que g(x) es identicamente cero.
Soluci on. Daremos una solucion mejor e inusual, simplemente para ilustrar el uso
del teorema del Valor Medio. Comenzaremos por considerar el intervalo [0, 1]. Sea x
6.6. TEOREMA DEL VALOR MEDIO 253
Figura 6.8.
un punto arbitrario en (O, 1) Por el teorema del Valor Medio, existe un punto c1 en
(0, x) tal que
g

(c1) =
(x) g(0)
x 0
Se sigue que |g(x)| = |xg

(c1)| = |x| |g

(c1)| |x| |g(c1)| .


Similarmente existe un punto c2 en (0, c1) tal que |g(c1)| |c1| |g(c2)| .
Continuando de esta manera encontramos n umeros c1, c2, ..., cn, 0 < cn < ... < c2 <
c1 < x < 1, tales que |g(x)| |x| |g(c1)| |cn1| |g(cn)| . Como g es continua sobre
[0, 1], es acotada (entre sus valores mnimo y maximo, que existen por el teorema del
valor extremo) y ademas, como el lado derecho de la ultima desigualdad puede hacerse
arbitrariamente peque na tomando n sucientemente grande (cada uno de los |ci| es
menor que 1), aun mas para el caso en que g(x) = 0. Luego, g(x) es identicamente
igual a cero sobre [0, 1].
El mismo argumento podemos ahora aplicar al intervalo [1,2] (para x en (1,2) existe un
c1 en (1.x) tal que |g(x)| |x l| |g(c
l
)| , etc.). Como consecuebcia de este argumento,
tendremos que g es igual a cero sobre [1,2].
Por un argumento inductivo, tenemos que g es igual a cero sobre [n, n + 1] para todo
entero n. Luego, g es identicamente cero. (Notese que no usamos la hipotesis de que
g

fuera continua)
El teorema del valor medio tiene un importante n umero de corolarios que pondremos
en juego, ellos son los siguientes:
Supongamos que f y g son funciones continuas sobre [a, b] y diferenciables sobre (a, b).
(i) Si f

(x) = 0 para todo x en (a, b), entonces f es una constante.


(ii) Si f

(x) = g

(x) para todo x en (a, b), entonces existe una constante c tal que
f(x) = g(x) +C (
254 CAP

ITULO 6. AN

ALISIS REAL INTERMEDIO


(iii) Si f

(x) 0 para toda x en (a, b), entonce f es una funcion creciente. Similar-
mente si f

(x) 0 para toda x en (a.b) entonces f es una funcion decreciente.


( No-decreciente, no-creciente, respectivamente sobre (a, b). [Para algunas apli-
caciones ver seccion 7.4]
Prueba de (i) : Sea x (a, b). Por el teorema del valor medio, existe un n umero c en
(a, b) tal que [f(x) f(a)]/[x a] = f

(c) = O. Se sigue que f(x) = f(a) para todo x


en (a, b).
Prueba de (ii) : Aplicando (i) a la funcion h(x) = f(x) g(x).
Prueba de (iii) : Considerar x, y (a, b), x < y. Por el teorema del valor medio existe
un nn umero c en (x, y) tal que [f(y) f(x)]/[y x] = f

(c) 0 por lo cual se sigue


que f(y) f(x), y f es cresiente.
6.6.2 Sea f : R R tal que para todo x e y en R, Si |f(y) f(x)| (x y)
2
.
Probar que f es una constante.
Soluci on. Por el primero de los corolarios presedentes, es suciente probar que
f

(x) = 0 para todo x. Luego, argunmentamos como sigue:

(x)

limyx
f(y) f(x)
y x

= limyx

f(y) f(x)
y x

= limyx
|f(y) f(x)|
|y x|
limyx
(y x)
2
|y x|
= limyx |y x|
= 0.
6.6.3 Supongase que f : R R es dos veces diferenciable con f

(x) 0 para todo


x. Probar que para todas a y b, a < b,
f(
a +b
2
)
f(a) +f(b)
2
Soluci on. La Figura 6.9 muestra la conclusion, pero el uso del teorema del valor
medio permite pasar de una propiedad local, f

(x) > 0 (f

esta determinada para


algunos valores dados de x), en una propiedad global (vertdadera para todas a y
b independientemente de su proximidad). por el teorema del valor medio existe un
n umero x1 en (a,
1
2
(a +b)) tal que
f(
1
2
(a +b)) f(a)
1
2
(a +b) a
= f

(x1)
6.6. TEOREMA DEL VALOR MEDIO 255
y un n umero x2 en (
1
2
(a +b), b) tal que
Figura 6.9.
f(b) f(
1
2
(a +b))
b
1
2
(a +b)
= f

(x2).
Pero f

(x) 0 para todo x en (x1, x2), luego, f

es una funcion no-decreciente. De


aqu f

(x2) f

(x1) o equivalentemente
f(b) f(
1
2
(a +b))
b
1
2
(a +b)

f(
1
2
(a +b)) f(a)
1
2
(a +b) a
f(
1
2
(a +b))
f(a) +f(b)
2
En el resto de esta seccion trataremos problemas en donde usaremos todos o la
mayora de los teoremas de existencia considerados en este captulo: el teorema del
valor intermedio, el teorema del valor extremo, el teorema de Rolle y el teorema del
valor medio.
6.6.4 Sea f una funcion diferenciable con f

continua sobre [a, b]. Probar que existe


un n umero c en (a, b] tal que f

(c) = 0, entonces podemos encontrar un n umero en


(a, b) tal que
f

() =
f() f(a)
b a
Soluci on. Entraremos al campo de la geometra para tratar el problema: considerar
la gr

fica en la Figura 6.10, donde B esta localizado de forma que la recta RC es


horizontal. Para un punto x entre a y b el lado derecho de la ecuacion, es decir,
f(x) f(a)
b a
representa la pendiente de la recta AB, ademas el lado izquierdo f

(x) representa la
pendiente de la tangente a la curva en C. considerar entonces las funcion
F(x) = f

(x)
f(x) f(a)
b a
256 CAP

ITULO 6. AN

ALISIS REAL INTERMEDIO


Esta es una funcion continua de x (aqu hacemos uso del hecho de que f

es continua,)
Figura 6.10.
luego por el teorema del valor intermedio, existe un punto en (a, b) tal que F() = 0
debido a que podemos encontrar puntos x1 y x2 (a, b) tales que F(x1) > 0 y F(x2) < 0.
Observese que F(x) va tomando valores positivos a partir de x = a y negativos a
partir de x = c. Aceptemos esto, o algo similar, siempre serel caso? Supongase que
f(c) > f(a). Entonces f

(c) = 0, y [f(c) f(a)]/[b a] > 0, de aqu


F(c) = f

(c)
f(c) f(a)
b a
< 0.
Por el teorema del valor medio, existe un punto d en (a, c) tal que f

(d) =
f(c)f(a)
ca
.
De donde
F(d) = f

(d)
f(d) f(a)
b a
=
f(c) f(a)
c a

f(d) f(a)
b a

f(c) f(a)
b a

f(d) f(a)
b a
=
f(c) f(d)
b a
.
Ahora, veamos que pasapara el caso en que f(c) > f(d). Desafortunadamente, no
podemos asegurar nada, como lo indica la graca de la Figura 6.11. Para salvar esta
dicultad, podemos proceder como sigue. Considerar la funcion f sobre el intervalo
[a, c]. Por el teorema del valor extremo, se alcanza un valor maximo sobre este intervalo,
digamos en x = s (puede ser igual a c). Como hemos asumido que f(c) > f(a), sabemos
que a < s c. Si s = c entonces f

(s) = f

(c) = O, de otra forma si a < s < c entonces


f

(s) = O por 6.3.7. Ahora procedamos como sigue: Existe un punto d en (a, s) tal
6.6. TEOREMA DEL VALOR MEDIO 257
Figura 6.11.
que f

(d) = [f(s) f(a)]/(s a), y


F(d) = f

(d)
f(d) f(a)
b a
=
f(s) f(a)
s a

f(d) f(a)
b a

f(s) f(a)
b a

f(d) f(a)
b a
=
f(s) f(d)
b a
.
y esta ultima expresion es no-negativa, ya que f(s) f(d) por las caractersticas de
s. Estyo completa la prueba para este caso: El argumento es similar para los casos
f(c) < f(a) y f(c) = f(a).
6.6.5 supongase que f es una funcion de valores reales dos veces continuamente
diferenciable denida sobre todos los n umeros relaes tal que |f(x)| 1 parsa todos los x
y (f(0))
2
+(f

(0))
2
= 4. Probar que existe un n umero real x0 tal que f(x0)+f(x0) = 0.
Soluci on. Existen dos formas naturales de aprovechar nuestros esfuerzos. Una con-
siste en aplicar el teorema del valor intermedio; es decir, considerar la funcion F(x) =
f(x) + f

(x) y encontrar a y b para las cuales F(a) > 0 y F(b) < 0. Desafortunada-
mente, la condicion (f(0))
2
+ (f

(0))
2
= 4. no es suciente para aprovechar esto.
Otra idea es ver si G(x) = (f(x))
2
+ (f

(x))
2
tiene un extremo en el interior de
alg un intervalo. Para un tal extremo, G

(x) = 0. Notese que G

(x) = 2f(x)f

(x) +
2f

(x)f

(x) = 2f

(x)[f(x) +f

(x)]. Esto parece mejor!.


Esforcemonos en probar que existen puntos a y b, 2 < a < 0, 0 < b < 2. tales que
|G(a)| 2 y |G(b)| 2. Como G(0) = 4 se sigue que G(x) tiene un maximo en un
punto x0 en (a, b), y que en este punto G

(x0) = 0.
258 CAP

ITULO 6. AN

ALISIS REAL INTERMEDIO


Del teorema del valor medio existe un punto a en (2,0) y b en (0.2) tales que
f

(a) =
f(0) f(2)
2
y f

(b) =
f(2) f(0)
2
se sigue que

(a)

f(0) f(2)
2

|f(0)| +|f(2)|
2

1 + 1
2
= 1.

(b)

f(2) f(0)
2

|f(2)| +|f(0)|
2

1 + 1
2
= 1.
Luego
|G(a)| =

(f(a))
2
+ (f

(a))
2

(f(a))
2

(f

(a))
2

2,
|G(b)| =

(f(b))
2
+ (f

(b))
2

(f(b))
2

(f

(b))
2

2.
Sea x0 el punto en (a, b) donde G(x0) es un m ximo. Entonces G

(x0) = 2f

(x0)[f(x0)+
f

(x0)] = 0.
Si f

(x0) = 0, entonces G(x0) = (f(x0))


2
+ (f

(x0))
2
= (f(x0))
2
1. Pero G(x0) 4,
ya que G(0) = 4. De donde f

(x0) = 0 y de aqu se tiene que f(x0) +f

(x0) = 0. Esto
completa la prueba.
6.6.6 Sea f(x) diferenciable sobre [0.l] con f(0) = 0 y f(1) = l. Para cada entero
positivo n, probar que existen distintos puntos x1, x2, ..., xn en [0, 1] tales que
n

i=1
1
f

(xi)
= n.
Soluci on. Ayuda para generar ideas,considerar el caso n = 1. Podemos encontrar x
en [0, 1] tal que 1/f

(x1) = 1. Esto es posible por el teorema del valor medio, ya que


sobre el intervalo [0, 1] existe un punto x1 tal que f

(x1) = 1.
Considerar el caso n = 2. Considerar los subintervalos [0, x] y [x, 1] donde x es alg un
n umero entre 0 y 1 determinado. Por el teorema del valor medio, existen un x1 (0, x)
y x2en (x, 1) tales que
f

(x1) =
f(x) f(0)
x
y f

(x2) =
f(1) f(x)
1 x
.
Luego,
1
f

(x1)
+
1
f

(x2)
= 2
Si y solo si
x
f(x)
+
1 x
1 f(x)
= 2
x(1 f(x)) + (1 x)f(x) = 2f(x) 2(f(x))
2
,
6.6. TEOREMA DEL VALOR MEDIO 259
x xf(x) +f(x) xf(x) 2f(x) + 2(f(x))
2
= 0
x 2xf(x) f(x) + 2(f(x))
2
= 0
x(1 2f(x)) f(x)(1 2f(x)) = 0
(x f(x))(1 2f(x)) = 0
Ahora, debemos encontrar x en (0, 1) tal que f(x) = 1/2 (esto es inmediato por el
teorema del valor intermedio), la prueba se completa regresando la serie de pasos
previa.
Con base en este trabajo podemos considerar el caso para un entero positivo arbitrario
n. Sea ci el menor n umero en [0,1] tal que f(ci) = i/n (la existencia de este n umero
es consecuencia del teorema del valor intermedio conjuntamente con la suposicion de
continuidad). Entonces 0 < c1 < c2 < ... < cn1 < 1. Denamos c0 = 0 y cn = 1, y
para cada intervalo (ci1, ci), i = 1, 2, ..., n, tomar xi tal que
f

(xi) =
f(ci) f(ci1)
ci ci1
(esto es posible por el teorema del valor medio). Entonces
f

(xi) =
i
n

i1
n
ci ci1
=
1
n(ci ci1)
luego
n

i=1
1
f

(xi)
=
n

i=1
n(ci ci1) = n.
Problemas
6.6.7 .
(a) Probar que
F(x) =
senx +sen(x +a)
cosx cos(x +a)
es una funcion constante probando que F(x) = 0. (Este problema aparecio en
1.2.1.)
(b) Si P(x) es un polinomio de grado tres en x, y y
2
= P(x), probar que
D(y
3
D
2
y)
y
2
es una constante, donde D denota el operador derivada. (Sugerencia: Primero
escribir la expresion anterior en terminos de P y sus derivadas.)
6.6.8 .
260 CAP

ITULO 6. AN

ALISIS REAL INTERMEDIO


(a) Si Y = f(x) es una solucion de la ecuacion diferencial y

+ y = 0, probar que
f
2
+ (f

)
2
es una constante.
(b) Usar la parte (a) para probar que cada solucion de y

+ y = 0 es de la forma
y = Acosx+Bsenx. (Sugerencia: Es facil probar que todas las funciones Acosx+
Bsenx satisfacen la ecuacion dferencial. Sea f(x) una solucion. Para que f(x)
tenga la forma f(x) = Acosx + Bsenx es necesario que A = f(0) y que B =
f

(0). Ahora considereF(x) = f(x) f(0)cosx f

(0)senx. Aplicar la parte (a)


a F(x), haciendo uso del hecho de que F(0) = 0 = F

(0).)
(c) Usar la parte (b) para probar las formulas de adicion
sin(x +y) = senxcosy +cosxseny,
cos(x +y) = cosxcosy senxseny.
6.6.9 Sea f(x) diferenciable sobre [0, 1] con f(0) = 0 y f(l) = 1. Para cada en-
tero positivo n y n umeros positivos arbitrarios dados k1, k2, ..., kn probar que existen
distintos x1, x2, ..., xn tales que
n

i=1
ki
f

(xi)
=
n

i=1
ki.
Ejemplos Adicionales
6.9.6,6.9.10, Seccion 7.4.
6.7. Regla de LHopital
Aqu asumiremos la familiaridad del lector con variadas formas de la Regla de LHopital.
6.7.1 Evaluar
lm
x
_
1
x
a
x
1
a 1
_
1/x
.
Soluci on. Reescribimos la expresion en la forma equivalente
_
1
x
a
x
1
a 1
_
1/x
= exp
_
1
x
log
_
1
x
a
x
1
a 1
__
.
De esta manera el problema ha sido transformado para la evaluacion de
lm
x
_
log
1
x
a
x
1
a1
x
_
,
6.7. REGLA DE LH

OPITAL 261
o equivalentemente
lm
x
_
log
1
x
x
_
+ lm
x
_
log(a
x
1)
x
_
lm
x
_
log(a 1)
x
_
,
probando que cada uno de estos lmites existe. Claramente, lmx(Iog(a1))/x = 0,
y por la Regla de LHopital,
lm
x
_
log
1
x
x
_
= lm
x
_
logx
x
_
= lm
x
_
1
x
1
_
= 0
Tambien, por la Regla de LHopital,
lm
x
_
log(a
x
1)
x
_
= lm
x
_
a
x
loga
a
x
1
_
= loga.
Se sigue que
lm
x
_
1
x
a
x
1
a 1
_
1/x
= exploga = a.
6.7.2 Supongase f es una funcion con con dos derivadas continuas y f(0) = 0.
Probar que la funcion g denida por g(0) = f

(0), g(x) = f(x)/x para x = 0 tiene una


derivada continua.
Soluci on. Para x = 0
g

(0) = limx0
_
g(x) g(0)
x 0
_
y como f

es continua, luego tambien lo es g para x = 0. Solamente resta vericar que


g tiene una derivada en x = 0, y si g

(O) existe, ver si g

es continua en x = O.
Para la existencia de g

(O) podemos examinar lo siguiente:


g

(0) = limx0
_
g(x) g(0)
x 0
_
= limx0
_
f(x)/x f

(0)
x
_
= limx0
_
f(x) xf

(0)
x
2
_
Como f(x) xf

(0) 0 cuando x 0 y como f y f

son diferenciables podemos


aplicar la Regla de LHopital para obtener el lmite
g

(0) = limx0
_
f

(x) f

(0)
2x
_
=
1
2
limx0
_
f

(x) f

(0)
x
_
=
1
2
f

(0).
262 CAP

ITULO 6. AN

ALISIS REAL INTERMEDIO


(Este ultimo paso se debe a la denicion de f

(0).) Luego g

(0) existe.
Para vericar la continuidad de g

en 0 tenemos
limx0g

(x) = limx0
_
xf

(x) f(x)
x
2
_
= limx0
_
f

(x) +xf

(x) f

(x)
2x
_
= limx0
_
+f

(x)
2
_
=
1
2
f

(0).
El ultimo paso se debe a que tenemos dado que f tiene una segunda derivada continua.
Luego limx0g

(x) = g

(0), y la prueba esta completa.


Problemas
6.7.3 Evaluar
limn4
n
_
1 cos

2
n
_
.
6.7.4 Evaluar los siguientes lmites:
(a) limn
_
1 +
1
n
_
n
.
(b) limn
_
n+1
n+2
_
n
.
(c) limn
_
1 +
1
n
2
_
n
.
(d) limn
_
1 +
1
n
_
n
2
.
(e) limn
2pnPn
pn+Pn
, donde pn =
_
1 +
1
n
_
n
y Pn =
_
1 +
1
n
_
n+1
.
6.7.5 Si 0 < a < b. Evaluar
limt0
__
1
0
_
bx +a(1 x)
t

dt
_
1/t
6.7.6 Calcular
limxx
_
x
0
e
t
2
x
2
dt
6.7.7 Probar que la funcion y = (x
2
)
x
, y(0) = 1, es continua en x = 0.
6.8. LA INTEGRAL 263
6.8. La Integral
Considerar que deseamos conocer la suma
1
n
+
1
n + 1
+... +
1
2n 1
cuando n . Uno puede comenzar interpretandola geometricamente; construyendo
los rectangulos sobre [n, 2n] como muestra la Figura 6.12. De la gura es claro que
1
n
+
1
n + 1
+... +
1
2n 1
>
_
2n
n
1
x
dx = logx|
2n
n
= log2n logn = log2.
Similarmente, de la Figura 6.13 se sigue que
1
n + 1
+
1
n + 2
+... +
1
2n
<
_
2n
n
1
t
dt = log2.
De ambos resultados tenemos
Figura 6.12.
log2 <
1
n
+
1
n + 1
+... +
1
2n 1
<
_
1
n

1
2n
_
+log2.
Ahora, como n se sigue que la suma en cuestion se aproxima a log2. Otra forma
de ver esto es reescribir la suma en la forma
n1

k=0
1
n +k
=
n1

k=0
_
1
1 +
k
n
_
1
n
y tomar cada termino
_
1
1 +
k
n
_
1
n
como el area del rectangulo con base [k/n, (k + 1)/n] y altura 1/(1 + k/n). En esta
forma, la suma represena el area de los rectangulos sombreados como se muestra
264 CAP

ITULO 6. AN

ALISIS REAL INTERMEDIO


Figura 6.13.
en la Figura 6.14. Cuando n , estas areas se aproximan al area acotada por
y = 1/(1 +x), y = 0, x = 0, x = 1. Esto es
limn
n1

k=0
1
n +k
= limn
n1

k=0
_
1
1 +
k
n
_
1
n
=
_
1
0
1
1 +x
dx = log2.
Figura 6.14.
6.8.1 Evaluar
limn
1
n

k=1
___
2n
k
__
2
__
n
k
__
_
.
Soluci on. El problema requiere de la evaluacion de la integral denida
_
1
0
__
2
x
__
2
__
1
x
__
dx.
6.8. LA INTEGRAL 265
hagamos esto geometricamente calculando el area bajo la graca de f(x) =
__
2
x

2
__
1
x

entre x = 0 y x = 1. Los puntos de discontinuidad de f(x) en (0, 1) ocurren


cuando 2/x o 1/x son enteros. en el primer caso 2/x = n cuando x = 2/n y en
el segundo caso 1/x = n cuando x = 1/n. Luego, concentrandonos en los puntos
1 > 2/3 > 2/4 > 2/5 > 2/6 > . Es facil vericar que para cada n,
f(x) =
_
0 si x(
2
2n+1
,
2
2n
]
1 si x(
2
2n+2
,
2
2n+1
]
La graca es como se muestra en la Figura 6.15. La integral de esta es igual a
Figura 6.15.
_
2
3

2
4
_
+
_
2
5

2
6
_
+
_
2
7

2
8
_
+ ,
o
2
_
1
3

1
4
+
1
5

1
6
+
1
7

1
8
_
+ ,
Ahora recordemos que
log(1 +x) = x
x
2
2
+
x
3
3

x
4
4
+ , 1 < x 1.
Esto signica que
2
_
1
3

1
4
+
1
5

1
6
+
1
7

1
8
_
+ = 2
_
log2 1 +
1
2
_
= log4 1
y la solucion esta completa.
6.8.2 Evaluar
limn
1
n
4
2n

i=1
_
n
2
+ 1
2
_
1/n
.
266 CAP

ITULO 6. AN

ALISIS REAL INTERMEDIO


Soluci on. Podemos cambiar el producto en una forma equivalente escribiendo
1
n
4
2n

i=1
_
n
2
+i
2
_
1/n
= exp
_
log
1
n
4
2n

i=1
_
n
2
+i
2
_
1/n
_
=
2n

i=1
log
1
n
_
n
2
+i
2
_
logn
4
.
Luego, examinaremos
limn
_
2n

i=1
1
n
log
_
n
2
+i
2
_
logn
4
_
= limn
_
2n

i=1
1
n
logn
2
_
n
2
+i
2
n
2
_
logn
4
_
= limn
_
2n

i=1
1
n
_
logn
2
+log
_
1 +
_
i
n
_
2
__
logn
4
_
= limn
_
2n

i=1
1
n
logn
2
+
2n

i=1
1
n
log
_
1 +
_
i
n
_
2
_
logn
4
_
= limn
_
2n
n
logn
2
+
2n

i=1
1
n
log
_
1 +
_
i
n
_
2
_
logn
4
_
= limn
_
2n

i=1
log
_
1 +
_
i
n
_
2
_

1
n
_
.
Reorganicemos la ultima expresion como la integral denida
_
2
0
log(1 +x
2
)dx.
Usando la integracion por partes tenemos
_
2
0
log(1 +x
2
)dx = xlog(1 +x
2
)|
2
0
2
_
2
0
x
2
1 +x
2
dx
= 2log5 2
_
2
0
_
1
1
1 +x
2
_
dx
= 2log5 2 [x arctanx] |
2
0
= 2log5 2 [2 arctan2] .
Luego, el lmite original es
exp [2log5 4 + 2arctan2] ,
o equivalentemente
25exp (2arctan2 4) .
6.8. LA INTEGRAL 267
6.8.3 Probar que:
n

k=0
(1)
k
_
n
k
_
1
k +m+ 1
=
m

k=0
(1)
k
_
n
k
_
1
k +n + 1
.
Soluci on. La clave es observar que
1
k +m+ 1
=
_
1
0
t
k+m
dt.
Usando esto, encontramos
n

k=0
(1)
k
_
n
k
_
1
k +m+ 1
=
n

k=0
(1)
k
_
n
k
_
_
1
0
t
k+m
dt
=
_
1
0
n

k=0
(1)
k
_
n
k
_
t
k+m
dt
=
_
1
0
t
m
(1 t)
n
dt.
Ahora usando cambio de variable: sea s = 1 t. Continuando desde la ultima integral
=
_
1
0
(1 s)
m
s
n
dt
=
_
1
0
s
n
m

k=0
(1)
k
_
m
k
_
s
k
dt
=
m

k=0
(1)
k
_
m
k
_
_
1
0
s
n+k
dt
=
m

k=0
(1)
k
_
m
k
_
1
k +n + 1
dt.
Problemas
6.8.4 Evaluar los siguientes lmites:
(a) limn
_
1
2n+1
+
1
2n+2
+ +
1
3n
_
.
(b) limn
_
1
a
+2
a
++n
a
n
1+a
_
, a > 1.
(c) limn
_
n
1
2
+n
2
+
n
2
2
+n
2
+ +
n
n
2
+n
2
_
.
(d) limn
__
1 +
1
n
_ _
1 +
2
n
_

_
1 +
n
n
_
1/n
.
268 CAP

ITULO 6. AN

ALISIS REAL INTERMEDIO


6.8.5 Evaluar cada uno de los siguientes:
(a) limnn
3/2

n
k=1

k.
(b) limn

n
k=1
1

k
2
+n
2
.
6.8.6 Encontrar la suma parcial

10
9
n=1
n
2/3
. (Sugerencia: Comparar el area bajo
la curva de f(x) = x
3/2
sobre el intervalo [1, 10
9
+ 1] con el area bajo la curva
f(x) = (x 1)
3/2
sobre el intervalo [2, 10
9
+ 1].)
6.8.7 Supongase que f y g son funciones continuas sobre [0, a], y supongase que
f(x) = f(a x) y g(x) + g(a x) = k para todo x en [0, a), donde k es un n umero
jo. Probar que
_
a
0
f(x)g(x)dx =
1
2
k
_
a
0
f(x)dx.
Usar este hecho para evaluar
_

0
xsenx
1 +cos
2
x
dx.
6.8.8 .
(a) Sea
A =
_

0
cosx
(x + 2)
2
dx.
Computar
_
/2
0
senxcosx
x + 1
dx
en terminos de A.
(b) Sea
f(x) =
_
x
0
logt
1 +t
dt para x > 0.
Computar f(x) +f(1/x).
6.8.9 Encontrar todas las funciones continuas positivas f(x), para 0 x l, tales
que
_
1
0
f(x)dx = 1,
_
1
0
xf(x)dx = a,
_
1
0
x
2
f(x)dx = a
2
, donde a es un n umero real
dado.
6.8.10 Sea f(x, y) una funcion continua sobre el cuadrado
S = (x, y) : 0 x l, 0 y l.
Para cada punto (a.b) en el interior de S, sea S
(a,b)
el cuadrado ms grande contenido
en S centrado en (a, b) y cuyos lados son paralelos a los lados de S. Si la doble integral
6.9. TEOREMA FUNDAMENTAL DEL C

ALCULO 269
_ _
f(x, y)dxdy es cero sobre cada cuadrado S
(a,b)
, f(x, y) sera identicamente cero en
S?
Ejemplos Adicionales
1.4.4, 1.6.3, 1.12.3, 1.12.6, 2.5.15, 6.2.2, 6.2.9, 7.6.3.
6.9. Teorema Fundamental del Calculo
El teeorema fundamental del calculo hace referencia a la relacion que existe entre
las operaciones inversas de diferencicion e integracion. El teorema fundamental para
integrales de derivadas establece que si F(x) tiene una derivada continua sobre el
intervalo (a, b), entonces
_
b
a
F

(t)dt = F(b) F(a)


En otras palabras, la diferenciacion seguida de la integracion recupera la funcion, en
el sentido de que
F(x) =
_
x
0
F

(t)dt +C)
donde C = F(0).
Por ejemplo la derivada de F(t) = sen
2
t is F(t) = 2sentcost. la integracion de F

(t)
sobre [0, x]
sen
2
x =
_
x
0
2sentcostdt.
En este caso recuperamos integramente la funcion debido a que F(0) = 0. pero tambien
obsevamos que la integracion puede ser efectuada de otra manera: veamos (sea U =
cost),
_
x
0
2sentcostdt = cos
2
t|
x
0
= cos
2
x + 1.
Se sigue que sen
2
x = cos
2
x+1, o equivalentemente, sen
2
x+cos
2
x = I para toda x.
6.9.1 Encontrar todas las funciones diferenciables f denidas para x > 0 que satis-
facen
f(xy) = f(x) +f(y), para x, y > 0.
Soluci on. Cuando x = y = 1 tenemos que f(1) = f(1 1) = f(1) + f(1), y se sigue
que f(1) = 0.
Si x = 0 tenemos 0 = f(1) = f(x 1/x) = f(x) +f(1/x), y de aqu f(1/x) = f(x).
Se sigue que f(x/y) = f(x) f(y). Ahora la idea es ver la derivada de f y recobrar f
270 CAP

ITULO 6. AN

ALISIS REAL INTERMEDIO


mediante la integracion:
f

(x) = lim
h0
f(x +h) f(x)
h
= lim
h0
f(x +h)/x
h
= limt0
f(1 +t)
tx
, donde t =
h
x
= limt0
1
x
f(1 +t) f(1)
t
=
1
x
f

(1).
De aqu por el teorema fundamental del calculo,
f(x) = f(x) f(1) =
_
x
1
f

(t)dt =
_
x
1
f

(1)
t
dt = f

(1)logx
Luego, las funciones que buscamos son de la forma Alogx donde A es una constante
arbitraria.
6.9.2 Encontrar la suma de la serie
1
1
5
+
1
7

1
11
+ +
1
6n 5

1

6n 1 + .
Soluci on. Considerar la funcion denida por la serie innita
f(x) = x
x
5
5
+
x
7
7

x
11
11
+ +
x
6n5
6n 5

x
6n1
6n 1
+
para 0 < x 1, la serie es absolutamnete convergente para |x| < 1, y por tanto,
podemos reordenarlos terminos:
f(x) =
_
x +
x
7
7
+
x
13
13
+ +
x
6n5
6n 5
+
_

_
x
5
5
+
x
11
11
+ +
x
6n1
6n 1
+
_
Aprovechando la diferenciabilidad de f, cambiando su forma, y recobrando f a traves
de la integfracion por el teorema fundamental del calculo. Tenemos para 0 < x 1,
f

(x) =
_
1 +x
6
+x
12
+ +x
6n6
+
_

_
x
4
+x
10
+ +x
6n2
+ ,
_
=
1
1 x
6

x
4
1 x
6
=
(1 x
2
)(1 +x
2
)
(1 x
2
)(1 +x
2
+x
4
)
=
(1 +x
2
)
(1 +x
2
+x
4
)
Integrando (los detalles no son de interes aqu), y notando que f(0) = 0, tenemos
f(x) =
1

3
_
arctan
2x 1

3
+arctan
2x + 1

3
_
.
6.9. TEOREMA FUNDAMENTAL DEL C

ALCULO 271
Como la representacion en series de f converge para x = 1, el teorema de Abel (seccion
5.4) implica que la serie original converge a
f(x) =
1

3
_
arctan
1

3
+arctan

3
_
=

2

3
.
El teorema fundamental del calculo para derivadas de integrales establece que si f es
una funcion continuaen unintervalo [a, b], entonces para cualquier x en (a, b)
d
dx
_
x
a
f(t)dt = f(x).
ncion. En otras palabras, la integracion seguida de la derivada recobra exactamente la
fu
6.9.3 Sean a(x), b(x), c(x) y d(x) polinomios en x probar que
_
x
1
a(x)c(x)dx
_
x
1
b(x)d(x)dx
_
x
1
a(x)d(x)dx
_
x
1
b(x)c(x)dx
es divisible por (x 1)
4
.
Soluci on. Denotar la expresion en cuestion por F(x). Notese F(x) es un polinomio
en x. Tambien notese que F(1) = 0, y por tanto (x 1) es un factor de F(x).
Cmo F es un polinomio, sabemos que (x 1)
4
divide a F(x) si y solo si F

(1) = 0.
Podemos computar F

usando el teorema fundamental del calculo:


F

(x) = ac
_
x
1
bd +bd
_
x
1
ac ad
_
x
1
bc bc
_
x
1
ad
(Note que F

(1) = 0 y de aqu que (x 1)


2
divide a F(x).) las derivadas F

y F

se realizan de manera similar, de forma que F

(1) = (ac)

bd + (bd)

ac (ad)

bc
(bc)

ad|x=1 = 0. Esto completa la prueba.


Los siguientes tres ejemplos combinan varias ideas de este captulo.
6.9.4 Sea f : [0, ) R diferenciable y asumamos que f(x) + f

(x) 0, cuando
x . Probar que f(x) 0, cuando x .
Soluci on. Primero una disgresion: si p(x) y q(x) so funciones continuas, la ecuacion
dy
dx
+p(x)y = q(x)
puede ser resuelta de la siguiente manera. Multilicar cada lado de la ecuacion por
m(x) = e
_
p(x)dx y notese que el resultado es una ecuacion que se puede tomar de la
forma
d
dx
(ym(x)) = m(x)q(x)
272 CAP

ITULO 6. AN

ALISIS REAL INTERMEDIO


Luego, por el teorema fundamental del calculo, para cada constante a existe una
cconstante C tal que
ym(x) =
_
x
a
m(t)q(t)dt +C
De aqu podemos resolver para y.
Ahora, retornemos al problema y sea g(x) = f

(x) +f(x). Recordando el razonamien-


to del ultimo paragrafo, podemos resolver f(x) (en terminos de g(x)) pero primero
multipliquemos cada lado por e
x
. Como anteriormente, esto da origen a la siguiente
ecuacion
f(x)e
x
=
_
x
a
e
t
g(t)dt +C
o equivalentemente
f(x) = e
x
_
x
a
e
t
g(t)dt +e
x
C
Sea > 0. Como g(x) 0, cuando x , tomemos a tal que |g(x)| < para toda
x > a. Entonces
|f(x)| e
x

_
x
a
e
t
g(t)dt

Ce
x

e
x
_
x
a
e
t
|g(t)| dt +

Ce
x

e
x
_
x
a
e
t
dt +

Ce
x

e
x
(e
x
e
a
) +

Ce
x


_
1 e
ax
_
+

Ce
x

Ahora, para x sucientemente grande, tendremos que |f(x)| < 2. Se sigue que f(x)
0, cuando x .
6.9.5 Evaluar
limx0
_
x
0
(1 +sen2t)
1/t
dt.
Soluci on. Parece que debemos aplicar la regla de LHopital, pero no sin relizar alg un
trabajo previo. Primero existe una pregunta concerniente a la existencia de la integral,
debido a que el integrando es idenido cuando t = 0. Sin embargo
limx0 (1 +sen2x)
1/x
= limx0
_
exp
1
x
log (1 +sen2x)
_
= exp
_
limx0
log (1 +sen2x)
x
_
el cual por la regla de LHopital es
exp
_
limx0
2cos2x
1 +sen2x
_
= exp2 = e
2
6.9. TEOREMA FUNDAMENTAL DEL C

ALCULO 273
Luego, si denimos
f(x) =
_
(1+sen2x)
1/x
si x=0
e
2
si x=0
la funcion f es continua y
_
x
0
(1 +sen2t)
1/t
dt =
_
x
0
f(t)dt.
Antes de otra aplicacion de la regla de LHopital podemos probar que
_
x
0
(1 +sen2t)
1/t
dt
0 cuando x 0. Para esto sea K una cota superior para f(x) para toda x en (1, 1).
Entonces para toda x en (1, 1),

_
x
0
(1 +sen2t)
1/t
dt

_
x
0
|(1 +sen2t)|
1/t
dt K|x| .
Se sigue que
_
x
0
(1 +sen2t)
1/t
dt 0 cuando x 0.
Podemos ahora aplicar la regla de LHopital al problema original
limx0
_
x
0
(1 +sen2t)
1/t
dt
x
= limx0 (1 +sen2x)
1/x
= e
2
.
6.9.6 Supongamos que f : [0, 1] R tiene una segunda derivada continua, que
f(0) = 0 = f(1), y que f(x) > O para todo x en (O, 1). Probar que
_
1
0

(x)
f(x)

> 4.
Soluci on. Si X denota un punto en (0,1) donde f(x) es un maximo, y supongase que
Y = f(X). Entonces
_
1
0

(x)
f(x)

dx >
1
|Y |
_
1
0

(x)

dx

1
|Y |

_
1
0
f

(x)dx

(1) f

(0)
Y

.
No nos par4ce inmediato, debido a que ciertamente no es necesario que en ese punto
estimado |f

(1) f

(0)| 4 |Y | . Sin embargo, por el teorema del valor medio existen


puntos a en (O, X) y b en (X, 1) tales que
f

(a) =
f(X) f(0)
X 0
=
f(X)
X
=
Y
X
,
y
f

(b) =
f(1) f(X)
1 X
=
f(X)
1 X
=
Y
1 X
.
Luego
_
1
0

(x)
f(x)

dx
_
b
a

(x)
f(x)

dx
1
|Y |

_
b
a
f

(x)dx

,
274 CAP

ITULO 6. AN

ALISIS REAL INTERMEDIO


luego aplicando el teorema fundamental a la ultima integral, tenemoa
_
1
0

(x)
f(x)

dx >
1
|Y |

(b) f

(a)

=
1
|Y |

Y
1 X

Y
X

=
1
|Y |

Y
1 X
+
Y
X

1
X (1 X)

.
Pero el maximo valor que puede tomar x(1 x) es 1/4 (cuando x = 1/2) y por tanto
_
1
0

(x)
f(x)

dx >
1
|X (1 X)|
4.
Problemas
6.9.7 Que funciones estan denidas por la ecuacion
f(x) =
_
x
0
f(t)dt + 1.
Figura 6.16.
6.9.8 Sea f[0, 1] (0, 1) continua. Probar que la ecuacion
2x
_
x
0
f(t)dt = 1
tiene una y solo una solucion en el intervalo [0, 1].
6.9. TEOREMA FUNDAMENTAL DEL C

ALCULO 275
6.9.9 Supongamos que f es una funcion continua para toda x que satisface la ecuacion
_
x
0
f(t)dt =
_
1
x
t
2
f(t)dt +
x
16
8
+
x
18
9
+C
donde C es una constante. Encontrar una forma explcita para f(x) y encontrar el
valor de la constante C.
6.9.10 Sean C1 y C2 curvas que pasan a traves del origen como se muestra en la
Figura 6.16. Se dice que una curva C bisecta en area la region entre C1 y C2 si para
cada punto P en C las dos regiones sombreadas A y B mostradas en la gura tienen
areas iguales. Determine la curva superior C2 cuya curva bisectante C tiene la ecuacion
y = x
2
y la curva inferior C1 tiene la ecuacion y =
1
2
x
2
.
6.9.11 Suma la serie 1 +
1
3

1
5

1
7
+
1
9
+
1
11
.
6.9.12 Supongase que f es diferenciable, y que f

(x) es estrictamente cresciente para


x 0. Si f(0) = 0, probar que f(x)/x es estrictamente cresciente para x > 0.
Ejemplos Adicionales
1.5.1, 5.1.3, 5.1.9, 5.1.11, 5.4.6, 7.6.5.
276 CAP

ITULO 6. AN

ALISIS REAL INTERMEDIO


Captulo 7
Desigualdades
Las desigualdades son muy utiles virtualmente en todas las areas de matematicas y
los problemas de desigualdades son de lo mas hermoso. Debemos concentrarnos solo
en dos problemas de desigualdades entre todas las posibles que podriamos considerar;
La desigualdad entre la media aritmetica y la media geometrica en la seccion 7.2 y
la de Cauchy-Schwarz en la seccion 7.3.Ademas debemos considerar varias tecnicas
geometricas y algebraicas en la seccion 7.1 y tecnicas analticas en las secciones 7.4.
y 7.5. En la seccion nal 7.6 veremos como las desigualdades pueden ser usadas para
evaluar lmites.
7.1. Propiedades basicas de las desigualdades.
Los primeros pasos para establecer una desigualdad es recurrir a una maniobra alge-
braica o a una interpretacion geometrica. Por ejemplo la desigualdad entre las medias
aritmetica y geometrica,
a +b
2

ab, 0 < a b,
puede ser establecida algebracamente escribiendolo en su forma equivalente
_

b
_
2
0,
o geometricamente considerando el semicrculo en la gura 7.1. (El semicrculo es
construido con el diametro AB de longitud a + b,y C es un punto escogido de modo
que AC =a y CB= b. Una perpendicular a AB desde C corte a la circunferencia en
D. Los triangulos ACD y CBD son semejantes y por lo tanto a/CD = CD/b muestra
que CD =

ab. Claramente,

ab al radio del crculo = (a +b) /2.) Ambos caminos


demuestran que la igualdad se da s y solo s a = b. En esta seccion consideraremos
277
278 CAP

ITULO 7. DESIGUALDADES
Figura 7.1.
ejemplos de desigualdades que pueden ser vericadas usando solo ideas algebraicas y
geometricas.
7.1.1 Muestra que para los n umeros positivos a, b, c,
a
2
+b
2
+c
2
ab +bc +ca
Soluci on. Trabajando en retrospectiva
a
2
+b
2
+c
2
ab +bc +ca,
2a
2
+ 2b
2
+ 2c
2
2ab + 2bc + 2ca,
_
a
2
2ab +b
2
_
+
_
b
2
2bc +c
2
_
+
_
c
2
2ca +a
2
_
0,
(a b)
2
+ (b c)
2
+ (c a)
2
0.
Esta ultima desigualdad es obviamente verdadera, debido a que los pasos son re-
versibles, la solucion esta completa. (La prueba tambien deja claro que la igualdad se
presenta s y solo s a = b = c.)
Este ejemplo ilustra un tema com un: desarrollando la expresion de tal forma hasta
llegar a una armacion de la que estemos convencidos de su veracidad (en este caso,
que un n umero al cuadrado no es negativo).
7.1.2 Prueba que para 0 < x <
1
2
, cos
2
x +xsin x < 2.
Soluci on. Considerando la funcion
f (x) = 2 cos
2
xsin x,
7.1. PROPIEDADES B

ASICAS DE LAS DESIGUALDADES. 279


y desarrollando los siguientes pasos:
f (x) = 1 +
_
1 cos
2
x
_
xsin x
= 1 +sin
2
x xsin x
=
_
1 2 sin x = sin
2
x
_
xsin x + 2 sin x
= (1 sin x)
2
+ (2 x) sin x
De esta forma vemos que la desigualdad deseada es cierta siempre que 0 < x < 2.
7.1.3 Si 0 a, b, c 1, muestra que
a
b +c + 1
+
b
c +a + 1
+
c
a +b + 1
+ (1 a) (1 b) (1 c) 1.
Soluci on. Aqu, la expansion directamente algebraica nos lleva a complicaciones nada
claras y horrendas. Una simplicacion es asumir sin perder la generalidad que 0 a
b c 1. Entonces por ejemplo tenemos
a
b +c + 1
+
b
c +a + 1
+
c
a +b + 1

a +b +c
a +b + 1
y trataremos de probar que
a +b +c
a +b + 1
+ (1 a) (1 b) (1 c) 1.
Este problema algebraicamente es facil, pero contin ua confuso y claro podramos haber
ido mas lejos (es decir, esta desigualdad podra no ser verdadera). Sin embargo tenemos
lo siguiente:
a +b +c
a +b + 1
+ (1 a) (1 b) (1 c)
=
a +b + 1
a +b + 1
+
c 1
a +b + 1
+ (1 a) 1 b (1 c)
= 1
_
1c
a+b+1
_
[1 (1 +a +b) (1 a) (1 b)] .
La desigualdad deseada sigue de esta expresion despues de se nalar que
(1 +a +b) (1 a) (1 b) (1 +a +b +ab) (1 a) (1 b)
= (1 +a) (1 +b) (1 a) (1 b)
=
_
1 a
2
_ _
1 b
2
_
1.
7.1.4 Siendo n un n umero positivo y ai 1 para i = 1, 2, ..., n. Muestra que
(1 +a1) (1 +a2) ... (1 +an)
2
n
n + 1
(1 +a1 +... +an) .
280 CAP

ITULO 7. DESIGUALDADES
Soluci on. La induccion es una estrategia natural aqu y no es difcil de manejarlo de
esta manera pero el siguiente argumento es mas divertido:
(1 +a1) (1 +a2) ... (1 +an)
= 2
n
_
1
2
+
a
1
2
_ _
1
2
+
a
2
2
_
...
_
1
2
+
an
2
_
= 2
n
_
1 +
a
11
2
_ _
1 +
a
2
1
2
_
...
_
1 +
an1
2
_
2
n
(1 +
a
1
1
2
1 +
a
2
1
2
+... +
a
n1
2
)
2
n
_
1 +
a
11
n+1
+
a
21
n+1
+... +
a
n1
n+1
_
=
2
n
n+1
(n = 1 +a1 1 +a2 1 +... +an 1)
=
2
n
n+1
(1 +a1 +a2 +... +an) .
7.1.5 Para cada n entero positivo. Probemos que
_
1 +
1
n
_
n
<
_
1 +
1
n + 1
_
n+1
Soluci on. Esta es una desigualdad importante que podemos probar en una variedad de
formas (ver 7.1.11,7.2.28,7.4.18). Aqu daremos una prueba basada en la comparacion
de terminos correspondientes en la expansion binominal de cada lado. Sobre el lado
izquierdo,
_
1 +
1
n
_
n
=
n

k=0
_
n
k
_
_
1
n
_
k
=
n

k=0
n(n 1) (n 2) ... (n k + 1)
n.n.n...n
1
k!
=
n

k=0
1
k!
_
1
1
n
__
1
2
n
_
...
_
1
k 1
n
_
.
De una manera similar,
_
1 +
1
n + 1
_
n+1
=
n+1

k=0
1
k!
_
1
1
n + 1
__
1
2
n + 1
_
...
_
1
k 1
n + 1
_
=
_
1
n + 1
_
n+1
+
n

k=0
1
k!
_
1
1
n + 1
__
1
2
n + 1
_
...
_
1
k 1
n + 1
_
.
La desigualdad es obvia ahora, dado que comparando los coecientes de 1/k! en estas
7.1. PROPIEDADES B

ASICAS DE LAS DESIGUALDADES. 281


expresiones vemos que para cada k = 0, 1, 2, ..., n,
_
1
1
n
__
1
2
n
_
...
_
1
k 1
n
_
<
_
1
1
n + 1
__
1
2
n + 1
_
...
_
1
k 1
n + 1
_
.
Vale la pena notar que
_
1 +
1
n
_
n
=
n

k=0
1
k!
_
1
1
n
__
1
2
n
_
...
_
1
k 1
n
_
<
n

k=0
1
k!
= 1 +
n

k=1
1
k!
< 1 +
n

k=1
1
2
k1
= 1 +
n1

k=0
1
2
k
< 1 +

k=0
1
2
k
= 3
De esta manera ,la sucesion (1 + 1/n)
n
es creciente y acotada superiormente por
3.(Puede demostrarse que la sucesion converge al n umero e.)
El siguiente resultado es importante teoricamente y es muy util (e.g.,ver 7.4.9 y 7.4.20).
7.1.6 Suponiendo que f : R R satisface
f
_
x +y
2
_
<
f (x) +f (y)
2
para toda x y y en un intervalo(a, b) , x = y. Prueba que
f
_
x1 +x2 +... +xn
n
_
<
f (x1) +f (x2) +... +f (xn)
n
Siempre que las xi esten en (a, b), con xi = xj para por lo menos un par (i, j) .
Soluci on. Asumiendo que el resultado es cierto siempre que n = m; mostraremos que
tambien lo es para n = 2m. Disponiendolo de otro modo si es necesario, asumiremos
que x1 = x2 y tenemos
f
_
x1 +... +x2m
2m
_
=
= f
_
1
2
_
x1 + +xm
m
+
xm+1 + +x2m
m
_
_

1
2
_
f
_
(x
1
)++(xm)
m
_
+f
_
(x
m+1
)++(x
2m
)
m
__
<
1
2
_
f (x1) + +f (xm)
m
+
f (xm+1) + +f (x2m)
m
_
=
f (x1) +f (x2) + +f (x2m)
2m
.
282 CAP

ITULO 7. DESIGUALDADES
De esta manera por induccion, el resultado se satisface para toda potencia positiva de
2.
Ahora supongase que n > 2 y n no es una potencia de 2; esto es suponer que 2
m1
<
n < 2
m
para alg un entero m. Tomar k = 2
m
n, y hacer yi = (x1 +... +xn) /n para
i = 1,2, ..., k. Entonces x1, x2, ..., xn, y1, y2, ..., y
k
son 2
m
n umeros en el intervalo (a, b) ,
y en consecuencia nuestro argumento anterior implica que
f
_
x1 + +xn +y1 + +y
k
2
m
_
<
f (x1) + +f (y
k
)
2
m
Pero notemos que
f
_
x1 + +xn +y1 + +y
k
2
m
_
= f
_
x
1
++xn+k(x
1
++xn)/n
2
m
_
= f
_
n(x
1
++xn)+(2
m
n)(x
1
++xn)
n2
m
_
= f
_
x
1
++xn
n
_
.
Realizando esta sustitucion dentro de la ultima desigualdad;
f
_
x1 +... +xn
n
_
<
f (x1) + +f (xn) +f (y1) +... +f (y
k
)
2
m
=
f (x1) +... +f (xn) +kf ((x1 +... +xn) /n)
2
m
Multiplicando cada lado por 2
m
se tiene
2
m
fx1 +... +xnn < f (x1) +... +f (xn)
+ (2
m
n) fx1 +... +xnn
y de esto obtenemos la desigualdad deseada para n:
f(
x1 +... +xn
n
) <
f (x1) +... +f (xn)
n
.
Problemas
7.1.7 Suponer que a, b, c son n umeros positivos. Probar que:
(a) (a +b) (b +c) (c +a) 8abc.
(b) a
2
b
2
+b
2
c
2
+c
2
a
2
abc (a +b +c) ,1
(c) Si a +b +c = 1,entonces ab +bc +ca
1
3
.
7.1. PROPIEDADES B

ASICAS DE LAS DESIGUALDADES. 283


7.1.8 Prueba que
1
2
.
3
4
.
5
6
...
999999
1000000
<
1
1000
.
(Sugerencia: Encuadra cada parte y de a poco para crear un producto condensado en
que encajen unas partes en otras.)
7.1.9 .
(a) Si a y b son n umeros reales distintos de cero, prueba que por lo menos una de las
siguientes desigualdades es cierta:

a +

a
2
+ 2b
2
2b

< 1,

a
2
+ 2b
2
2b

< 1.
(b) Si los n n umeros x1, x2, ..., xn estan en el intervalo (0, 1), prueba que por lo menos
una de las siguientes desigualdades es verdadera:
x1x2...xn 2
n
, (1 x1) (1 x2) ... (1 xn) 2
n
.
7.1.10 .
(a) Estableciendo que a
1/
b1,a2/b2, ..., an/bn sean n fracciones con bi > 0 para i =
1, 2, ..., n. Muestra que la fraccion
a1 +a2 +...an
b1 +b2 +... +bn
es un n umero entre la mas grande y la mas peque na de estas fracciones. (Notar
el caso especial en el cual todas las fracciones ai/bi son iguales.)
(b) Si
a +b
b +c
=
c +d
d +a
prueba que tambien a = c o a +b +c +d = 0.
7.1.11 .
(a) Para 0 < a < b, muestra que
(n + 1) (b a) a
n
< b
n+1
a
n+1
< (n + 1) (b a) b
n
.
(b) Aplicar esta desigualdad para el caso especial a = 1 + 1/ (n + 1) y b = 1 + 1/n
para demostrar que
(1 + 1/n)
n
< (1 + 1/ (n + 1))
n+1
.
284 CAP

ITULO 7. DESIGUALDADES
7.1.12 Prueba que para todo n.
_
n
e
_
n
< n! < e
_
n
2
_
n
.
7.1.13 (Desigualdad de Cauchy-Schwarz). Por induccion matematica sobre n, prueba
que para todos los n umeros reales a1,..., an, b1, ..., bn,
_
n

k=1
a
k
bk
_
2

_
n

k=1
a
2
k
__
n

k=1
b
2
k
_
.
7.1.14 En un cuadrilatero convexo (las dos diagonales son interiores al cuadrilatero)
prueba que la suma del largo de las diagonales es menor que el permetro pero mayor
que la mitad del perimetro.
7.1.15 Prueba que para cualquier n umero entero positivo n,
n

n < 1 +
_
2/n.
Ejemplos adicionales.
1.3.3., 1.7.4, 1.8.2.,1.8.5., 1.8.6., 1.12.7., 2.1.5., 2.1.6., 2.2.4., 2.2.6., 2.4.1., 2.4.4., 2.4.6.,
5.3.8., 6.1.3., 7.3.1., 7.4.8., 7.4.9., 7.4.21., 7.4.20, 7.4.22., 7.4.23.
7.2. La desigualdad entre la media aritmetica y
la media geometrica
Supongamos que x1 > 0 para i = 1, 2, ...n. La media aritmetica de x1, x2, ..., xn es el
n umero
x1 +x2 +... +xn
n
,
y la media geometrica de x1, x2, ..., xn es el n umero
(x1x2...xn)
1/n
.
La desigualdad de la media aritmetica-geometrica establece que
(x1x2...xn)
1/n

x1 +x2 +... +xn


n
,
Con la igualdad s y solo s todas las x
,
i
s son iguales.
El caso especial n = 2 fue vericado algebraica y geometricamente empezando los
parrafos de la seccion 7.1. Una prueba para valores mayores de n puede ser manejada
por induccion matematica (ver 7.2.5. o 2.5.7),o considerando la funcion f (t) = log t
7.2. LADESIGUALDADENTRE LAMEDIAARITM

ETICAYLAMEDIAGEOM

ETRICA285
(ver 7.4.20). Sin embargo una heurstica mas ilustrativa (no obstante no es una prueba)
puede ser hecha como sigue.
Considera la media geometrica (x1x2...xn)
1/n
y la media aritmetica(x1 +... +xn) /n
si no todas lasx
,
i
son iguales reemplaza la mas grande y la mas peque na por ellas,
decimos xM y +xm, respectivamente, por
1
2
(xM +xm) , entonces debido a que
1
2
(xM+
xm)+
1
2
(xM +xm) = xM + xm, y
_
1
2
(xM +xm)

2
> xM + xm, el resultado de este
reemplazo es que la media geometrica ha crecido mientras la media aritmetica se ha
mantenido sin cambio. Si la nueva serie de n umeros n no son todos iguales, podemos
repetir el proceso anterior. Repitiendo este proceso sucientemente podemos hacer
las cantidades tan aproximadas como favorezcan.(este paso necesita justicacion adi-
cional, pero no nos preocuparemos de eso aqu.) En cada paso de este proceso, la
media geometrica es incrementada y la aritmetica sigue sin cambio. Podra suceder
que todos los n umeros se volvieran iguales (esto quizas nunca suceda, en consecuencia;
por ejemplo, tomamos x1 = 1, x2 = 3, x3 = 4) las dos medias coincidiran. Este puede
ser el caso, por lo tanto la media geometrica es menor o igual que la media aritmetica,
con la igualdad si y solo si todos los n umeros son iguales.
Un ejemplo de este proceso, considerar el caso x1 = 2, x2 = 4, x3 = 8, x4 = 12. El
algoritmo descrito proporciona las siguiente secuencia: {2, 4, 8, 12} {7, 4, 8, 7, }
{7, 6, 6, 7, }
_
13
2
,
13
2
,
13
2
,
13
2
_
. La media geometrica de la correspondiente serie se
incrementa a
13
2
;la media aritmetica se mantiene ja en
13
2
.
7.2.1 Prueba que el cubo es el paraleleppedo rectangular con m aximo volumen para
una area de supercie dada y un mnimo de area de supercie para un volumen
dado.
Soluci on. Supongamos que las longitudes de los tres lados adyacentes son a, b,y
c.Supongamos que A y V denotan el area de la supercie y el volumen respectiva-
mente del paraleleppedo. Entonces
A = 2 (ab +bc +ca) y V = abc.
Por desigualdad de las medias aritmetica-geometrica,
V
2
= a
2
b
2
c
2
= (ab) (bc) (ca)
_
ab +bc +ca
3
_
3
= 2 (ab +bc +ca)6
3
= A6
3
.
De esta manera para todo a, b, c.
6V
2/3
A.
Ademas, 6V
2/3
< A en todos los casos excepto para cuando ab = bc = ca (o al
equivalente ,cuando a = b = c ) y en este caso 6V
2/3
= A. Entonces si A es ja,
obtenemos el volumen mayor (es decir V = (A/6)
3/2
) cuando a = b = c (un cubo), y
cuando V es jo, obtenemos la menor supercie del area (es decir A = 6V
2/3
) cuando
a = b = c (un cubo).
7.2.2 Prueba la siguiente desigualdad :
n
_
(n + 1)
1/n
1
_
< 1 +
1
2
+
1
3
+... +
1
n
< n (n 1) n
1/(n1)
.
286 CAP

ITULO 7. DESIGUALDADES
Soluci on. Suponer que sn = 1 +
1
2
+ ... + 1/n. La desigualdad de la izquierda es
equivalente a probar que
n +ss
n
> (n + 1)
1/n
la cual parece una aplicacion de la desigualdad de las medias aritmetica-geometrica.
Podemos desarrollar la idea trabajando de la siguiente manera:
n = sn
n
=
n + (1 + 1/2 +... + 1/n)
n
=
(1 + 1) + (1 + 1/2) +... + (1 + 1/n)
n
=
2 + 3/2 + 4/3 +... + (n + 1) /n
n
>
_
2.
3
2
.
4
3
...
n + 1
n
_
1/n
= (n + 1)
1/n
Para la desigualdad de la derecha necesitamos mostrar que
n sn
n 1
> n
1/(n1)
.
Otra vez,usando la desigualdad de las medias aritmetica-geometrica, tenemos
n ss
n 1
=
n 1 (1 + 1/2 = 1/3 +... + 1/n)
n 1
=
(1 1) + (1 1/2) +... + (1 1/n)
n 1
=
1/2 + 2/3 +... + (n 1) /n
n 1
>
_
1
2
2
3
3
4
...
n 1
n
_
1/(n1)
=
_
1
n
_
1/(n1)
= n
1/(n1)
.
7.2.3 Si a, b, c,son n umeros positivos tales que (1 +a) (1 +b) (1 +c) = 8. Prueba que
abc 1.
Soluci on. Tenemos que
1 + (a +b +c) + (ab +bc +ca) +abc = 8.
Por la desigualdad de las medias aritmetica-geometrica,
a +b +c 3 (abc)
1/3
y ab +bc +ca 3 (abc)
2/3
,
7.2. LADESIGUALDADENTRE LAMEDIAARITM

ETICAYLAMEDIAGEOM

ETRICA287
Con la igualdad s y solo si a = b = c. Entonces,
8 1 + 3 (abc)
1/3
+ 3 (abc)
2/3
+abc
=
_
1 + (abc)
1/3
_
3
.
Esto muesta que
(abc)
1/3
(2 1) = 1,
o equivalentemente,
abc 1
con la igualdad s y solo si a = b = c = 1.
7.2.4 Suponer que xi > 0, = 1, 2, ...n y suponer que xn+1 = x1. Muestra que
n

i=1
_
xi+1
xi
_

i=1
_
xi
xi+1
_
n
.
Soluci on. Considerar el caso n = 3. Por la desigualdad de las medias aritmetica-
geometrica, tenemos
x2
x1
=
x2
x3
.
x3
x1
,1
1
3
_
x2
x3
_
3
+
1
3
_
x3
x1
_
3
+
1
3
,
x3
x2
=
x1
x2
.
x3
x1
,1
1
3
(
x1
x2
)
3
+ +
1
3
(
x3
x1
)
3
+
1
3
,
x1
x3
=
x1
x2
.
x2
x3
,1
1
3
(
x1
x2
)
3
+
1
3
(
x2
x3
)
3
+ +
1
3
.
Tambien
1 =
x1
x2
.
x2
x3
.
x3
x1

1
3
(
x1
x2
)
3
+
1
3
( x2x3)
3
+
1
3
(
x3
x1
)
3
.
Sumando estas desigualdades nos da el resultado deseado. El caso para un n umero
arbitrario entero positivo n es similar.
Problemas
288 CAP

ITULO 7. DESIGUALDADES
7.2.5 Completando los pasos de la siguiente prueba inductiva de la desigualdad de las
medias aritmetica-geometrica: Para cada k supongamos que A
k
= (x1 +x2 +... +x
k
) /k,
y G
k
= (x1x2...x
k
)
1/k
. Asumimos que hemos demostrado que A
k
G
k
.
Supongamos que
A =
x
k+1
+ (k 1) A
k=1
k
y G =
_
x
k+1
A
k1
k+1
_
1/k
.
Entonces, usando la hipotesis inductiva, tenemos A G y se concluye que A
k+1
=
1
2
(A
k
+A) (A
k
A) 0
1/2
(G
k
G)
1/2
=
_
G
k+1
k+1
A
k1
k+1
_
1/(2k)
. A partir de esto tenemos
que A
k+1
G
k+1
. Sobre las bases de este argumento es muy facil comprobar que la
igualdad se da s y solo si todas las x
,
i
s son iguales.
7.2.6 Si a, b, c son n umeros positivos ,probamos que
_
a
2
b +b
2
c +c
2
a
_ _
a
2
c +b
2
a +c
2
b
_
9a
2
b
2
c
2
.
7.2.7 Suponer que a1..., an son n umeros positivos y b1, ..., bn es un nuevo arreglo de
a1, ..., an. Muestra que
a1
b1
+
a2
b2
+... +
an
bn
n.
7.2.8 .
(a) Para n umeros positivos a y b, a = b, prueba que
(ab
n
)
1/(n+1)
<
a +nb
n + 1
(b) En el apartado a, consideramos el caso a = 1 y b = 1 + 1/n y mostramos que
_
1 +
1
n
_
n
<
_
1
n+1
_
n+1
.
(c) En el apartado a, reemplazamos n por n+1,suponiendo que a = 1 y b = n(n + 1)
y mostremos que
_
1 +
1
n
_
n+1
>
_
1 +
1
n + 1
_
n+2
.
7.2. LADESIGUALDADENTRE LAMEDIAARITM

ETICAYLAMEDIAGEOM

ETRICA289
7.2.9 Para cada n umero entero n > 2, probemos que
(a)
n

k=0
_
n
k
_

_
2
n
2
n 1
_
n1
,
(b) n! <
_
n+1
2
_
n
,
(c) 1 3 5 ... (2n 1) < n
n
.
7.2.10 Dando todas las races de x
6
6x
5
+ax
4
+bx
3
+cx
2
+dx+1 = 0 son positivas,
encontrar a, b, c, d.
7.2.11 .
(a) Suponer que xi > 0 para i = 1, 2, ...n, y suponer que p1, p2, ..., pn son n umeros
enteros positivos. Probemos que
_
x
p1
1
x
p2
2
...x
pn
n
_
1/(p
1
+...pn)

p1x1 +...pnxn
p1 +... +pn
.
(b) Probar el resultado del apartado (a) cuando las p
,
i
s son n umeros racionales pos-
itivos.
7.2.12 Usa la desigualdad de las medias aritmetica-geometrica para cada uno de lo
siguientes:
(a) Un tanque con una base rectangular y lados rectangulares abierto por arriba.
Sera construido con una amplitud de 4 metros y su volumen de 36 metros c ubicos.
La construccion del tanque cuesta $10 por metro cuadrado de base y $ 5 por
metro cuadrado por sus lados Cuanto es lo menos que cuesta el tanque ?
(b) Un granjero con un terreno adyacente a la rivera del ro desea cercar una region
para pastar. Si por el lado del ro no necesita cerca y el tiene 1000 de alam-
bre, cuales seran las dimensiones de este terreno y cual es su area maxima?
(Sugerencia: su equivalente es aumentar el area dos veces.)
(c) Un granjero con 1000 metros de alambre, desea construir un corral y dividirlo
en dos peque nos espacios rectangulares a nadiendo una cerca en com un de abajo
hacia la mitad. Cuales seran las dimensiones en general del corral a n de
maximizar el area total?
(d) Probar que el cuadrado es el rectangulo del area maxima dada al permetro y el
permetro mnimo al area dada.
(e) Probar que el triangulo equilatero es el area maxima del triangulo para un permetro
dado, y un permetro mnimo para una area dada. (Sugerencia: El area del
triangulo es relacionada al permetro del triangulo por la formula A = (s (s a) (s b) (s c))
1/2
,
donde a, b, c, son las longitudes de los lados del triangulo y s =
1
2
P, P es el
permetro del triangulo .)
290 CAP

ITULO 7. DESIGUALDADES
Ejemplos adicionales
Introduccion a la seccion 7.6; 7.3.1, 8.1.4.
7.3. Desigualdad Cauchy-Schwarz.
Suponiendo que ai > 0 y bi > 0 para i = 1, 2, ..., n. La desigualdad Cauchy-Schwarz
arma que
n

i=1
aibi
_
n

i=1
a
2
i
_
1/2
_
n

i=1
b
2
i
_
1/2
,
con la igualdad s y solo si a1/b1 = a2/b2 = ...an/bn.
Un prueba puede ser dada usando induccion matematica (ver 7.1.13). Pero un sencillo
metodo es considerar el polinomio cuadratico P (x) =

n
i=1
(aix bi)
2
. Observese
que P (x) 0 Para todas las x; en efecto, P (x) = 0 solo bajo las condiciones en las
cuales a1/b1 = a2/b2 = ... = an/bn y x = bi/ai. Ahora
P (x) =
n

i=1
_
a
2
i
x
2
2aibix +b
2
i
_
=
_
n

i=1
a
2
i
_
x
2
2
_
n

i=1
aibi
_
x +
n

i=1
b
2
i
,
y desde que P (x) 0,el distintivo de P no puede ser positivo y en realidad igualara a
cero solo cuando P (x) = 0. Entonces,
_
2
n

i=1
aibi
_
2
4
_
n

i=1
a
2
i
__
n

i=1
b
2
i
_
0,
o al equivalente,
n

i=1
aibi
_
n

i=1
a
2
i
_
1/2
_
n

i=1
b
2
i
_
1/2
,
con igualdad s y solo si a1/b1 = ...an/bn.
En esta desigualdad, note que el requerimiento de que ai y bi sean positivos es redun-
dante, ya que para todos ai,bi
n

i=1
aibi
n

i=1
|ai| |bi|
_
n

i=1
a
2
i
_
1/2
_
n

i=1
b
2
i
_
1/2
.
7.3.1 Si a, b, c > 0, es verdad que a cos
2
+ b sin
2
< c implica que

a cos
2
+

b sin
2
<

c?
7.3. DESIGUALDAD CAUCHY-SCHWARZ. 291
Soluci on. Por la desigualdad de Cauchy-Schwarz

a cos
2
+

bsen
2

[(

a cos )
2
+ (

bsen)
2
]
1/2
(cos
2
+sen
2
)
1/2
= (a cos
2
+bsen
2
)
1/2
<

c
Hay tambien una solucion plausible basada en la desigualdad de las medias aritmetica-
geometrica:
_

a cos
2
+

b sin
2

_
2
= a cos
4
+ 2

b cos
2
sin
2
+b sin
4

a cos
4
+ (a +b) cos
2
sin
2
+b sin
4

=
_
a cos
2
+b sin
2

_ _
cos
2
+ sin
2

_
< c.
Otra solucion, de naturaleza mas geometrica, esta dada en 7.4.19.
7.3.2 Supongamos que P es el punto interior del triangulo ABC, y supongamos que
r1,r2, r3 denotan las distancias de P a los lados a1, a2, a3 del triangulo respectivamente.
Supongamos que R denota el circunradio de ABC. Muestra que

r1 +

r2 +

r3
1

2R
_
a
2
1
+a
2
2
+a
2
3
_
1/2
con la igualdad s y solo si ABC es equilatero y P es el incentro.
Soluci on. Por la desigualdad de Cauchy-Schwarz

r1 +

r2 +

r3 =

a1r1

a2r2 +

a2r2
_
1/a2 +

a3r3
_
1/a3
(a1r1 +a2r2 +a3r3)
1/2
_
1
a1
+
1
a2
+
1
a3
_
1/2
con igualdad s y solo si

a1r1
_
1/a1
=

a2r2
_
1/a2
=

a3r3
_
1/a3
,
o el equivalente s y solo si
a
2
1
r1 = a
2
2
r2 = a
2
3
r3.
En la desigualdad que precede ,reconocemos que a1r1 + a2r2 + a3r3 = 2A, donde A
es el area del triangulo. Tambien, sabemos que el area del triangulo, en terminos del
radio del circunradio R, esta dada por A = a1a2a3/4R (ver 8.1.12). Por lo tanto,
a1r1 +a2r2+ a3r3 = a1a2a3/2R y tenemos

r1 +

r2 +

r3
_
a1a2a3
2R
_
1/2
_
1
a1
+
1
a2
+
1
a3
_
1/2
=
_
a1a2a3
2R
_
1/2
_
a2a3 +a3a1 +a1a2
a1a2a3
_
1/2
=
1

2R
(a2a3 +a3a1 +a1a2)
1/2
.
292 CAP

ITULO 7. DESIGUALDADES
Ahora, nuevamente por la desigualdad de Cauchy-Schwarz,
a2a3 +a3a1 +a1a2
_
a
2
2
+a
2
3
+a
2
1
_
1/2
_
a
2
3
+a
2
1
+a
2
2
_
1/2
=
_
a
2
1
+a
2
2
+a
2
3
_
con igualdad s y solo si
a2/a3 = a3/a1 = a1/a2 (= (a2 +a3 +a1) / (a3 +a1 +a2) = 1; ver 7.1.10), o equiva-
lentemente, s y solo si
a1 = a2 = a3.
Luego, tenemos

r1 +

r2 +

r3
1

2R
_
a
2
1
+a
2
2
+a
2
3
_
1/2
con igualdad s y solo si a
2
1
r1 = a
2
2
r2 = a
2
3
r3 y a1 = a2 = a3;esto es s y solo si
a1 = a2 = a3 y r1 = r2 = r3. Esto completa la prueba.
7.3.3 Dado que a, b, c, d, e son n umeros reales tales que
a +b +c +d +e = 8,
a
2
+b
2
+c
2
+d
2
+c
2
= 16,
determina el maximo valor de e.
Soluci on. Las ecuaciones dadas pueden ser puestas dentro de la forma
8 e = a +b +c +d,
16 e
2
= a
2
+b
2
+c
2
+d
2
.
Deseamos encontrar una desigualdad involucrando solo e; la desigualdad de Cauchy-
Schwarz provee una manera, ya que
(a +b +c +d) (1 + 1 + 1 + 1)
1/2
_
a
2
+b
2
+c
2
+d
2
_
1/2
.
Realizando las sustituciones, y elevandolas al cuadrado, tenemos
(8 e)
2
4
_
16 e
2
_
,
64 16e +e
2
64 4e
2
,
5e
2
16e 0,
e (5e 16) 0.
De aqu se sigue que 0 e
16
5
. La cota maxima,
16
5
, se tiene cuando a = b = c = d =
6
5
.
7.3.4 Supongamos que a1, a2, ...an son reales (n > 1) y
A+
n

i=1
a
2
i
<
1
n 1
_
n

i=1
ai
_
2
.
7.3. DESIGUALDAD CAUCHY-SCHWARZ. 293
prueba que A < 2aiaj para 1 i < j n.
Soluci on. Por la desigualdad de Cauchy-Schwarz
_
n

i=1
ai
_
2
= [(a1 +a2) +a3 +.... +an]
2
(1 +... + 1) ((a1 +a2)
2
+a
2
3
+... +a
2
n
)
= (n 1)
_
n

i=1
a
2
i
+ 2a1a2
_
.
Esto, junto con la desigualdad dada, implica que
A <
_
n

i=1
a
2
i
_
+
1
n 1
_
n

i=1
ai
_
2
<
_
n

i=1
a
2
i
_
+
1
n 1
_
(n 1)
_
n

i=1
a
2
i
+ 2a1a2
__
= 2a1a2.
De forma parecida, A < 2aiaj para 1 i < j n.
7.3.5 Suponer que xi > 0 para i = 1, 2, ..., n. Para cada entero no negativo k, probar
que
x
k
1
+... +x
k
n
n

x
k+1
1
+... +x
k+1
n
x1 +... +xn
.
Soluci on. Podremos asumir sin perder la generalidad que x1 + ... + xn = 1, si no,
podemos reemplazar xi por Xi = xi/ (x1 +... +xn) .
El resultado se mantiene cuando k = 0. Asumase que el resultado se mantiene para
todos los enteros no negativos menores que k.Por la desigualdad de Cauchy-Schwarz,
n

i=1
x
k
i
n
=
n

i=1
x
(k+1)/2
i
x
(k1)/2
i
n

_
n

i=1
x
k+1
i
_
1/2
_
n

i=1
x
k1
i
n
2
_
1/2
.
Asumiendo por induccion,

n
i=1
x
k1
i
/n

n
i=1
x
k
i
, y de aqu continuando con la
` ultima desigualdad, tenemos
_
n

i=1
x
k+1
i
_
1/2
_
n

i=1
x
k1
i
n
2
_
1/2

_
n

i=1
x
k+1
i
_
1/2
_
n

i=1
x
k
i
n
_
1/2
.
Luego,
n

i=1
x
k
i
n

_
n

i=1
x
k+1
i
_
1/2
_
n

i=1
x
k
i
n
_
1/2
,
294 CAP

ITULO 7. DESIGUALDADES
_
n

i=1
x
k
i
n
_
1/2

_
n

i=1
x
k+1
i
_
1/2
,
n

i=1
x
k
i
n

n

i=1
x
k+1
i
.
Por induccion la prueba esta completa.
Problemas
7.3.6 Usar la desigualdad Cauchy-Schwarz para probar que si a1, ..., an son n umeros
reales tales que a1 +... +an = 1, entonces a
2
1
+... +a
2
n
1/n.
7.3.7 Usa la desigualdad Cauchy-Schwarz para probar lo siguiente:
(a) Si p1,..., pn, x1, ..., xn son 2n n umeros positivos,
(p1x1 +... +pnxn)
2
(p1 +... +pn)
_
p1x
2
1
+... +pnx
2
n
_
.
(b) Si a, b, c,son n umeros positivos,
_
a
2
b +b
2
c +c
2
a
_ _
ab
2
+bc
2
+ca
2
_
9a
2
b
2
c
2
.
(c) Si x
k
, y
k
, k = 1, 2, ..., n, son n umeros positivos,
n

k=1
x
k
y
k

_
n

k=1
kx
2
k
_
1/2
_
n

k=1
y
2
k
/k
_
1/2
.
(d) Si a
k
, b
k
, c
k
, k = 1, 2, ..., n, son n umeros positivos,
_
n

k=1
a
k
b
k
c
k
_
4

_
n

k=1
a
4
k
__
n

k=1
b
4
k
__
n

k=1
c
2
k
_
2
.
(e) Si C
k
=
_
n
k
_
para n > 2, 1 k n,
n

k=1

C
k

n2
n
1.
7.3.8 Para n un entero positivo, supongase que (a1, a2, ...an) y (b1, b2, ..., bn) son dos
permutaciones (no necesariamente diferentes ) de (1, 2, ..., n). Encontrar la mayor cota
inferior y la menor cota superior para a1b1 +... +anbn.
7.4. CONSIDERACIONES FUNCIONALES. 295
7.3.9 Si a, b, c, d son n umeros positivos tales como c
2
+d
2
=
_
a
2
+b
2
_
3
. Probar que
a
3
c
+
b
3
d
1,
con la igualdad s y solo si ad = bc. (Sugerencia: Muestra que
_
a
3
/c +b
3
/d
_
(ac +bd)
_
a
2
+b
2
_
2
ac +bd.)
7.3.10 Supongase que P es un punto en el interior del triangulo ABC , y supong-
amos que r1, r2, r3, denotan las distancias de P a los lados a1, a2, a3 del triangulo
respectivamente. Usar la desigualdad Cauchy-Schwarz para mostrar que el mnimo
valor de
a1
r1
+
a2
r2
+
a3
r3
ocurre cuando P es el incentro del triangulo ABC. (Sugerencia: ai =

airi
_
ai/ri.)
Ejemplo adicional
7.6.14.
7.4. Consideraciones Funcionales.
En esta seccion daremos ejemplos para mostrar como las tecnicas del analisis, partic-
ularmente de diferenciacion, pueden ser usadas con ecacia en una amplia variedad de
problemas de desigualdad.
7.4.1 Dados n umeros positivos p, q y r. Tales que 2p = q +r, q = r. Muestra que
p
q+r
q
q
r
r
< 1.
Soluci on. Supongase que q y r son enteros positivos y considerar los n umeros q,
1/q, ..., 1/q y los n umeros r,1/r, ..., 1/r. Por la desigualdad de las medias aritmetica-
geometrica
_
1
q
q
.
1
r
r
_
1/(q+r)
<
q (1/q) +r (1/r)
q +r
=
1
p
,
el cual es equivalente a la desigualdad deseada.
Por supuesto, este metodo se rompe si cualquiera q o r no son enteros ,entonces como
podemos proceder?. Una idea es reescribir la desigualdad de la siguiente manera:
p
q+r
< q
q
r
r
_
q +r
2
_
q+r
< q
q
r
r
,
296 CAP

ITULO 7. DESIGUALDADES
12
q+r
< (
q
q +r
)
q
(
r
q +r
)
r
,
1
2
< qq +r
q/(q+r)
rq +r
r/(q+r)
.
Si x = q/ (q +r) y y = r/ (q +r) Observar que x + y = 1 y 0 < x, y < 1. Entonces
el problema es equivalente al probar que
F (x) x
x
(1 x)
1x
>
1
2
, 0 < x < 1, x =
1
2
.
Introduciendo la funcion de esta manera ,somos capaces de usar los metodos de anali-
sis. La idea es encontrar el valor mnimo de F sobre (0, 1). Para simplicar la difer-
enciacion ,consideraremos la funcion G(x) = log F (x) . Para encontrar los puntos
crticos, derivamos:
G

(x) =
d
dx
[xlog x + (1 x) log (1 x)]
= (log x + 1) 1 log (1 x)
= log
x
1 x
.
Vemos que G

(x) = 0 s y solo si x =
1
2,
Ademas ,G

(x) < 0 sobre el intervalo


_
0,
1
2
_
,y
G

(x) > 0 sobre el intervalo


_
1
2
, 1
_
. Por lo tanto G

(x) toma su valor mnimo en (0, 1)


para x =
1
2
. Entonces el valor mnimo de F (x)en (0, 1)es F
_
1
2
_
= (
1
2
)
1/2
(
1
2
)
1/2
=
1
2
.
De la siguiente manera F (x) >
1
2
para toda x en (0, 1) , x =
1
2
y la prueba esta termi-
nada.
7.4.2 Si p y q son n umeros positivos con p +q = 1. Muestra que para toda x,
pe
x/p
+qe
x/q
e
x
2
/8p
2
q
2
.
Soluci on. Considere la funcion
F (x) =
pe
x/p
+qe
x/q
e
x
2
/8p
2
q
2
Nuestro problema es probar que F (x) 1para toda x . Debido a la simetra del
problema, es suciente probar que F (x) 1 para toda x 0.
Notamos que F (0) = 1. Por corolario (iii) del teorema del valor medio (ver la discusion
anterior 6.6.2) es suciente probar que F

(x) 0 para toda x. Para simplicar el


calculo, considerar la funcion G(x) = log F (x). Del procedimiento de diferenciacion
y simpli- cacion algebraca resulta
G

(x) =
F

(x)
F (x)
=
e
x/p
e
x/q
pe
x/p
+qe
x/q

x
4p
2
q
2
=
e
x/pq
1
pe
x/pq
+q

x
4p
2
q
2
Como F (x) > 0 para toda x 0, F

(x) 0 s y solo si G

(x) 0. Desafortunada-
mente, la expresion anterior de G

(x) hace difcil determinar si G

(x) 0 o no, por


7.4. CONSIDERACIONES FUNCIONALES. 297
lo tanto manejaremos el analisis con otra perspectiva. Es decir, G

(0) = 0 y (otra vez


dejando a un lado los detalles)
G

(x) =
_
pe
x/pq
q
_
2
4p
2
q
2
(pe
x/pq
+q)
2
.
Aqu es claro que G

(x) 0.Para toda x 0. Esto junto con G

(0) = 0,implica que


G

(x) 0para toda x 0, y esto implica que F

(x) 0 para toda x 0. Por lo tanto


F (0) = 1, d edonde F (x) 1 para toda x 0,y la prueba esta terminada.
El procedimiento usado en el problema anterior es muy com un, para recapitular como
sigue: Probar una desigualdad de la forma
f (x) g (x) , x a,
es equivalente probar que
Q(x)
f (x)
g (x)
1, x a,
o que
D(x) f (x) g (x) 0, x a.
Cualquiera puede ser vericada probando la desigualdad para x = a y entonces probar
que Q

(x) 0 (o D

(x) 0 respectivamente) para toda x a.


En el ejemplo previo, si hubieramos considerado la funcion
D(x) = e
x
2
/8p
2
q
2
pe
x/p
qe
x/q
,
este analisis no hubiera sido concluido: sin embargo D(0) = 0, no es necesariamente
el caso de D

(x) 0 (por ejemplo cuando p =


1
3
, q =
2
3
, x =
1
2
).
7.4.3 Probar que para todos los n umeros reales a y b
|a +b|
p
|a|
p
+|b|
p
, 0 p 1
Soluci on. La desigualdad es trivial en casos especiales. Por ejemplo el resultado
es cierto s a = 0,o si a y b tienen signos opuestos. Tambien, si p = 0 o p = 1, el
resultado es verdadero. Por lo tanto es suciente mostrar que el resultado es verdadero
cuando a y b son positivos y 0 < p < 1.
Para cada a y b y p, supongamos que x = b/a. Entonces, el problema es demostrar
que
(1 +x)
p
1 +x
p
, x > 0, 0 < p < 1.
Para esto, supongamos que D(x) = 1+x
p
(1 +x)
P
. Tenemos D(0) = 0 y D

(x) =
px
p1
p (1 +x)
p1
> 0, entonces por las observaciones anteriores, la prueba esta ter-
minada. (Note que si p > 1, la desigualdad podra ser contraria.)
298 CAP

ITULO 7. DESIGUALDADES
7.4.4 En [0, 1], si f tiene derivada continua cumpliendo que 0 < f

(t) 1. Y
suponiendo que f (0) = 0. Probar que
__
1
0
f (t) dt
_
2

_
1
0
[f (t)]
3
dt
Soluci on. Aqu, como en ultimo ejemplo, no esta claro como hacer uso de la difer-
enciacion. La idea es introducir una variable y probar un resultado mas general. Para
0 x 1, sea
F (x)
__
x
0
f (t) dt
_
2

_
x
0
(f (t))
3
dt.
Entonces F (0) = 0. y
F

(x) = 2
__
x
0
f (t) dt
_
f (x) [f (x)]
3
= f (x)
_
2
_
x
0
f (t) dt [f (x)]
2
_
.
Sabemos que f (x) 0 para 0 < x < 1 (ya que estamos dando f (0) = 0 y f

(x) > 0);


sin embargo no esta claro que el segundo factor en la ultima expresion para F

es no
negativo. Por lo tanto, tomemos
G(x) = 2
x

0
f (t) dt [f (x)]
2
, 0 x 1.
Entonces G(0) = 0 y
G

(x) = 2f (x) f

(x)
= 2f (x)
_
1 f

(x)

0
(la ultima desigualdad se tiene porque f (x) 0 y por hipotesis ,1 f

(x) 0). Se
sigue por los argumentos que F (x) 0 para toda x, 0 x 1;en particular ,F (1) 0
y la prueba esta completa.
7.4.5 Demostrar que si x es positivo entonces log (1 + 1/x) > 1/ (1 +x) .
Soluci on. Suponer que f (x) = log (1 + 1/x)1/ (1 +x) (= log (1 +x) log x 1/ (1 +x)) .
Entonces
f

(x) =
1
1 +x

1
x
+
1
(1 +x)
2
=
x(1 +x) (1 +x)
2
+x
x(1 +x)
2
=
1
x(1 +x)
2
< 0 para x > 0
Ademas, lmxf (x) = 0 y esto, junto con f

(x) < 0 para x > 0, implica que


f (x) > 0 para x > 0.
7.4. CONSIDERACIONES FUNCIONALES. 299
7.4.6 Encontrar todos los n enteros positivos tales que
3
n
+ 4
n
+... + (n + 2)
n
= (n + 3)
n
.
Soluci on. Un calculo directo muestra que obtenemos la igualdad cuando n = 2 y
cuando n = 3. Un argumento de paridad muestra que no se da ni cuando n = 4 o
cuando n = 5. Basado en este principio, podramos suponer que la clave podra involu-
crar un trabajo en aritmetico modular. Sin embargo estas pruebas no son fructferas
y podramos buscar otro metodo. Mostraremos que
3
n
+ 4
n
+... + (n + 2)
n
< (n + 3)
n
para n 6, y de esta manera la igualdad se presenta solo cuando n = 2 o n = 3.
La desigualdad que queremos probar puede ser escrita en la siguiente forma:
_
3
n + 3
_
n
+
_
4
n + 3
_
n
+... +
_
n + 2
n + 3
_
n
< 1,
_
1
n
n + 3
_
n
+
_
1
n 1
n + 3
_
n
+... +
_
1
1
n + 3
_
n
< 1,
o revirtiendo el orden convenientemente,
_
1
1
n + 3
_
n
+
_
1
2
n + 3
_
n
+... +
_
1
n
n + 3
_
n
< 1.
Para probar esta desigualdad es suciente mostrar que
_
1
k
n + 3
_
n
< (
1
2
)
k
, k = 1, 2, ..., n.
Para entonces,
_
1
1
n + 3
_
n
+
_
1
2
n + 3
_
n
+... +
_
1
n
n + 3
_
n
<
1
2
+ (
1
2
)
2
+... + (
1
2
)
n
< 1.
Sigue, entonces, probar que
_
1
k
n + 3
_
n
< (
1
2
)
k
, k = 1, 2, ..., n.
Por la desigualdad Bernoulli (una muy util desigualdad; ver 7.4.10),
_
1
1
n + 3
_
k

_
1
k
n + 3
_
,
y por lo tanto,
_
1
k
n + 3
_
n

_
1
1
n + 3
_
kn
=
__
1
1
n + 3
_
n
_
k
300 CAP

ITULO 7. DESIGUALDADES
El paso nal es mostrar que
_
1
1
n + 3
_
n

1
2
cuando n 6
Para esto, considerese la funcion
F (x) =
_
1
1
x + 3
_
x
.
Esto es directamente muestra que F

(x) < 0 para x 6, y que F (6) <


1
2
.
As, la prueba esta completa.
7.4.7 Probar que para 0 a < b <
1
2
,
b a
cos
2
a
< tan b tan a <
b a
cos
2
b
.
Soluci on. Considerar la funcion f (x) = tan x sobre [a, b, ] . De acuerdo al teorema
del valor medio hay un punto c en (a, b) tal que
f (b) f (a)
b a
= f

(c) .
en este caso,signica que
tan b tan a
b a
= sec
2
c
para alguna c en (a, b) . La desigualdad deseada se basa en el principio que sec
2
a <
sec
2
c < sec
2
b para 0 a < b < /2.
Muchas desigualdades pueden ser establecidas considerando una apropiada funcion
convexa (o concava).La idea esta basada en el resultado de 6.6.3: si f : R R es tal
que f

(x) 0, entonces
f
_
x +y
2
_

f (x) +f (y)
2
,
y si f

(x) 0, entonces
f
_
x +y
2
_

f (x) +f (y)
2
Por ejemplo para n umeros reales x y y,
_
x +y
2
_
2

x
2
+y
2
2
Ya que f (x) = x
2
es una funcion convexa . Como otro ejemplo, si 0 < x, y < ,
sin
_
x +y
2
_

sin x + sin y
2
.
Pues f (x) = sin x es una funcion concava en (0, ) .
7.4. CONSIDERACIONES FUNCIONALES. 301
7.4.8 Probar que si a y b son n umeros positivos tales como a +b = 1, entonces
_
a +
1
a
_
2
+
_
b +
1
b
_
2

25
2
.
Soluci on. Hemos visto que
x
2
+y
2
2

_
x +y
2
_
2
.
Tomar x = a + 1/a y y = b + 1/b. Entonces
1
2
_
_
a +
1
a
_
2
+
_
b +
1
b
_
2
_

_
1
2
__
a +
1
a
_
+
_
b +
1
b
___
2
=
_
1
2
_
1 +
1
a
+
1
b
__
2
.
Pero por la desigualdad de Cauchy-Schwarz (1/a + 1b) (a +b) (1 + 1)
2
= 4,de man-
era que
_
1
2
_
1 +
1
a
+
1
b
__
2

_
1
2
_
1 +
4
a +b
__
2
=
_
1 + 4
2
_
2
=
25
4
.
El resultado se encuentra despues poniendo junto las dos desigualdades anteriores y
multiplicando cada parte por 2.
7.4.9 Si 0 < xi < , i = 1, ..., n y sea x = (x1 +x2 +... +xn) /n. Probar que
n

i=1
_
sin xi
xi
_
(
sin x
x
)
n
.
Soluci on. El problema es equivalente a probar que
n

i=1
log
sin xi
xi
nlog
sin x
x
.
Considerar la funcion
f (t) = log
sin t
t
.
Este es un resultado directo para mostrar que f es concava (f

(t) < 0) sobre el inter-


valo (0, ). Por lo tanto
f
_
x1 +x2
2
_

f (x1) +f (x2)
2
.
De manera completamente analoga a la prueba de 7.1.6., se sigue que
f(
x1 +... +xn
n
)
f (x1) +... +f (xn)
n
.
302 CAP

ITULO 7. DESIGUALDADES
La sustitucion directa dentro de esta desigualdad completa la prueba :
log(
sin x
x
)
1
n
_
log
sin x1
x1
+... + log
sin xn
xn
_
.
Problemas
7.4.10 (Desigualdad de Bernoulli). Probar que para 0 < a < 1,
(1 +x)
a
1 +ax, x 1.
Como debera ir la desigualdad cuando a < 0,o cuando a > 1?
7.4.11 Probar que
x
1 +x
< log (1 +x) <
x(x + 2)
2 (x + 1)
, x > 0.
7.4.12 (Desigualdad de Huygen). Probar que
2 sin x + tan x 3x, 0 < x < /2.
7.4.13 Para toda x > 0, (2 + cos x) x > 3 sin x.
(a) Probar esta desigualdad considerando la funcion F (x) = x (3 sin x) / (2 + cos x) .
(b) Probar esta desigualdad considerando la funcion F (x) = (2 + cos x) x 3 sin x.
7.4.14 Probar que
0
xlog x
x
2
1

1
2
, x > 0, x = 1.
7.4. CONSIDERACIONES FUNCIONALES. 303
7.4.15 Probar que
log
_
1
1
x + 3
_
+
x
(x + 2) (x + 3)
< 0, x > 2.
7.4.16 Probar que
_
a + 1
b + 1
_
b+1
>
_
a
b
_
b
, a, b > 0, a = b.
7.4.17 Probar que
sin a
sin b
<
a
b
<
tan a
tan b
, 0 < b < a <
1
2
.
7.4.18 Usar los metodos de esta seccion para probar que para cada n entero positivo,
_
1 +
1
n
_
n
<
_
1 +
1
n + 1
_
n+1
(Esto es, mostrar que f (x) = (1 + 1/x)
x
es una funcion creciente.)
7.4.19 Usar la concavidad de f (x) =

x para probar que si a, b, c, son positivos,
entonces a cos
2
+b sin
2
< c implica

a cos
2
+

b sin
2
<

c. (Sugerencia: Detalla
la graca de f (x) =

x. En el dominio, donde esta el punto a cos


2
+b sin
2
, y en
el rango, donde esta

a cos
2
+

b sin
2
?)
7.4.20 Suponer xi > 0 para i = 1, 2, ..., n. Considerar la funcion F (t) = log t.y en
una manera similar que la usada en 7.4.9, probar que
(x1x2...xn)
1/n

x1 +x2 +... +xn


n
con la igualdad s y solo si todas las xi son iguales.
7.4.21 .
(a) Suponer que xi > 0 para i = 1, 2, ...n.Usar el resultado de 7.4.20.para mostrar
que
n
1
x1
+
1
x2
+... +
1
xn
(x1x2...xn)
1/n
304 CAP

ITULO 7. DESIGUALDADES
(b) Para n umeros positivos a, b, c tales como 1/a + 1/b + 1/c = 1, mostrar que
(a 1) (b 1) (c 1) 8.
7.4.22 Muestra que si a, b, c son n umeros positivos con a +b +c = 1, entonces
_
a +
1
a
_
2
+
_
b +
1
b
_
2
+
_
c +
1
c
_
2

100
3
.
7.4.23 Suponer que a, b, c denota las longitudes de los lados del triangulo. Muestra
que
3
2

a
b +c
+
b
c +a
+
c
a +b
2.
Ejemplos adicionales
6.4.6. 6.4.7.
7.5. Desigualdades por series
Otra manera para probar una desigualdad de la forma
f (x) g (x) , 0 < x < c
(ver la discusion anterior 7.4.3) es expander f y g en series de potencias, es decir
f (x) =

n=0
anx
n
y g (x) =

n=0
bnx
n
, para x en el intervalo (d, d). Si sucediera
que an bn para todas las n , entonces es obvio que f (x) g (x) para todas las x en
el intervalo (0, d).
7.5.1 Para cuales n umeros reales c es
1
2
_
e
x
+e
x
_
e
cx
2
para todos los reales x?
Soluci on. Si la desigualdad se mantiene para todad las x entonces
0 e
cx2

1
2
_
e
x
+e
x
_
=

n=0
c
n
x
2n
n!

n=0
x
2n
2
n
n!
=

n=0
_
c
n

1
2
n
_
x
2n
n!
=

n=0
_
c
n

1
2
n
_
x
2n
n!
.
Ver que c
1
2
,divide cada lado por x
2
y suponer que x = 0.
7.5. DESIGUALDADES POR SERIES 305
Por otro lado, si c
1
2
,
1
2
_
e
x
+e
x
_
=

n=0
x
2n
(2n)!

n=0
x
2n
2
n
n!
= e
x
2
/2
e
cx
2
.
Esta ultima desigualdad es cierta todas las x s y solo si c
1
2
.
Otra importante tecnica de series como aplicacion a problemas de desigualdad es
la concerniente a series alternadas. Recordar que si a0, a1, a2, ... es una sucesion de
n umeros positivos, entonces la serie

n=0
(1)
n
an converge probando que los termi-
nos disminuyen a cero (esto es ., an+1 < an y an 0 como n ). Lo mas importante
para nuestros propositos aqu, la suma de la serie cae entre cualesquiera dos parciales
sumas sucesivas. (si S denota la suma de las series y Sn denota la enesima suma par-
cial, entonces {S2n + 1} es una sucesion creciente, {S2n}es una sucesion decreciente,
y para todo n, S2n+1 < S < S2n.)
7.5.2 Mostrar que para toda x,
1 +x +
x
2
2!
+... +
x
2n
(2n)!
> 0.
Soluci on. La proposicion es cierta cuando x es positivo o cero. Supongase que x es
negativo y que k es un entero no negativo por lo cual 1 |x|

x
2
/2

...

x
k
/k!

x
k
/k!

x
k+1
/ (k + 1) 1

.... El resultado es ciertamente verdadero si 2n k,


considerando si 2n > k, el razonamiento del problema anterior implica que
1 +x +
x
2
2!
+... +
x
2n
(2n)!
> 1 +x +
x
2
2!
+... +
x
2n
(2n)!
+... = e
x
> 0.
7.5.3 Probar que (2 + cos x) x > 3 sin x, x > 0.
Soluci on. Este es el mismo problema que el 7.4.13, pero aqu daremos la solucion
basada en consideraciones de series. Del lado izquierdo de la desigualdad deseada,
sabemos que para x > 0,
(2 + cos x) x >
_
2 + 1
x
2
2!
+
x
4
4!

x
6
6!
_
x,
306 CAP

ITULO 7. DESIGUALDADES
y del lado derecho
3 sin x < 3
_
x
x
3
3!
+
x
5
5!.
_
De aqu, es suciente probar que
3x
x
3
2!
+
x
5
4!

x
7
6!
> 3
_
x
x
3
3!
+
x
5
5!
_
.
Esto es verdad para x > 0 s y solo si
x
5
4!

x
7
6!
>
3x
5
5!
,
_
1
4!

3
5!
_
>
1
6!
x
2
,
x
2
< 6!
_
2
5!
_
= 12.
Esto prueba la desigualdad deseada para el caso en el que 0 < x <

12. Pero la
desigualdad es obvia para x

12, por tanto es verdadera para todos x > 0, y la


prueba esta completa.
En la prueba anterior quizas uno se pregunte porque fueron escogidos varios terminos
de las series innitas .Porque no mas o menos? Para mantener las desigualdades vamos
en la direccion correcta, necesitamos subestimar cos x y sobreestimar sin x,de esta
manera dictaminando las se nales de los terminos nales en las series de aproximaciones.
La estimacion mas vasta podra reemplazar al cos x por 1x
2
/2 y reemplazar al sin x
por x. Esto nos dirige a investigar
_
3
x
2
2
_
x > 3x,
el cual es equivalente a

x
3
2
> 0,
y esto no es verdad para cualquier valor positivo de x.
Como el n umero de terminos en la series decrece, las aproximaciones mejoran, entonces
el siguiente trata quizas de reemplazar cos x por 1 x
2
/2 + x
4
/4! x
6
/6! y sin x
x
3
/3! +x
5
/5!. Esto nos lleva a la solucion como la presentada.
7.5.4 Probar que
_
sin x
x
_
3
cos x, 0 < x
1
2
.
7.5. DESIGUALDADES POR SERIES 307
Soluci on. Para x > 0,
_
sin x
x
_
3
>
_
1
x
2
3!
_
3
= 1
x
2
2
+
x
4
12

x
6
216
,
y
cos x < 1
x
2
2
+
x
4
4!

x
6
6!
+
x
8
8!
.
Por lo tanto ,esto es suciente para mostrar que
1
x
2
2
+
x
4
12

x
6
216
> 1
x
2
2
+
x
4
4!

x
6
6!
+
x
8
8!
o el equivalente,
1
4!
+
_

1
216
+
1
720
_
x
2

1
8!
x
4
> 0.
El lado izquierdo esta disminuyendo sobre el intervalo (0,
1
2
} y por lo tanto toma su
mnimo cuando x =
1
2
. En particular, para 0 < x /2,
1
4!
+
_

1
216
+
1
720
_
x
2

1
8!
x
4

1
4!
+
_

1
216
+
1
720
_
_

2
_
2

1
8!
_

2
_
4
>
1
4!
+
_

1
216
_
(2)
2

1
8!
(2)
4
> 0.
Esto completa la prueba.
Problemas
7.5.5 Use series innitas para probar las siguientes desigualdades:
(a) e
x
> 1 + (1 +x) log (1 +x) , x > 0.
(b) (1 +x) / (1 x) > e
2x
, 0 < x < 1.
(c) arcsin x < x/
_
1 x
2
_
, 0 < x < 1.
7.5.6 Probar que
3

1 +x 1
1
3
x +
1
9
x
2
<
5
81
x
3
, x > 0.
308 CAP

ITULO 7. DESIGUALDADES
7.5.7 Probar que
x <
1
3
(2 sin x + tan x), x > 0
[Sugerencia: Muestra la desigualdad equivalente,
sin x(2 cos x + 1) > 3xcos x, x > 0.]
7.5.8 Mostrar que sin
2
x < sin x
2
para 0 < x <
_
/2.
7.6. El principio del Emparedado
En esta seccion veremos como las consideraciones de desigualdad pueden jugar un
importante papel en los lmites de evaluacion. La idea principal (la cual tiene muchas
variables ) es expresada en el siguiente resultado.
El principio del Emparedado. Si {an} , {bn} , {cn} son sucesiones innitas tales
que an bn cn para todas las n sucientemente grandes y si {an} y {cn} convergen
al mismo n umero L, entonces {bn} tambien converge a L
Tan trivial como parece este principio (obviamente no hay alternativa para {bn}; al
estar entre {an} y {cn}, y ambos converg1endo al mismo lmite, es sorprendente que
puede ser tan util en la solucion de problemas. No obstante es aplicable en la siguiente
situacion .Supongase que deseamos evaluar el lmite de una sucesion {bn} y supong-
amos que las bn estan desafortunadamentes muy complicadas y no pueden ser tratadas
directamente. El principio del Emparedado sugiere que tratemos de colocar {bn} entre
dos sucesiones simples {an} y {cn} .
Por ejemplo, considerar la sucesion
_
n
1/n
_
. Podramos evaluar este lmite por la regla
de LHopital; considerando el siguiente argumento. Por la desigualdad de las medias
aritmetica-geometrica,
1 n
1/n
=
_
_
1 1 ... 1
. .
n2

n
_
_
1/n

(n 2) + 2

n
n
= 1 + 2
_
1

n

1
n
_
.
Ahora por el principio del emparedado (con an = 1, y cn = 1 +2 (1/

n 1/n) vemos
que n
1/n
forzosamente converge a 1.
7.6. EL PRINCIPIO DEL EMPAREDADO 309
7.6.1 Probar o refutar que el conjunto de todos los n umeros racionales positivos
pueden ser arreglados en una sucesion innita {bn} tales que
_
(bn)
1/n
_
es conver-
gente.
Soluci on. Empezamos por ordenar los n umeros racionales siguiendo el util sendero
de serpentina a traves del cuadro formado por racionales mostrados en la gura 7.2,
donde omitimos todas las fracciones no reducidas a sus terminos mnimos. La suce-
sion empieza de esta manera, 1,
1
2
, 2, 3,
1
3
,
1
4
,
2
3
,
3
2
, 4, 5,
1
5
,
1
6
, ....Si bn denota el enesimo
termino de su sucesion, nos gustara probar que
_
b
1/n
n
_
converge a 1.
1 1/2 1/3 1/4 1/5

2 2/2 2/3 2/4 2/5

3 3/2 3/3 3/4 3/5

4 4/2 4/3 4/4 4/5

5 5/2 5/3 5/4 5/5
Figura 7.2
En la gura 7.2., se observa que cada elemento en la ultima la es menor o igual a n
y cada elemento en la enesima columna es mayor o igual a 1/n. Tambien si bn toma
lugar en la la i y en la columna j, entonces i n y j n.
Por lo tanto
1
n

1
j
bn i n para todas las n
y en consecuencia,
an
1
n
1/n
=
_
1
n
_
1/n
b
1/n
n
n
1/n
cn
Ahora por el principio del emparedado,
_
b
1/n
n
_
converge a 1.
7.6.2 Suponer que f (x) es una funcion real valuada ,ddenida por 1 < x < 1 tal
que f

(0) existe .Suponer que {an} , {bn} son dos sucesiones tales que
1 < an < 0 < bn < 1, lm
n
an = 0 = lm
n
bn
Probar que
lm
n
f (bn) f (an)
bn an
= f

(0) .
310 CAP

ITULO 7. DESIGUALDADES
Soluci on. El cociente
f (bn) f (an)
bn an
puede ser interpretado geometricamente como la pendiente del segmento de la lnea
Pn (an,f (an)) , Qn (bn, f (bn)) (ver gura 7.3.)
Si R es el punto (0, f (0)) . Cualquiera que sea y la interseccion del segmento de recta
PnQn es igual o menor a f (0) (caso 1 ), o es mayor que f (0) (caso 2).
En el primer caso,
Pendiente RQn Pendiente PnQn Pendiente PnR,
o su equivalente
f (bn) f (0)
bn 0

f (bn) f (an)
bn an

f (an) f (0)
an 0
.
En el segundo caso,
Figura 7.3.
Pendiente PnR Pendiente PnQn Pendiente RQn,
o su equivalente,
f (an) f (0)
an 0

f (bn) f (an)
bn an

f (bn) f (0)
bn 0
.
En el caso 2, las desigualdades son justamente contrarias a aquellas del caso 1. Para
corregir esto deniremos dos nuevas sucesiones que son opuestas a los roles de an y
bn en el caso 2. As dejamos a {cn} y {dn}denidas por
cn = bn y dn = an Si el caso 1 es cierto para an y bn,
cn = an y dn = bn Si el caso 2 es cierto para an y bn.
7.6. EL PRINCIPIO DEL EMPAREDADO 311
Entonces para toda n,
f (cn) f (0)
cn 0

f (bn) f (an)
bn an

f (dn) f (0)
dn 0
.
desde que f

(0) existe y desde que lmncn = 0 = lmndn,


lm
n
f (cn) f0
cn 0
= f

(0) y lm
f (dn) f (0)
dn 0
= f

(0) .
El resultado se sigue del principio del emparedado.
Otra solucion instructiva, tambien basada en el mismo principio, se da por el hecho
de que si a y b son n umeros reales, a < b, entonces
a ra +sb b
para todos los n umeros positivos r y s que sumados den 1 (ver 1.2.11). En este prob-
lema, se tiene
f (bn) f (an)
bn an
=
_
f (bn) f (0)
bn
__
bn
bn an
_
+
_
f (an) f (0)
an
__
an
bn an
_
.
y tomando r = bn/ (bn an) y s = an/ (bn an) . Entonces r 0, s 0 y
r + s = 1. Por lo tanto [f (bn) f (an)] / [bn an] caen entre [f (bn) f (0)] /bn y
[f (an) f (0)] /an. Ya que estos ultimos cocientes convergen a f

(0) y tambien lo
hace [f (bn) f (an)] / [bn an] por el principio del emparedado.
7.6.3 Evaluar
lm
n
n
2

j=1
n
n
2
+j
2
.
Soluci on. La suma
n
2

j=1
n
n
2
+j
2
=
n
2

j=1
_
1/n
1 + (j/n)
2
_
puede ser considerada como una suma de Riemannn para la funcion f (x) = 1/
_
1 +x
2
_
sobre el intervalo [0, n] (ver gura 7.4 ) Desafortunadamente no es una suma Riemann,
porque el intervalo sobre el cual esta tomada no es un intervalo reparado; as cuando
n , no obtenemos una integral denida. Podemos decir sin embargo que para
toda n
n
2

j=1
_
n
n
2
+j
2
_

_
n
2
0
dx
1 +x
2
= arctan n
2
.
312 CAP

ITULO 7. DESIGUALDADES
Figura 7.4.
Para obtener una cota inferior para la suma en consideracion, sea k un entero positivo
jo y el intervalo respectivo [0, k] . Entonces, para cualquier n mas grande que k,
kn

j=1
n
n
2
+j
2
=
kn

j=1
1/n
1 + (j/n)
2
es una suma Riemann por f (x) = 1/
_
1 +x
2
_
sobre el intervalo [0, k] .Tambien.
kn

j=1
n
n
2
+j
2
<
n
n
2
+j
2
.
Poniendo todo esto junto, tenemos
kn

j=1
n
n
2
+j
2
<
n
2

j=1
n
n
2
+j
2
< arctan n
2
.
entonces por el principio del emparedado.
lm
n
kn

j=1
1/n
1 + (j/n)
2
lm
n
n
2

j=1
n
n
2
+j
2
lm
n
arctan n
2
,
k

0
dx
1 +x
2
lm
n
n
2

j=1
n
n
2
+j
2

1
2
.
Esto nos lleva a los lmites deseados iguales a
1
2
.
7.6. EL PRINCIPIO DEL EMPAREDADO 313
Otra importante aplicacion de desigualdades para la evaluacion de lmites esta basada
en el hecho siguiente.
Toda sucesion monotona y acotada converge.
Es decir, si {an} es una sucesion de n umeros reales tales que an+1 an para toda n
sucientemente grande (o an+1 an para toda n sucientemente grande) y si para
alguna constante K, an K para toda n (o an K respectivamente), entonces la
sucesion {an}converge.
Por ejemplo, para probar que la sucesion (1 + 1/n)
n
converge, es suciente probar que
esta es monotona creciente (incrementando en este caso ) y acotada superiormente
(por 3; ver 7.1.5).
7.6.4 Si an es una sucesion tal que para n 1
(2 an) an+1 = 1.
probar que lmnan existe y es igual a 1.
Soluci on. Primero probaremos que si la sucesion converge, debe converger a 1. El
argumento es estandar cuando una sucesion es denida recursivamente como aqu.
Supongase que lmn = L. Entonces tomando el lmite de cada lado de la relacion
de recurrencia (2 an) an+1 = 1,vemos que (2 L) L = 1, o equivalente, (L1)
2
= 0,
de lo cual concluimos que L = 1.
Ahora, para probar que la sucesion converge, probaremos que esta acotada y que
eventualmente se vuelve monotona. (Para otra solucion a este problema, ver 1.1.11)
Supongase que para algunas an, 0 < an < 1. Entonces
an+1 an =
1
2 an
an =
1 (2 an) an
2 an
=
(1 an)
2
2 an
> 0
y
an+1 = 1/ (2 an) < 1
Luego, an < an+1 < an+2 < ... < 1, entonces la sucesion es monotona y acotada, por
lo tanto converge. De esta manera es suciente probar que para algunas n, 0 < an < 1.
Hay algunos casos:
Si a1 < 0, entonces 0 < a2 = 1/ (2 a1) < 1, lo hemos hecho anteriormente.
Si a1 > 2, entonces a2 = 1/ (2 a1) < 0, volvemos al caso anterior.
Si a1 = 1, entonces an = 1 para todas las n.
314 CAP

ITULO 7. DESIGUALDADES
Queda por revisar el caso 1 < a1 2 , experimentando con algunos casos especiales
en este intervalo lleva a los siguientes (cada uno de los cuales puede ser probado por
induccion ).
La sucesion no esta denida si a1tiene la forma (n + 1) /n. Si a1 = (n + 1) /n, entonces
(uno puede mostrar) que an = 2 y consecuentemente an+1no esta denida. Si a1
pertenece al intervalo
_
n+1
n
,
n
n1
_
para n > 1, entonces (uno puede mostrar que )
an+1 cae en el intervalo (0, 1) y la prueba esta completa por el razonamiento previo .
De esta manera, en todos los casos (para los cuales la sucesion esta denida) la sucesion
converge.
7.6.5 Sea f(x) una funcion tal que f(1) = 1 y para x 1
f

(x) =
1
x
2
+f
2
(x)
.
Probar que lmxf (x) existe y es menor que 1 +
1
4
.
Soluci on. Por el teorema fundamental de calculo
f (x) f (1) =
_
x
1
dx
x
2
+f
2
(x)

_
x
1
dx
1 +x
2
= arctan x |
x
1
= arctan x arctan 1
<
1
2

1
4
=
1
4

De esta manera ,f (x) esta creciendo y rebotando arriba de 1 +


1
4
y en consecuencia
lmxf (x) existe y es menor que 1 +
1
4
.
7.6.6 Considerar todos los n umeros naturales los cuales representados en el sistema
decimal no tienen 9 entre sus dgitos. Probar que las series formadas por los recprocos
de estos n umeros convergen.
Soluci on. Suponer que smdenota la milesima suma parcial de la serie en consid-
eracion. La sucesion {sm} esta aumentando monotonamente, entonces para probar su
convergencia, solamente necesitamos probar que la sucesion esta acotada.
Para una suma parcial dada sm, supongase que n denota el n umero de dgitos en el
entero m. El n umero de enteros de exactamente n dgitos los cuales no tienen 9 en su
representacion decimal es 8 9
n1
(el primer dgito no puede ser cero).Por lo tanto la
suma de sus recprocos es menor que 8 9
n1
/10. De esta manera
sm < 8 + 8
9
10
+ 8
_
9
10
_
2
+... + 8
_
9
10
_
n1
< 8
_
1 +
9
10
+
_
9
10
_
2
+
_
= 80,
7.6. EL PRINCIPIO DEL EMPAREDADO 315
y la prueba esta completa.
Problemas
7.6.7 Probar las desigualdades, las cuales son aplicacion del principio del emparedado
para evaluar un lmite:
(a)
n

n
2
+n
<
n

i=1
1

n
2
+i
<
n

n
2
+ 1
.
(b) b < (a
n
+b
n
)
1/n
< b (2)
1/n
, 0 < a < b.
(c) e
11/(2n)
< (1 + 1/n)
n
< e
11/(2n)+1/(3n
2
).
7.6.8 Probar que cada una de las siguientes sucesiones convergen y encontrar su
lmite:
(a)

1,
_
1 +

1,
_
1 +
_
1 +

1,
_
1 +
_
1 +
_
1 +

1, ....
(b)

2,
_
2 +

2,
_
2 +
_
2 +

2,
_
2 +
_
2 +
_
2 +

2, ....
7.6.9 Probar que la sucesion {an} denida por
an = 1 +
1
2
+... +
1
n
log n
converge.
7.6.10 Probar que la sucesion {an} denida por
an+1 =
6 (1 +an)
7 +an
converge y encuentra su lmite.
7.6.11 Si a1 y b1 son cualesquiera dos n umeros positivos y defnase {an} y {bn} por
an =
2an1bn1
an1 +bn1
, bn =
_
an1bn1.
Probar que las sucesiones {an} .y {bn} convergen y tienen el mismo lmite.
316 CAP

ITULO 7. DESIGUALDADES
7.6.12 S1 = log a y Sn =

n1
i=1
log (a Si) , n > 1. Muestra que
lm
n
Sn = a 1.
(Sugerencia: Notar que Sn+1 = Sn + log (a Sn) .)
7.6.13 La sucesion Qn (x) de polinomios esta denida por
Q1 (x) = 1 +x, Q2 (x) = 1 + 2x,
y para m 1,
Q2m+1 (x) = Q2m (x) + (m+ 1)xQ2m1 (x) ,
Q2m+2 (x) = Q2m+1 (x) = (m+ 1)xQ2m (x) .
Sea xn la solucion real mas grande de Qn (x) = 0.Probar que {xn}es una sucesion
creciente y que lmnxn = 0.
7.6.14 Probar que si

n=1
a
2
n
convergen, entonces lo hace tambien

n=1
(an/n) .
7.6.15 Probar que
lm
n
2
2
4
2
6
2
... (2n)
2
(1 3) (3 5) ... ((2n 1) (2n + 1))
= lm
n
2
2
4
2
6
2
... (2n)
2
1
2
3
2
5
2
... (2n 1)
2
1
2n + 1
=
1
2
.
(Sugerencia: Para 0
1
2
,
_
/2
0
sin
2n+1
d
_
/2
0
sin
2n
d
_
/2
0
sin
2n1
d.
Aplicar el resultado de 2.5.14 junto con el principio del emparedado.)
Ejemplos adicionales
6.1.5.,6.3.7.,6.4.4.,6.6.2., Seccion 6.8, 6.9.4. Tambien ver los ejemplos de biseccion
repetida en la seccion 6.1.
Captulo 8
GEOMETR

IA.
En este captulo veremos algunas de la tecnicas mas comunes para resolver problemas
en geometra euclidiana. Ademas de los metodos de Eucldes clasicos, veremos como
el algebra, la trigonometra, el analisis, el algebra vectorial y los n umeros complejos
pueden ser herramientas utiles en el estudio de la geometra.
8.1. Geometra Clasica Plana
En esta seccion revisaremos las ideas y metodos caractersticos de la geometra plana
clasica; es decir, el estudio de las propiedades de los trangulos, cuadrilateros y crculos
que son invariantes ante el movimiento, (es decir, traslacion, rotacion y reexion).
Estaremos interesados en la geometra sintetica que se construye sobre la comprension
de las notaciones basicas de congruencia, similitud, proporcion, concurrencia, arcos y
cuerdas de crculos, angulos inscritos, etc. En adicion, enfocaremos nuestra atencion
en la importancia de las tecnicas algebraicas y trigonometricas para llegar a resultados
en la geometra plana euclidiana tradicional.
8.1.1 Encontrar el area de un octagono convexo inscrito en un crculo y que tiene
cuatro lados consecutivos de 3 unidades de longitud y otros cuatro de longitud 2. Dar
la respuesta en la forma r +s

t , con r, s y t enteros positivos.


Daremos algunas soluciones a este problema para ilustrar la variedad de metodos de
que se dispone.
Soluci on 1. Sean A B C D E F G H los vertices mostrados en la gura
8.1.2 donde AB = BC = GH = HA = 3 y CD = DE = EF = FG = 2. Sea O el
317
318 CAP

ITULO 8. GEOMETR

IA.
Figura 8.1.
centro del crculo.
Encontraremos el area de OAB y de ODE. Para esto es suciente encontrar las
alturas OK y OJ. Notese por OK =
1
2
EB; Ya que O es el punto medio de EA y K
es el punto medio de AB. Similarmente OJ =
1
2
AD y por lo tanto, es suciente para
encontrar DI, IA, EI y IB, donde I es la interseccion de AD y EB, por ALA, DCB

= ABI y por lo tanto DI = 2 y IB = 3, mas a un ya que ADE y ABE estan


inscritos en un semicrculo, ADE y ABE son angulos rectos. Por lo tanto, IBA
y EDI son triangulos isoceles rectangulos, y entonces IA = 3

2 y EI = 2

2.
Podemos encontrar el area del octagono
Area = 4
_
1
2
3
_
3 + 2

2
2
__
+ 4
_
1
2
2
_
2 + 3

2
2
__
= 13 + 12

2
Solucion 2. Quiza la Solucion mas simple esta basada en observar que el area del
octagono es igual a la mostrada en la gura 8.2, la cual tiene lados alternados de
longitudes 2 y 3. El area puede calcularse sustrayendo cuatro renglones triangulares
de un cuadrado, o sumando las areas de un cuadrado, cuatro rectangulos, y cuatro
triangulos. As, para el diagrama de la izquierda tenemos
_
x =
3
2

2
_

Area del octagono = (2x + 2)


2
4
_
1
2
x
2
_
= 2x
2
+ 8x + 4
8.1. GEOMETR

IA CL

ASICA PLANA 319


Figura 8.2.
Figura 8.3.
= 2
_
3
2

2
_
2
+ 8x
3
2

2 + 4
= 13 + 12

2
o, trabajando sobre la gura de la derecha
_
y =

2
_

Area del octagono = 9 + 4 (3y) + 4


_
1
2
y
2
_
320 CAP

ITULO 8. GEOMETR

IA.
= 9 + 12

2 + 2 2
= 13 + 12

2
Solucion 3. Sea R el radio del crculo, el area del octagono es igual a 4 veces el
area en el cuadrilatero OABC (ver gura 8.3), es claro que

Area OABC =

Area
OAC+

Area ABC
Mediante la formula de Heron para el area de un triangulo,

Area ABC=
_
s (s 2) (s 3)
_
s

2R
_
donde
s =
1
2
_
2 + 3 +

2R
_
=
5
2
+
1
2

2R.
Esto conduce a

Area OABC =
1
2
R
2

_
5
2
+
1
2

2R
__
1
2
+
1
2

2R
__

1
2
+
1
2

2R
__
5
2

1
2

2R
_
=
1
2
R
2
+

_
25
4

1
2
R
2
__

1
4
+
1
2
R
2
_
Por la ley de los cosenos (usando B en ABC) obtenemos
2R
2
= 4 + 9 2 2 3 cos 135

= 13 + 12
1
2

2
8.1. GEOMETR

IA CL

ASICA PLANA 321


y por lo tanto
R
2
=
13
2
+ 3

2
El resultado nal se obtiene despues de substituir este valor para R
2
en la ecuacion
anterior para el area OABC.
Solucion 4. En la gura 8.4, D y E son los pies de las perpendiculares desde B
hasta OA y OC respectivamente. Sea x = OE y y = OD y sea R el radio del crculo.
Entonces:

Area del octagono = 4 (area del cuadrilatero OABC)


= 4 (area OAB + area OCB)
= 4
_
1
2
Rx +
1
2
Ry
_
= 2R(x +y)
Figura 8.4.
Nuestro plan es expresar x+y en terminos de R, y luego usar el hecho de que R
2
=
13
2
+
3

2 (ver Solucion anterior). El teorema de Pitagoras aplicado a ABD conduce a x


2
+
(R y)
2
= 4 o, equivalentemente, 2R(R y) = 4 (Nota: x
2
+y
2
= R
2
). Similarmente,
de EBC tenemos y
2
= 9 (R x)
2
, o equivalentemente, 2R(R x) = 9. Sumando
322 CAP

ITULO 8. GEOMETR

IA.
R y =
4
2R
y R x =
9
2R
se obtiene 2R (x +y) =
13
(2R)
. O equivalentemente
x +y =
(4R
2
13)
(2R)
. Sustituyendo, encontramos:

Area del octagono = 2R


_
4R
2
13
2R
_
= 4R
2
13
= 4
_
13
2
+ 3

2
_
13 = 13 + 12

2
Solucion 5. El octagono puede cortarse como un pastel en ocho pedazos triangulares,
con lados de longitud R (igual al radio del crculo circunscrito) con bases 3 y 2. Sea
H y h las alturas de esos triangulos como se muestra en la gura 8.5. Luego

Area del octagono = 4


_
1
2
3H
_
4
_
1
2
2 h
_
= 6 H + 4 h
Con y como se muestra en la gura 8.5, tenemos las siguientes relaciones: + =

4
;
sin =
3
(2R)
; cos =
H
R
; sin =
1
R
; cos =
h
R
.
De aqu, encontramos:
R =
1
sin
=
1
sin
_
1
4

_ =
1
1
2

2 cos
1
2

2 sin
=
2

2
_
1
cos sin
_
=
2

2
_
1
4
R

3
2
R
_
=
2

2
_
2R
2H 3
_
Se sigue que
1 =
4

2 (2H 3)

O, equivalentemente:
H =
3
2
+

2
8.1. GEOMETR

IA CL

ASICA PLANA 323


Figura 8.5.
Usando esto
h = Rcos = R
_
cos
_
1
4

__
= R
_
1
2

2 cos +
1
2

2 sin
_
=
1
2

2R
_
H
R

3
212
_
=
1
4

2 [2H + 3]
=
1
4

2
_
2
_
3
2
+

2
_
+ 3
_
= 1 +
3
2

2
Sustituyendo

Area del octagono = 6


_
3
2
+

2
_
+ 4
_
1 +
3
2

2
_
= 13 + 12

2
8.1.3 Si A y B son puntos en un crculo dado y XY es un diametro variable del
mismo crculo, determinar los puntos en la interseccion de las lneas AX y BY . (Se
debe asumir que AB no es un diametro).
324 CAP

ITULO 8. GEOMETR

IA.
Soluci on . Considere la gura 8.6, donde A y B son puntos en la circunferencia de
un crculo dado. B

es el punto diametralmente opuesto al punto B. P y P

denotan
respectivamente la interseccion de AX y BY cuando esta interseccion queda dentro o
fuera del crculo (dependiendo a que lado de la linea BB

quede el punto X; ver gura


8.6)
En el primer caso, APB = 90

+
1
2
(arco AB), y este es un valor constante para todos
los diametros que resulten dentro de la interseccion del punto P. Esto implica que P
esta sobre el circulo formado por aquellos puntos que forman un angulo constante (es
decir, 90

1
2
(ArcoAB)) con la base constante AB.
En el segundo caso, AP

B = 90

1
2
(arco AB) , y este es un valor constante para
todos los diametros
Figura 8.6.
Figura 8.7.
As, P

permanece sobre el crculo que pasa por A y B. Mas a un, APB y AP

B
son suplementarios ( APB +AP

B = 90

+
1
2
(ArcoAB)+)90

1
2
(ArcoAB), y
por lo tanto APBP

es un cuadrilatero cclico; es decir, P y P

estan sobre el mismo


crculo por el que pasa A y B.
8.1. GEOMETR

IA CL

ASICA PLANA 325


8.1.4 P es un punto interior del angulo cuyos lados son los segmentos OA y OB.
Localizar X sobre AB y Y sobre OB , de tal manera que el segmento XY contiene a
P y tal que el producto de las distancias (PX) (PY )es minimo.
Soluci on . sea OC la bisectriz del anguloAOB, y sea L la linea que pasa por
P y que es perpendicular a OC sean X yY las intersecciones de L con OA y OB
respectivamente (ver gura 8.7).
luego, OX igual a OY , asi que hay un circulo tangente a OA en X y OB en Y . Sean
X1Y1cualquier otro segmento que contenga a P con X1sobre OA y Y1 sobre O. Sean X2
y Y2 las intersecciones de X1Y1 con el circulo. Entonces (PX) (PY ) = (PX2) (PY2) <
(PX1) (PY1),de tal manera que (PX) (PY )es el mnimo.
8.1.5 Sea P un punto interior del triangulo ABC, y sean x, y, z las distancias de P
a BC,AC, AB respectivamente. Donde debe localizarse C para maximizar el producto
XY Z?
Soluci on . sean a, b, c las longitudes de los lados BC, AC, AB respectivamente (gu-
ra8.8). Por la desigualdad media aritmetica-media geometrica,
3
_
(ax)(by)(cz)
ax +by +cz
3
Figura 8.8.
Pero sabemos que ax + by + cz = 2A,donde A es el area del triangulo. As, el valor
maximo de xyz es 8A
3
/(27abc), y esto ocurre si y solo si ax = by = cz.
mostraremos que ax = by = cz si y solo si P esta locaolizado en el centroide de ABC.
Para esto, supongase que CP intersecta a AB en D. Sean , , los angulos como se
muestra en la gura 8.9. Se sabe que
b sen
a sen
=
AD
DB
326 CAP

ITULO 8. GEOMETR

IA.
(Esta relacion es util en muchos problemas. Para ver que esto es cierto, aplicar la ley
de los senos a ADC y a ADC para encontrar
AD
sen
=
b
sen
y
DB
sen
=
a
sen
.
Usando estas ecuaciones se sigue que
b sen
a sen
=
AD sen
DB sen
=
AD
DB
,
ya que y son suplementarios.)
Usando las ecuaciones anteriores, tenemos
AD
DB
=
b sen
a sen
=
by/(CP)
ax/(CP)
=
by
ax
y se sigue que AD = DB si y solo by = ax. As, ax = by si y solo si P esta sobre la
mediana desde C.
Figura 8.9.
De manera similar, ax = cz si y solo si P esta sobre la mediana desde B. se concluye
que ax = by = cz si y solo si P es el centroide de ABC.
Problemas
8.1. GEOMETR

IA CL

ASICA PLANA 327


Figura 8.10.
8.1.6 Mostrar que un triangulo debe ser equilatero si cualquier par de los siguientes
centros coincide: incentro, circumcentro, centroide, ortocentro.
8.1.7 Un triangulo acutangulo esta inscrito en un crculo. Los tres arcos menores
resultantes son reejados sobre los lados correspondientes del triangulo (es decir, el
arco AB es reejado sobre el lado AB, etc.; ver gura 8.10). son concurrentes los
arcos reejados?
8.1.8 Sean C1 y C2 crculos de radio 1, tangentes entre si y tangentes al eje-x, con
el centro de C1 sobre el eje-y. Ahora construir una secuencia de crculos Cn tales que
Cn+1 es tangente a Cn1, Cn, y al eje-x.
(a) Encontrar el radio rn de Cn.
(b) Mostrar que la longitud de la tangente incluida entre los contactos con dos crcu-
los consecutivos Cn, Cn+1 es
_
n
2
_
1
para n 2.
(c) Para la parte (b) y la geometra del problema, mostrar que

n=2
_
n
2
_
1
= 2.
8.1.9 Si a, b, c son los lados de un triangulo ABC, ta, t
b
, tc son las bisectrices, y
Ta, T
b
, Tc son las extensiones de las bisectrices hasta que son cuerdas del crculo cir-
cumscrito del triangulo ABC, probar que
abc =

TaT
b
Tctat
b
tc.
(Sugerencia: Probar que Tata = bc, etc.)
8.1.10 .
328 CAP

ITULO 8. GEOMETR

IA.
Figura 8.11.
(a) Sea P un punto interior al angulo XOY . Sea AB un segmento que pase por P
con A sobre OX y B sobre OY , y tal que AP = PB. Sea MN otro segmento
a traves de P con M sobre OX y N sobre OY . Probar que el area del triangulo
MON es mayor o igual al area del triangulo AOB.
(b) Sean las tangentes desde un punto A fuera de un crculo dado que tocan al
crculo en los puntos D y E. tomar P en cualquier punto sobre el arco menor
DE, y sea las tangentes en P que intersectan a AD en B y a AE en C. Mostrar
que el permetro de ABC es mas peque no que el permetro de AMN.
(c) En el inciso anerior, sea MN cualquier lnea que pase por P y que interseque
a AD y a AE en M y N repectivamente. Probar que el permetro de ABC es
menor que el permetro de AMN.
8.1.11 Un cuadrilatero ABCD es inscrito en un crculo (gura 8.11). Sea x = BD,
y y = AC, y a, b, c, d las longitudes de los lados indicados. Construir CDE igul a
ADB.
(a) Probar que CDEADB y de ahi que EC x = ac.
(b) Probar que ADEBCD y de ahi que AE x = bd.
(c) A partir de (a) y (b), probar el teorema de Ptolomeo (un hecho importante
acerca de los cuadrilateros cclicos): En un cuadrilatero cclico el producto de
las diagonales es igual a la suma de los productos de los lados opuestos.
8.1.12 .
(a) Una lnea desde el vertice A de un triangulo equila tero ABC cae sobre el lado
opuesto BC en el punto P y en el c rculo circumscrito en Q. Probar que
1
PQ
=
1
EQ
=
1
CQ
.
8.1. GEOMETR

IA CL

ASICA PLANA 329


(b) Usando la notacion de la parte (a), probar que AQ
4
+BQ
4
+CQ
4
es constante
para todas las posiciones de Q en el arco menor BC. (Sugerencia: mediante una
aproximacion trigonometrica, sean
x = AQ, y = BQ, z = CQ, y = BAQ
Mostrar que x = (2/

3)sen, z = (1/

3)(cos sen), y = x + y, Tambien,


Figura 8.12.
ver 8.4.6.)
8.1.13 En la gura 8.12 tenemos un triangulo inscrito ABC. Sea R el radio de la
circunferencia circunscrita: denotar la altitud AD como ha.
(a) Mostrar que los triangulos ABD y ALC son semejantes, y de ah que ha 2R =
bc.
(b) Mostrar que el area de ABC es abc/4R.
8.1.14 El radio del crculo inscrito de un triangulo es 4, y los segmentos en los cuales
un lado es dividido por el punto de contacto miden 6 y 8. Determinar los otros dos
lados.
8.1.15 Los triangulos ABC y DEF estan inscritos en el mismo crculo. Probar que
senA+senB +senC = senD +senE +senF
si y solo si los permetros de los triangulos dados sean iguales.
8.1.16 En la siguiente gura, CD es una media cuerda perpendicular al di ametro
AB del semicrculo con centro O. Un crculo con centro P esta inscrita como se
muestra en la gura 8.13, tocando AB en E y al arco BD en F. Probar que EDA
es isosceles. (Sugerencia: hacer la gura y usar el teorema de pitagoras.)
330 CAP

ITULO 8. GEOMETR

IA.
Figura 8.13.
8.1.17 Encontrar la longitud de un lado de un triangulo equilatero en el cual las
distancias desde sus vertices hasta un punto interior son 5, 7 y 8.
Ejemplos Adicionales
1.2.1, 1.3.14, 1.4.2, 1.6.1, 1.6.10, 1.8.3, 1.8.7.
8.2. Geometra Analtica
La introduccion de un sistema coordenado hace posible atacar muchos problemas de
geometra por medio de algebra y analisis.
8.2.1 Sea P un punto sobre una elipse con focos F1yF2, y sea d la distancia desde el
centro de la elipse en P (gura 8.14). Probar que (PF1)(PF2)d
2
es constante cuando
P se mueve sobre la elipse.
Soluci on. Colocar las coordenadas sobre el plano de tal manera que la elipse tenga
la ecuacion
x
2
a
2
+
y
2
b
2
= 1, 0 < b a.
La tecnica es poderosa: calcular PF1, PF2, y d (como funciones de la coordenada x
de P), y observar si el producto requerido es una constante.
Sean (, ) las coordenadas de P. los puntos focales tienen coordenadas (c, 0), donde
c
2
= a
2
b
2
. Por lo tanto tenemos
PF =
_

2
+ ( c)
2
PF =
_

2
+ ( c)
2
8.2. GEOMETR

IA ANAL

ITICA 331
Figura 8.14.
para encontrar d, es necesario escribir la ecuacion de la tangente a la elipse en P(, ).
Para encontrar la pendiente de la tangente en P, calculamos la derivada:
2x
a
2
+
2y
b
2
y

= 0
as que
y

=
2x/a
2
2y/b
2
=
b
2
x
a
2
y
.
entonces la ecuacion de la tangente en P(, ) es
y =
b
2

a
2

(x ) .
o equivalentemente
a
2
y +b
2
x = b
2

2
+a
2

2
Pero
2
/a
2
+
2
/b
2
= 1, ya que P(, ) es un punto sobre la elipse, y por lo tanto

2
b
2
+a
2

2
= a
2
b
2
. de aqu la ecuacion de la tangente en P(, ) es
b
2
x +a
2
y a
2
b
2
= 0
Ahora recordando la formula para la distancia D de un punto Q(c, d) a la lnea Ax +
By +C = 0
D =
|Ac +Bd +C|

A
2
+B
2
en nuestro caso, la distancia d desde el origen a la lnea tangente es
d =
a
2
b
2
_

2
b
4
+
2
a
4
Ahora necesitamos examinar el producto d
2
(PF1)(PF1). Podemos eliminar en cada
uno de esos factores, ya que

2
=
a
2
b
2

2
b
2
a
2
tenemos
d
2
=
a
4
b
4

2
b
4
+ ((a
2
b
2

2
b
2
) /a
2
) a
4
332 CAP

ITULO 8. GEOMETR

IA.
=
a
4
b
4

2
b
4
+a
4
b
2

2
a
2
b
2
=
a
4
b
4
(
2
b
2

2
a
2
)b
2
+a
4
b
2
=
a
4
b
4
(c
2

2
)b
2
+a
4
b
2
=
a
4
b
2
a
4
c
2

2
,
y
PF
2
1
=
2
+ ( +c)
2
=
a
2
b
2

2
b
2
a
2
+
2
+ 2c +c
2
=
a
2
b
2

2
b
2
+a
2

2
+ 2a
2
c +a
2
c
2
a
2
=
a
2
_
b
2
+c
2
_
+
2
_
a
2
b
2
_
+ 2a
2
c
a
2
=
a
4
+ 2a
2
c +
2
c
2
a
2
=
_
a
2
+c
_
2
a
2
similarmente
PF
2
1
=
_
a
2
c
_
2
a
2
as
(PF1)(PF2)d
2
=
a
4
b
2
a
4
c
2

2
_
_
a
2
+c
_
a
2
__
_
a
2
a
2
c
_
a
2
_
= a
2
b
2
.
Esto completa la prueba.
8.2.2 Supongase que (x1, y1) , (x2, y2) , (x3, y3) son tres puntos de la parabola y
2
= ax
la cual tiene la propiedad de que sus lneas normales se intersecan en un punto com un.
Probar que y1+y2+y3 = 0. Soluci on. Sean (, ) las coordenadas de la interseccion de
las tres lneas normales (gura 8.15), y sean (x,y) un punto arbitrario sobre la parabola.
La pendiente de la lnea que va de (x,y) a (, ) es (y )/(x ). la pendiente de
la tangente a la parabola en (x, y) es y

= a/ (2y), y por lo tanto, la pendiente de la


lnea normal en (x, y) es 2y/a. Se sigue que (x1, y1) , (x2, y2) , (x3, y3) , satisfacen la
ecuacion
y
x
=
_
2y
a
_
reemplazando x por y
2
/a , esta ecuacion es
y =
_
2y
a
__
y
2
a

_
a
2
(y ) = 2y
3
+ 2ay.
8.2. GEOMETR

IA ANAL

ITICA 333
Figura 8.15.
As y1, y2, y3 son las tres raices de la ecuacion c ubica
2y
3
+a (a 2) y a
2
= 0.
Ahora, recordando como los coecientesde una ecuacion c ubica estan relacionados a
las raices, observamos que y1 +y2 +y3 = 0 (el coeciente de y
2
es cero).
8.2.3 Una lnea recta corta las asntotas de una hiperbola en los puntos A y B y a
la curva en los puntos P y Q. Probar que AP = BQ.
Soluci on. Podemos suponer que la hiperbola y la lnea recta tienen las ecuaciones
xy = 1
x
a
+
y
b
= 1
respectivamente.(La hiperbola puede tomar esta forma mediante un escalamiento
Figura 8.16.
334 CAP

ITULO 8. GEOMETR

IA.
apropiado seguido de una rotacion, cada una de estas transformaciones reproduce una
recta en una recta y conserva las proporciones de los segmentos.)
Las asntotas de la hiperbola son los ejes x y y (gura 8.16); de tal manera que A
es la interseccion x de la lnea y B es la interseccion y. Sean (x1, y1) y (x2, y2) las
coordenadas de P y Q. sustituyendo y = 1/x en (2) tenemos
x
2
ax +a/b = 0
y dado que x1 y x2 son las raices de la ecuacion, sabemos que
x1 +x2 = a
Similarmente, sustituyendo x = 1/y en (2) tenemos
y
2
ay +b/a = 0
y esto implica que
y1 +y2 = b
entonces
AP
2
= (x1 a)
2
+y
2
1
= (a x2 a)
2
+ (by
1
)
2
= x
2
2
+ (b y2)
2
= BQ
2
y el resultado se sigue inmediatamente.
8.2.4 Determinar todas las lneas rectas que estan sobre la supercie z = xy.
Soluci on. La ecuacion parametrica de la lne que pasa por (a1, a2, a3) con direccion
(d1, d2, d3) esta dado por
x = a1 +d1t
y = a2 +d2t
z = a3 +d3t
para cada lnea sobre la supercie z = xy es necesaro y suceinte que para toda t,
a3 +d3t = a2(a1 +d1t)
o
z = a2x
si d1 = 0 entonces
a3 +d3t = a1(a2 +d2t)
o
z = a1y
As, las unicas lneas sobre la supercie z = xy son de la forma z = ax, y = a o de la
forma z = ay, x = a, donde a es una constante arbitraria.
8.2. GEOMETR

IA ANAL

ITICA 335
8.2.5 Un triangulo equilatero ABC es proyectado ortogonalmente desde un plano
P a otro plano P

. Mostrar que la suma de los cuadrados de los lados del triangulo


resultante A

(Figura 8.17.) es independiente de la orientacion del triangulo ABC


en P.
Soluci on. Primero, algunas observaciones acerca de como las longitudes se transfor-
man bajo esta proyeccion. Supongase que AB es un segmento de lnea en P de longitud
uno, y que forma un angulo con la lnea L de interseccion de P y P

. Sea el angulo
entre los planos. Colocar C de tal manera que ABC es rectangulo, y AC es paralelo
a L (ver gura). El triangulo ABC se proyectara en un triangulo rectangulo A

.
Mas aun, AC y A

tienen la misma longitud, y B

= BC cos . Ya que AC = cos


y BC = sen, entonces
A

=
_
(cos )
2
+ (sencos )
2
Ahora, denotemos ABC como un triangulo equilatero arbitrario en P. Podemos supon-
er que la longitud del lado es 1. Sopungamos que AB forma un angulo con L. Luego
Figura 8.17.
BC y AC formaran angulos +
1
3
y +
2
3
con L. Aplicando el resultado obtenido
arriba, encontramos que la suma de los cuadrados de los lados del triangulo A

es
_
(cos )
2
+ (sencos )
2

+
_
_
cos( +
1
3
)
_
2
+
_
sen( +
1
3
) cos
_
2
_
+
_
_
cos( +
2
3
)
_
2
+
_
sen( +
2
3
) cos
_
2
_
esto se reduce a
3 cos
2
+
3
2
sen
2

336 CAP

ITULO 8. GEOMETR

IA.
lo cual no depende de .
Problemas
8.2.6 Sea el triangulo ABC inscrito en un crculo, sea P el centroide del triangulo,
y sea O el circuncentro. Suongase que A, B, C tienen coordenadas (0, 0), (a, 0),y (b, c)
respectivamente.
(a) Expresar las coordenadas de P y O en terminos de a, b, c.
(b) Extender los segmentos AP, BP, CP hasta encontrar el crculo en los puntos D,
E, y F respectivamente. Mostrar que
AP
PD
+
BP
PE
+
CP
PF
= 3.
(Sugerencia: Una manera de proceder e la siguiente: llamemos x a OP, y R el
radio de la circunferencia circunscrita. Entonces
AP
PD
+
BP
PE
+
CP
PF
=
AP
2
+BP
2
+CP
2
R
2
x
2
.
Luego expresar cada uno de los terminos del lado derecho en terminos de a, b, c
[usando el resultado (a).])
8.2.7 Encontrar la relacion que debe existir entre los parametros a, b, c, tales que la
lnea x/a +y/b = 1 la cual es tangente al crculo x
2
+y
2
= c
2
.
8.2.8 Triangulos equilateros cuyos lados son 1,3,5,7,.... se encuentran alineados sobre
una lnea recta de tal manera que sus bases se tocan por las esquinas uno despues del
otro. Mostrar que los vertices caen sobre una parabola y que ademas se encuentra a
una distancia entera desde el foco.
8.2.9 .
(a) Se dibujan las tangentes desde dos puntos (a,b) y (c,d) sobre la parabola y=x
2
.
Encontrar las coordenadas de la interseccion.
(b) Dos lneas tangentes, L1 y L2se dibujan desde un punto T hasta una parabola;
sean P y Q los puntos de tangencia de L1 y L2 respectivamente. Sean L cualquier
otra tangente a la parabola, y suponer que L intersecta a L y L en R y S
respectivamente. Probar que
TR
TP
+
TS
TQ
= 1
8.2. GEOMETR

IA ANAL

ITICA 337
8.2.10 Una Parabola con ecuacion y
2
= ax es cortada en cuatro puntos por el crculo
(x-h)
2
+(y-k)
2
=r
2
. Determinar el producto de las distancias de los cuatro puntos de
interseccion dede el eje de la parabola.
8.2.11 Sean b y c n umeros reales jos, y sean diez puntos (j, yj), j = 1, 2, ...., 10,
sobre la parabola y = x
2
+bx+c. Para j = 1, 2, ....., 9,sean Ij los puntos de interseccion
de las tangentes de la parabola dada en (j, yj) y (j + 1, yj+1). Determinar lafuncion
polinomial y = g(x) del ultimo grado cuya graca pase a traves de los nueve puntos
Ij.
8.2.12 Probar o desaprobar: Hay al menos una lnea recta normal a la graca de
y = coshx en el punto (a, cosha) y tambien normal a la graca de y = senhx en un
punto (c, senhc) .
8.2.13 .
(a) Mostrar que las lneas tangentes a la elipse x
2
/a
2
+y
2
/b
2
= 1 tienen la forma
y = x
_
a
2

2
+b
2
_ 1
2
y varian en posicion con diferentes valores de .(el porque de la gran utilidad de
esta forma, particularmente en problemas de tangencia los cuales no involucran
la consideracion del punto decontacto, esta se denomina la ecuacion magica de
la tangente.)
(b) Encontrar la ecuacion de las tangentes a la elipse 3x+y=3 la cual tiene una
pendiente de uno.
(c) Encontrar el area del triangulo formado por una tangente a la elipse (digamos
de pendiente m) y los dos ejes coordenados.
8.2.14 .
(a) Sea D el disco x
2
+ y
2
< 1. sea el punto A con coordenadas (r, 0), donde 0 <
r < 1. Describir el conjunto de puntos P en D tales que el disco abierto cuyo
centro es el punto medio de AP y con radio AP/2 es un subconjunto de D.
(b) sea D el disco x
2
+ y
2
< 1. sean los puntos A y B selecciondos aleatoriamente
en D. Encontrar la probabilidad de que el disco abierto con centro en el punto
medio de AB y radio AB/2 sea un subconjunto de D.
8.2.15 Dada una elipse x
2
/a
2
+y
2
/b
2
= 1 , a = b, encontrar la ecuacion del conjunto
de todos los puntos desde los cuales hay dos tangentes a la elipse cuyas pendientes sean
recprocas.
8.2.16 Si dos cuerdas de una conica se estan bisectando m utumente, probar que la
conica no puede ser una parabola.
338 CAP

ITULO 8. GEOMETR

IA.
8.2.17 Probar que la graca de una ecuacion c ubica es simetrica respecto a su punto
de inexion. (nota: Si la ecuacion c ubica es f(x) = ax
3
+bx
2
+cx +d, la coordenada
x del punto de inexion es b/3a.)
Ejemplos Adicionales
1.3.11, 1.5.3, 1.5.8, 1.6.4, 3.1.4, 4.3.6, 4.3.7.
8.3. Geometra Vectorial
En esta seccion veremos a los vectores como cantidades que tienen magnitud y di-
reccion. Ejemplos de estas cantidades vectoriales son fuerza, velocidad y aceleracion.
Veremos que los vectores tambien pueden ser usados ventajosamente en problemas de
geometra. Representaremos a los vectores por echas (es decir, segmentos de recta
dirigidos) en el plano Euclidiano. La direccion de la echa indica la direccion delk vec-
tor, y la longitud de la echa indica la magnitud del vector. Dos vectores son iguales si
tienen la misma longitud y la misma direccion. Es importante destaca que dos vectores
pueden ser iguales a pesar de ser colineales. Si P y Q son dos puntos, el vector de P a
Q sera denotado por

PQ . La longitud h o magnitud de

PQ sera denotada por |PQ| .


La suma,

A +

B, de los vectores

A y

B esta dada por la ley del paralelogramo (ver


Figura 8.18), o equivalentemente, completando el triangulo como en la Figura 8.19.
La diferencia,

B,

B de

A se muestra en la gura 8.20. Con respecto a las coorde-


Figura 8.18.
nadas cartesianas y denotando el origen por O. Cada punto P del plano determina un
vector unico,

OP, llamado el vector de posicion de P; Para simplicar lo denotaremos


por

P (en vez de

OP). Supongamos que

P y

Q son vectores de posicion de los dos


puntos P y Q (Figura 8.21). Si R es un punto en el segmento de recta dirigido PQ
que divide a PQ en la razon m:n (Figura 8.22). Entonces el vector de posicion de R
8.3. GEOMETR

IA VECTORIAL 339
Figura 8.19.
Figura 8.20.
esta dado por

R =

P +
m
m+n
(

P)
=
(m+n)

P +m(

P)
m+n
=
n
m+n

P +
m
m+n

Q
Es instructivo pensar en R en terminos fsicos de la siguiente manera. Imagine la
barra PQ con una masa de n/(m + n) en P y una masa de (m/n + m) en Q. El
centro de masas del sistema resultante sera el punto X sobre PQ donde (los pesos
estan balanceados) esto es, el punto X donde
n
m+n
PX =
m
m+n
QX
Pero esto es lo mismo que
PX : XQ = m : n
340 CAP

ITULO 8. GEOMETR

IA.
Figura 8.21.
Figura 8.22.
Luego X divide al segmento PQ en la razon m:n, en otras palabras,

X=

R= (n/(m +
n))

P +(m/(m + n))

Q . Los coecientes n/(m + n) y (m/(m + n) pueden ser iden-


ticados como los factores de pesoC reciendo la proporcion de pesoconforme R se
mueve hacia el punto P y decrece la razon m:n, etc..
8.3.1 En un triangulo ABC los puntos D, E, F trisectan los lados tal que BC =
3BD, CA = 3CE, y AB = 3AF (Figura 8.23). Muestre que los triangulos ABC y
DEF tienen el mismo centroide.
Soluci on. Primero probaremos que el vector de posicion del centroide de un triangulo
arbitrario PQR esta dado por 1/3

P +1/3

Q +1/3

R. Veamos, recuerde que el


centroide de un triangulo esta localizado dos tercios desde el vertice P al punto medio
de QR De la discusion precedente sabemos que el vector de posicion del punto medio
de QR esta dado por 1/2

Q +1/2

R, y de aqu tenemos que el vector de posicion del


centroide esta dado por 1/3

P +2/3(1/2

Q +1/2

R) y de aqu el vector de posicion


del triangulo PQR es 1/3

P +1/3

Q +1/3

R, como dijimos. Debido a como fueron


8.3. GEOMETR

IA VECTORIAL 341
Figura 8.23.
denidos D, E y F, tenemos

D=
2
3

B +
1
3

E=
2
3

C +
1
3

F=
2
3

A +
1
3

B
y de aqu
el centroide del triangulo DEF =
=
1
3

D +
1
3

E +
1
3

F
=
1
3
(
2
3

B +
1
3

C) +
1
3
(
2
3

C +
1
3

A)
+
1
3
(
2
3

A +
1
3

B)
=
1
3

A +
1
3

B +
1
3

C
= centroide del triangulo ABC
8.3.2 Probar que es posible construir un triangulo con lados iguales y paralelos a las
medianas de un triangulo dado.
Soluci on. Considerar el triangulo ABC, y sean D, E, F los puntos medios de los lados
BC, AC, y AB respectivamente (ver Figura 8.24). Entonces

AD=

AB +
1
2

BC

BE=

BC +
1
2

AC

CF=

CA +
1
2

AB
342 CAP

ITULO 8. GEOMETR

IA.
Sumando estos, encontramos que

AD +

BE +

CF=

AB +

BC +

CA +1/2(

AC
+

AB +

BC) =

0 +1/2

0= 0 esto implica que los vectores

AD,

BE,

CF forman un
triangulo. Pero

AD,

BE,

CF son iguales en direccion y magnitud a las medianas del


triangulo ABC .
Ahora consideremos el siguiente ejemplo para desarrollar el siguiente principio basico.
Supongamos que P, Q y R son puntos no colineales (Figura 8.25). Si a

PQ +b

PR=
Figura 8.24.
Figura 8.25.
c

PQ +d

PR,entonces a = c y b = d. Para la condicion anterior y si a = c, entonces

PQ= [
db
ac

PR, ]
esto implica que los puntos P, Q y R son colineales (los vectores

PQ y

PR tienen en
com un al punto P) lo cual es una contradiccion. Por lo tanto a = c. De la misma
manera b = d.
8.3. GEOMETR

IA VECTORIAL 343
8.3.3 Probar que la recta que une un vertice de un paralelogramo con el punto medio
del lado opuesto triseca a una diagonal del paralelogramo.
Soluci on. Denotar al paralelogramo por A, B, C, D como muestra la Figura 8.26, sea
F el punto medio de CD y sea E la interseccion de AF y BD. Note que

AB=

DC
y

AD=

BC, debido a que los vectores tienen la misma magnitud y la misma direccion
El punto E como la interseccion de dos lneas. Podemos expresarlo algebraicamente
conociendo la existencia de constantes a y b tales que

AE = a

AF

AE =

AB +b

BD
De aqu,
a

AF=

AB +b

BD
La idea es expresar cada uno de los vectores de esta ultima ecuacion en terminos de

AB y

AD, y entonces haremos uso del principio discutido antes de este problema
Figura 8.26.
Luego tenemos

AB +b(

AD

AB) = a(

AD +1/2

AB)
(1 b)

AB +b

AD =
1
2
a

AB +a

AD
Se sigue que
(1 b) =
1
2
a
b = a
Esta ultima ecuacion implica que a = b = 1/2, y el resultado se sigue.
8.3.4 En el triangulo ABC (Figura 8.27), Sean D y E Los puntos de triseccion del
lado BC con D entre B y E, Sea F el punto medio del lado AC y sea G el punto
344 CAP

ITULO 8. GEOMETR

IA.
Figura 8.27.
medio del lado AB. Sea H la interseccion de los segmentos EG y DF. Encontrar la
razon EH : HG
Soluci on. El plan es exactamente como en el problema precedente, sean a y b con-
stantes tales que

AG +a

GE=

AF +b

FD
Ahora expresemos cada uno de los vectores en terminos de

AB y

AC .

AG =
1
2

AB

GE =

GB +

BE=
1
2

AB +
2
3

BC
=
1
2

AB +
2
3
(

AC

AB) =
2
3

AC
1
6

AB

AF =
1
2

AC

FD =

FA +

AB +

BD=
1
2

AC +

AB +
1
3

BC
=
1
2

AC +

AB +
1
3
(

AC

AB) =
2
3

AB
1
6

AC .
Sustituyendo esto en la ecuacion previa, tenemos
1
2

AB +a(
2
3

AC
1
6

AB) =
1
2

AC +b(
2
3

AB
1
6

AC)
(
1
2

1
6
a)

AB +
2
3
a

AC =
2
3
b

AB +(
1
2

1
6
b)

AC
Se sique que
1
2

1
6
a =
2
3
b
2
3
a =
1
2

1
6
b
o equivalentemente
a + 4b = 3,
4a +b = 3.
8.3. GEOMETR

IA VECTORIAL 345
La solucion de este sistema es a = b = 3/5, y se sigue que EH : HG = 2 : 3.
Figura 8.28.
8.3.5 Dado un triangulo ABC, construir triangulos semejantes ABC

y ACB

exte-
riores sobre las bases respectivas AB y AC, y BCA

interior sobre la base BC (Figura


8.28). Probar que AB

es un paralelogramo.
Soluci on. En lenguaje de vectores, nuestro problema es probar que

AB

AC

AA

.
Sean D, E, F los puntos medios de los lados AB, BC, AC respectivamente. Entonces

AB

AF +

FB

= 1/2

AC +

FB

AC

AD +

DC

= 1/2

AB +

DC

AA

AE +

EB +

BA

AB +1/2(

AC

AB)+

EA

= 1/2

AB +1/2

AC +

EA

.
Para

FB

DC

EA

en terminos de

AB y

AC, introduciremos la siguiente notacion


(usada com unmente en otros problemas, como veremos). Dados puntos P y Q deno-
tamos por

PQ al vector obtenido al rotar el vector

PQ, conservando su magnitud,


un angulo recto en direccion positiva, como en la Figura 8.29. Ahora, supongase que
los triangulos isosceles levantados sobre los lados del triangulo ABC tienen la razon
346 CAP

ITULO 8. GEOMETR

IA.
Figura 8.29.
altura-base igual a k, es decir, FB

/AC = DC

/AB = EA

/BC = k. Entonces,

AB

= 1/2

AC +

FB

= 1/2

AC +k

AC

AC

= 1/2

AB +

DC

= 1/2

AB k

AB

AA

= 1/2

AB +1/2

AC +

EA
= 1/2

AB +1/2

AC +k

BC
= 1/2

AB +1/2

AC +k

AC

AB)
= 1/2

AB +1/2

AC +k

AC k

AB
Estas expresiones para

AB

AC

AA

prueban que

AB

AC

AA

y la solucion
esta completa.
Dados vectores

PQ y

RS el producto punto

PQ

RS esta dado por la formula

PQ

RS= |PQ| |RS| cos


Donde es el angulo entre los vectores, 0 180
o
Se puede probar que para vectores arbitrarios

A,

B,

B=

A
Y

A (

B +

C) =

B +

C
Notese que si

A y

B son perpendiculares, entonces

B= 0. Inversamente si

B= 0,
entonces

A= 0,

B= 0 o

A y

B son perpendiculares. Tambien notese que

A= |A|
2
8.3. GEOMETR

IA VECTORIAL 347
Figura 8.30.
8.3.6 En el triangulo ABC (Figura 8.30), AB = AC, D es el punto medio de BC,
E es el pie de perpendicular desde D a AC y F el punto medio de DE. Probar que
AF es perpendicular a BE.
Soluci on. Este es el mismo problema que 1.5.3, pero aqu daremos una prueba usando
notacion vectorial. Tenemos

AF

BE = (

AE +

EF) (

BD +

DE)
=

AE

BD +

EF

BD +

AE

DE +

EF

DE
= (

AD +

DE)

BD +

EF

BD +

EF

DE
=

AD

BD +

DE

BD +

EF

BD +

EF

DE
=

DE

DC

DE

DC
2

DE

DE
2
=

DE

DC
2

DE

DE
2
=

DE

DC

DE
2
=

DE

EC
2
= 0
El concepto de vector tiene sentido en el espacio Euclidiano justamente como en el
plano Euclidiano, como en el caso del plano, los vectores tienen magnitud y direccion
y son representados por echas o segmentos dirigidos (pero ahora en el espacio de tres
dimensiones). Son sumados por la ley del paralelogramo, y pueden ser manipulados
para probar resultados en geometra de solidos.
8.3.7 Si dos alturas de un tetraedro son coplanares, la arista que une los vertices de
estas dos alturas es perpendicular a la arista opuesta.
348 CAP

ITULO 8. GEOMETR

IA.
Soluci on. Supongase que AP y BZ son las alturas desde A y B respectivamente y
supongase que se intersectan en un punto H (Figura 8.31).

AH es ortogonal a cada
Figura 8.31.
uno de

BC,

CD, y

BD y

BH es ortogonal a

CD,

AD,

AC . Debemos probar que

AB
es ortogonal a

CD . Para esto calculemos el producto punto

AB

CD= (

HB

HA)

CD=

HB

CD

HA

CD= 0 0 = 0.
Esto completa la prueba.
8.3.8 Probar que si los lados opuestos de un cuadrilatero torcido (noplano) tienen las
mismas longitudes. Entonces las lneas que unen los puntos medios de dos diagonales
es perpendicular a esas diagonales.
Soluci on. Denotemos por A, B, C, D los vertices del cuadrilatero, y sean P y Q los
puntos medios de AC y BD respectivamente (Figura 8.32). Sabemos que |AB| = |CD|
y |AC| = |BD|. Elevando al cuadrado, y trasladando esto al lenguaje del producto
punto tenemos que

AD

AD=

BC

BC,

AB

AB=

CD

CD,
o equivalentemente,
(

A) (

A) = (

B) (

B) (8.1)
(

A) (

A) = (

C) (

C) (8.2)
Debemos probar que PQ es perpendicular a AC y BD; en lenguaje de vectores,
debemos probar que

PQ

AC= 0
,

PQ

BD= 0
8.3. GEOMETR

IA VECTORIAL 349
Figura 8.32.
O equivalentemente,
(

P) (

A) = 0
,
(

P) (

B)
Sustituyendo

P= 1/2(

A +

C) y Q = 1/2(

B +

D), estas ecuaciones quedan como sigue


(

B +

C) (

A) = 0 (8.3)
(

B +

C) (

B) = 0 (8.4)
Nuestro problema entonces es equivalente a probar que las ecuaciones (1) implican las
ecuaciones (2). Expandiendo (1) tenemos

D 2

D +

A=

C 2

C +

B 2

B +

A=

D 2

D +

C
Sumando estas, tenemos
2

D 2

B +2

A= 2

C 2

C 2

D
(

B +

D)

A +

A= (

B +

D)

C +

C
(

B +

D) (

A) (

A) = 0
(

B +

D) (

A) (

C +

A) (

A) = 0
(

B +

A) (

A) = 0
Esta es la primera de las dos ecuaciones de (2). La segunda de las ecuaciones de (2) se
obtiene tomando la diferencia de las ecuaciones (1). Los detalles son justamente como
en el calculo previo. De manera similar, sumando y restando las ecuaciones en (2) se
obtienen las ecuaciones en (1), mostrando que el inverso del teorema es verdadero: en
otras palabras, si la recta que une los puntos medios de las diagonales es perpendicular
a ellas, entonces los lados opuestos del cuadrilatero tienen la misma longitud.
Problemas
350 CAP

ITULO 8. GEOMETR

IA.
8.3.9 En un triangulo ABC los puntos D, E, y F trisecan los lados tal que BC =
3BD, CA = 3CE, y B = 3AF. Similarmente los puntos G, H, e / trisecan los lados
del triangulo DEF tal que EF = 3EG, FD

= 3FH, y DE = 3D/, Probar que los


lados del triangulo GHI son paralelos a los del triangulo ABC y que cada uno de los
lados del triangulo pequeo es 1/3 del lado paralelo respectivo del triangulo mayor.
Figura 8.33.
8.3.10 Los lados AD, AB, CB, CD del cuadrilatero ABCD estan divididos por los
puntos E, F, G, H tal que AE : ED = AF : FB = CG : GB = CH : HD. Probar que
EFGH es un paralelogramo.
8.3.11 .
(a) En el triangulo ABC (Figura 8.33), los puntos D y E dividen los lados BC y
AC de tal modo que BD/DC = 3 y AE/EC = 3/2. Si P denota la interseccion
de AD y BE. Encontrar la razon BP : PE.
(b) En el triangulo ABC (Figura 8.33), los puntos E y F dividen los lados AC y
AB respectivamente tal que AE/EC = 4 y AF/FB = 1. Supongase que D es
un punto sobre el lado BC, sea G la interseccion de EF y AD, y supongamos
que D esta situado tal que AG/GD = 3/2, Encontrar la razon BD/DC.
8.3.12 Sobre los lados de un paralelogramo arbitrario ABCD,. Se construyen cuadra-
dos exteriores a el. Probar que los centros M1, M2, M3, M4 son a su vez los vertices de
un cuadrado.
8.3.13 Sobre los lados de un cuadrilatero convexo arbitrario ABCD se construyen
triangulos equilateros ABM1, BCM2, CDM3, y DAM4 tales que el primero y el ter-
cero de ellos son exteriores al cuadrilatero, mientras que el segundo y el cuarto estan
sobre el mismo lado que los lados BC y AD en el mismo cuadrilatero. Probar que el
cuadrilatero M1M2M3M4 es un paralelogramo.
8.3. GEOMETR

IA VECTORIAL 351
8.3.14 Sobre los lados de un cuadrilatero convexo arbitrario ABCD , se construyen
cuadrados todos externos al cuadrilatero, con centros M1, M2, M3, M4- Probar que
Figura 8.34.
M1M3 = M2M4 y que M1M3 es perpendicular a M2M4
8.3.15 Triangulos semejantes BCX, CAY, y ABZ son construidos externamente so-
bre los lados de un triangulo ABC. Probar que el centroide del triangulo ABC y el
centroide del triangulo XY Z coinciden.
8.3.16 Las alturas del triangulo ABC son extendidas externamente a los puntos
A

, B

y C

respectivamente, donde AA

= k/ha, BB

= k/h
b
y CC

= k/hc. Aqu,
k es una constante y ha denota la longitud de la altura del triangulo ABC desde el
vertice A, etc. Probar que el centroide del triangulo A

coincide con el centroide


del triangulo ABC.
8.3.17 Sea ABC un triangulo acutangulo. Construir cuadrados externamente sobre
los tres lados. Extiendase las alturas desde los tres vertices hasta los lados opuestos en
el cuadrado. Entonces el cuadrado queda cortado en dos rectangulos adjuntos. Probar
que los rectangulos adyacentesde diferentes cuadrados tienen la misma area. Esto es,
probar que area i=area i

, para i = 1, 2, 3 (ver Figura 8.35). (Usar el producto punto


dado en al prueba lneas arriba). Que sucede si ABC es un triangulo rectangulo
8.3.18 En un tetrahedro, dos pares de aristas opuestas son ortogonales. Probar que
el tercer par de aristas opuestas tambien son ortogonales.
8.3.19 Sea O un punto dado, sean P1, P2, ..., Pn los vertices de un n-agono regular,
nmayoroigualque7, y sean Q1, Q2, , Qn dados por
OQi = OPi +Pi+1Pi+2, i = 1, 2, ..., n
Pn+1 = P1, Pn+2P2 probar que Q1, Q2, , Qn son los vertices de un n-agono regular.
352 CAP

ITULO 8. GEOMETR

IA.
Figura 8.35.
8.4. Geometra en N umeros Complejos
En esta seccion haremos geometra sobre los n umeros complejos introducidos en la
seccion 3.5
8.4.1 A1, A2, ..., An son los vertices de un polgono regular inscrito en un crculo de
radio r y centro O, P es un punto sobre OA1 mas alla de A1. Probar que
n

k=1
PAk = OP
n
r
n
Soluci on. Considerar la Figura 8.36 como la representacion del plano complejo con el
origen en el centro del crculo, y con los vertices Ai como las n races de z
n
r
n
= 0. Es-
peccamente, en nuestro conjunto, el representante de A
k
sera z
k
= re
2pi(k1)i/n
(El
representante a un punto Qen el plano complejo es el n umero complejo que corresponde
a Q). Con estas coordenadas, P corresponde a un n umero real, que sera denotado por
8.4. GEOMETR

IA EN N

UMEROS COMPLEJOS 353


z. Entonces
n

k=1
PA
k
=
n

k=1
|z z
k
|
=

k=1
(z z
k
)

= |z
n
r
n
|
= z
n
r
n
, (zyrsonrealesyz > r)
= OP
n
r
n
8.4.2 Dado un punto P sobre la circunferencia de un crculo unitario y los vertices
A1, A2, ..., An de un polgono regular inscrito de n lados, probar que PA
2
1
+PA
2
2
+... +
PA
2
n
, es una constante.
Figura 8.36.
Soluci on . . Nuevamente, sean A1, A2, ..., An las correspondientes n races de la
unidad, especcamente el complejo asociado a A
k
es z
k
= e
2ki/n
, k = 1, 2, , n. Si a P
le corresponde el complejo z . Entonces, como para cualquier complejo w |w|
2
= ww,
tenemos
n

k=1
PA
2
k
=
n

k=1
|z z
k
|
2
n

k=1
(z z
k
)(z z
k
)
=
n

k=1
(zz z
k
z z
k
z +z
k
z
k
)
=
n

k=1
zz (
n

k=1
z
k
)z z(
n

k=1
z
k
) +
n

k=1
z
k
z
k
354 CAP

ITULO 8. GEOMETR

IA.
Pero

n
k=1
z
k
= 0, ya que z
k
son las races de z
n
1 = 0 y el coeciente de z
n1
es
cero. Luego,
n

k=1
PA
2
k
=
n

k=1
zz +
n

k=1
z
k
z
k
=
n

k=1
|z|
2
+
n

k=1
|z
k
|
2
= n +n(|z| = 1, |z
k
| = 1)
= 2n.
8.4.3 Probar que si los puntos del plano complejo correspondientes a distintos n umeros
complejos z1 y z2 son los vertices de un triangulo equilatero, entonces el tercer vertice
corresponde a wz1 w
2
z2 donde w es una raz c ubica imaginaria de la unidad.
Soluci on. Los puntos z1, z2, z3 forman un triangulo equilatero si y solamente si z3
z1 = (z2 z1)e
i/3
. Luego dados z1 y z2, z3 debe ser de la forma
z3 = (1 e
i/3
)z1 +e
i/3
z2
= [1 +e
i/3
]z1 [e
i/3
]z2
Podemos ver geometricamente la interpretacion de estas cantidades (Figura 8.37) que
1+e
i/3
y e
i/3
son las races c ubicas imaginarias de la unidad. Alternativamente
podemos vericar esto algebraicamente:
1 +e
i/3
= 1 +cos(pi/3) +isen(pi/3)
= 1 + 1/2 +i

3/2
= 1/2 +i

3/2
y
e
i/3
= [cos(/3) +isen(/3)]
= 1/2 i

3/2
Inversamente, supongase que z3 = wz1 w
2
z2 donde w es una raz c ubica imaginaria
de la unidad. Entonces
w = 1 +e
i/3
yw
2
= e
i/3
o
w = 1 +e
i/3
yw
2
= e
i/3
Y los argumentos previos, prueban que z1, z2, z3 forman un triangulo equilatero.
8.4.4 Triangulos equilateros son levantados sobre los lados de un triangulo arbitrario
ABC. Probar que los centros (centroides) de estos tres triangulos equilateros forman
un triangulo equilatero.
8.4. GEOMETR

IA EN N

UMEROS COMPLEJOS 355


Figura 8.37.
Soluci on. Sean a, b, c los representantes de A, B, C respectivamente (en el plano com-
plejo), con x, y, z los representantes de los centros de los triangulos equilateros como
se muestra en la Figura 8.38. Si w = e
2i/3
. Entonces w
2
+ w + 1 = 0 ( w es una
raz c ubica de la unidad , es decir, 0 = w
3
1 = (w 1)(w
2
+ w + 1) ). Tambien
note que w
2
= e
i/3
y w = e
i/3
El centroide del triangulo ABC tiene asignado
1/3(a +b +c), de manera similar x, y y z estan dadas por
x = 1/3[a +c + [a w
2
(c a)]] = 1/3[(2 +w
2
)a + (1 w
2
)c],
y = 1/3[a +b + [a w(b a)]] = 1/3[(2 +w)a + (1 w)b],
z = 1/3[b +c + [b w(c b)]] = 1/3[(2 +w)b + (1 w)c]
Para probar que x, y, z forman un triangulo equilatero es suciente probar que
z x = w
2
(y x)
Tenemos que
3(z x) = (2 +w
2
)a + (2 +w)b + +(w +w
2
)c
3w
2
(y x) = 3w
2
(x y) = (w
4
w
3
)a (w
2
w
3
)b + (w
2
w
4
)c
Pero
w
4
w
3
= w 1 = (1 w
2
) 1 = (2 +w
2
)
(w
2
w
3
) = (w
2
1) = (1 +w) + 1 = 2 +w
w
2
w
4
= w
2
w
y de aqu, los coecientes de a, b, c en las expresiones de z x y .w
2
(y x) son iguales.
Se sigue que x, y, z forman unb triangulo equilatero.
Problemas
356 CAP

ITULO 8. GEOMETR

IA.
Figura 8.38.
8.4.5 Si A0, A1, A2, A3, A4 dividen a un crculo unitario (crculo de radio 1) en cinco
partes iguales. Probar que las cuerdas A0A1, A0A2 satisfacen
(A0A1 A0A2)
2
= 5
8.4.6 Dado un punto P sobre la circunferencia de un crculo unitario y los vertices
A1, A2, , , An de un polgono regular de n lados, probar que PA
4
1
+ PA
4
2
+ + PA
4
n
es
constante (es decir, independiente de la posicion de P en la circunferencia).
8.4.7 Si G denota el centroide del triangulo ABC. Probar que
3(GA
2
+GB
2
+GC
2
) = AB
2
+BC
2
+CA
2
.
8.4.8 Sea ABCDEF un hexagono en un crculo de radio r. Probar que si AB =
CD = EF =, entonces los puntos medios de r de BC, DE, y FA son los vertices de
un triangulo equilatero.
8.4.9 Si z1, z2, z3 son tales que va(z1) = va(z2) = va(z3) = 1 y z1 + z2, +z3 = 0,
probar que z1, z2, z3 son los vertices de un triangulo equilatero iscrito en un crculo
unitario.
8.4.10 Probar que z1, z2, z3 forman un triangulo equilatero si y solo si
z
2
1
+z
2
2
+z
2
3
= z1z2 +z2z3 +z3z1
8.4. GEOMETR

IA EN N

UMEROS COMPLEJOS 357


8.4.11 Los tres puntos en el plano complejo que corresponden a las races de la
ecuacion
z
3
3pz
2
+ 3qz r = 0
son los vertices de un triangulo.
(a) Probar que el centroide del triangulo es el punto correspondiente a p.
(b) Probar que el triangulo es equilatero si y solamente si p
2
= q.

Anda mungkin juga menyukai